229
El Auditor Interno Certificado Preguntas Examen Modelo 2004

CIA Examen modelo 2004

Embed Size (px)

DESCRIPTION

Examen modelo del año 2004

Citation preview

El Auditor Interno Certificado

Preguntas Examen Modelo 2004

Declaración Copyright © 2004 by The Institute of Internal Auditors Research Foundation, 247 Maitland Avenue, Altamonte Springs, Florida 32701-4201. Todos los derechos reservados. Ninguna parte de este material puede ser reproducido en cualquier formato sin el previo consentimiento por escrito del editor. Permiso ha sido obtenido del sostenedor del copyright para publicar esta traducción, que es igual en todos los respectos del material al original. The IIA publica este documento para fines educativos y de información. Este documento tiene la intención de suministrar información, pero no es un sustituto para asesoramiento legal o contable. The IIA no suministra dicho asesoramiento ni garantiza ningún resultado legal o contable por la publicación de este documento. Cuando surjan asuntos legales o contables, deberá buscarse y mantenerse asistencia profesional. El Marco para la Práctica Profesional fue diseñado por el Grupo Guía del Consejo de Directores del The IIA para organizar apropiadamente la gama total de pautas de práctica existentes y en desarrollo para la profesión. Basados en la definición de auditoría interna, el Marco para la Práctica Profesional comprende Éticas y Normas, Consejos para la práctica y Ayudas para la Práctica y el Desarrollo, y allana el camino hacia la auditoría interna de clase mundial. Estas pautas se adecuan al Marco con el encabezado Ayudas para la Práctica y el Desarrollo.

mentzminger
Text Box
ISBN 0-89413-555-4

iii

Prólogo

Desde que el Consejo de Regentes de The Institute of Internal Auditors (IIA) inició su programa denominado “Auditor Interno Certificado” (Certified Internal Auditor® -CIA®) en diciembre de 1972, el examen CIA fue revisado en ciertas ocasiones a fin de reflejar los cambios en la profesión y en la metodología de prueba. Las preguntas del modelo de examen 2004 reflejan los cambios en el contenido debido a las modificaciones del programa del CIA, que se puso en vigencia con el ciclo de pruebas de mayo de 2004. Las preguntas del modelo de examen 2004 incluyen sólo 100 preguntas por parte; sin embargo, en mayo de 2004, el examen CIA tendrá 125 preguntas por parte. Entre las 125 preguntas, se incluirán hasta 25 preguntas no registradas, que se utilizarán para propósitos de investigación. Dichas preguntas no registradas se insertarán sin identificar entre las registradas. Por lo tanto, los candidatos deben responder las 125 preguntas lo mejor posible. Las preguntas del modelo de examen 2004 constituyen un medio para familiarizar a los interesados con el contenido y el formato del examen CIA. No tienen el propósito de reemplazar el material suministrado por cualquiera de los proveedores de materiales de revisión. Las preguntas de esta publicación, ya sean nuevas o adaptadas de exámenes anteriores, sólo representan el formato, la longitud y el contenido de las preguntas que un candidato CIA puede encontrar en futuros exámenes. El éxito o fracaso que un candidato CIA actual o futuro haya logrado al responderlas no debe considerarse como garantía de los resultados de ese candidato en un examen CIA real. En el caso de producirse cambios significativos en el formato o contenido del examen CIA en el futuro, el Departamento de Certificaciones del IAI los dará a conocer mediante la página web del IAI www.theiia.org o mediante el envío de información por correo a los actuales candidatos CIA. Para mayor información sobre el programa, visite “Certification” en la página web del IAI, o solicite un folleto informativo al Centro de Servicios al Cliente del IAI: The Institute of Internal Auditors Customer Service Center 247 Maitland Avenue Altamonte Springs, Florida 32701-4201, USA Phone: +1-407-937-1111 Fax: +1-407-937-1101 E-mail: [email protected]

v

Contenido

Prólogo................................................................................................................. …iii Temas evaluados.................................................................................................... vii Parte I: El rol de la actividad de auditoría interna en el gobierno, el riesgo y el control Preguntas del examen modelo ............................................................................ I - 1 Soluciones (con referencias cruzadas con los temas evaluados) ..................... I - 39 Parte II: Como conducir el trabajo de auditoría interna Preguntas del examen modelo ........................................................................... II - 1 Soluciones (con referencias cruzadas con los temas evaluados) .................... II - 38

Parte III: Análisis del negocio y tecnología de la información Preguntas del examen modelo .......................................................................... III - 1 Soluciones (con referencias cruzadas con los temas evaluados) ................... III - 29

Parte IV: Destrezas de la administración del negocio Preguntas del examen modelo .......................................................................... IV - 1 Soluciones (con referencias cruzadas con los temas evaluados) ................... IV - 26

vii

Temas de prueba del examen para

Auditor Interno Certificado (CIA)

Las siguientes páginas proveen un detalle de los temas de cada parte del examen para Auditor Interno Certificado (CIA). Se aconseja a los candidatos que planifiquen sus estudios en base a los temas detallados a continuación, más que limitarse a estudiar aquellos que aparecen en el presente Examen Modelo. El examen CIA comprueba los conocimientos de los temas especificados en tres niveles de competencia, según se define a continuación: Nivel bàsico – El candidato muestra sus conocimientos sobre terminología/hechos básicos y una apreciación de la naturaleza y los fundamentos amplios de tema que se está evaluando. No se espera que los candidatos tengan un conocimiento detallado de los temas enumerados. Nivel superior – El candidato puede mostrar la habilidad de aplicar conocimientos específicos a áreas que probablemente aborde y tratar estas áreas sin recurrir a asistencia ni investigaciones técnicas. Se espera que los candidatos demuestren una comprensión profunda de los principios, prácticas y procedimientos de los temas que se están evaluando. El detalle de temas presentado a continuación indica el nivel de competencia requerido para cada una de las áreas. Por favor, observe que el examen CIA evalúa el Marco para la Práctica Profesional (MPP) de The IIA. El MPP consiste en tres categorías de pautas. La primera categoría (Pauta obligatoria) consiste en materiales centrales: el Código de Ética del IAI y las Normas Internacionales para el Ejercicio Profesional de Auditoría Interna (con el glosario). Dichos materiales serán obligatorios. (Por ejemplo, ¿cuál de los siguientes se requiere de acuerdo con las Normas?) Las pautas de la segunda categoría (Consejos para la Práctica) son firmemente recomendadas y aprobadas por The IIA, pero no son obligatorias. Si bien los Consejos para la Práctica no son obligatorios, se espera que los candidatos los conozcan al nivel superior. Al responder las preguntas del examen, los candidatos deben observar que Los Consejos para la Práctica se tratarán como prácticas correctas y se aceptarán como el método apropiado de desempeño. La tercera categoría de pautas (Ayudas para el Desarrollo & la Práctica) no se evaluará específicamente como parte del MPP. Sin embargo, algunos de estos materiales pueden utilizarse como referencias para las preguntas sobre las áreas que cubran.

viii

PARTE I: EL ROL DE LA ACTIVIDAD DE AUDITORÍA INTERNA EN EL GOBIERNO, EL RIESGO Y EL CONTROL

A. Cumplir con las normas sobre atributos del IAI (15-25%) [Nivel superior]

1. Definir el propósito, la autoridad y la responsabilidad de la actividad de auditoría interna a. Determinar si el propósito, la autoridad y la responsabilidad de la actividad de auditoría

interna se documentan y aprueban con claridad. b. Determinar si se comunican el propósito, la autoridad y la responsabilidad de la actividad

de auditoría interna a los clientes del trabajo. c. Demostrar que se comprenden el propósito, la autoridad y la responsabilidad de la

actividad de auditoría interna 2. Mantener la independencia y la objetividad

a. Fomentar la independencia 1) Comprender la independencia organizacional 2) Reconocer la importancia de la independencia organizacional 3) Determinar si la actividad de auditoría interna se alinea apropiadamente para lograr la

independencia organizacional b. Fomentar la objetividad

1) Establecer políticas para promover la objetividad 2) Evaluar la objetividad individual 3) Mantener la objetividad individual 4) Reconocer y mitigar los impedimentos a la independencia y objetividad

3. Determinar si se dispone de los conocimientos, las aptitudes y otras competencias a. Comprender los conocimientos, aptitudes y competencias que un auditor interno debe

poseer b. Identificar los conocimientos, aptitudes y competencias requeridas para cumplir con las

responsabilidades de la actividad de la auditoría interna 4. Desarrollar y/o procurar los conocimientos, aptitudes y competencias necesarios

colectivamente requeridos por la actividad de auditoría interna 5. Ejercer el debido cuidado profesional 6. Promover el desarrollo profesional continuo

a. Desarrollar e implementar un plan para el desarrollo profesional continuo del personal de auditoría interna

b. Mejorar la competencia individual a través del desarrollo profesional continuo 7. Promover el aseguramiento de calidad y la mejora de la actividad de auditoría interna

a. Establecer y mantener un aseguramiento de calidad y un programa de mejora b. Supervisar la eficacia del aseguramiento de calidad y el programa de mejora c. Informar los resultados del aseguramiento de calidad y el programa de mejora al consejo o

a otro organismo gubernamental d. Realizar procedimientos de aseguramiento de calidad y recomendar mejoras al

desempeño de la actividad de auditoría interna 8. Respetar y promover el cumplimiento del Código de Ética del IAI

B. Establecer un plan basado en el riesgo para determinar las prioridades de la actividad de

auditoría interna (15-25%) [Nivel superior] 1. Establecer un marco para evaluar el riesgo 2. Utilizar el marco para:

a. Identificar las fuentes de posibles trabajos (por ejemplo, universo de auditoría, requerimiento de la gerencia, mandato regulador)

b. Evaluar el riesgo al nivel de la organización c. Solicitar los temas de posibles trabajos a partir de diversas fuentes d. Recopilar y analizar datos de los trabajos propuestos e. Clasificar y validar las prioridades de los riesgos

(CONTINÚA EN LA PRÓXIMA PÁGINA)

ix

3. Identificar los requerimientos de los recursos de auditoría interna 4. Coordinar los esfuerzos de la actividad de auditoría interna con:

a. El auditor externo b. Los organismos reguladores de supervisión c. Otras funciones de aseguramiento interno (por ej., departamento de salud y seguridad)

5. Seleccionar trabajos a. Participar en el proceso de selección de trabajos b. Seleccionar trabajos c. Comunicar y obtener la aprobación del plan de trabajos por parte del consejo

C. Comprender el rol de la actividad de auditoría interna en el gobierno organizacional (10-20%)

[Nivel superior] 1. Obtener la aprobación del estatuto de auditoría por parte del consejo 2. Comunicar el plan de trabajos 3. Informar las cuestiones significativas de auditoría 4. Comunicar los indicadores clave de desempeño al consejo en forma regular 5. Conversar sobre las áreas de riesgo significativo 6. Respaldar al consejo en la evaluación de riesgos al nivel empresarial 7. Revisar el posicionamiento de la función de auditoría interna en el marco de administración de

riesgo dentro de la organización 8. Vigilar el cumplimiento del código corporativo de las prácticas de conducta/del negocio 9. Informar sobre la eficacia del marco de control 10. Asistir al consejo en evaluar la independencia del auditor externo 11. Evaluar el clima ético del consejo 12. Evaluar el clima ético de la organización 13. Evaluar el cumplimiento de las políticas en áreas específicas (por ej., derivativas) 14. Evaluar el mecanismo de informes de la organización al consejo 15. Realizar un seguimiento e informar sobre la respuesta de la gerencia a las revisiones de los

organismos reguladores 16. Realizar un seguimiento e informar sobre la respuesta de la gerencia a la auditoría externa 17. Evaluar la adecuación del sistema de medición de desempeño, el logro del objetivo corporativo 18. Respaldar una cultura de toma de conciencia del fraude y estimular para informar

incongruencias D. Realizar otros roles y responsabilidades de auditoría interna (0-10%) [Nivel superior]

1. Ética/cumplimiento a. Investigar y recomendar la resolución para quejas relacionadas con la ética/el

cumplimiento b. Determinar la disposición de las violaciones a la ética c. Fomentar un clima ético saludable d. Mantener y administrar la política de conducta del negocio (por ej., conflicto de intereses) e. Informar sobre el cumplimiento

2. Administración de riesgo a. Desarrollar e implementar un marco de riesgos y controles al nivel de la organización b. Coordinar la evaluación de riesgos al nivel empresarial c. Informar la evaluación de riesgos corporativos al consejo d. Revisar el proceso de planificación de la continuidad del negocio

3. Privacidad a. Determinar las vulnerabilidades de la privacidad b. Informar sobre el cumplimiento

4. Información o seguridad física a. Determinar las vulnerabilidades de la seguridad b. Determinar la disposición de las violaciones a la seguridad c. Informar sobre el cumplimiento

(CONTINÚA EN LA PRÓXIMA PÁGINA)

x

E. Elementos del conocimiento del gobierno, del riesgo y del control (15-25%) 1. Principios del gobierno corporativo [Nivel básico] 2. Marcos de control alternativos [Nivel básico] 3. Vocabulario y conceptos de riesgo [Nivel superior] 4. Técnicas de administración de riesgo [Nivel superior] 5. Implicancias del riesgo/control de diferentes estructuras organizacionales [Nivel superior] 6. Implicancias del riesgo/control de diferentes estilos de liderazgo [Nivel básico] 7. Administración de cambios [Nivel básico] 8. Administración de conflictos [Nivel básico] 9. Técnicas de control gerencial [Nivel superior] 10. Tipos de control (por ej., preventivo, detectivo, entrada, salida) [Nivel superior]

F. Planificar trabajos (15-25%) [Nivel superior]

1. Iniciar una comunicación preliminar con el cliente del trabajo 2. Realizar una encuesta preliminar del área del trabajo

a. Obtener el aporte del cliente del trabajo b. Realizar revisiones analíticas c. Realizar “benchmarking” d. Realizar entrevistas e. Revisar informes de auditoría anteriores y otra documentación pertinente f. Mapear procesos g. Desarrollar listas de verificación

3. Completar una evaluación de riesgos detallada del área (fijar prioridades de los factores de riesgo/control o evaluarlos)

4. Coordinar los esfuerzos del trabajo de auditoría con a. El auditor externo b. Los organismos reguladores de supervisión

5. Establecer/perfeccionar los objetivos del trabajo e identificar/finalizar el alcance del trabajo 6. Identificar o desarrollar los criterios para los trabajos de aseguramiento (criterios mediante los

cuales se debe auditar) 7. Considerar la posibilidad de fraude cuando se planifica un trabajo

a. Estar bien informado sobre los factores de riesgo y las banderas rojas del fraude b. Identificar los tipos comunes de fraude asociados con el área del trabajo c. Determinar si el riesgo de fraude requiere una consideración especial cuando se realiza un

trabajo 8. Determinar los procedimientos del trabajo 9. Determinar el nivel del personal y los recursos necesarios para el trabajo 10. Establecer la planificación y supervisión adecuada del trabajo 11. Preparar el programa del trabajo

Formato: 125 preguntas de opción múltiple

xi

PARTE II: CÓMO CONDUCIR EL TRABAJO DE AUDITORÍA INTERNA A. Realizar trabajos (25-35%) [Nivel superior]

1. Investigar y aplicar las normas apropiadas: a. Marco para la Práctica Profesional del IAI (Código de Ética, Normas, Consejos para la

Práctica) b. Otras normas profesionales, legales y reguladoras

2. Mantener la toma de conciencia de la posibilidad de fraude cuando se realiza un trabajo a. Observar los indicadores o los síntomas de fraude b. Diseñar los pasos apropiados del trabajo para abordar un riesgo significativo de fraude c. Utilizar las pruebas de auditoría para detectar fraudes d. Determinar si cualquier sospecha de fraude merece una investigación

3. Recopilar datos 4. Evaluar la pertinencia, suficiencia y competencia de la evidencia 5. Analizar e interpretar datos 6. Desarrollar los papeles de trabajo 7. Revisar los papeles de trabajo 8. Comunicar el progreso provisional 9. Sacar conclusiones 10. Desarrollar recomendaciones cuando resulte apropiado 11. Informar los resultados del trabajo

a. Realizar una reunión de cierre b. Preparar un informe u otra comunicación c. Aprobar el informe del trabajo d. Determinar la distribución del informe e. Obtener una respuesta al informe por parte de la gerencia

12. Realizar una encuesta de satisfacción del cliente 13. Completar las evaluaciones de desempeño del personal del trabajo

B. Realizar trabajos específicos (25-35%) [Nivel superior]

1. Realizar trabajos de aseguramiento a. Investigación de fraude

1) Determinar partes apropiadas para que participen en la investigación 2) Establecer los hechos y el alcance del fraude (por ej., entrevistas, interrogatorios y

análisis de datos) 3) Informar los resultados a las partes apropiadas 4) Completar la revisión de un proceso para mejorar los controles a fin de evitar el

fraude y recomendar cambios b. Auto-evaluación de riesgos y controles

1) Enfoque a) Enfoque hacia el cliente b) Enfoque hacia la auditoría

2) Enfoque de cuestionario 3) Enfoque de auto-certificación

c. Auditorías de terceras partes y auditorías contractuales d. Trabajos de auditoría de calidad e. Trabajos de auditoría de debido cumplimiento (“due diligence”) f. Trabajos de auditorías de seguridad g. Trabajos de auditorías de privacidad h. Trabajos de auditorías de desempeño (indicadores clave de desempeño) i. Trabajos de auditoría operativa (eficiencia y eficacia) j. Trabajos de auditoría financiera

(CONTINÚA EN LA PRÓXIMA PÁGINA)

xii

k. Trabajos de auditoría de tecnología de la información (TI) 1) Sistemas operativos

a) Unidad central b) Puestos de trabajo c) Servidor

2) Desarrollo de la aplicación a) Autenticación de la aplicación b) Metodología de desarrollo de sistemas c) Control de cambios d) Informática del usuario final

3) Comunicaciones/conexiones de datos y en red (por ej., LAN, VAN y WAN) 4) Comunicaciones por voz 5) Seguridad de sistemas (por ej., muros refractarios, control de acceso) 6) Planificación de contingencias 7) Bases de datos 8) Áreas funcionales de las operaciones de TI (e.j., operaciones de los centros de datos) 9) Infraestructura de la red 10) Licencias de software 11) Transferencia electrónica de fondos (EFT)/Intercambio electrónico de datos (EDI) 12) e-Commerce 13) Protección de la información/virus 14) Claves de encriptación 15) Software de planificación de recursos al nivel empresarial (ERP) (por ej., SAP R/3)

l. Trabajos de auditorías de cumplimiento 2. Realizar trabajos de consultoría

a. Capacitación de control interno b. Revisión de procesos del negocio c. “Benchmarking” d. Tecnología de la información (TI) y desarrollo de sistemas. e. Diseño de sistemas de medición del desempeño

C. Vigilar los resultados del trabajo (5-15%) [Nivel superior]

1. Determinar la actividad de seguimiento apropiada por parte de la actividad de auditoría interna 2. Identificar un método apropiado para vigilar los resultados del trabajo 3. Realizar una actividad de seguimiento 4. Comunicar el plan y los resultados del monitoreo

D. Elementos para detectar el fraude (5-15%)

1. Muestreo de descubrimiento [Nivel básico] 2. Técnicas de interrogatorios [Nivel básico] 3. Auditoría legal [Nivel básico] 4. Uso de computadoras al analizar datos [Nivel superior] 5. Banderas rojas [Nivel superior] 6. Tipos de fraude [Nivel superior]

E. Herramientas del trabajo (15-25%)

1. Muestreo [Nivel básico] a. No estadístico (según el criterio) b. Estadístico

2. Análisis estadísticos (técnicas de control de los procesos) [Nivel básico] 3. Herramientas de recopilación de datos [Nivel superior]

a. Entrevistas b. Cuestionarios c. Listas de verificación

(CONTINÚA EN LA PRÓXIMA PÁGINA)

xiii

4. Técnicas analíticas de revisión [Nivel superior] a. Estimación de razón b. Análisis de varianza (por ej., presupuesto vs. real) c. Otras pruebas de razonabilidad

5. Observación [Nivel superior] 6. Solución de problemas [Nivel superior] 7. Auto-evaluación de riesgos y controles (CSA) [Nivel básico] 8. Herramientas y técnicas computarizadas de auditoría [Nivel superior]

a. Módulos de auditoría incluidos b. Técnicas de extracción de datos c. Software genérico de auditoría (por ej., ACL, IDEA) d. Análisis de hojas electrónicas e. Papeles de trabajo automatizados (por ej., Notas de Lotus, Asistente del Auditor)

9. Mapeo de procesos en el que se incluyen los diagramas de flujo [Nivel superior] Formato: 125 preguntas de opción múltiple

xiv

PARTE III: ANÁLISIS DEL NEGOCIO Y TECNOLOGÍA DE LA INFORMACIÓN

A. Procesos del negocio (15-25%)

1. Gestión de calidad (por ej., TQM) [Nivel básico] 2. Marco de las normas ISO [Nivel básico] 3. Pronóstico [Nivel básico] 4. Técnicas de administración del proyecto [Nivel superior] 5. Análisis de los procesos del negocio (por ej., análisis del circuito de producción y de la

capacidad limitante, teoría de las restricciones) [Nivel superior] 6. Técnicas y conceptos de la administración de inventarios [Nivel superior] 7. Objetivos y políticas de Marketing – fijación de precios [Nivel básico] 8. Marketing – administración de la cadena de suministros [Nivel básico] 9 Recursos humanos (Administración y medición del desempeño individual; supervisión; factores

ambientales que afectan al desempeño; técnicas de comunicación; fuentes/dotación del personal; capacitación y desarrollo; seguridad) [Nivel superior]

10. Tablero de comando [Nivel básico] B. Contabilidad financiera y finanzas (15-25%)

1. Conceptos básicos y principios subyacentes de la contabilidad financiera (por ej., declaraciones, terminología, relaciones) [Nivel superior]

2. Conceptos intermedios de la contabilidad financiera (por ej., bonos, contratos de arrendamiento, pensiones y jubilaciones, activos intangibles, Investigación & Desarrollo) [Nivel básico]

3. Conceptos avanzados de contabilidad financiera (por ej., consolidación, sociedades, transacciones con moneda extranjera) [Nivel básico]

4. Análisis de estados financieros [Nivel superior] 5. Costo de la evaluación del capital [Nivel superior] 6. Tipos de deudas y capital social [Nivel básico] 7. Instrumentos financieros (por ej., derivativos) [Nivel básico] 8. Gestión de caja (funciones del tesoro) [Nivel básico] 9. Modelos de valuación [Nivel básico]

a. Valuación del inventario b. Valuación del negocio

10. Ciclos de vida del desarrollo del negocio [Nivel básico] C. Contabilidad gerencial (10-20%)

1. Conceptos de costos (por ej., costo total, variable, fijo) [Nivel superior] 2. Presupuestación del capital [Nivel básico] 3. Presupuesto operativo [Nivel superior] 4. Precios de transferencia [Nivel básico] 5. Análisis de costos-volúmenes-ganancias [Nivel básico] 6. Costo pertinente [Nivel básico] 7. Sistemas de costos (por ej., basados en la actividad, estándar) [Nivel básico] 8. Contabilidad de responsabilidad [Nivel básico]

D. Regulación, asuntos legales y economía (5-15%) [Nivel básico]

1. Impacto de la legislación y regulación gubernamentales en el negocio 2. Legislación y regulaciones comerciales 3. Sistemas tributarios 4. Contractos 5. Naturaleza y reglas de la evidencia legal 6. Indicadores económicos clave

(CONTINÚA EN LA PRÓXIMA PÁGINA)

xv

E. Tecnología de la información (TI) (30-40%) [Nivel básico] 1. Marcos de control (por ej., SAC, COBIT) 2. Comunicaciones/conexiones de datos y en red (por ej., LAN , VAN, y WAN) 3. Transferencia electrónica de fondos (EFT) 4. e-Commerce 5. Intercambio electrónico de datos (EDI) 6. Áreas funcionales de las operaciones de TI (por ej., operaciones de los centros de datos) 7. Claves de encriptación 8. Protección de la información (por ej., virus, privacidad) 9. Evaluar la inversión en la TI (costo de propiedad) 10. Software de planificación de recursos al nivel empresarial (ERP) (por ej., SAP R/3) 11. Sistemas operativos 12. Desarrollo de la aplicación 13. Comunicaciones por voz 14. Planificación de contingencias 15. Seguridad de sistemas (por ej., muros refractarios, control de acceso) 16. Bases de datos 17. Licencias de software 18. Infraestructura de la red

Formato: 125 preguntas de opción múltiple

xvi

PARTE IV: DESTREZAS DE ADMINISTRACIÓN EN LOS NEGOCIOS A. Administración estratégica (20-30%) [Nivel básico]

1. Técnicas analíticas globales a. Análisis estructural de las industrias b. Estrategias competitivas (por ej., modelo de Porter) c. Análisis competitivo d. Señales del mercado e. Evolución de la industria

2. Entornos de la industria a. Estrategias competitivas relacionadas con:

1) Industrias fragmentadas 2) Industrias emergentes 3) Industrias en declinación

b. Competencia en industrias globales 1) Fuentes/impedimentos 2) Evolución de los mercados globales 3) Alternativas estratégicas 4) Tendencias que afectan a la competencia

3. Decisiones estratégicas a. Análisis de las estrategias de integración b. Expansión de la capacidad c. Ingreso en nuevos negocios

4. Técnicas de cartera del análisis competitivo 5. Ciclos de vida de productos

B. Entornos globales del negocio (15-25%) [Nivel básico]

1. Entornos culturales/legales/políticos a. El equilibrio de los requerimientos globales y los imperativos locales b. Modos de pensar globales (características/competencias personales) c. Fuentes y métodos para administrar complejidades y contradicciones d. La administración de equipos multiculturales

2. Entornos económicos/financieros a. Globales, multinacionales, internacionales y multilocales comparados y contrastados b. Requerimientos para ingresar en el mercado global c. La creación de la adaptabilidad organizacional d. La administración de la capacidad y el desarrollo

C. Conducta organizacional (15-25%) [Nivel básico]

1. Motivación a. Pertinencia e implicancia de diversas teorías b. Impacto del diseño del trabajo, las recompensas, los programas laborales, etc.

2. Comunicación a. El proceso b. Dinámica organizacional c. Impacto de la computarización

3. Desempeño a. Productividad b. Eficacia

4. Estructura a. Centralizada/descentralizada b. Departamentalización c. Nuevas configuraciones (por ej., hourglass, cluster, networks)

(CONTINÚA EN LA PRÓXIMA PÁGINA)

xvii

D. Destrezas administrativas (20-30%) [Nivel básico] 1. Dinámica de grupos

a. Rasgos (por ej., cohesión, roles, normas, pensamiento grupal) b. Etapas del desarrollo grupal c. Política organizacional d. Criterios y factores determinantes de la eficacia

2. Formación de equipos a. Métodos utilizados en la formación de equipos b. Evaluación del desempeño de los equipos

3. Destrezas de liderazgo a. Teorías comparadas y contrastadas b. Cuadrícula de liderazgo (topología de los estilos de liderazgo) c. El mentor

4. Administración del tiempo personal E. Negocios (5-15%) [Nivel básico]

1. Resolución de conflictos a. Competitivos/cooperativos b. Convenir, competir o contender, ceder o conceder, etc.

2. Negociación de valor agregado a. Descripción b. Pasos específicos

Formato: 125 preguntas de opción múltiple

El no cumplir con estas instrucciones y con las guías de "Instrucciones para candidatos" puede afectar seriamente

su derecho para recibir los resultados de este examen y su futura participación el programa de Auditor Interno Certificado.

Todos los documentos presentados a la terminación de cualquiera de las

partes de este examen son propiedad exclusiva de The Institute of Internal Auditor, Inc. Los candidatos no puede revelar el contenido de este examen a menos que cuenten con la

autorización del Departamento de Certificación.

I - 1

Preguntas del examen modelo para el Auditor Interno Certificado (CIA)

Parte I – El rol de la actividad de auditoría interna

en el gobierno, el riesgo y el control

Preguntas del examen modelo parte I: 100

Preguntas del examen CIA real parte I: 125 (ver explicación en el “prólogo”, página iii)

Tiempo permitido para terminar el examen CIA parte I: 210 minutes

Instrucciones como las que a continuación se enuncian serán las que aparezcan

en la portada de cada examen CIA. Por favor, léanlas con cuidado.

1. Anote su número de candidato en la hoja de respuestas en el espacio proporcionado.

2. No haga anotaciones irrelevantes en la hoja de respuestas.

3. Aségurese de que todo cambio de respuesta sea completamente borrado.

4. Todas las referencias al Marco para la Práctica Profesional se refieren al Marco para la Práctica Profesional del IAI, que incluye las Normas y los Consejos para la Práctica. Todas las referencias a las Normas se refieren a las Normas Internacionales para el Ejercicio Profesional de la Auditoría Interna resumidas en el Marco para la Práctica Profesional del IAI.

I - 2

1. ¿Cuál de las siguientes características no es verdad con respecto al estatuto de auditoría interna? a. Define las autoridades y

responsabilidades para la actividad de auditoría interna.

b. Especifica los recursos mínimos necesarios para la actividad de auditoría interna.

c. Provee una base para evaluar la actividad de auditoría interna.

d. Debe ser aprobado por la alta gerencia y el consejo.

2. ¿Qué herramienta para planificar auditorías es

de carácter general y se usa para asegurar una cobertura adecuada con el tiempo? a. La programación a largo plazo. b. El programa de trabajos. c. El presupuesto de la actividad de

auditoría. d. El estatuto (charter) de la actividad de

auditoría interna. 3. La función de auditoría interna, relacionada

con los informes financieros internos, sería: a. Asegurar el cumplimiento de los

procedimientos de información. b. Revisar los conceptos de gastos y cotejar

cada concepto con los gastos en los cuales se incurrió.

c. Determinar si hay empleados que gastan fondos sin autorización.

d. Identificar controles inadecuados que incrementan la probabilidad de gastos no autorizados.

4. Es más probable que los comités de auditoría

participen en la aprobación de: a. Promociones y aumentos salariales del

personal de auditoría. b. Las observaciones y recomendaciones

incluidas en los informes de auditoría interna.

c. Los programas de trabajos de auditoría. d. La designación del director ejecutivo de

auditoría. 5. De acuerdo con el Marco para la Práctica

Profesional, la independencia de la actividad de auditoría interna se logra mediante: a. Integración y supervisión del personal. b. Desarrollo profesional continuo y debido

cuidado profesional. c. Relaciones humanas y comunicaciones. d. Objetividad y nivel organizacional.

1. Which of the following is not true with regard to the internal audit charter? a. It defines the authorities and

responsibilities for the internal audit activity.

b. It specifies the minimum resources needed for the internal audit activity.

c. It provides a basis for evaluating the internal audit activity.

d. It should be approved by senior management and the board.

2. Which engagement-planning tool is general in

nature and is used to ensure adequate audit coverage over time? a. The long-range schedule. b. The engagement program. c. The audit activity’s budget. d. The audit activity’s charter.

3. The function of internal auditing, as related to

internal financial reports, would be to: a. Ensure compliance with reporting

procedures. b. Review the expenditure items and match

each item with the expenses incurred. c. Determine if there are any employees

expending funds without authorization. d. Identify inadequate controls that increase

the likelihood of unauthorized expenditures.

4. Audit committees are most likely to participate

in the approval of: a. Audit staff promotions and salary

increases. b. The internal audit report observations and

recommendations. c. Audit work schedules. d. The appointment of the chief audit

executive. 5. According to the Professional Practices

Framework, the independence of the internal audit activity is achieved through: a. Staffing and supervision. b. Continuing professional development and

due professional care. c. Human relations and communications. d. Organizational status and objectivity.

I - 3

6. ¿Cuál de las siguientes acciones constituiría una violación a la independencia del auditor? a. Continuar un trabajo de auditoría en una

división de la cual el auditor pronto será responsable, como resultado de un inminente ascenso.

b. Reducir el alcance de un trabajo debido a restricciones presupuestarias.

c. Participar en un equipo de trabajo que recomiende normas de control para un nuevo sistema de distribución.

d. Revisar el borrador del contrato con un agente de compras antes de que se formalice.

7. Como parte de un programa de

reconocimiento de méritos, una división ofreció a un auditor interno un premio de valor económico significativo en reconocimiento de los ahorros que se lograron por las recomendaciones del auditor. De acuerdo con el Marco para la Práctica Profesional, ¿cuál es la acción más apropiada que el auditor debe tomar? a. Aceptar el premio, pues el trabajo ya

concluyó y se emitió el informe. b. Aceptar el premio con la condición de que

se done lo recaudado a una institución de beneficencia.

c. Informar a la gerencia de auditoría y consultarle si debe aceptar el premio.

d. Rechazar el premio e informar al gerente de la división.

8. ¿En cuál de las siguientes situaciones es

probable que el auditor carezca de objetividad? a. Un auditor revisa los procedimientos para

una nueva conexión de intercambio electrónico de datos (EDI) con un cliente importante antes de que se implementen.

b. Un ex-ayudante de compras lleva a cabo una revisión de controles internos sobre la función de compras cuatro meses después de ser transferido al departamento de auditoría interna.

c. Un auditor recomienda normas de control y medidas de desempeño respecto de un contrato con una organización de servicio, para el procesamiento de la nómina de sueldos y prestaciones a empleados.

d. Un empleado de contabilidad de sueldos ayuda a un auditor en la verificación del inventario físico de motores pequeños.

6. Which of the following actions would be a violation of auditor independence? a. Continuing on an audit assignment at a

division for which the auditor will soon be responsible as the result of a promotion.

b. Reducing the scope of an engagement due to budget restrictions.

c. Participating on a task force which recommends standards of control for a new distribution system.

d. Reviewing a purchasing agent's contract drafts prior to their execution.

7. As part of a company-sponsored award

program, an internal auditor was offered an award of significant monetary value by a division in recognition of the cost savings that resulted from the auditor's recommendations. According to the Professional Practices Framework, what is the most appropriate action for the auditor to take? a. Accept the gift since the engagement is

already concluded and the report issued. b. Accept the award under the condition that

any proceeds go to charity. c. Inform audit management and ask for

direction on whether to accept the gift. d. Decline the gift and advise the division

manager's superior. 8. In which of the following situations would an

auditor potentially lack objectivity? a. An auditor reviews the procedures for a

new electronic data interchange connection to a major customer before it is implemented.

b. A former purchasing assistant performs a review of internal controls over purchasing four months after being transferred to the internal audit activity.

c. An auditor recommends standards of control and performance measures for a contract with a service organization for the processing of payroll and employee benefits.

d. A payroll accounting employee assists an auditor in verifying the physical inventory of small motors.

I - 4

9. Un auditor interno certificado (CIA), que trabaja como director de compras, firma un contrato que asigna un pedido importante a un proveedor que ofrece el mejor precio, calidad y desempeño. Poco tiempo después de firmar el contrato, el proveedor se presenta ante el CIA con un obsequio de valor monetario significativo. ¿Cuál de las siguientes declaraciones referentes a la aceptación del obsequio es correcta? a. Aceptar el obsequio estaría prohibido sólo

si no fuera habitual. b. Aceptar el obsequio constituiría una

violación al Código de Ética del IAI y estaría prohibido para un CIA.

c. Ya que el CIA no está trabajando como auditor interno, la aceptación del obsequio se regiría únicamente por el código de conducta de la organización.

d. Como el contrato fue firmado antes de que se ofreciera el obsequio, la aceptación de éste no violaría el Código de Ética del IAI ni el código de conducta de la organización.

10. Un auditor interno al que se le asignó la tarea

de auditar el cumplimiento de un proveedor con las normas de calidad de un producto es el hermano del controller de dicho proveedor. El auditor debe: a. Aceptar la tarea, pero evitar contacto con

el controller durante el trabajo de campo. b. Aceptar la tarea, pero informar sobre la

relación en la comunicación final del trabajo.

c. Notificar al proveedor sobre el posible conflicto de intereses.

d. Notificar al director ejecutivo de auditoría sobre el posible conflicto de intereses.

9. A CIA, working as the director of purchasing, signs a contract to procure a large order from the supplier with the best price, quality, and performance. Shortly after signing the contract, the supplier presents the CIA with a gift of significant monetary value. Which of the following statements regarding the acceptance of the gift is correct? a. Acceptance of the gift would be prohibited

only if it were non-customary. b. Acceptance of the gift would violate the

IIA Code of Ethics and would be prohibited for a CIA.

c. Since the CIA is not acting as an internal auditor, acceptance of the gift would be governed only by the organization’s code of conduct.

d. Since the contract was signed before the gift was offered, acceptance of the gift would not violate either the IIA Code of Ethics or the organization’s code of conduct.

10. An internal auditor assigned to audit a

vendor’s compliance with product quality standards is the brother of the vendor’s controller. The auditor should: a. Accept the assignment, but avoid contact

with the controller during fieldwork. b. Accept the assignment, but disclose the

relationship in the engagement final communication.

c. Notify the vendor of the potential conflict of interest.

d. Notify the chief audit executive of the potential conflict of interest.

I - 5

11. ¿Cuáles de las siguientes destrezas deben poseer los auditores internos según lo solicitado por las Normas?

I. Los auditores internos deben comprender

las relaciones humanas y estar capacitados para tratar a las personas.

II. Los auditores internos deben poder reconocer y evaluar la materialidad e importancia de las desviaciones de las buenas prácticas de negocios.

III. Los auditores internos deben ser expertos en temas tales como economía, leyes comerciales, impuestos, finanzas y tecnología de la información.

IV. Los auditores internos deben estar capacitados para la comunicación oral y escrita.

a. II solamente. b. I y III solamente. c. III y IV solamente. d. I, II, y IV solamente.

12. El comité de auditoría solicitó a un director

ejecutivo de auditoría que condujera un trabajo en una fábrica química lo antes posible. El trabajo debe incluir revisiones de la administración y los procesos de protección ambiental, de seguridad y salud. El director ejecutivo de auditoría sabe que la actividad de auditoría interna no posee los conocimientos necesarios sobre protección ambiental, de seguridad y salud para conducir un trabajo de esa índole. El director debe: a. Comenzar el trabajo e incorporar

capacitación en protección ambiental, de seguridad y salud a la planificación del próximo año a fin de prepararse para un trabajo de seguimiento.

b. Sugerir al comité de auditoría que el propio personal de la fábrica conduzca el trabajo.

c. Solicitar permiso al comité de auditoría para obtener el apoyo apropiado de un profesional de protección ambiental, de seguridad y salud.

d. Postergar el trabajo y explicar al comité de auditoría que llevará varios meses capacitar al personal de auditoría interna para un trabajo de esa índole.

11. The Standards require that internal auditors possess which of the following skills?

I. Internal auditors should understand

human relations and be skilled in dealing with people.

II. Internal auditors should be able to recognize and evaluate the materiality and significance of deviations from good business practices.

III. Internal auditors should be experts on subjects such as economics, commercial law, taxation, finance, and information technology.

IV. Internal auditors should be skilled in oral and written communication.

a. II only. b. I and III only. c. III and IV only. d. I, II, and IV only.

12. A chief audit executive (CAE) has been

requested by the audit committee to conduct an engagement at a chemical factory as soon as possible. The engagement will include reviews of health, safety, and environmental (HSE) management and processes. The CAE knows that the internal audit activity does not possess the HSE knowledge necessary to conduct such an engagement. The CAE should: a. Begin the engagement and incorporate

HSE training into next year’s planning to prepare for a follow-up engagement.

b. Suggest to the audit committee that the factory’s own HSE staff conduct the engagement.

c. Seek permission from the audit committee to obtain appropriate support from an HSE professional.

d. Defer the engagement and tell the audit committee that it will take several months to train internal audit staff for such an engagement.

I - 6

13. Para asegurarse de haber ejercido en todo momento el debido cuidado profesional durante un trabajo, el auditor interno siempre debe: a. Asegurarse de incluir toda la información

financiera relacionada con la auditoría en el plan y examinarla para detectar incumplimientos o irregularidades.

b. Asegurarse de documentar totalmente todas las pruebas de auditoría.

c. Considerar la posibilidad de incumplimientos o irregularidades en todo momento durante el trabajo.

d. Comunicar sin demora cualquier incumplimiento o irregularidad descubiertos durante un trabajo al comité de auditoría.

14. En un trabajo de aseguramiento de

operaciones del tesoro, se solicita a un auditor interno que considere todas las siguientes cuestiones excepto que: a. El comité de auditoría solicitó

aseguramiento sobre el cumplimiento por parte del departamento del tesoro con una nueva política sobre el uso de instrumentos financieros.

b. La gerencia del tesoro no instituyó ninguna política de administración de riesgos.

c. Debido a la reciente venta de una división, la cantidad de efectivo y valores comerciables administrados por el departamento del tesoro aumentó en un 350 por ciento.

d. Los auditores externos indicaron que tuvieron algunas dificultades para obtener las confirmaciones sobre las cuentas.

13. To ensure that due professional care has been taken at all times during an engagement, the internal auditor should always: a. Ensure that all financial information

related to the audit is included in the audit plan and examined for nonconformance or irregularities.

b. Ensure that all audit tests are fully documented.

c. Consider the possibility of nonconformance or irregularities at all times during an engagement.

d. Communicate any noncompliance or irregularity discovered during an engagement promptly to the audit committee.

14. In an assurance engagement of treasury

operations, an internal auditor is required to consider all of the following issues except: a. The audit committee has requested

assurance on the treasury department’s compliance with a new policy on use of financial instruments.

b. Treasury management has not instituted any risk management policies.

c. Due to the recent sale of a division, the amount of cash and marketable securities managed by the treasury department has increased by 350 percent.

d. The external auditors have indicated some difficulties in obtaining account confirmations.

I - 7

15. Para promocionar una imagen positiva dentro de una organización, un director ejecutivo de auditoría planificó conducir trabajos de aseguramiento que pusieron énfasis en lograr posibles reducciones de costos. Las observaciones negativas debían omitirse en las comunicaciones finales de los trabajos. ¿Cuál de las acciones tomadas por el director se considerarían una violación de las Normas?

I. Se cambió el enfoque de los trabajos de

auditoría sin modificar el estatuto ni consultar al comité.

II. Se omitieron las observaciones negativas en las comunicaciones finales de los trabajos.

III. Se puso énfasis en las recomendaciones de reducciones de costos expresadas en las comunicaciones finales de los trabajos.

a. I solamente. b. I y II solamente. c. I y III solamente. d. II y III solamente.

16. Un director ejecutivo de auditoría de un

departamento de auditoría interna muy pequeño acaba de recibir un pedido de la gerencia para efectuar una auditoría de un área extremadamente compleja en la cual ni el director ni el departamento tienen pericia alguna. La naturaleza del trabajo de auditoría se encuentra dentro del alcance de las actividades de auditoría interna. La gerencia expresó un deseo de conducir el trabajo en un futuro muy cercano debido al alto nivel de riesgo afectado. ¿Cuál de las siguientes respuestas dadas por el director ejecutivo de auditoría constituiría una violación a las Normas? a. Conversar con la gerencia sobre la

posibilidad de tercerizar la auditoría de este área compleja.

b. Agregar un consultor externo al personal de auditoría para ayudar en el desempeño del trabajo de auditoría.

c. Aceptar el trabajo de auditoría y comenzar de inmediato, ya que se trata de un área de alto riesgo.

d. Conversar con la gerencia sobre la línea temporal del trabajo de auditoría a fin de determinar si se dispone de tiempo suficiente para desarrollar la pericia apropiada.

15. To promote a positive image within an organization, a chief audit executive (CAE) planned to conduct assurance engagements that highlighted potential costs to be saved. Negative observations were to be omitted from engagement final communications. Which action taken by the CAE would be considered a violation of the Standards?

I. The focus of the audit engagements was

changed without modifying the charter or consulting the audit committee.

II. Negative observations were omitted from the engagement final communications.

III. Cost savings recommendations were highlighted in the engagement final communications.

a. I only. b. I and II only. c. I and III only. d. II and III only.

16. A chief audit executive (CAE) for a very small

internal audit department has just received a request from management to perform an audit of an extremely complex area in which the CAE and the department have no expertise. The nature of the audit engagement is within the scope of internal audit activities. Management has expressed a desire to have the engagement conducted in the very near future because of the high level of risk involved. Which of the following responses by the CAE would be in violation of the Standards? a. Discuss with management the possibility

of outsourcing the audit of this complex area.

b. Add an outside consultant to the audit staff to assist in the performance of the audit engagement.

c. Accept the audit engagement and begin immediately, since it is a high-risk area.

d. Discuss the timeline of the audit engagement with management to determine if sufficient time exists in which to develop appropriate expertise.

I - 8

17. Un auditor que está a punto de terminar un trabajo descubre que el director de marketing tiene el hábito del juego. Este aspecto no está directamente relacionado con el trabajo en curso y se ejerce presión para terminarlo. El auditor anota el problema y pasa la información al director ejecutivo de auditoría, pero no le da seguimiento. Las acciones del auditor pueden: a. Ser una violación del Código de Ética del

The Institute of Internal Auditors por retener información importante.

b. Ser una violación a las Normas porque el auditor no dio debido seguimiento a una bandera roja que podría indicar la existencia de fraude.

c. No violar el Código de Ética del The IIA ni las Normas.

d. Tanto a como b. 18. Al seleccionar una estrategia de instrucción

para desarrollar al personal de auditoría interna, un director ejecutivo de auditoría debe comenzar por revisar lo siguiente: a. Objetivos organizacionales. b. Contenido de aprendizaje. c. Disposición de quienes aprenden. d. Restricciones del presupuesto.

19. ¿Cuáles de las siguientes actividades están

diseñadas para proporcionar retroalimentación en cuanto a la eficacia de una función de auditoría interna?

I. Supervisión apropiada. II. Capacitación apropiada. III. Evaluaciones internas. IV. Evaluaciones externas. a. I, II, y III solamente. b. I, II, y IV solamente. c. I, III, y IV solamente. d. II, III, y IV solamente.

20. La razón más importante para que el director

ejecutivo de auditoría garantice que el departamento de auditoría interna posee los recursos adecuados y suficientes es: a. Asegurar que la función se encuentre

adecuadamente protegida contra la tercerización.

b. Demostrar capacidad suficiente para cumplir con los requerimientos del plan de auditoría.

c. Establecer credibilidad con el comité de auditoría y la gerencia.

d. Satisfacer la necesidad de una planificación en serie eficaz.

17. An auditor, nearly finished with an engagement, discovers that the director of marketing has a gambling habit. The gambling issue is not directly related to the existing engagement and there is pressure to complete the current engagement. The auditor notes the problem and forwards the information to the chief audit executive but performs no further follow-up. The auditor’s actions would: a. Be in violation of the IIA Code of Ethics

for withholding meaningful information. b. Be in violation of the Standards because

the auditor did not properly follow up on a red flag that might indicate the existence of fraud.

c. Not be in violation of either the IIA Code of Ethics or Standards.

d. Both a and b. 18. In selecting an instructional strategy for

developing internal audit staff, a chief audit executive should begin by reviewing: a. Organizational objectives. b. Learning content. c. Learners’ readiness. d. Budget constraints.

19. Which of the following activities are designed

to provide feedback on the effectiveness of an internal audit function?

I. Proper supervision. II. Proper training. III. Internal assessments. IV. External assessments.

a. I, II, and III only. b. I, II, and IV only. c. I, III, and IV only. d. II, III, and IV only.

20. The most important reason for the chief audit

executive to ensure that the internal audit department has adequate and sufficient resources is to: a. Ensure that the function is adequately

protected from outsourcing. b. Demonstrate sufficient capability to meet

the audit plan requirements. c. Establish credibility with the audit

committee and management. d. Fulfill the need for effective succession

planning.

I - 9

21. ¿Cuál de las siguientes acciones forma parte de un programa de aseguramiento de la actividad de auditoría interna, más que encontrarse incluida como parte de otras responsabilidades del director ejecutivo de auditoría? a. El director ejecutivo de auditoría

proporciona información y permite a los auditores externos acceder a los papeles de trabajo de la auditoría interna, para permitirles comprender y determinar el grado en el cual pueden confiar en el trabajo de la auditoría interna.

b. La gerencia aprueba un documento formal (estatuto) que establece el propósito, autoridad y responsabilidad de la actividad de auditoría interna.

c. El desempeño individual de cada auditor interno se evalúa cuando menos una vez al año.

d. La supervisión de la labor de un auditor interno se realiza a través de cada trabajo de auditoría.

22. Un director ejecutivo de auditoría utiliza un

modelo de evaluación de riesgos para establecer el plan de auditoría anual. ¿Cuál de las siguientes sería una acción apropiada para que tome el director?

I. Mantener un diálogo continuo con la

gerencia y el comité de auditoría. II. Asegurar que el programa de prioridades

de auditoría no sufra cambios. III. Utilizar sólo métodos cuantitativos para

determinar los pesos de los riesgos. IV. Revisar la evaluación de riesgos y las

prioridades de auditoría según lo autorizado.

a. III solamente. b. I y II solamente. c. I y IV solamente. d. III y IV solamente.

21. Which of the following is part of an internal audit activity’s quality assurance program, rather than being included as part of other responsibilities of the chief audit executive (CAE)? a. The CAE provides information about and

access to internal audit workpapers to the external auditors to enable them to understand and determine the degree to which they may rely on the internal auditors' work.

b. Management approves a formal charter establishing the purpose, authority, and responsibility of the internal audit activity.

c. Each individual internal auditor's performance is appraised at least annually.

d. Supervision of an internal auditor's work is performed throughout each audit engagement.

22. A chief audit executive (CAE) uses a risk

assessment model to establish the annual audit plan. Which of the following would be an appropriate action by the CAE?

I. Maintain ongoing dialogue with

management and the audit committee. II. Ensure that the schedule of audit priorities

remains unchanged. III. Employ only quantitative methods to

determine risk weightings. IV. Revise the risk assessment and audit

priorities as warranted.

a. III only. b. I and II only. c. I and IV only. d. III and IV only.

I - 10

23. Cuando se ha empleado el proceso de evaluación de riesgos para integrar un programa de auditoría, ¿cuál de las siguientes situaciones debiera recibir atención en primer lugar? a. Los auditores externos han requerido

apoyo para la realización de su próxima auditoría anual.

b. Un nuevo sistema de cuentas por pagar se prueba actualmente por parte del departamento de tecnología de información.

c. La gerencia ha solicitado que se investigue una posible sobreposición o traslape en las cuentas por cobrar.

d. El sistema de cuentas por pagar en uso no fue auditado el año pasado.

24. Un director ejecutivo de auditoría está

revisando el siguiente mapa de riesgo al nivel empresarial:

PROBABILIDAD Remota Posible Probable

Critico Reisgo A Reisgo B Mayor Reisgo D

IMPA

CTO

Menor Reisgo C

¿Cuál de las siguientes es la secuencia correcta de prioridades de riesgos, si se consideran los recursos limitados en la actividad de auditoría interna? a. Riesgo B, Riesgo C, Riesgo A, Riesgo D. b. Riesgo A, Riesgo B, Riesgo C, Riesgo D. c. Riesgo D, Riesgo B, Riesgo C, Riesgo A. d. Riesgo B, Riesgo C, Riesgo D, Riesgo A.

25. ¿Cuál de las siguientes representa la mejor

técnica de evaluación de riesgos? a. Evaluación de los niveles de riesgos para

acontecimientos futuros basados en el grado de incertidumbre de esos acontecimientos y su impacto en el logro de objetivos organizacionales a largo plazo.

b. Evaluación de riesgos inherentes y de controles y su impacto sobre el grado de aserciones financieras erróneas.

c. Evaluación de los niveles de riesgos de acontecimientos en curso y futuros, su efecto en el logro de los objetivos organizacionales y sus causas subyacentes.

d. Evaluación de los niveles de riesgos de acontecimientos en curso y futuros, su impacto en la misión de la organización y la probabilidad de eliminar factores de riesgo posibles o existentes.

23. When a risk assessment process has been used to construct an audit engagement schedule, which of the following should receive attention first? a. The external auditors have requested

assistance for their upcoming annual audit.

b. A new accounts payable system is currently undergoing testing by the information technology department.

c. Management has requested an investigation of possible lapping in receivables.

d. The existing accounts payable system has not been audited over the past year.

24. A chief audit executive is reviewing the

following enterprise-wide risk map:

LIKELIHOOD Remote Possible Likely

Critical Risk A Risk B Major Risk D

IMPA

CT

Minor Risk C

Which of the following is the correct prioritization of risks, considering limited resources in the internal audit activity? a. Risk B, Risk C, Risk A, Risk D. b. Risk A, Risk B, Risk C, Risk D. c. Risk D, Risk B, Risk C, Risk A. d. Risk B, Risk C, Risk D, Risk A.

25. Which of the following represents the best risk

assessment technique? a. Assessment of the risk levels for future

events based on the extent of uncertainty of those events and their impact on achievement of long-term organizational goals.

b. Assessment of inherent and control risks and their impact on the extent of financial misstatements.

c. Assessment of the risk levels of current and future events, their effect on achievement of the organization’s objectives, and their underlying causes.

d. Assessment of the risk levels of current and future events, their impact on the organization’s mission, and the potential for elimination of existing or possible risk factors.

I - 11

26. ¿Cuál de las siguientes es la mejor razón para que el director ejecutivo de auditoría considere el plan estratégico para desarrollar el plan anual? a. Garantizar que el plan de auditoría

interna respalde los objetivos globales del negocio.

b. Garantizar que el plan de auditoría interna tenga la aprobación de la alta gerencia.

c. Hacer recomendaciones para mejorar el plan estratégico.

d. Poner énfasis en la importancia de la función de auditoría interna.

27. Al evaluar el riesgo organizacional en un

ambiente de manufactura, ¿cuál de las siguientes cuestiones tendría el impacto de mayor rango en la organización? a. Programa de producción. b. Política de inventarios. c. Calidad del producto. d. Presupuesto de publicidad.

28. Al evaluar el riesgo asociado con una actividad

un auditor interno debe: a. Determinar cómo administrar mejor el

riesgo. b. Proveer aseguramiento en la

administración de riesgo. c. Actualizar el proceso de administración

de riesgos sobre la base de las exposiciones al riesgo.

d. Diseñar controles para mitigar los riesgos identificados.

26. Which of the following is the best reason for the chief audit executive to consider the strategic plan in developing the annual audit plan? a. To ensure that the internal audit plan

supports the overall business objectives. b. To ensure that the internal audit plan will

be approved by senior management. c. To make recommendations to improve

the strategic plan. d. To emphasize the importance of the

internal audit function. 27. In assessing organizational risk in a

manufacturing environment, which of the following would have the most long-range impact on the organization? a. Production scheduling. b. Inventory policy. c. Product quality. d. Advertising budget.

28. When assessing the risk associated with an

activity, an internal auditor should: a. Determine how the risk should best be

managed. b. Provide assurance on the management of

the risk. c. Update the risk management process

based on risk exposures. d. Design controls to mitigate the identified

risks.

I - 12

Utilice la siguiente información para responder a las preguntas 29 y 30.

Durante la fase de planificación, el director de auditoría interna se encuentra evaluando la realización de cuatro auditorías basándose en los siguientes factores: la capacidad de la auditoría para reducir riesgos a la organización, la capacidad de la auditoría para ahorrar dinero a la organización y la magnitud de los cambios ocurridos en el área desde que se realizó la última auditoría. El director ha calificado las auditorías por cada uno de estos factores en una escala de bajo a alto, asignándoles puntos y calculando una calificación total. Los resultados se muestran enseguida con los puntos asignados entre paréntesis.

Nro. Reducción de riesgo

Reducciones de costos

Cambios

1 Alto (3) Medio (2) Bajo (1) 2 Alto (3) Bajo (1) Alto (3) 3 Bajo (1) Alto (3) Medio (2) 4 Medio (2) Medio (2) Alto (3)

29. ¿Cuáles auditorías debiera proceder a realizar

el director si todos los factores tuvieran la misma importancia? a. 1 y 2 solamente. b. 1 y 3 solamente. c. 2 y 4 solamente. d. 3 y 4 solamente.

30. Si la organización ha solicitado al director que

considere el ahorro en costos como un factor que representa el doble de importancia que los otros dos factores, ¿cuáles auditorías debería proceder a realizar? a. 1 y 2 solamente. b. 1 y 3 solamente. c. 2 y 4 solamente. d. 3 y 4 solamente.

31. Si un departamento que se encuentra fuera de

la actividad de auditoría interna es responsable de revisar una función o proceso, los auditores internos deben: a. Considerar el trabajo del otro

departamento cuando evalúen la función o el proceso.

b. Ignorar el trabajo del otro departamento y proseguir con una auditoría independiente.

c. Reducir el alcance de la auditoría ya que el otro departamento ya realizó el trabajo.

d. Ceder la responsabilidad de evaluar la función o el proceso al otro departamento.

Use the following information to answer questions 29 through 30.

During the planning phase, a chief audit executive (CAE) is evaluating four audit engagements based on the following factors: the engagement’s ability to reduce risk to the organization, the engagement’s ability to save the organization money, and the extent of change in the area since the last engagement. The CAE has scored the engagements for each factor from low to high, assigned points, and calculated an overall ranking. The results are shown below with the points in parenthesis:

AuditRisk

Reduction Cost

Savings

Changes 1 High (3) Medium (2) Low (1) 2 High (3) Low (1) High (3) 3 Low (1) High (3) Medium (2) 4 Medium (2) Medium (2) High (3)

29. Which audit engagements should the CAE

pursue if all factors are weighed equally? a. 1 and 2 only. b. 1 and 3 only. c. 2 and 4 only. d. 3 and 4 only.

30. If the organization has asked the CAE to

consider the cost savings factor to be twice as important as any other factor, which engagements should the CAE pursue? a. 1 and 2 only. b. 1 and 3 only. c. 2 and 4 only. d. 3 and 4 only.

31. If a department outside of the internal audit

activity is responsible for reviewing a function or process, the internal auditors should: a. Consider the work of the other

department when assessing the function or process.

b. Ignore the work of the other department and proceed with an independent audit.

c. Reduce the scope of the audit since the work has already been performed by the other department.

d. Yield the responsibility for assessing the function or process to the other department.

I - 13

32. ¿Quién tiene la responsabilidad primordial de proveer información al comité de auditoría sobre los beneficios profesionales y organizacionales de coordinar las actividades de aseguramiento y consultoría de auditoría interna con otras actividades de aseguramiento y consultoría? a. El auditor externo. b. El director ejecutivo de auditoría. c. El Director General. d. Cada función de aseguramiento y

consultoría. 33. El uso del departamento de auditoría interna

para coordinar los esfuerzos de los examinadores reguladores resulta beneficioso para la organización porque los auditores internos pueden: a. Influir en la interpretación que los

examinadores reguladores dan a la ley para combinarla con la práctica corporativa.

b. Recomendar cambios en el alcance para limitar la influencia de los examinadores reguladores.

c. Realizar trabajo de campo para los examinadores reguladores y, de esta manera, reducir la cantidad de tiempo que los examinadores reguladores se encuentran en el lugar.

d. Proveer evidencia de pruebas adecuadas de cumplimiento mediante los papeles de trabajo e informes de auditoría interna.

34. Un director ejecutivo de auditoría muy

probablemente utilizaría la evaluación de riesgos para la planificación de auditorías porque ésta le proporciona: a. Un proceso sistemático para evaluar e

integrar el juicio profesional respecto a condiciones adversas probables.

b. Un listado de efectos potencialmente adversos en la organización.

c. Un listado de actividades que pueden auditarse en la organización.

d. La probabilidad de que un evento o acción pueda afectar de manera adversa a la organización.

32. Who has primary responsibility for providing information to the audit committee on the professional and organizational benefits of coordinating internal audit assurance and consulting activities with other assurance and consulting activities? a. The external auditor. b. The chief audit executive. c. The chief executive officer. d. Each assurance and consulting function.

33. Using the internal audit department to

coordinate regulatory examiners’ efforts is beneficial to the organization because internal auditors can: a. Influence the regulatory examiners’

interpretation of law to match corporate practice.

b. Recommend changes in scope to limit bias by the regulatory examiners.

c. Perform fieldwork for the regulatory examiners and thus reduce the amount of time regulatory examiners are on-site.

d. Supply evidence of adequate compliance testing through internal audit workpapers and reports.

34. A chief audit executive would most likely use

risk assessment for audit planning because it provides: a. A systematic process for assessing and

integrating professional judgment about probable adverse conditions.

b. A listing of potentially adverse effects on the organization.

c. A list of auditable activities in the organization.

d. The probability that an event or action may adversely affect the organization.

I - 14

35. Al decidir si resulta más conveniente programar el departamento de compras o el de personal para un trabajo de auditoría, ¿cuál de los siguientes sería el factor menos importante? a. Hubo cambios fundamentales en las

operaciones de uno de los departamentos.

b. El personal de auditoría acaba de agregar a un individuo con pericia en una de las áreas.

c. Existen más oportunidades para lograr los beneficios operativos en uno de los departamentos que en el otro.

d. La posibilidad de pérdida es significativamente mayor en un departamento que en el otro.

36. La actividad de auditoría interna de una

corporación de gran envergadura estableció su plan operativo y presupuesto para el año próximo. El plan operativo se limita a las siguientes categorías: un listado prioritario de todos los trabajos, del personal y un presupuesto detallado de gastos, además de la fecha de comienzo de cada trabajo. ¿Cuál de las siguientes situaciones describe mejor la deficiencia principal de este plan operativo? a. No se consideran los pedidos de

proyectos especiales por parte de la gerencia.

b. Se ignoran las oportunidades para lograr los beneficios operativos.

c. No se proveen los criterios de medición y fechas programadas de finalización.

d. Se ignoran el conocimiento, las destrezas y las disciplinas requeridas para realizar el trabajo.

37. Para mejorar la eficiencia de la auditoría, los

auditores internos pueden confiar en el trabajo de auditores externos: a. Que se realice después del trabajo de

auditoría interna. b. Que se relacione primordialmente con los

objetivos y actividades operativos. c. Que se coordine con la actividad de

auditoría interna. d. Que se conduzca de acuerdo con el

Código de Ética del IAI.

35. In deciding whether to schedule the purchasing or the personnel department for an audit engagement, which of the following would be the least important factor? a. There have been major changes in

operations in one of the departments. b. The audit staff has recently added an

individual with expertise in one of the areas.

c. There are more opportunities to achieve operating benefits in one of the departments than in the other.

d. The potential for loss is significantly greater in one department than in the other.

36. The internal audit activity of a large corporation

has established its operating plan and budget for the coming year. The operating plan is restricted to the following categories: a prioritized listing of all engagements, staffing, a detailed expense budget, and the commencement date of each engagement. Which of the following best describes the major deficiency of this operating plan? a. Requests by management for special

projects are not considered. b. Opportunities to achieve operating

benefits are ignored. c. Measurability criteria and targeted dates

of completion are not provided. d. Knowledge, skills, and disciplines

required to perform work are ignored. 37. To improve audit efficiency, internal auditors

can rely upon the work of external auditors that is: a. Performed after the internal audit

engagement. b. Primarily concerned with operational

objectives and activities. c. Coordinated with internal audit activity. d. Conducted in accordance with the IIA

Code of Ethics.

I - 15

38. La actividad de auditoría interna acaba de experimentar la partida de dos auditores internos que no pueden reemplazarse de inmediato debido a las restricciones del presupuesto. ¿Cuál de las siguientes es la opción menos deseable para finalizar futuros trabajos con eficiencia, dada esta reducción de recursos? a. Utilizar los cuestionarios de auto-

evaluación para abordar objetivos de auditoría.

b. Emplear tecnología de la información en la planificación de auditoría, el muestreo y la documentación.

c. Eliminar los proyectos de consultoría del programa de trabajos.

d. Llenar las vacantes con personal de los departamentos operativos que no se están auditando.

39. Si el plan de auditoría anual no permite una

revisión adecuada del cumplimiento con todas las reglamentaciones materiales que afectan a la compañía, la actividad de auditoría interna debe: a. Asegurar que el consejo directivo y la alta

gerencia sean conscientes de la limitación.

b. Incluir un memo con el archivo de planificación de auditoría que detalle las razones de la falta de cobertura.

c. Documentar que las reglamentaciones no incluidas se revisarán al año siguiente.

d. Disminuir el alcance de las auditorías operativas y financieras para disponer de tiempo adicional de auditoría.

40. ¿Cuáles de los siguientes comentarios son

correctos en relación con la evaluación del riesgo asociado con dos proyectos que están compitiendo por la limitación de los recursos de auditoría?

I. Las actividades que requiere el comité de

auditoría deben siempre considerarse de mayor riesgo que las que son requeridas por la gerencia.

II. Las actividades con presupuestos financieros más altos deben siempre considerarse de mayor riesgo que aquellas con presupuestos más bajos.

III. El riesgo debe siempre medirse por el potencial monetario o exposición adversa a la organización.

a. I solamente. b. II solamente. c. III solamente. d. I y III solamente.

I - 16

38. The internal audit activity has recently experienced the departure of two internal auditors who cannot be immediately replaced due to budget constraints. Which of the following is the least desirable option for efficiently completing future engagements, given this reduction in resources? a. Using self-assessment questionnaires to

address audit objectives. b. Employing information technology in audit

planning, sampling, and documentation. c. Eliminating consulting engagements from

the engagement work schedule. d. Filling vacancies with personnel from

operating departments that are not being audited.

39. If the annual audit plan does not allow for

adequate review of compliance with all material regulations affecting the company, the internal audit activity should: a. Ensure that the board of directors and

senior management are aware of the limitation.

b. Include a memo with the audit planning file listing the reasons for the lack of coverage.

c. Document that regulations not included will be reviewed in the subsequent year.

d. Decrease the scope of operational and financial audits to make additional audit time available.

40. Which of the following comments is correct

regarding the assessment of risk associated with two projects that are competing for limited audit resources?

I. Activities that are requested by the audit

committee should always be considered higher risk than those requested by management.

II. Activities with higher dollar budgets should always be considered higher risk than those with lower dollar budgets.

III. Risk should always be measured by the potential dollar or adverse exposure to the organization.

a. I only. b. II only. c. III only. d. I and III only.

I - 17

41. ¿Cuál de las siguientes actividades llevadas a cabo por el auditor interno podría entrar en conflicto con la norma de independencia? a. Consultoría de la administración de

riesgo. b. Líder del equipo de desarrollo de

productos. c. Defensor de la ética. d. Coordinación con la auditoría externa.

42. La actividad de auditoría interna debe

contribuir al proceso de gobierno de la organización evaluando los procesos a través de los cuales:

I. Se promueven la ética y los valores. II. Se garantizan la administración y

responsabilidad en el desempeño organizacional eficaz.

III. Se comunica la información sobre riesgos y controles.

IV. Se coordinan las actividades de la gerencia y de los auditores externos e internos.

a. I solamente. b. IV solamente. c. II y III solamente. d. I, II, III, y IV.

43. En un ambiente gerencial bien desarrollado, la

actividad de auditoría interna: a. Informaría los resultados de un trabajo de

auditoría tanto a la gerencia de línea como a la alta gerencia.

b. Conduciría auditorías iniciales de los nuevos sistemas computarizados una vez que comienzan a operar.

c. Intercambiaría impresiones primordialmente con la alta gerencia, minimizando las interacciones con los gerentes de línea que constituyen el objeto del trabajo de auditoría interna.

d. Pondrían énfasis especialmente en la administración de activos e informaría los resultados al comité de auditoría.

41. Which of the following activities undertaken by the internal auditor might be in conflict with the standard of independence? a. Risk management consultant. b. Product development team leader. c. Ethics advocate. d. External audit liaison.

42. The internal audit activity should contribute to

the organization’s governance process by evaluating the processes through which:

I. Ethics and values are promoted. II. Effective organizational performance

management and accountability are ensured.

III. Risk and control information is communicated.

IV. Activities of the external and internal auditors and management are coordinated.

a. I only. b. IV only. c. II and III only. d. I, II, III, and IV.

43. In a well-developed management environment, the internal audit activity would: a. Report the results of an audit engagement

to line management as well as to senior management.

b. Conduct initial audits of new computer systems after they have begun operating.

c. Interface primarily with senior management, minimizing interactions with line managers who are the subjects of internal audit work.

d. Focus primarily on asset management and report results to the audit committee.

I - 18

44. ¿Cuál de las siguientes situaciones describe mejor el propósito de un auditor interno al revisar los procesos existentes de administración de riesgos, control y gobierno de la organización? a. Ayudar a determinar la naturaleza, la

oportunidad y el alcance de las pruebas necesarias para lograr los objetivos del trabajo.

b. Asegurarse de corregir las debilidades del sistema de control interno.

c. Proveer la seguridad razonable de que los procesos harán posible que se alcancen los objetivos y metas de la organización de manera eficiente y económica.

d. Determinar si los procesos garantizan que los registros contables son correctos y los estados financieros se declaran de manera justa.

45. ¿Cuál de las siguientes representa la mejor

estructura de gobierno? Gerencia Gerencia Auditoría operativa ejecutiva interna

a. Responsabilidad Rol de Rol de para el riesgo supervisión consultor

b. Rol de Responsab. Rol de supervisión para el riesgo consultor

c. Responsabilidad Rol de Rol de para el riesgo consultor supervisión

d. Rol de Rol de Responsab. supervisión consultor para el riesgo

46. ¿Cuál de las siguientes no es una

responsabilidad del director ejecutivo de auditoría? a. Comunicar los requerimientos de los

planes y recursos de la actividad de auditoría interna a la alta gerencia y al consejo para su revisión y aprobación.

b. Coordinar con otros proveedores internos y externos de servicios de auditoría y consulta para garantizar la cobertura adecuada y minimizar la duplicación.

c. Para vigilar el establecimiento, la administración y la evaluación del sistema de procesos de administración de riesgos de la organización.

d. Hacer un seguimiento para determinar si se tomaron las acciones gerenciales apropiadas sobre riesgos significativos registrados.

44. Which of the following best describes an internal auditor's purpose in reviewing the organization’s existing risk management, control, and governance processes? a. To help determine the nature, timing, and

extent of tests necessary to achieve engagement objectives.

b. To ensure that weaknesses in the internal control system are corrected.

c. To provide reasonable assurance that the processes will enable the organization's objectives and goals to be met efficiently and economically.

d. To determine whether the processes ensure that the accounting records are correct and that financial statements are fairly stated.

45. Which of the following represents the best

governance structure? Operating Executive Internal Management Management Auditing

a. Responsibility Oversight Advisory for risk role role

b. Oversight Responsibility Advisory role for risk role

c. Responsibility Advisory Oversight for risk role role

d. Oversight Advisory Responsibility role role for risk

46. Which of the following is not a responsibility of

the chief audit executive? a. To communicate the internal audit

activity’s plans and resource requirements to senior management and the board for review and approval.

b. To coordinate with other internal and external providers of audit and consulting services to ensure proper coverage and minimize duplication.

c. To oversee the establishment, administration, and assessment of the organization’s system of risk management processes.

d. To follow up on whether appropriate management actions have been taken on significant reported risks.

I - 19

47. ¿Qué afirmación describe con mayor exactitud la manera en que se establecen los criterios para que los utilicen los auditores internos al determinar si se alcanzaron las metas y los objetivos? a. La gerencia es responsable de establecer

los criterios. b. Los auditores internos deben usar las

normas profesionales o las reglamentaciones gubernamentales para establecer los criterios.

c. La industria en la cual opera una compañía establece criterios para cada compañía miembro mediante parámetros y mejores prácticas para esa industria.

d. Como criterios deben utilizarse normas contables o de auditoría apropiadas, entre las cuales se incluyan las normas internacionales.

48. ¿Cuál de los siguientes no es un rol de la

actividad de auditoría interna en las actividades gubernamentales de las mejores prácticas? a. Respaldar al consejo en la evaluación de

riesgos al nivel empresarial. b. Garantizar la implementación oportuna de

las recomendaciones de auditoría. c. Vigilar el cumplimiento del código de

conducta corporativo. d. Conversar sobre áreas de riesgos

significativos. 49. Las evaluaciones de la independencia de los

auditores externos de una organización: a. Deben llevarse a cabo sólo cuando se

designe al auditor externo. b. No deben incluir participación alguna por

parte de la actividad de auditoría interna. c. Deben incluir la actividad de auditoría

interna sólo cuando se designe al auditor externo.

d. Deben incluir la actividad de auditoría interna en el momento de la designación y luego con regularidad.

47. Which statement most accurately describes how criteria are established for use by internal auditors in determining whether goals and objectives have been accomplished? a. Management is responsible for

establishing the criteria. b. Internal auditors should use professional

standards or government regulations to establish the criteria.

c. The industry in which a company operates establishes criteria for each member company through benchmarks and best practices for that industry.

d. Appropriate accounting or auditing standards, including international standards, should be used as the criteria.

48. Which of the following is not a role of the

internal audit activity in best practice governance activities? a. Support the board in enterprise-wide risk

assessment. b. Ensure the timely implementation of audit

recommendations. c. Monitor compliance with the corporate

code of conduct. d. Discuss areas of significant risks.

49. Assessments of the independence of an

organization’s external auditors should: a. Be carried out only when the external

auditor is appointed. b. Not include any participation by the

internal audit activity. c. Include the internal audit activity only

when the external auditor is appointed. d. Include the internal audit activity at the

time of appointment and regularly thereafter.

I - 20

50. Durante una revisión de contratos, el director ejecutivo de auditoría sospecha que a un proveedor se le había concedido una ventaja no equitativa para concursar por un contrato. Al enterarse de que el director general de la compañía es miembro del consejo directivo del proveedor, ¿cómo debiera proceder dicho ejecutivo? a. Remitir un borrador del informe a la alta

dirección, excluyendo al director general. b. Entrar en contacto con los auditores

externos de la organización para obtener ayuda.

c. Obtener documentación comprobatoria y presentar la situación al presidente del comité de auditoría.

d. Notificar inmediatamente al consejo directivo.

51. La compañía A tiene un código corporativo

formal, mientras que la compañía B, no. El código de ética cubre aspectos tales como los acuerdos de compras y las relaciones con los proveedores, así como muchas otras cuestiones para guiar la conducta individual dentro de la compañía. ¿Cuál de las siguientes afirmaciones puede deducirse lógicamente?

I. La compañía A muestra un nivel más alto

de conducta ética que la compañía B. II. La compañía A estableció los criterios

mediante los cuales pueden evaluarse las acciones de un empleado.

III. La ausencia de un código de ética corporativo formal en la compañía B impediría lograr el desarrollo de una auditoría de conducta ética en esa compañía.

a. II solamente. b. III solamente. c. I y II solamente. d. II y III solamente.

50. During a review of contracts, a chief audit executive (CAE) suspects that a supplier was given an unfair advantage in bidding on a contract. After learning that the chief executive officer (CEO) of the company is a member of the supplier's board of directors, how should the CAE proceed? a. Submit a draft report to senior

management, excluding the CEO. b. Contact the organization's external

auditors for assistance. c. Obtain supporting documentation and

present the finding to the chairperson of the audit committee.

d. Immediately notify the board of directors. 51. Company A has a formal corporate code of

ethics while company B does not. The code of ethics covers such things as purchase agreements and relationships with vendors as well as many other issues to guide individual behavior within the company. Which of the following statements can be logically inferred?

I. Company A exhibits a higher standard of

ethical behavior than does company B. II. Company A has established objective

criteria by which an employee’s actions can be evaluated.

III. The absence of a formal corporate code of ethics in company B would prevent a successful audit of ethical behavior in that company.

a. II only. b. III only. c. I and II only. d. II and III only.

I - 21

52. La gerencia y el consejo directivo son responsables de hacer un seguimiento de las observaciones y recomendaciones sugeridas por los auditores externos. ¿Qué rol, si es que existe alguno, debe tener la auditoría interna en este proceso? a. La actividad de auditoría interna no debe

tener rol alguno en este proceso para asegurar la independencia.

b. La actividad de auditoría interna sólo debe participar si el director ejecutivo de auditoría tiene suficiente evidencia de que no se está realizando el seguimiento.

c. La actividad de auditoría interna debe establecer un proceso de monitoreo para vigilar la adecuación y eficacia de las acciones de seguimiento de la gerencia.

d. La actividad de auditoría interna debe participar sólo si lo requiere específicamente la gerencia o el consejo directivo.

53. La razón principal por la cual un banco

mantendría una función de cumplimiento separada es para: a. Administrar mejor los altos riesgos

percibidos. b. Fortalecer los controles sobre las

inversiones bancarias. c. Asegurar la independencia de la gerencia

de línea y la alta gerencia. d. Responder mejor a las expectativas de

los accionistas. 54. La función del director de riesgos es más

eficaz cuando: a. Administra el riesgo como miembro de la

alta gerencia. b. Comparte la administración de riesgos

con la gerencia de línea. c. Comparte la administración de riesgos

con el director ejecutivo de auditoría. d. Supervisa el riesgo como parte del equipo

de administración de riesgos empresariales.

55. Para minimizar las posibles pérdidas

financieras asociadas con los activos físicos, los activos deben asegurarse por una cantidad que: a. Tenga el respaldo de las evaluaciones

periódicas. b. El consejo directivo determine. c. Se ajuste automáticamente mediante un

indicador económico tal como el índice de precios al consumidor.

d. Sea igual al valor contable de los activos individuales.

52. Management and the board of directors are responsible for following up on observations and recommendations made by the external auditors. What role, if any, should the internal audit activity have in this process? a. The internal audit activity should have no

role in this process in order to ensure independence.

b. The internal audit activity should only become involved if the chief audit executive has sufficient evidence that the follow-up is not occurring.

c. The internal audit activity should establish a monitoring process to review the adequacy and effectiveness of management’s follow-up actions.

d. The internal audit activity should become involved only if specifically requested by management or the board of directors.

53. The primary reason that a bank would maintain

a separate compliance function is to: a. Better manage perceived high risks. b. Strengthen controls over the bank’s

investments. c. Ensure the independence of line and

senior management. d. Better respond to shareholder

expectations. 54. The function of the chief risk officer (CRO) is

most effective when the CRO: a. Manages risk as a member of senior

management. b. Shares the management of risk with line

management. c. Shares the management of risk with the

chief audit executive. d. Monitors risk as part of the enterprise risk

management team. 55. To minimize potential financial losses

associated with physical assets, the assets should be insured in an amount that is: a. Supported by periodic appraisals. b. Determined by the board of directors. c. Automatically adjusted by an economic

indicator such as the consumer price index.

d. Equal to the book value of the individual assets.

I - 22

56. ¿Cuál o cuáles de las siguientes declaraciones son correctas en relación con los sistemas corporativos de remuneración y bonos (bonificaciones) relacionados?

I. Un sistema de bonos debe considerarse

formando parte del ambiente de control de una organización y debe incluirse al formularse un informe sobre control interno.

II. Los sistemas de remuneración no forman parte del sistema de control de una organización y no debe informarse respecto a ellos.

III. Una auditoría del sistema de remuneración de una organización debe llevarse a cabo independiente de una auditoría al sistema de control de otras funciones que afecten a los bonos corporativos.

a. I solamente. b. II solamente. c. III solamente. d. II y III solamente.

57. ¿Cuál de las siguientes afirmaciones respecto

del gobierno corporativo no es correcta? a. Entre los mecanismos corporativos de

control se incluyen los internos y los externos.

b. El esquema de compensación para la gerencia es parte de los mecanismos corporativos de control.

c. La dilución de la riqueza de los accionistas provocada por las opciones de compra de acciones de los empleados o de sus dividendos constituye una cuestión contable más que una cuestión de gobierno corporativo.

d. El auditor interno de una compañía tiene más responsabilidad que el consejo directivo del gobierno corporativo de la compañía.

58. La actividad de comerciar futuros con el

objetivo de reducir o controlar el riesgo se denomina: a. Seguros. b. Operaciones de protección cambiaria. c. Venta al descubierto. d. Factoraje.

56. Which of the following statements is correct regarding corporate compensation systems and related bonuses?

I. A bonus system should be considered

part of the control environment of an organization and should be considered in formulating a report on internal control.

II. Compensation systems are not part of an organization's control system and should not be reported as such.

III. An audit of an organization’s compensation system should be performed independently of an audit of the control system over other functions that impact corporate bonuses.

a. I only. b. II only. c. III only. d. II and III only.

57. Which of the following statements regarding

corporate governance is not correct? a. Corporate control mechanisms include

internal and external mechanisms. b. The compensation scheme for

management is part of the corporate control mechanisms.

c. The dilution of shareholders’ wealth resulting from employee stock options or employee stock bonuses is an accounting issue rather than a corporate governance issue.

d. The internal auditor of a company has more responsibility than the board for the company’s corporate governance.

58. The activity of trading futures with the objective

of reducing or controlling risk is called: a. Insuring. b. Hedging. c. Short-selling. d. Factoring.

I - 23

59. La administración de riesgos empresariales: a. Garantiza el logro de los objetivos

organizacionales. b. Requiere el establecimiento de

actividades de riesgo y control por parte de los auditores internos.

c. Implica la identificación de acontecimientos con impactos negativos sobre los objetivos organizacionales.

d. Incluye la selección de la mejor respuesta de riesgo para la organización.

60. ¿Qué es el riesgo residual?

a. EL impacto del riesgo. b. El riesgo que se encuentra bajo control. c. El riesgo que no se administra. d. El riesgo subyacente en el ambiente.

Utilice la siguiente información para contestar las preguntas 61 y 62. El departamento de marketing de una importante compañía minorista asigna a gerentes de producto por separado para cada línea de producto. Los gerentes de producto son responsables de ordenar productos y determinar precios minoristas. El gerente de marketing determina el presupuesto de compras de cada gerente de producto. Los productos se entregan a un centro de distribución donde se segregan las mercaderías para su distribución en 52 locales de los departamentos de la compañía. Como se registran los recibos en el centro de distribución, la compañía no mantiene una función de recepción en cada local. Se evalúa a los gerentes de producto sobre una combinación de ventas y utilidad bruta generada de sus líneas de producto. Muchos productos son de temporada y es probable que los gerentes individuales de los locales soliciten que se quiten dichos productos para hacer lugar a los de la próxima temporada. 61. ¿Cuál de las siguientes es una deficiencia de

control en esta situación? a. El gerente del local puede requerir que se

quiten productos, afectando así a la posible evaluación de desempeño de los gerentes individuales de producto.

b. El gerente de producto negocia el precio de compra y fija el precio de venta.

c. La evaluación de los gerentes de producto por las ganancias brutas totales generadas por la línea de producto llevará a una conducta disfuncional.

d. No hay ninguna función de recepción ubicada en los locales individuales.

59. Enterprise risk management: a. Guarantees achievement of

organizational objectives. b. Requires establishment of risk and control

activities by internal auditors. c. Involves the identification of events with

negative impacts on organizational objectives.

d. Includes selection of the best risk response for the organization.

60. What is residual risk?

a. Impact of risk. b. Risk that is under control. c. Risk that is not managed. d. Underlying risk in the environment.

Use the following information to answer questions 61 through 62.

The marketing department for a major retailer assigns separate product managers for each product line. Product managers are responsible for ordering products and determining retail pricing. Each product manager’s purchasing budget is set by the marketing manager. Products are delivered to a central distribution center where goods are segregated for distribution to the company’s 52 department stores. Because receipts are recorded at the distribution center, the company does not maintain a receiving function at each store. Product managers are evaluated on a combination of sales and gross profit generated from their product lines. Many products are seasonal and individual store managers can require that seasonal products be removed to make space for the next season's products. 61. Which of the following is a control deficiency in

this situation? a. The store manager can require items to

be removed, thus affecting the potential performance evaluation of individual product managers.

b. The product manager negotiates the purchase price and sets the selling price.

c. Evaluating product managers by total gross profit generated by product line will lead to dysfunctional behavior.

d. There is no receiving function located at individual stores.

I - 24

62. Los requerimientos de compras que excedan los inicialmente presupuestados deberán ser aprobados por el gerente de marketing. Este procedimiento:

I. Debe proveer la asignación más eficiente

de escasos recursos organizacionales. II. Es un procedimiento de control de

detección. III. Resulta innecesario porque se evalúa a

cada gerente de producto sobre las utilidades generadas.

a. I solamente. b. III solamente. c. II y III solamente. d. I, II, y III.

63. La gerencia de una organización percibe la

necesidad de hacer cambios importantes. ¿Cuál de los siguientes factores es el que con menor probabilidad la gerencia puede ser capaz de cambiar? a. Los miembros de la organización. b. La estructura de la organización. c. El ambiente de la organización. d. La tecnología de la organización.

64. Muchas organizaciones utilizan la

transferencia electrónica de fondos para pagar a sus proveedores en lugar de emitir cheques. Con respecto a los riesgos asociados con la emisión de cheques, ¿cuál de las siguientes técnicas de administración de riesgos representa esta situación? a. Controlar. b. Aceptar. c. Transferir. d. Evitar.

65. ¿Cuál de las siguientes metas determina

estrategias de administración de riesgos a un nivel óptimo? a. Minimizar costos. b. Maximizar la porción del mercado. c. Minimizar pérdidas. d. Maximizar el valor neto.

62. Requests for purchases beyond those initially budgeted must be approved by the marketing manager. This procedure: I. Should provide for the most efficient

allocation of scarce organizational resources.

II. Is a detective control procedure. III. Is unnecessary because each product

manager is evaluated on profit generated. a. I only. b. III only. c. II and III only. d. I, II, and III.

63. An organization's management perceives the

need to make significant changes. Which of the following factors is management least likely to be able to change? a. The organization's members. b. The organization's structure. c. The organization's environment. d. The organization's technology.

64. Many organizations use electronic funds

transfer to pay their suppliers instead of issuing checks. Regarding the risks associated with issuing checks, which of the following risk management techniques does this represent? a. Controlling. b. Accepting. c. Transferring. d. Avoiding.

65. Which of the following goals sets risk

management strategies at the optimum level? a. Minimize costs. b. Maximize market share. c. Minimize losses. d. Maximize shareholder value.

I - 25

66. De las siguientes razones por las cuales los empleados se resisten a un cambio fundamental en los procesos organizacionales, ¿cuál es menos probable? a. Amenaza de pérdida de empleos. b. Asistencia requerida a clases de

capacitación. c. Separación de grupos existentes de

trabajo. d. Imposición de nuevos procesos por parte

de la alta gerencia sin conversaciones previas.

67. Todo lo siguiente formaría parte de un sistema

de control para evitar la liberación de consumos de agua no reciclables que no cumplan con las normas de descarga excepto: a. Realizar análisis químicos del agua

previos a la descarga para ver si los componentes están especificados en la autorización.

b. Especificar (por medio de emitir una política, proporcionar capacitación y colocar rótulos) cuáles sustancias pueden ser desalojadas vía vertederos y drenajes dentro de las instalaciones.

c. Periódicamente lavar los vertederos y drenajes con un gran volumen de agua limpia para asegurar que los agentes contaminantes de diluyan en forma suficiente.

d. Establecer un programa de mantenimiento preventivo para el sistema de pretratamiento.

68. Una organización está cambiando hacia la

implementación de un programa de aseguramiento de calidad que incorpora la calidad en todo el proceso. Es muy diferente de los años en que se dependía del control de calidad al terminar el proceso. Este tipo de cambio es: a. Un cambio cultural. b. Un cambio de producto. c. Un cambio estructural. d. Un cambio organizacional.

66. Of the following reasons for employees to resist a major change in organizational processes, which is least likely? a. Threat of loss of jobs. b. Required attendance at training classes. c. Breakup of existing work groups. d. Imposition of new processes by senior

management without prior discussion. 67. All of the following would be part of a factory’s

control system to prevent release of waste water that does not meet discharge standards except: a. Performing chemical analysis of the

water, prior to discharge, for components specified in the permit.

b. Specifying (by policy, training, and advisory signs) which substances may be disposed of via sinks and floor drains within the factory.

c. Periodically flushing sinks and floor drains with a large volume of clean water to ensure pollutants are sufficiently diluted.

d. Establishing a preventive maintenance program for the factory’s pretreatment system.

68. An organization is changing to a quality

assurance program that incorporates quality throughout the process. This is very different from its years of dependence on quality control at the end of the process. This type of change is a: a. Cultural change. b. Product change. c. Structural change. d. Organizational change.

I - 26

69. Un director ejecutivo de auditoría planifica implementar cambios que pueden ser percibidos negativamente por el personal de auditoría. La mejor manera de reducir la resistencia sería: a. Desarrollar el nuevo enfoque en forma

total antes de presentarlo al personal de auditoría.

b. Pedirle al director general que apruebe los cambios y hacer que éste asista a la reunión del personal departamental cuando se presenten dichas modificaciones.

c. Abordar al personal con la idea general y hacerlo participar en el desarrollo de los cambios.

d. Hacer que los clientes de la actividad de auditoría interna respalden los cambios.

70. Durante una reunión del equipo del proyecto

de auditoría interna, dos miembros disienten, y uno acusa al otro de tratar de hacer prevalecer los intereses personales sobre los intereses de la auditoría. El gerente de auditoría debe: a. Disciplinar a ambos auditores después de

la reunión por su falta de conducta profesional.

b. Continuar la reunión, pero hablar con el acusador más tarde con respecto a la conducta inapropiada.

c. Encontrarse con ambos auditores después de la reunión para resolver el conflicto y la conducta inapropiada.

d. Detener la reunión y abordar la cuestión con todo el equipo para debatirla.

71. El control que más probablemente aseguraría

que los cheques para el pago de la nómina se emiten sólo por las cantidades autorizadas es: a. Conducir la verificación periódica de los

empleados de la nómina. b. Solicitar la devolución al cajero de los

cheques no entregados. c. Requerir la aprobación de las tarjetas

reloj de los empleados. d. Presenciar la distribución de los cheques

para los pagos por nómina periódicamente.

69. A chief audit executive plans to make changes that may be perceived negatively by the audit staff. The best way to reduce resistance would be to: a. Develop the new approach fully before

presenting it to the audit staff. b. Ask the chief executive officer (CEO) to

approve the changes and have the CEO attend the departmental staff meeting when they are presented.

c. Approach the staff with the general idea and involve them in the development of the changes.

d. Get the internal audit activity’s clients to support the changes.

70. During a meeting of an internal audit project

team, two members of the team disagree, and one accuses the other of trying to advance personal interests over the interests of the audit. The audit manager should: a. Discipline both auditors after the meeting

for their lack of professional conduct. b. Continue the meeting but speak to the

accusing auditor later regarding the inappropriate conduct.

c. Meet with both auditors after the meeting to resolve the conflict and the inappropriate behavior.

d. Stop the meeting and refer the matter to the entire team for discussion.

71. The control that would most likely ensure that

payroll checks are written only for authorized amounts is to: a. Conduct periodic floor verification of

employees on the payroll. b. Require the return of undelivered checks

to the cashier. c. Require supervisory approval of

employee time cards. d. Periodically witness the distribution of

payroll checks.

I - 27

72. ¿Cuál de los siguientes controles evitaría pedir cantidades que excedan las necesidades de una organización? a. La revisión de todos los requerimientos

de compra por parte de un supervisor en el departamento del usuario antes de presentarlos al departamento de compras.

b. La nueva emisión automática de un pedido por parte del departamento de compras cuando el sistema indique un bajo nivel de inventario.

c. Una política que requiera la revisión de la orden de compra antes de recibir un nuevo despacho.

d. Una política que requiera la aceptación del informe de recepción y del detalle de empaque antes de archivar nuevos recibos.

73. ¿Cuál de las siguientes observaciones por

parte de un auditor es más probable que indique la existencia de debilidades de control sobre la salvaguarda de activos? I. La ubicación de un departamento de

servicio no es adecuada para ofrecer un servicio apropiado a las otras unidades.

II. Los empleados contratados para posiciones sensibles no se encuentran sujetos a verificaciones de antecedentes.

III. Los gerentes no tienen acceso a informes que perfilen el desempeño global en comparación con otras organizaciones.

IV. La gerencia no tomó ninguna acción correctiva para resolver observaciones anteriores del trabajo en relación con controles de inventario.

a. I y II solamente. b. I y IV solamente. c. II y III solamente. d. II y IV solamente.

74. Un control que probablemente evite que los

agentes de compras favorezcan a proveedores específicos es: a. Solicitar la revisión por parte de la

gerencia de un informe mensual que detalle los montos abonados por cada comprador.

b. Solicitar a los compradores que cumplan con las especificaciones detalladas de los materiales.

c. Rotar las tareas del comprador en forma periódica.

d. Supervisar la cantidad de órdenes solicitadas por cada comprador.

72. Which of the following controls would prevent the ordering of quantities in excess of an organization’s needs? a. Review of all purchase requisitions by a

supervisor in the user department prior to submitting them to the purchasing department.

b. Automatic reorder by the purchasing department when low inventory level is indicated by the system.

c. A policy requiring review of the purchase order before receiving a new shipment.

d. A policy requiring agreement of the receiving report and packing slip before storage of new receipts.

73. Which of the following observations by an

auditor is most likely to indicate the existence of control weaknesses over safeguarding of assets? I. A service department's location is not well

suited to allow adequate service to other units.

II. Employees hired for sensitive positions are not subjected to background checks.

III. Managers do not have access to reports that profile overall performance in relation to other benchmarked organizations.

IV. Management has not taken corrective action to resolve past engagement observations related to inventory controls.

a. I and II only. b. I and IV only. c. II and III only. d. II and IV only.

74. A control likely to prevent purchasing agents

from favoring specific suppliers is: a. Requiring management’s review of a

monthly report of the totals spent by each buyer.

b. Requiring buyers to adhere to detailed material specifications.

c. Rotating buyer assignments periodically. d. Monitoring the number of orders placed

by each buyer.

I - 28

75. ¿Cuál de las siguientes opciones minimizaría los defectos de los productos terminados provocados por materias primas de baja calidad? a. Procedimientos documentados para el

manejo apropiado del inventario de los trabajos en proceso.

b. Especificaciones de materiales requeridas para todas las compras.

c. Seguimiento oportuno de todas las variantes desfavorables de uso.

d. Determinación de la cantidad de desechos que quedan al terminar el proceso de manufactura.

76. El requerimiento de que las compras se hagan

a proveedores de una lista autorizada constituye un ejemplo de: a. Un control preventivo. b. Un control detectivo. c. Un control correctivo. d. Un control de monitoreo.

77. El control interno apropiado para la sucursal

de una corporación multinacional que posee una unidad de transferencia monetaria requiere que: a. El individuo que inicia transferencias

cablegráficas no concilie el resumen de cuenta.

b. El gerente de sucursal reciba todas las transferencias cablegráficas.

c. Que dos empleados diferentes computen las tasas de traducción de moneda extranjera por separado.

d. La gerencia corporativa apruebe la contratación de empleados de la unidad de transferencia monetaria.

78. ¿Cuál de los siguientes describe mejor una

encuesta preliminar? a. Un cuestionario estandarizado que se use

para comprender los objetivos de la gerencia.

b. Una muestra estadística para revisar las actitudes, las destrezas y el conocimiento clave.

c. Un “walk-through” del sistema de controles financieros para identificar los riesgos y los controles que puedan abordar dichos riesgos.

d. Un proceso utilizado para familiarizarse con actividades y riesgos a fin de identificar áreas para dar énfasis a los trabajos.

75. Which of the following would minimize defects in finished goods caused by poor quality raw materials? a. Documented procedures for the proper

handling of work-in-process inventory. b. Required material specifications for all

purchases. c. Timely follow-up on all unfavorable usage

variances. d. Determination of the amount of spoilage

at the end of the manufacturing process. 76. The requirement that purchases be made from

suppliers on an approved vendor list is an example of a: a. Preventive control. b. Detective control. c. Corrective control. d. Monitoring control.

77. Appropriate internal control for a multinational

corporation’s branch office that has a monetary transfer unit requires that: a. The individual who initiates wire transfers

not reconcile the bank statement. b. The branch manager receive all wire

transfers. c. Foreign currency rates be computed

separately by two different employees. d. Corporate management approve the

hiring of monetary transfer unit employees.

78. Which of the following best describes a

preliminary survey? a. A standardized questionnaire used to

obtain an understanding of management objectives.

b. A statistical sample to review key employee attitudes, skills, and knowledge.

c. A walk-through of the financial control system to identify risks and the controls that can address those risks.

d. A process used to become familiar with activities and risks in order to identify areas for engagement emphasis.

I - 29

79. Durante una encuesta preliminar, un auditor descubrió que diversos comprobantes de cuentas a pagar emitidos para los proveedores más importantes requerían ajustes por el pago duplicado de facturas anteriores. Esta situación indicaría: a. Una necesidad de pruebas adicionales

para determinar controles afines y la exposición actual a pagos duplicados hechos a los proveedores.

b. La posibilidad de pasivos no registrados para la cantidad de pagos en exceso.

c. Controles insuficientes en el área de recepción para asegurar el envío de una notificación oportuna al área de cuentas a pagar en la cual se especifique que se recibieron e inspeccionaron las mercaderías.

d. La existencia de un sistema sofisticado de cuentas a pagar que correlacione los pagos en exceso con las facturas abiertas y así se eviten más preocupaciones por parte de auditoría.

80. ¿Cuál de los siguientes procedimientos se

desarrollaría como parte de una revisión preliminar en una auditoría de las actividades de inversiones y préstamos de un banco? a. Revisar informes de auditorías realizadas

por auditores reguladores y externos desde el último trabajo de auditoría interna.

b. Entrevistar a la gerencia para identificar los cambios implementados en las políticas con respecto a inversiones o préstamos.

c. Revisar las minutas de las reuniones del consejo directivo a fin de identificar los cambios implementados en las políticas que afectan a inversiones y préstamos.

d. Todos los anteriores. 81. Durante una evaluación de riesgos asociada

con contratos de ventas y comisiones afines, ¿cuál de los siguientes factores probablemente provocaría una expansión del alcance del trabajo? a. Un incremento en las ventas de

productos, junto con un aumento en las comisiones.

b. Un incremento en los ingresos por ventas, junto con un aumento en las comisiones.

c. Un decrecimiento en las comisiones por ventas, junto con una disminución en las ventas de productos.

d. Un decrecimiento en los ingresos por ventas, junto con un aumento en las ventas de productos.

I - 30

79. During a preliminary survey, an auditor found that several accounts payable vouchers for major suppliers required adjustments for duplicate payment of prior invoices. This would indicate: a. A need for additional testing to determine

related controls and the current exposure to duplicate payments made to suppliers.

b. The possibility of unrecorded liabilities for the amount of the overpayments.

c. Insufficient controls in the receiving area to ensure timely notice to the accounts payable area that goods have been received and inspected.

d. The existence of a sophisticated accounts payable system that correlates overpayments to open invoices and therefore requires no further audit concern.

80. Which of the following procedures should be

performed as part of a preliminary review in an audit of a bank’s investing and lending activities? a. Review reports of audits performed by

regulatory and outside auditors since the last internal audit engagement.

b. Interview management to identify changes made in policies regarding investments or loans.

c. Review minutes of the board of directors’ meetings to identify changes in policies affecting investments and loans.

d. All of the above. 81. During an assessment of the risk associated

with sales contracts and related commissions, which of the following factors would most likely result in an expansion of the engagement scope? a. An increase in product sales, along with

an increase in commissions. b. An increase in sales returns, along with

an increase in commissions. c. A decrease in sales commissions, along

with a decrease in product sales. d. A decrease in sales returns, along with an

increase in product sales.

I - 31

82. En una entrevista, un auditor experimentado en temas de calidad del aire descubrió que el gerente del departamento de protección ambiental, de seguridad y salud no conocía todos los requerimientos legales para el control de las emisiones en la atmósfera. El auditor debe: a. Modificar el alcance de la auditoría para

centrarse en las actividades asociadas con las emisiones en la atmósfera.

b. Compartir su amplio conocimiento con el gerente de protección ambiental, de seguridad y salud.

c. Tomar nota de las debilidades y formular preguntas adicionales para determinar el posible efecto de la falta de conocimiento.

d. Informar las posibles violaciones en esta área al organismo gubernamental apropiado.

83. ¿Cuál de las siguientes es una afirmación

apropiada de un objetivo del trabajo de auditoría? a. Observar el recuento del inventario físico. b. Determinar si las existencias son

suficientes para cumplir con las ventas proyectadas.

c. Buscar existencias obsoletas mediante una rotación de inventario computarizada por línea de productos.

d. Incluir información sobre las existencias de salida en la comunicación final del trabajo.

84. Un auditor interno planifica conducir una

auditoría para verificar la adecuación de los controles sobre las inversiones en nuevos instrumentos financieros. ¿Cuál de las siguientes opciones no se requeriría como parte de un trabajo de esta índole? a. Determinar si existen políticas que

describan los riesgos que puede tomar el tesorero y los tipos de instrumentos en los cuales puede invertir el tesorero.

b. Determinar el alcance de la supervisión por parte de la gerencia de las inversiones en instrumentos sofisticados.

c. Determinar si el tesorero obtiene tasas más altas o más bajas del rendimiento de las inversiones con respecto a los tesoreros de otras organizaciones.

d. Determinar la naturaleza de los controles establecidos por el tesorero para vigilar los riesgos que puede haber en las inversiones.

82. An auditor, experienced in air-quality issues, discovered a significant lack of knowledge about legal requirements for controlling air emissions while interviewing the manager of the environmental, health, and safety (EHS) department. The auditor should: a. Alter the scope of the engagement to

focus on activities associated with air emissions.

b. Share extensive personal knowledge with the EHS manager.

c. Take note of the weakness and direct additional questions to determine the potential effect of the lack of knowledge.

d. Report potential violations in this area to the appropriate regulatory agency.

83. Which of the following is an appropriate

statement of an audit engagement objective? a. To observe the physical inventory count. b. To determine whether inventory stocks

are sufficient to meet projected sales. c. To search for the existence of obsolete

inventory by computing inventory turnover by product line.

d. To include information about stockouts in the engagement final communication.

84. An internal auditor plans to conduct an audit of

the adequacy of controls over investments in new financial instruments. Which of the following would not be required as part of such an engagement? a. Determine if policies exist which describe

the risks the treasurer may take and the types of instruments in which the treasurer may make investments.

b. Determine the extent of management oversight over investments in sophisticated instruments.

c. Determine whether the treasurer is getting higher or lower rates of return on investments than are treasurers in comparable organizations.

d. Determine the nature of controls established by the treasurer to monitor the risks in the investments.

I - 32

85. Si las normas de operación de un auditado son imprecisas y por consiguiente se encuentran sujetas a interpretación, el auditor debe: a. Buscar un acuerdo con el auditado

respecto de los criterios por utilizar para medir el desempeño operativo.

b. Determinar las mejores prácticas en esta área y utilizarlas como norma.

c. Interpretar las normas en su sentido más estricto, porque de otro modo son sólo medidas mínimas de aceptación.

d. Omitir cualquier comentario sobre las normas y el desempeño del auditado en relación con ellas, ya que un análisis de esa índole no tendría sentido.

86. Si a raíz de la evaluación preliminar de

controles internos efectuada por un auditor se observa que los controles pueden resultar inadecuados, el siguiente paso sería: a. Expandir el trabajo de auditoría antes de

preparar la comunicación final del trabajo. b. Preparar un diagrama de flujo que

represente el sistema de controles internos.

c. Observar una excepción en la comunicación final del trabajo si surgieron pérdidas.

d. Implementar los controles deseados.

85. If a department’s operating standards are vague and thus subject to interpretation, an auditor should: a. Seek agreement with the departmental

manager as to the criteria needed to measure operating performance.

b. Determine best practices in the area and use them as the standard.

c. Interpret the standards in their strictest sense because standards are otherwise only minimum measures of acceptance.

d. Omit any comments on standards and the department’s performance in relationship to those standards, because such an analysis would be inappropriate.

86. If an auditor's preliminary evaluation of internal

controls results in an observation that controls may be inadequate, the next step would be to: a. Expand audit work prior to the preparation

of an engagement final communication. b. Prepare a flowchart depicting the internal

control system. c. Note an exception in the engagement

final communication if losses have occurred.

d. Implement the desired controls.

I - 33

Utilice la siguiente información para contestar las preguntas 87 y 88.

El gerente de una línea de productos tiene la autoridad de pedir y recibir los repuestos de toda la maquinaria que requiere mantenimiento periódico. El auditor interno recibió un informe anónimo en el cual se detallaba que el gerente había pedido muchos más repuestos de los necesarios a un miembro de su familia que trabajaba en el negocio de suministro de piezas. Los repuestos innecesarios nunca se entregaron. En cambio, el gerente procesó los documentos de recepción y cobró las partes a las cuentas de mantenimiento de maquinarias. Los pagos de los repuestos innecesarios se enviaron al proveedor; el dinero se dividió entre el gerente y el miembro de su familia. 87. ¿Cuál de los siguientes controles internos

podría haber evitado este fraude? a. Establecer niveles predefinidos de gastos

para todos los proveedores durante el proceso de licitación.

b. Segregar la función de la autorización de compras de repuestos.

c. Comparar el conocimiento de embarque emitido para los repuestos con la orden de compra aprobada.

d. Utilizar el sistema de inventario de la compañía para comparar las cantidades requeridas con las recibidas.

88. ¿Cuál de las siguientes pruebas asistiría mejor

al auditor al decidir si debe seguir investigando este informe anónimo? a. La comparación de los gastos de

mantenimiento del trimestre actual con la actividad de períodos anteriores.

b. Las pruebas del inventario físico de los repuestos para constatar su existencia y valuación.

c. El análisis de repuestos cobrados a mantenimiento para revisar la razonabilidad de la cantidad de artículos reemplazados.

d. La revisión de un modelo de prueba de las facturas de los repuestos a fin de obtener la autorización y recepción adecuadas.

Use the following information to answer questions 87 through 88.

The manager of a production line has the authority to order and receive replacement parts for all machinery that require periodic maintenance. The internal auditor received an anonymous tip that the manager ordered substantially more parts than were necessary from a family member in the parts supply business. The unneeded parts were never delivered. Instead, the manager processed receiving documents and charged the parts to machinery maintenance accounts. The payments for the undelivered parts were sent to the supplier, and the money was divided between the manager and the family member. 87. Which of the following internal controls would

have most likely prevented this fraud from occurring? a. Establishing predefined spending levels

for all vendors during the bidding process. b. Segregating the receiving function from

the authorization of parts purchases. c. Comparing the bill of lading for

replacement parts to the approved purchase order.

d. Using the company’s inventory system to match quantities requested with quantities received.

88. Which of the following tests would best assist

the auditor in deciding whether to investigate this anonymous tip further? a. Comparison of the current quarter’s

maintenance expense with prior-period activity.

b. Physical inventory testing of replacement parts for existence and valuation.

c. Analysis of repair parts charged to maintenance to review the reason-ableness of the number of items replaced.

d. Review of a test sample of parts invoices for proper authorization and receipt.

I - 34

89. Al encontrarse con una limitación impuesta en el alcance del trabajo, el director ejecutivo de auditoría debe: a. Diferir el trabajo hasta que se elimine la

limitación del alcance. b. Comunicar los posibles efectos de la

limitación del alcance al comité de auditoría y al consejo directivo.

c. Aumentar la frecuencia de auditoría para la actividad en cuestión.

d. Asignar el trabajo a personal con más experiencia.

90. Se asignó a un auditor la tarea de analizar la

eficacia de un conjunto de programas de rehabilitación. Los programas están en uso desde hace diez años y no se evaluaron. La organización que provee los datos del programa afirma que dichos datos están incompletos. El auditor: a. Debe realizar el análisis de todas

maneras, evaluando los efectos de los datos incompletos, pero también debe aclarar que no se hace responsable de la confiabilidad de los datos.

b. Debe rastrear un conjunto de registros elegidos al azar y compararlos con los archivos fuente para evaluar si los datos suministrados son exactos y están completos.

c. No debe realizar el análisis. d. Debe posponer el análisis hasta que los

datos estén completos. 91. ¿Cuál de las siguientes contabilizaciones

fraudulentas es más probable que se haga para ocultar el robo de un activo? a. Debitar gastos y acreditar el activo. b. Debitar el activo y acreditar otra cuenta

de activos. c. Debitar los ingresos y acreditar el activo. d. Debitar otra cuenta de activos y acreditar

el activo. 92. Es más probable que un sistema adecuado de

controles internos detecte irregularidades perpetradas por: a. Un grupo de empleados en

confabulación. b. Sólo un empleado. c. Un grupo de gerentes en confabulación. d. Sólo un gerente.

89. When faced with an imposed scope limitation, a chief audit executive should: a. Delay the engagement until the scope

limitation is removed. b. Communicate the potential effects of the

scope limitation to the audit committee of the board of directors.

c. Increase the frequency of auditing the activity in question.

d. Assign more experienced personnel to the engagement.

90. An auditor has been assigned to analyze the

effectiveness of a set of rehabilitation programs. The programs have been in operation for ten years and have not been evaluated. The organization providing the program data asserts that the data are incomplete. The auditor should: a. Perform the analysis anyway, assessing

the effects of the incomplete data, but disclaim any assertion regarding data reliability.

b. Trace a randomly chosen set of records to source files to assess the accuracy and completeness of the data provided.

c. Not perform the analysis. d. Postpone the analysis until data are

complete. 91. Which of the following fraudulent entries is

most likely to be made to conceal the theft of an asset? a. Debit expenses, and credit the asset. b. Debit the asset, and credit another asset

account. c. Debit revenue, and credit the asset. d. Debit another asset account, and credit

the asset. 92. An adequate system of internal controls is

most likely to detect an irregularity perpetrated by a: a. Group of employees in collusion. b. Single employee. c. Group of managers in collusion. d. Single manager.

I - 35

93. La administración divisional expresó que el aumento en el margen bruto se debió a una mayor eficiencia en las operaciones de manufactura. ¿Cuál de los siguientes procedimientos de auditoría sería el más pertinente a esta aseveración? a. Realizar un conteo físico del inventario. b. Tomar una muestra de productos y

comparar los costos por unidad de este año con los del año pasado, examinar las estructuras de costos y analizar las variaciones de los costos estándar.

c. Tomar una muestra del inventario físico del equipamiento para determinar si hubo cambios importantes.

d. Tomar una muestra del inventario de productos terminados y, para determinar la exactitud del precio registrado por las materias primas, rastrear su costo buscándolo en los precios de compra.

Utilice la siguiente información para contestar las preguntas 94 y 95. Una compañía mantiene los datos de producción en computadoras personales interconectadas en una red de área local (LAN), y utiliza los datos para la generación automática de órdenes de compra vía intercambio electrónico de datos. Las compras se hacen a proveedores autorizados según los planes de producción del mes siguiente y según el plan aprobado de requerimiento de materiales (MRP), que identifica las partes que se requieren para cada unidad de producción. 94. La línea de producción ha sufrido

interrupciones porque algunas partes no se encontraban disponibles. ¿Cuál de los siguientes procedimientos de auditoría sería el mejor para identificar la causa de dicho problema? a. Determinar si los controles de acceso son

suficientes para restringir el ingreso de datos incorrectos en la base de datos de producción.

b. Utilizar un software general de auditoría para preparar una lista completa de las partes faltantes que originaron cada una de las interrupciones y analizar estos datos.

c. Tomar una muestra al azar de las partes disponibles según la base de datos de las computadoras personales y compararla con las partes que realmente se encuentran disponibles.

d. Tomar una muestra al azar de la información de producción que se recopiló durante determinados días y rastrear su ingreso en la base de datos de producción mantenida en la red (LAN).

I - 36

93. Divisional management stated that a recent gross margin increase was due to increased efficiency in manufacturing operations. Which of the following audit procedures would be most relevant to that assertion? a. Obtain a physical count of inventory. b. Select a sample of products, then

compare costs-per-unit this year to those of last year, test cost buildups, and analyze standard cost variances.

c. Take a physical inventory of equipment to determine if there were significant changes.

d. Select a sample of finished goods inventory and trace raw materials cost back to purchase prices in order to determine the accuracy of the recorded raw materials price.

Use the following information to answer questions 94 through 95.

A company maintains production data on personal computers, connected by a local area network (LAN), and uses the data to generate automatic purchases via electronic data inter-change. Purchases are made from authorized vendors based on production plans for the next month and on an authorized materials requirements plan (MRP) which identifies the parts needed for each unit of production. 94. The production line has experienced shut-

downs because needed production parts were not on hand. Which of the following audit procedures would best identify the cause of the parts shortages? a. Determine if access controls are sufficient

to restrict the input of incorrect data into the production database.

b. Use generalized audit software to develop a complete list of the parts shortages that caused each of the production shutdowns, and analyze this data.

c. Select a random sample of parts on hand per the personal computer databases and compare with actual parts on hand.

d. Select a random sample of production information for selected days and trace input into the production database maintained on the LAN.

I - 37

95. ¿Cuál de los siguientes procedimientos de auditoría resultaría más eficaz para determinar si los requerimientos de compra han sido actualizados para los cambios en las técnicas de producción? a. Recalcular las partes requeridas

basándose en estimados de la producción actual y en el plan autorizado de requerimientos de materiales para las técnicas de producción revisadas. Comparar estas necesidades con las órdenes de compra generadas por el sistema durante el mismo período.

b. Desarrollar datos de prueba para ingresarlos en la red de área local (LAN) y comparar las órdenes de compra generadas por dichos datos con las órdenes de compra generadas por los datos de producción.

c. Utilizar un software general de auditoría para preparar un informe de inventario en exceso. Comparar el inventario con el volumen actual de producción.

d. Tomar una muestra de los estimados de producción y del plan autorizado de requerimiento de materiales por varios períodos y rastrearlos dentro del sistema a fin de determinar que el ingreso es exacto.

96. ¿Cuál de los siguientes factores se

consideraría el menos importante con respecto a decidir si los recursos de la auditoría deben cambiarse de una auditoría de cumplimiento en curso a un trabajo de auditoría de una división solicitada por la gerencia? a. Una auditoría financiera de la división

realizada por el auditor externo hace un año.

b. La posibilidad de fraude asociada con el trabajo en curso.

c. Un incremento en el nivel de gastos de la división durante el año anterior.

d. La posibilidad de fuertes multas impuestas por organismos reguladores asociadas con el trabajo en curso.

95. Which of the following audit procedures would be most effective in determining if purchasing requirements have been updated for changes in production techniques? a. Recalculate parts needed based on

current production estimates and the MRP for the revised production techniques. Compare these needs with purchase orders generated from the system for the same period.

b. Develop test data to input into the LAN and compare purchase orders generated from test data with purchase orders generated from production data.

c. Use generalized audit software to develop a report of excess inventory. Compare the inventory with current production volume.

d. Select a sample of production estimates and MRPs for several periods and trace them into the system to determine that input is accurate.

96. Which of the following factors would be

considered the least important in deciding whether existing internal audit resources should be moved from an ongoing compliance audit engagement to a division audit engagement requested by management? a. A financial audit of the division performed

by the external auditor a year ago. b. The potential for fraud associated with the

ongoing engagement. c. An increase in the level of expenditures

experienced by the division for the past year.

d. The potential for significant regulatory fines associated with the ongoing engagement.

I - 38

97. Por lo general, para controlar los proyectos y evitar excesos en el presupuesto, la revisión del presupuesto de una auditoría debe hacerse: a. Inmediatamente después de concluir la

encuesta preliminar. b. Cuando se ha comprobado una

deficiencia significativa. c. Cuando se asignan auditores sin

experiencia a un trabajo. d. Inmediatamente después de incrementar

las pruebas para establecer la confiabilidad de las observaciones.

98. Determinar que se alcanzaron los objetivos de

la auditoría es finalmente la responsabilidad: a. Del auditor interno. b. Del comité de auditoría. c. Del supervisor de auditoría interna. d. Del director ejecutivo de auditoría.

99. ¿Para cuál de las siguientes situaciones un

programa de auditoría interna estandarizado no sería apropiado? a. Un ambiente operativo estable que

experimente sólo cambios mínimos. b. Un ambiente operativo complejo o con

cambios. c. Múltiples lugares con operaciones

parecidas. d. Auditorías de inventario consecutivas

desarrolladas en el mismo lugar. 100. Los programas de auditoría que examinan

controles internos deben: a. Hacerse a la medida de la auditoría de

cada operación. b. Generalizarse para cubrir todas las

situaciones sin importar las líneas departamentales.

c. Generalizarse para poder usarlos en toda la organización.

d. Reducir la costosa duplicación de esfuerzos al asegurarse de examinar cada aspecto de una operación.

FIN DE LAS PREGUNTAS PARTE I POR FAVOR OBSERVE: La parte I del examen CIA actual constará de 125 preguntas de examen. Entre las 125 preguntas, se incluirán hasta 25 preguntas no registradas, que se utilizarán para propósitos de investigación. Dichas preguntas no registradas se insertarán sin identificar entre las registradas. Por lo tanto, los candidatos debe responder a las 125 preguntas lo mejor posible.

97. As a means of controlling projects and avoiding time-budget overruns, decisions to revise time budgets for an audit engagement should normally be made: a. Immediately after completing the

preliminary survey. b. When a significant deficiency has been

substantiated. c. When inexperienced audit staff members

are assigned to an engagement. d. Immediately after expanding tests to

establish reliability of observations. 98. Determining that engagement objectives have

been met is ultimately the responsibility of the: a. Internal auditor. b. Audit committee. c. Internal audit supervisor. d. Chief audit executive.

99. A standardized internal audit engagement

program would not be appropriate for which of the following situations? a. A stable operating environment

undergoing only minimal changes. b. A complex or changing operating

environment. c. Multiple branches with similar operations. d. Subsequent inventory audit engagements

performed at the same location. 100. Audit engagement programs testing internal

controls should: a. Be tailored for the audit of each operation. b. Be generalized to fit all situations without

regard to departmental lines. c. Be generalized so as to be usable at

various international locations of an organization.

d. Reduce costly duplication of effort by ensuring that every aspect of an operation is examined.

END OF PART I QUESTIONS

PLEASE NOTE: The actual CIA exam Part I will contain 125 exam questions. The 125 questions will include up to 25 unscored questions, which will be used for research purposes. These unscored questions will be interspersed with the scored questions and will not be identified as unscored questions. Candidates should therefore answer all 125 questions to the best of their ability.

I - 39

Soluciones a la Parte I - El rol de la actividad de auditoría interna en el gobierno,

el riesgo y el control

Las soluciones y explicaciones sugeridas para la parte I de las preguntas del Examen modelo para Auditor Interno Certificado se proporcionan en las siguientes páginas.

La tabla que a continuación se presenta asocia los números de las preguntas de la parte I con los temas

evaluados:

Tema Evaluado Número de Pregunta

Cumplimiento con las Normas de Atributo del IAI 1 – 21

Establecer un plan basado en el riesgo para determinar las prioridades de la actividad de auditoría interna.

22 – 40

Comprender el rol de la actividad de auditoría interna en el gobierno de las organizaciones

41 – 52

Otros roles y responsabilidades de la auditoría interna

53 – 55

Elementos sobre el conocimiento del gobierno, riesgo y control.

56 – 77

Planificación de los trabajos 78 – 100

I - 40

1. Solución: b a. Incorrecta. El estatuto de auditoría interna define las autoridades y responsabilidades

necesarias. b. Correcta. El manual de auditoría interna y el plan de auditoría anual ayudan a determinar los

requerimientos de los recursos. c. Incorrecta. El estatuto de auditoría interna define el rol y la responsabilidad de la actividad de

auditoría interna y actúa como parámetro para evaluar la función de auditoría. d. Incorrecta. La alta gerencia y el consejo son quienes deben aprobar el estatuto de auditoría

interna. 2. Solución: a

a. Correcta. La programación a largo plazo provee evidencia de la cobertura de funciones clave en intervalos planeados.

b. Incorrecta. El programa de trabajos es limitado en alcance para un proyecto particular. c. Incorrecta. El presupuesto de la actividad de auditoría interna puede utilizarse para justificar la

cantidad de personal de auditoría, pero no se usa para asegurar una cobertura adecuada de auditoría en el tiempo.

d. Incorrecta: El estatuto de la actividad de auditoría interna no es una herramienta de planificación de trabajos.

3. Solución: d

a. Incorrecta. Las Normas no requieren que los auditores internos aseguren el cumplimiento de los procedimientos de información.

b. Incorrecta. No se espera que se haga un cotejo del flujo de fondos con los conceptos de gastos en un período determinado.

c. Incorrecta. Sería una función del personal y/o de los departamentos de finanzas. d. Correcta. Los auditores internos son responsables de identificar controles inadecuados para

evaluar la eficacia gerencial y señalar riesgos comunes. 4. Solución: d

a. Incorrecta. El director ejecutivo de auditoría de la compañía es responsable de las promociones del personal.

b. Incorrecta. El director ejecutivo de auditoría de la compañía es responsable de aprobar los informes de auditoría interna.

c. Incorrecta. Es parte de la función de planificación de la actividad de auditoría interna. d. Correcta. La independencia de la actividad de auditoría interna se intensifica cuando el comité

de auditoría participa en el nombramiento del director ejecutivo de auditoría. 5. Solución: d

a. Incorrecta. La integración y la supervisión se relacionan con la capacidad profesional del departamento de auditoría interna.

b. Incorrecta. El desarrollo profesional continuo y el debido cuidado profesional se relacionan con la capacidad profesional del auditor interno.

c. Incorrecta. Las relaciones humanas y las comunicaciones se relacionan con la capacidad profesional del auditor interno.

d. Correcta. De acuerdo con el Consejo para la práctica 1100-1-1, el nivel organizacional y la objetividad permiten a los miembros de la actividad de auditoría interna emitir juicios imparciales y objetivos, que resultan esenciales para la conducta apropiada de las auditorías.

I - 41

6. Solución: a a. Correcta. Un auditor que será promovido a un departamento operativo no debe continuar con

una auditoría de ese departamento. De acuerdo con el Consejo para la Práctica 1130-1.1, el director ejecutivo de auditoría debe reasignar a los auditores si existe alguna razón para presentir un conflicto de intereses o una parcialidad.

b. Incorrecta. Las restricciones presupuestarias no constituyen una violación a la independencia del auditor.

c. Incorrecta. El Consejo para la Práctica 1130.A1-1.4 expresa que un auditor puede recomendar normas de control para nuevos sistemas. Sin embargo, el diseño, la instalación u operación de dichos sistemas podrían perjudicar la objetividad.

d. Incorrecta. Un auditor puede revisar los contratos antes de que se formalice. 7. Solución: c

a. Incorrecta. La gerencia de auditoría debe estar siempre informada sobre cualquiera de esos ofrecimientos.

b. Incorrecta. La gerencia de auditoría debe estar siempre informada sobre cualquiera de esos ofrecimientos.

c. Correcta. Debe consultarse a la gerencia de auditoría para obtener pautas. d. Incorrecta. Podría desgastar la relación de la función de auditoría interna con la división en

cuestión. Debe informarse y consultarse a la gerencia de auditoría en primer lugar para obtener pautas.

8. Solución: b

a. Incorrecta. El Consejo para la Práctica 1130.A1-1.4 expresa que la objetividad del auditor interno no se ve adversamente afectada cuando el auditor revisa los procedimientos antes de implementarlos.

b. Correcta. El Consejo para la Práctica 1130.A1-1.3 expresa que no debe asignarse a las personas transferidas a la actividad de auditoría interna para auditar aquellas actividades que realizaron anteriormente, hasta que haya transcurrido un período razonable (al menos un año).

c. Incorrecta. El Consejo para la Práctica 1130.A1-1.4 expresa que la objetividad del auditor interno no se ve adversamente afectada cuando el auditor recomienda las normas de control para los sistemas antes de implementarlos.

d. Incorrecta. El uso de personal de otras áreas para ayudar al auditor interno no perjudica la objetividad, en especial cuando el personal no pertenece al área que se está auditando.

9. Solución: b

a. Incorrecta. Podría presumirse con facilidad que la aceptación del obsequio afectó la independencia y, por lo tanto, no sería aceptable.

b. Correcta. Mientras un individuo sea un Auditor Interno Certificado, él o ella deben guiarse por el Código de Ética de la profesión además del código de conducta de la organización. La Regla de Conducta 2.2 del Código de Ética del IAI no permitiría un obsequio de esa índole, ya que se presumiría que influyó en la decisión del individuo.

c. Incorrecta. Ver respuesta “b”. d. Incorrecta. Ver respuesta “b”. Más aún, no se da información suficiente para juzgar posibles

violaciones del código de conducta de la organización. Sin embargo, la acción podría fácilmente percibirse como una acción corrupta.

I - 42

10. Solución: d a. Incorrecta. Aunque el auditor evitase tener contacto con el controller, podría parecer que existe

un conflicto de intereses. b. Incorrecta. Las situaciones de posible conflicto de intereses o parcialidad deben evitarse, no

sólo revelarse. c. Incorrecta. Los conflictos de intereses deben informarse al director ejecutivo de auditoría, no al

proveedor ni al cliente del trabajo. d. Correcta. El Consejo para la Práctica 1130-1.1 de las Normas indica que los auditores internos

deben informar al Director Ejecutivo de Auditoría cualquier situación en la cual se presente o pueda suponerse que existe un conflicto de intereses o una parcialidad.

11. Solución: d (I, II, y IV solamente)

I, II, IV. Correcta. Se espera que los auditores internos puedan reconocer buenas prácticas de negocios, comprender las relaciones humanas y estar capacitados en comunicaciones orales y escritas.

III. Incorrecta. No se espera que los auditores internos sean expertos en una amplia variedad de campos relacionados con sus responsabilidades de auditoría.

12. Solución: c

a. Incorrecta. El director ejecutivo de auditoría (DEA) no debe comenzar la auditoría sin notificar al comité de auditoría sobre la cuestión del conocimiento y tratar de resolverla.

b. Incorrecta. No proveería al comité de auditoría una revisión independiente de la gerencia ni de los procesos de PASS.

c. Correcta. Cuando un DEA reconoce que la actividad de auditoría interna no posee el conocimiento ni las destrezas necesarias para un trabajo planificado o requerido, debe solicitar al comité de auditoría que apruebe el uso de los recursos independientes apropiados, de acuerdo con el Consejo para la Práctica 1210.A1-1.1.

d. Incorrecta. Esta demora puede tener serias consecuencias por la naturaleza de las cuestiones involucradas de PASS.

13. Solución: c

a. Incorrecta. La inclusión automática de información financiera en una auditoría no garantiza que el debido cuidado profesional se haya logrado para la auditoría como un todo.

b. Incorrecta. El mantener papeles de trabajo detallados no asegura que se haya tenido el debido cuidado profesional durante las pruebas.

c. Correcta. El considerar la posibilidad de incumplimientos o irregularidades materiales en todo momento durante un trabajo es la única manera de demostrar que se haya tenido el debido cuidado profesional en un trabajo de auditoría interna, de acuerdo con el Consejo para la Práctica 1220-1.2.

d. Incorrecta. El debido cuidado profesional no requiere que se informen todos los casos de incumplimiento o irregularidad al comité de auditoría.

14. Solución: d

a. Incorrecta. La Norma 1220.A1 expresa que el auditor debe considerar el alcance del trabajo necesario para alcanzar los objetivos del trabajo. Es un objetivo específico del trabajo.

b. Incorrecta. La Norma 1220.A1 expresa que auditor debe considerar la adecuación y la eficacia de los procesos de administración de riesgo.

c. Incorrecta. La Norma 1220.A1 expresa que el auditor debe considerar la significatividad y materialidad de las cuestiones a las cuales se aplican los procedimientos de aseguramiento. Es un incremento significativo.

d. Correcta. Es la responsabilidad de los auditores externos y no debe cambiar lo que debe considerar el auditor interno.

I - 43

15. Solución: b (I y II solamente) I, II. Correcta. El director ejecutivo de auditoría (DEA) cambió drásticamente la naturaleza de la

función de auditoría sin consultar al comité de auditoría ni modificar el estatuto de auditoría interna. La Norma sobre Atributos 1000 indica que el propósito, la autoridad y la responsabilidad de la actividad de auditoría interna deben estar formalmente definidos en un estatuto, de conformidad con las Normas, y aprobados por el Consejo. La Norma sobre Desempeño 2400 requiere que los auditores internos comuniquen los resultados del trabajo. La Norma sobre Desempeño 2420 indica que las comunicaciones deben ser precisas, objetivas, claras, concisas, constructivas, completas y oportunas. El Consejo para la Práctica 2420-1 indica que las comunicaciones completas no carecen de nada esencial para el auditorio meta e incluye toda la información y las observaciones significativas y pertinentes para respaldar las recomendaciones y las conclusiones.

III. Incorrecta. Poner énfasis en las posibles reducciones de costos resulta apropiado para la comunicación final de los trabajos.

16. Solución: c

a. Incorrecta. La tercerización sería una respuesta apropiada cuando los auditores no poseen los antecedentes o destrezas necesarios ni pueden desarrollar dichas destrezas de manera oportuna.

b. Incorrecta. Agregar un consultor sería una respuesta apropiada cuando los auditores no poseen los antecedentes o destrezas necesarios ni pueden desarrollar dichas destrezas de manera oportuna.

c. Correcta. La planificación y ejecución del trabajo de auditoría sin los antecedentes y destrezas apropiados violarían la Norma sobre Atributos 1210. La Norma sobre Atributos 1210 requiere que el departamento de auditoría interna provea aseguramiento de que la capacidad técnica y los antecedentes educativos de los auditores internos resulten apropiados para las auditorías que deban realizarse. Los auditores no tienen dicha pericia.

d. Incorrecta. Determinar si se dispone de tiempo y habilidad suficiente para desarrollar dichas destrezas sería una respuesta apropiada. Los auditores internos deben comprometerse a un aprendizaje de por vida y, por lo tanto, no sería irracional hacer que expandan sus conocimientos y destrezas.

17. Solución: c

a. Incorrecta. El auditor no está reteniendo información porque la entregó al director ejecutivo de auditoría. La información puede resultar útil en un trabajo subsiguiente del área de Marketing.

b. Incorrecta. El auditor documentó una bandera roja que puede ser importante en un trabajo subsiguiente, lo cual no viola las Normas.

c. Correcta. No hay violación del Código de Ética ni de las Normas. Ver respuestas “a” y “b.” d. Incorrecta. Ver respuestas “a” y “b.”

18. Solución: a

a. Correcta. Sin objetivos, no hay dirección para lograr la estrategia. b. Incorrecta. Sin una fijación de objetivos, no puede resumirse el contenido. c. Incorrecta. La disposición de quienes aprenden debe considerarse después de determinar los

objetivos. d. Incorrecta. Las restricciones del presupuesto deben considerarse más adelante durante el

proceso. 19. Solución: c (I, III y IV solamente)

I. Correcta. Los programas de aseguramiento de calidad están diseñados para proveer retroalimentación en cuanto a la eficacia de una función de auditoría interna. El programa de aseguramiento de calidad debe incluir la supervisión, que provee una retroalimentación cotidiana.

II. Incorrecta. La capacitación apropiada es importante, pero no provee retroalimentación. III. Correcta. Un programa de aseguramiento de calidad debe incluir evaluaciones internas. IV. Correcta. Un programa de aseguramiento de calidad debe incluir evaluaciones externas.

I - 44

20. Solución: b a. Incorrecta. La decisión de tercerizar la función de auditoría interna no se basa primordialmente

en los recursos existentes. b. Correcta. La Norma 2030 requiere que los recursos sean adecuados y suficientes. c. Incorrecta. La cantidad de recursos no constituye un factor significativo para establecer

credibilidad. d. Incorrecta. La planificación en serie no se relaciona con la cantidad de recursos de auditoría.

21. Solución: d

a. Incorrecta. Esta declaración se relaciona con la responsabilidad del director ejecutivo de auditoría (DEA) de coordinar con los auditores externos (Norma de Desempeño 2050 y Consejo para la Práctica relacionado 2050-1).

b. Incorrecta. La responsabilidad de un DEA de buscar la aprobación de un estatuto que establezca la autoridad, el propósito y la responsabilidad (Norma sobre Atributos 1000 y Consejo para la Práctica relacionado 1000-1) no forma parte de un programa de aseguramiento de calidad.

c. Incorrecta. Las evaluaciones de desempeño individuales forman parte de la responsabilidad de un DEA hacia la administración y el desarrollo del personal (Norma de Desempeño 2030 y Consejo para la Práctica relacionado 2030-1).

d. Correcta. La supervisión es un método de revisión continua, que forma parte del aspecto de evaluaciones internas del aseguramiento de calidad (Consejo para la Práctica 1311-1.1).

22. Solución: c (I y IV solamente)

I, IV. Correcta. Es una mejor práctica para que la evaluación de riesgos sea un proceso dinámico, que cambie con el tiempo y a medida que se identifiquen los nuevos riesgos, estrategias del negocio e información. La consulta continua con los miembros de la gerencia y del comité de auditoría es una manera de que la actividad de auditoría interna obtenga dicha información y armonice con los desarrollos organizacionales que puedan impactar en las prioridades de auditoría existentes. Para acomodar dichas prioridades emergentes, es probable que deba alterarse el programa del trabajo.

II. Incorrecta. Es probable que los programas de auditoría cambien regularmente para satisfacer las necesidades de la organización, en particular si se basan en un proceso eficaz de evaluación de riesgos.

III. Incorrecta. La determinación del riesgo es un ejercicio (juicio) tanto cuantitativo como cualitativo.

23. Solución: c

a. Incorrecta. Los requerimientos de auditoría externa de ayudar con el trabajo de campo deben subordinarse a las investigaciones de fraude.

b. Incorrecta. Como el nuevo sistema todavía se está gestando, esto puede esperar. c. Correcta. El requerimiento de la gerencia de investigar un posible fraude en la unidad de

cuentas por cobrar debe citar un precedente para las otras entidades. d. Incorrecta. Un requerimiento de la gerencia que implique un fraude debe tener prioridad frente

a un sistema que no se auditó el año pasado. 24. Solución: c

a. Incorrecta. El riesgo D citaría un precedente frente al riesgo A, ya que tiene una mayor probabilidad de ocurrir a pesar de que el impacto es más bajo.

b. Incorrecta. Es lo opuesto del orden correcto. c. Correcta. Este orden clasifica el riesgo mediante una combinación de probabilidad e impacto. d. Incorrecta. El riesgo D debe clasificarse más alto que el riesgo C, debido a la probabilidad y al

impacto.

I - 45

25. Solución: c a. Incorrecta. No es la mejor técnica, ya que sólo adopta el enfoque de dos niveles hacia la

administración de riesgo (es decir, evento e impacto). b. Incorrecta. No es la mejor técnica, ya que no adopta un enfoque amplio hacia la administración

de riesgo. c. Correcta. Es la mejor respuesta, ya que adopta un enfoque amplio hacia la administración de

riesgo; no sólo considera el evento y el impacto, sino también las causas. d. Incorrecta. Una vez más, esta opción adopta un enfoque de dos niveles y también explica la

eliminación de riesgos en lugar de su mitigación. 26. Solución: a

a. Correcta. Considerar el plan estratégico en el desarrollo del plan de auditoría interna asegurará que los objetivos de auditoría respalden los objetivos globales del negocio expresados en el plan estratégico.

b. Incorrecta. Esta acción puede hacer que el plan de auditoría interna se adecue mejor al plan estratégico, pero es probable que no provoque un efecto en la aprobación de la gerencia.

c. Incorrecta. Si bien el director ejecutivo de auditoría (DEA) puede hacer recomendaciones para mejorar el plan estratégico, no es el propósito primordial del DEA revisar el plan.

d. Incorrecta. Si bien la importancia de la función de auditoría interna puede incrementarse con dicha acción, no es la razón primordial de la acción.

27. Solución: c

a. Incorrecta. Rara vez tendría un impacto de gran rango. b. Incorrecta. Rara vez sería una preocupación de gran rango. c. Correcta. Sería un tema de planificación de gran rango porque afecta al posicionamiento del

mercado. d. Incorrecta. Sin duda, es una preocupación, pero tiene un impacto de menor rango que la

calidad del producto. 28. Solución: b

a. Incorrecta. Determinar cómo administrar un riesgo inaceptable es el rol de la gerencia. b. Correcta. Los servicios de aseguramiento implican la evaluación por parte del auditor interno de

los objetivos de las actividades de administración de riesgo de la gerencia y la medida en que resulten eficaces.

c. Incorrecta. El diseño y la actualización del proceso de administración de riesgo es el rol de la gerencia.

d. Incorrecta. El diseño de controles perjudicaría la independencia del auditor interno. 29. Solución: c

a. Incorrecta. El puntaje total es menor que el de los trabajos 2 y 4. b. Incorrecta. El puntaje total es menor que el de las otras opciones. c. Correcta. Los trabajos 2 y 4 tienen el puntaje global más alto. d. Incorrecta. Realizar los trabajos 3 y 4 implicaría saltear el trabajo 2, clasificado con el puntaje

global más alto junto con el trabajo 4. 30. Solución: d

a. Incorrecta. Esta opción implica el puntaje total más bajo. b. Incorrecta. El puntaje total es menor que el de los trabajos 3 y 4. c. Incorrecta. El puntaje total es menor que el de los trabajos 3 y 4. d. Correcta. Tiene el puntaje total más alto, y los trabajos tienen entre una mediana y alta

posibilidad de reducir costos.

I - 46

31. Solución: a a. Correcta. La revisión y las pruebas de los procedimientos del otro departamento pueden reducir

la cobertura de auditoría necesaria de la función o del proceso. b. Incorrecta. El concentrarse en la función o el proceso puede llevar a la duplicación de

esfuerzos. c. Incorrecta. El auditor interno no puede confiar en el trabajo de otros sin verificar los resultados. d. Incorrecta. La responsabilidad global de la actividad de auditoría interna para evaluar la función

o el proceso no se ve afectada por la cobertura del otro departamento. 32. Solución: b

a. Incorrecta. La responsabilidad para asegurar que las responsabilidades profesionales y organizacionales de la actividad de auditoría interna maximicen los beneficios que puedan lograrse por la coordinación con otras actividades de aseguramiento y consultoría es del director ejecutivo de auditoría, de acuerdo con el Consejo para la Práctica 2050-1.3. Los comentarios sobre este tema deben siempre formar parte de cualquier informe de la actividad que el director ejecutivo de auditoría dirija al comité de auditoría.

b. Correcta. El Consejo para la Práctica 2060-2.5 recomienda que el director ejecutivo de auditoría provea al comité de auditoría información sobre la coordinación y la supervisión de otras funciones de control y monitoreo.

c. Incorrecta. El CEO no sería normalmente responsable de la planificación, del trabajo ni de la coordinación relacionados con los trabajos de aseguramiento y consultoría de auditoría interna.

d. Incorrecta. No todas las otras actividades de aseguramiento y consultoría son organizacionalmente responsables ante el comité de auditoría por su trabajo, y es probable que no tengan la oportunidad de informar directamente al comité de auditoría.

33. Solución: d

a. Incorrecta. Los auditores internos no deben intentar influir en las interpretaciones de la ley por parte de los reguladores.

b. Incorrecta. Los auditores internos no deben intentar influir en el alcance del trabajo de los examinadores reguladores. No resultaría ético y sería una violación del Código de Ética del IAI.

c. Incorrecta. Los auditores internos no deben realizar trabajo de campo para los examinadores reguladores.

d. Correcta. Los auditores deben tener un acceso inmediato a los papeles de trabajo e informes, que pueden suministrar evidencia de las pruebas de cumplimiento a los examinadores reguladores.

34. Solución: a

a. Correcta. Es un motivo fundamental apropiado. b. Incorrecta. Un listado de esta índole podría convencer al director ejecutivo de auditoría acerca

de la necesidad de evaluación de riesgos, pero no lo provee el proceso. c. Incorrecta. Lo utiliza el proceso de evaluación de riesgos, pero no es un motivo fundamental

para utilizar la evaluación de riesgos. d. Incorrecta. Es una definición de riesgo.

35. Solución: b

a. Incorrecta. Es un factor importante, de acuerdo con el Consejo para la Práctica 2010-1.4. b. Correcta. Si bien deben considerarse las destrezas del auditor en el proceso de planificación,

las necesidades de auditoría – no la disponibilidad de las destrezas del auditor – deben conducir los programas de trabajo en un plan de auditoría basado en el riesgo.

c. Incorrecta. Es un factor importante, de acuerdo con el Consejo para la Práctica 2010-1.4. d. Incorrecta. Es un factor importante, de acuerdo con el Consejo para la Práctica 2010-1.4.

I - 47

36. Solución: c a. Incorrecta. Se consideraría este factor al fijar prioridades en los trabajos. b. Incorrecta. Al revisar la dotación del personal, la fijación de prioridades de los trabajos y de los

gastos, pueden obtenerse beneficios operativos. c. Correcta. El Consejo para la Práctica 2010-1.2 expresa que las metas de la actividad de

auditoría interna, de acuerdo con los planes operativos y presupuestos especificados, deben ir acompañadas de criterios de medición y fechas programadas de finalización.

d. Incorrecta. La dotación de personal para cada trabajo debe incluir esta consideración. 37. Solución: c

a. Incorrecta. Puede llevar a la duplicación en la cobertura de auditoría. b. Incorrecta. La auditoría interna abarca objetivos y actividades tanto financieros como

operativos. En consecuencia, la cobertura de auditoría interna podría proveerse mediante el trabajo de auditoría externa que incluía primordialmente actividades y objetivos financieros.

c. Correcta. La coordinación del trabajo de auditoría interna y externa ayuda a evitar la duplicación en la cobertura, mejorando así la eficiencia de la auditoría interna.

d. Incorrecta. El trabajo de auditoría externa se conduce de acuerdo con las Normas de Auditoría Generalmente Aceptadas.

38. Solución: c

a. Incorrecta. Los cuestionarios de auto-evaluación constituyen un medio de abordar los objetivos de ciertas auditorías internas de manera eficiente.

b. Incorrecta. El uso de la tecnología es un medio apropiado de lograr eficiencia en la ejecución de la auditoría.

c. Correcta. El programa de auditoría debe sólo reducirse como último recurso una vez exploradas todas las otras alternativas viables, entre las cuales se incluye el requerimiento de recursos adicionales.

d. Incorrecta. El uso de personal operativo con el interés de auditoría interna y la experiencia corporativa es una manera apropiada de mejorar los recursos de auditoría interna.

39. Solución: a

a. Correcta. La alta gerencia y el consejo directivo deben estar informados sobre las implicancias de las brechas en la cobertura de auditoría, entre las cuales se incluye la revisión del cumplimiento con leyes y reglamentaciones aplicables.

b. Incorrecta. No sólo la actividad de auditoría interna debe saber acerca de la cobertura incompleta de la auditoría.

c. Incorrecta. Es probable que resulte necesario revisar el cumplimiento de las reglamentaciones materiales una vez por año.

d. Incorrecta. La cobertura de auditoría en otras áreas no debe reducirse en forma automática. Es probable que la actividad de auditoría interna requiera recursos adicionales para proveer una cobertura adecuada de riesgos.

40. Solución: c (III solamente)

I. Incorrecta. El departamento de auditoría interna debe considerar tanto los requerimientos de la administración como los del comité de auditoría. Aunque los requerimientos del comité de auditoría son importantes, no son siempre los más importantes ni tampoco implican un mayor riesgo.

II. Incorrecta. El riesgo se mide por la posible exposición de la organización. El tamaño del presupuesto del departamento es un factor determinante para considerar, pero no es suficiente.

III. Correcta. El Consejo para la Práctica 2010-2 expresa que el grado o la materialidad de exposición es un componente importante del riesgo.

I - 48

41. Solución: b a. Incorrecta. No crea conflicto con la independencia de la actividad de auditoría interna. b. Correcta. En algunas circunstancias, tales como un equipo de desarrollo de productos, el rol

del líder o miembro del equipo puede crear conflicto con el atributo de independencia de la actividad de auditoría interna. El auditor puede participar como consultor, pero no debe participar como líder del equipo.

c. Incorrecta. Para mejorar el clima ético, el auditor interno debe asumir el rol de defensor de la ética que, en consecuencia, no crea conflicto con la independencia de la actividad de auditoría interna.

d. Incorrecta. No crea conflicto con la independencia de la actividad de auditoría interna, ya que las funciones de auditoría interna y externa comparten información y trabajan colaborando fuera de la influencia de la gerencia.

42. Solución: d (I, II, III y IV)

I. Correcta. El evaluar si se preservan la ética y los valores contribuiría al gobierno corporativo, de acuerdo con la Norma 2130.

II. Correcta. El evaluar la eficacia de la administración y responsabilidad organizacionales contribuirían al gobierno corporativo, de acuerdo con la Norma 2130.

III. Correcta. El evaluar cómo se comunica la información sobre riesgos y controles contribuiría al gobierno corporativo, de acuerdo con la Norma 2130.

IV. Correcta. El evaluar la coordinación de los auditores externos e internos y la gerencia contribuiría al gobierno corporativo, de acuerdo con la Norma 2130.

43. Solución: a

a. Correcta. En un sistema bien desarrollado por la gerencia, la función de auditoría interna se utiliza para proveer un beneficio más directo para alinear operaciones proveyendo respuestas a la gerencia operativa así como a la alta gerencia.

b. Incorrecta. Debe ponerse énfasis en las auditorías de los productos y sistemas propuestos. Estos primeros exámenes podrían utilizarse para determinar la posibilidad y/o la conveniencia de realizar cambios antes de implementarlos.

c. Incorrecta. El rol del auditor interno implica establecer interfases con la gerencia al nivel operativo así como al nivel superior.

d. Incorrecta. La administración de activos no constituiría un enfoque primordial de la actividad de auditoría interna.

44. Solución: c

a. Incorrecta. Es un propósito de la planificación de auditoría. b. Incorrecta. La corrección de las debilidades de control es una función de la gerencia, no del

auditor interno. c. Correcta. Es el propósito expresado en el Consejo para la Práctica 2100-1.1. d. Incorrecta. Es un objetivo básico desde una perspectiva contable y financiera y de auditoría,

pero no resulta lo suficientemente amplia para cubrir todo el propósito de revisión del auditor interno.

45. Solución: a

a. Correcta. La gerencia operativa es responsable de la administración de riesgo; la gerencia ejecutiva, de la supervisión y los auditores internos se ocupan de los roles de supervisión y consultoría.

b. Incorrecta. La gerencia operativa se desempeña en el rol de implementación de la administración de riesgo.

c. Incorrecta. En general, los auditores internos se desempeñan en el rol de aseguramiento y consultoría.

d. Incorrecta. La gerencia operativa no se desempeña en el rol de supervisión.

I - 49

46. Solución: c a. Incorrecta. Es una responsabilidad del director ejecutivo de auditoría (DEA), de acuerdo con la

Norma 2020. b. Incorrecta. Es una responsabilidad del DEA, de acuerdo con la Norma 2050. c. Correcta. El Consejo para la Práctica 2120.A1-1 expresa que es el rol de la alta gerencia, no

del DEA. d. Incorrecta. Es una responsabilidad del DEA, de acuerdo con la Norma 2500.

47. Solución: a

a. Correcta. Está respaldado por la Norma de Implementación 2120.A4. b. Incorrecta. En casos en que la gerencia no haya establecido los criterios o si, en la opinión del

auditor, los criterios establecidos se estiman poco adecuados, el auditor debe trabajar con la gerencia para desarrollar criterios de evaluación apropiados.

c. Incorrecta. Son fuentes de información que ayudarán a la gerencia a establecer metas y criterios objetivos, pertinentes y significativos.

d. Incorrecta. Las normas contables o de auditoría no resultaríanapropiadas para este propósito. 48. Solución: b

a. Incorrecta. La actividad de auditoría se desempeña en este rol. El consejo y la gerencia son responsables de la identificación de un modelo y metodología de riesgo apropiados.

b. Correcta. Es el rol de la gerencia asegurar la implementación oportuna de las recomendaciones de auditoría. La actividad de auditoría interna es responsable del desarrollo de un procedimiento oportuno para vigilar la disposición de las recomendaciones de auditoría. La actividad de auditoría interna trabaja con la alta gerencia y el comité de auditoría para asegurar que las recomendaciones de auditoría reciban la atención apropiada.

c. Incorrecta. La actividad de auditoría interna debe vigilar el cumplimiento del código de conducta corporativo establecido por el consejo y la gerencia.

d. Incorrecta. La actividad de auditoría interna es responsable de hablar sobre los riesgos y exposiciones significativos financieros, técnicos y operativos y sobre los planes para minimizar dichos riesgos.

49. Solución: d

a. Incorrecta. Esta evaluación debe llevarse a cabo por lo menos una vez por año. b. Incorrecta. El consejo puede solicitar al director ejecutivo de auditoría (DEA) que participe en la

evaluación del desempeño de los auditores externos, lo cual puede incluir la evaluación de la independencia.

c. Incorrecta. Ver respuestas “a” y “b”. d. Correcta. Ver respuestas “a” y “b”.

50. Solución: c

a. Incorrecta. El CEO es un miembro de la alta gerencia. Otros miembros de la alta gerencia pueden recibir un informe final ya revisado y aprobado por el consejero legal.

b. Incorrecta. No debe entrarse en contacto con los auditores externos. Puede dárseles un informe final a los auditores externos ya revisado y aprobado por el consejero legal.

c. Correcta. Debe presentarse un borrador del informe propuesto sobre fraude o situaciones de conflicto de intereses al presidente del comité de auditoría como próximo paso en vista de la posición del CEO en la compañía.

d. Incorrecta. Sería necesario obtener documentación de respaldo antes de informar al comité de auditoría o al consejo.

I - 50

51. Solución: a (II solamente) I. Incorrecta. La existencia de un código de ética corporativo, en sí mismo, no asegura altos

niveles de conducta ética. Debe estar complementado por políticas de seguimiento y actividades de monitoreo para asegurar el cumplimiento del código.

II. Correcta. Un código de ética corporativo formalizado presenta criterios objetivos mediante los cuales pueden evaluarse las acciones y, de esta manera, serviría como conjunto de criterios mediante los cuales puedan evaluarse las actividades.

III. Incorrecta. Las normas que influirían en las acciones individuales pueden ser otras distintas de las que contiene el código de ética corporativo. Por ejemplo, puede haber políticas definidas con respecto a las actividades de compra que pueden tener el mismo propósito que un código de ética. Dichas políticas también pueden servir como criterios mediante los cuales puedan evaluarse las actividades.

52. Solución: c

a. Incorrecta. Ver respuesta “c”. b. Incorrecta. Ver respuesta “c”. c. Correcta. Un director ejecutivo de auditoría debe establecer un proceso de seguimiento para

vigilar la adecuación, eficacia y oportunidad de las acciones tomadas por la gerencia acerca de las observaciones y recomendaciones informadas sobre el trabajo, entre las cuales se incluyen aquellas hechas por los auditores externos y otros.

d. Incorrecta. Ver respuesta “c”. 53. Solución: a

a. Correcta. Las organizaciones tales como los agentes de bolsa, los bancos y las compañías de seguros pueden considerar los riesgos lo suficientemente críticos como para garantizar vigilancia y monitoreo continuos.

b. Incorrecta. Una función de cumplimiento separada puede tener recomendaciones para ayudar a fortalecer los controles, pero no constituye su propósito primordial.

c. Incorrecta. La gerencia no es independiente, ya que la administración de riesgo es su responsabilidad directa.

d. Incorrecta. Ayudará a responder ante las necesidades de los accionistas, pero no es la razón primordial para establecer la función de cumplimiento.

54. Solución: d

a. Incorrecta. La alta gerencia tiene un rol de supervisión en la administración de riesgo. b. Incorrecta. El conocimiento del riesgo al nivel de línea sería específico sólo para esa área de la

organización. c. Incorrecta. El director ejecutivo de auditoría (DEA) no tiene la responsabilidad de administrar

riesgos. d. Correcta. El director de riesgos es más eficiente cuando lo respalda un equipo específico con la

pericia y experiencia necesarias relacionadas con el riesgo organizacional. 55. Solución: a

a. Correcta. Los tipos y cantidades de seguros deben tener el respaldo de las evaluaciones periódicas.

b. Incorrecta. La determinación de la cobertura de seguros no es una función del consejo directivo.

c. Incorrecta. En general, el índice de precios al consumidor no provee un factor de ajuste apropiado para los activos fijos.

d. Incorrecta. Los valores contables pueden no reflejar el reemplazo o el valor real de un activo.

I - 51

56. Solución: a (I solamente) I. Correcta. Los sistemas de remuneración influyen en la conducta y deben considerarse una

parte integral de la estructura de control de la organización. Así, debe considerarse una parte importante de la estructura de control sobre el intercambio comercial de derivados.

II. Incorrecta. Los sistemas de remuneración forman parte de los sistemas de control de la organización.

III. Incorrecta. Las auditorías de los sistemas de remuneración pueden combinarse con una auditoría sobre otras funciones que impacten en los dividendos de la corporación.

57. Solución: d

a. Incorrecta. Los mecanismos corporativos de control sí incluyen mecanismos internos y externos.

b. Incorrecta. El esquema de remuneración de la gerencia es parte de los mecanismos corporativos de control.

c. Incorrecta. La dilución de la riqueza de los accionistas provocada por las opciones de compra de acciones por parte de los empleados o de los dividendos en acciones de los empleados es una cuestión contable más que una cuestión de gobierno corporativo.

d. Correcta. El consejo es, en definitiva, el responsable del gobierno corporativo de la compañía, no los auditores internos.

58. Solución: b

a. Incorrecta. Los seguros constituyen una actividad de administración de riesgo. b. Correcta. Las operaciones de protección cambiaria constituyen el uso de futuros contratos para

limitar la exposición al riesgo sobre tipos de cambio. c. Incorrecta. Las ventas al descubierto se refieren a las ventas de productos básicos o de

acciones. d. Incorrecta. El factoraje se aplica a los descuentos de las cuentas por cobrar.

59. Solución: c

a. Incorrecta. Los procesos de administración de riesgo no pueden garantizar que se alcancen los objetivos en su totalidad.

b. Incorrecta. La participación de los auditores internos en establecer actividades de control perjudicaría su independencia y objetividad.

c. Correcta. Esta opción se incluye en el marco de la administración de riesgo. d. Incorrecta. La administración de riesgo empresarial no concierne a la selección de la mejor

respuesta de riesgo, sino a la selección de la respuesta de riesgo que se incluye en el apetito de riesgo de la empresa. Además, no es el comité de auditoría el que la selecciona, sino el equipo de la gerencia.

60. Solución: c

a. Incorrecta. El impacto del riesgo es su consecuencia. b. Incorrecta. El riesgo que se encuentra bajo control es el riesgo que se administra. c. Correcta. El riesgo residual es el riesgo que queda después de haber aplicado todos los

controles y las técnicas de administración de riesgo. d. Incorrecta. El riesgo subyacente es el riesgo absoluto.

61. Solución: d

a. Incorrecta. Los productos son estacionales y el espacio de almacenamiento es limitado. Es una restricción coherente con maximizar los ingresos y las utilidades de la organización.

b. Incorrecta. Se evalúa al gerente de producto sobre la base de las ventas y ganancias brutas; de esta manera, no se crea conflicto con la realización de ambas obligaciones.

c. Incorrecta. La evaluación de los gerentes de producto por las ganancias brutas y las ventas presupuestadas atribuye la responsabilidad al gerente.

d. Correcta. Existe la posibilidad de que los productos puedan desviarse del centro de distribución y no se entreguen al local minorista apropiado.

I - 52

62. Solución: a (I solamente) I. Correcta. La organización tiene dos recursos escasos para asignar: (a) su presupuesto de

compras (restringido por la habilidad financiera) y (b) el espacio disponible en los locales minoristas. De esta manera, existe la necesidad de un mecanismo para asignar estos dos recursos escasos a fin de maximizar el rendimiento global de la organización. Es el mecanismo apropiado.

II. Incorrecta. Es un control preventivo, no detectivo. III. Incorrecta. La evaluación de las ganancias brutas resulta eficaz para evaluar al gerente, pero

no aborda las dos restricciones principales identificadas en la declaración I. 63. Solución: c

a. Incorrecta. La gerencia puede cambiar a los miembros de la organización. b. Incorrecta. La gerencia puede cambiar la estructura de la organización. c. Correcta. El ambiente suele determinarse por fuerzas externas, fuera del control directo de la

organización. d. Incorrecta. La gerencia puede cambiar la tecnología de la organización.

64. Solución: d

a. Incorrecta. La eliminación de cheques no representa un control continuo. b. Incorrecta. La eliminación de cheques evita el riesgo asociado en lugar de aceptarlo. c. Incorrecta. El riesgo no se transfiere a ninguna otra persona; se elimina. d. Correcta. Al eliminar los cheques, la organización evita todos los riesgos asociados con ellos.

65. Solución: d

a. Incorrecta. No es un enfoque amplio hacia la administración de riesgo. b. Incorrecta. Ver respuesta “a”. c. Incorrecta. Ver respuesta “a”. d. Correcta. Es un enfoque amplio y se relacionará con las estrategias de administración de

riesgo en toda la empresa. 66. Solución: b

a. Incorrecta. La pérdida real o imaginaria de empleos es una razón común por la cual los empleados se resisten a cualquier cambio.

b. Correcta. Los programas de capacitación para los empleados facilitan la realización de los trabajos de una manera nueva o diferente.

c. Incorrecta. Los miembros de los grupos de trabajo suelen ejercerse presión de pares entre sí para resistirse a los cambios; en especial, si cambian las relaciones sociales.

d. Incorrecta. La falta de comunicación y el no hablar sobre la necesidad de cambiar amenaza el statu quo.

67. Solución: c

a. Incorrecta. Cada uno de estos controles individuales y, probablemente otros también, ayuden a la gerencia a alcanzar su objetivo de evitar la liberación de consumos de agua no reciclables que no cumplan con los límites permitidos o con otras condiciones. Cada uno de estos tres controles abordan el riesgo de diferentes maneras. Los resultados analíticos son los criterios para la decisión de descargar; el mantenimiento de contaminantes fuera del agua residual ayudará a reducir las concentraciones y el grado de pretratamiento necesario; además, es menos probable que falle el equipamiento si existe un programa de mantenimiento preventivo.

b. Incorrecta. Ver respuesta “a”. c. Correcta. Es probable que la dilución periódica no siempre evite los contaminantes que

excedan los límites de descarga. d. Incorrecta. Ver respuesta “a”.

I - 53

68. Solución: a a. Correcta. Es un cambio cultural porque implica un cambio de actitud y mentalidad. b. Incorrecta. El cambio de productos implica un cambio en los atributos físicos y la utilidad de un

producto para sus clientes. c. Incorrecta. Aquí no existe ningún cambio en los sistemas ni las estructuras. d. Incorrecta. No implica un cambio organizacional, ya que sólo se encuentra involucrado el

aseguramiento de calidad. 69. Solución: c

a. Incorrecta. Desarrollar el plan y luego presentarlo al personal de auditoría no ayudaría a reducir su resistencia al cambio.

b. Incorrecta. Lograr la participación del CEO no necesariamente reducirá la resistencia al cambio por parte del personal de auditoría.

c. Correcta. Lograr que el personal respalde los cambios desde el principio reducirá su resistencia al cambio.

d. Incorrecta. Lograr la participación de los clientes de la actividad de auditoría interna no necesariamente reducirá la resistencia al cambio por parte del personal de auditoría.

70. Solución: c

a. Incorrecta. El gerente no abordó el tema de la conducta y perdió la oportunidad de instruir a ambos miembros del personal y resolver el conflicto.

b. Incorrecta. Si bien uno de los auditores se comportó de manera inapropiada, ambos permitieron que se presentara la situación y deben participar en resolverla para proteger la moral y eficacia del equipo.

c. Correcta. Esta situación permite que ambas partes aborden y resuelvan sus diferencias bajo la supervisión del gerente de auditoría.

d. Incorrecta. No es una cuestión para que aborde todo el equipo. Puede informarse al equipo al respecto después de resolver el conflicto, pero el equipo no debe participar en una acción disciplinaria aplicada por el gerente.

71. Solución: c

a. Incorrecta. Puede incluirse a los empleados en la nómina de manera apropiada, pero es probable que los montos abonados no estén autorizados.

b. Incorrecta. Los cheques no entregados no proveen evidencia respecto de la validez de los montos.

c. Correcta. El supervisor del empleado se encontraría en la mejor posición para asegurar el pago del monto apropiado.

d. Incorrecta. Ser testigo de la distribución de los pagos por nómina no aseguraría que los montos abonados estén autorizados.

72. Solución: a

a. Correcta. La revisión por supervisión al nivel del departamento originario es un medio de control sobre la cantidad de artículos pedidos.

b. Incorrecta. Este procedimiento llevaría a comprar material en exceso, porque no considera planes futuros.

c. Incorrecta. Es un control para el riesgo de aceptar productos no pedidos. d. Incorrecta. Es un control para el riesgo de recibir una cantidad distinta de la pedida.

I - 54

73. Solución: d (II y IV solamente) I. Incorrecta. Es un síntoma de controles débiles para alcanzar metas y objetivos

organizacionales, pero no para salvaguardar activos. II. Correcta. Es un síntoma de controles débiles para salvaguardar activos. III. Incorrecta. Es un síntoma de controles débiles para alcanzar metas y objetivos

organizacionales, pero no para salvaguardar activos. IV. Correcta. El hecho de que la gerencia no haya tomado acción correctiva de acuerdo con

observaciones anteriores del trabajo, que se relacionaban con la salvaguarda de activos, es una debilidad relacionada con la salvaguarda de activos.

74. Solución: c

a. Incorrecta. El monto total comprometido no detectaría el favoritismo mostrado hacia proveedores individuales.

b. Incorrecta. Las especificaciones detalladas de los materiales no evitarán el favoritismo de los compradores al colocar pedidos.

c. Correcta. La rotación periódica de las tareas de los compradores limitará la oportunidad de cualquier comprador de mostrar favoritismo hacia un proveedor en particular.

d. Incorrecta. La cantidad de pedidos colocados no es pertinente para evitar el favoritismo. 75. Solución: b

a. Incorrecta. No aseguraría que las materias primas sean de suficiente calidad. b. Correcta. Las especificaciones para los materiales comprados proveen un medio objetivo a fin

de determinar que los materiales cumplan con un nivel mínimo de calidad para la producción. c. Incorrecta. Sólo ayudaría a asegurar que las materias primas se utilizan en las cantidades

apropiadas. d. Incorrecta. Sólo permitiría la determinación apropiada de la cantidad de desechos una vez que

las materias primas se hayan utilizado en la producción. 76. Solución: a

a. Correcta. Los controles preventivos son acciones tomadas antes de que se realicen las transacciones con la intención de evitar que se sigan cometiendo errores. El uso de una lista aprobada de proveedores es un control para evitar el uso de proveedores inaceptables.

b. Incorrecta. Un control detectivo es un control que identifica errores después de que se produjeron.

c. Incorrecta. Los controles correctivos corrigen los problemas identificados por los controles detectivos.

d. Incorrecta. Los controles de monitoreo se encuentran diseñados para asegurar la calidad del desempeño del sistema de controles en un tiempo dado.

77. Solución: a

a. Correcta. La conciliación independiente de cuentas bancarias resulta necesaria para el buen control interno.

b. Incorrecta. No es una consideración importante de control interno. c. Incorrecta. Las tasas de traducción de moneda extranjera no se computan, sino que se

verifican. Tener a dos empleados en el mismo departamento que realicen la misma tarea no mejorará el control interno de manera significativa.

d. Incorrecta. No es una consideración importante de control interno.

I - 55

78. Solución: d a. Incorrecta. Puede utilizarse, pero es sólo un medio para alcanzar el objetivo de una encuesta

preliminar. La respuesta “d” es la más completa. b. Incorrecta. Ver respuesta “a”. c. Incorrecta. Ver respuesta “a”. d. Correcta. El Consejo para la Práctica 2210.A1-1.2 expresa: “Cuando corresponda, debe

realizarse un estudio para familiarizarse con las actividades, riesgos y controles, para identificar las áreas en las que se deberá poner más énfasis en el trabajo y para lograr comentarios y sugerencias de parte de los clientes del trabajo...”

79. Solución: a

a. Correcta. La información de esta encuesta preliminar debe incitar al auditor a identificar la magnitud de dichos pagos duplicados.

b. Incorrecta. Los pasivos no registrados no resultarían. c. Incorrecta. La existencia de pagos duplicados no se relaciona con un problema del área de

recepción. d. Incorrecta. Los pagos duplicados no son pagos en exceso; son excepciones y deben

administrarse como tales. 80. Solución: d

a. Incorrecta. Ver respuesta “d”. b. Incorrecta. Ver respuesta “d”. c. Incorrecta. Ver respuesta “d”. d. Correcta. Deben desarrollarse todos los procedimientos. Ver Consejo para la Práctica 2210.A1-

1, que describe una revisión preliminar. 81. Solución: b

a. Incorrecta. Estas tendencias no provocarían una expansión del alcance, porque resultan compatibles.

b. Correcta. Estas tendencias pueden indicar cifras de ventas con inflación. c. Incorrecta. Estas tendencias no provocarían una expansión del alcance, porque resultan

compatibles. d. Incorrecta. Estas tendencias no provocarían una expansión del alcance, porque resultan

compatibles. 82. Solución: c

a. Incorrecta. Es importante mantener un alcance amplio y no reducirlo prematuramente. b. Incorrecta. Aunque el auditor pueda contribuir al conocimiento del gerente del departamento de

protección ambiental, de seguridad y salud (PASS) sobre temas relacionados con la calidad del aire, es mucho más importante, durante esta fase de la auditoría, saber qué hace el gerente.

c. Correcta. El auditor debe asegurar que el trabajo de campo esté diseñado para identificar posibles casos de falta de cumplimiento y, en la reunión de cierre, debe recomendar capacitación adicional para el gerente de PASS.

d. Incorrecta. Nunca es apropiado que un auditor informe violaciones o posibles violaciones a organismos reguladores. Dichos temas son responsabilidad del asesor legal de la compañía.

83. Solución: b

a. Incorrecta. Especifica parte de un paso del programa del trabajo. b. Correcta. Es algo que debe lograrse con el trabajo de auditoría. Asimismo, es específico, ya

que conecta el saldo del inventario con el criterio de satisfacer las necesidades proyectadas de los clientes.

c. Incorrecta. Es un paso del programa del trabajo. d. Incorrecta. Es una especificación para la comunicación final del trabajo.

I - 56

84. Solución: c a. Incorrecta. Como los nuevos instrumentos financieros resultaban muy riesgosos, el primer paso

de dicho trabajo debe ser determinar la naturaleza de las políticas establecidas para las inversiones.

b. Incorrecta. La supervisión realizada por un comité gerencial es un control importante. En consecuencia, el auditor debe determinar la naturaleza de la supervisión establecida para vigilar y autorizar dichas inversiones.

c. Correcta. Si bien podría ser informativo, no hay necesidad de desarrollar una comparación de rendimientos con otras organizaciones. En realidad, algunos escándalos sobre inversiones financieras muestran que dichas comparaciones puden resultar altamente engañosas porque los altos rendimientos se debieron a que se tomó un alto nivel de riesgo. Además, no es una prueba de la adecuación de los controles.

d. Incorrecta. Un concepto de control fundamental sobre activos de caja es que alguien establece un mecanismo para vigilar los riesgos.

85. Solución: a

a. Correcta. Es lo que requieren las Normas (Normas de Implantación 2120.A4 y Consejo para la práctica relacionado 2120.A4-1).

b. Incorrecta. El auditor debe lograr entender las normas operativas en la medida que se aplican a la organización. Asimismo, las mejores prácticas pueden producir estándares demasiado altos.

c. Incorrecta. Las Normas estipulan que si los auditores internos deben interpretar las normas, deben lograr llegar a un acuerdo con el cliente del trabajo.

d. Incorrecta. En primer lugar, el auditor debe llegar a un acuerdo con el gerente del departamento sobre las normas apropiadas.

86. Solución: a

a. Correcta. Si la evaluación preliminar indica problemas de control, el auditor, en general, decide realizar algunas pruebas expandidas.

b. Incorrecta. Si fuera necesario un diagrama de flujo, el auditor lo habría preparado durante la evaluación preliminar.

c. Incorrecta. El auditor no está preparado para hacer un informe hasta que se haya realizado más trabajo.

d. Incorrecta. Los auditores no implementan controles; es una función de la gerencia. 87. Solución: b

a. Incorrecta. Es probable que los niveles de gastos predefinidos ya incluyan los montos fraudulentos y sólo limiten la envergadura del fraude.

b. Correcta. La autorización adicional sería la opción más probable para evitar el fraude. c. Incorrecta. El conocimiento de embarque coincidiría con la orden de compra. La cantidad

recibida (verificada por un tercero) debe compararse tanto con el conocimiento de embarque como con la orden de compra.

d. Incorrecta. La comparación mediante la computadora sólo verificaría los papeles fraudulentos. 88. Solución: c

a. Incorrecta. Los gastos del trimestre actual serían iguales a la actividad de períodos anteriores, a menos que el gerente comience con este fraude. El auditor no tiene información sobre cuánto hace que esto podría haber estado ocurriendo.

b. Incorrecta. Las pruebas del inventario físico no ubicarían partidas no existentes que ya se habrían cobrado a mantenimiento.

c. Correcta. Un análisis de repuestos cobrado a mantenimiento cuantificaría la cantidad excesiva de artículos y detectaría que podría estarse cometiendo un abuso.

d. Incorrecta. La falta de segregación de obligaciones permitió que hubiera fraude. Se autorizó al gerente a procesar tanto la compra como el recibo; por lo tanto, la prueba sólo verificaría los papeles fraudulentos.

I - 57

89. Solución: b a. Incorrecta. El trabajo puede realizarse con una limitación del alcance. b. Correcta. El Consejo para la Práctica 1130-1.3 expresa que una limitación del alcance y sus

posibles efectos deben ser comunicados al comité de auditoría y al consejo directivo. c. Incorrecta. Una limitación del alcance no necesariamente provocaría la necesidad de trabajos

de auditoría más frecuentes. d. Incorrecta. Una limitación del alcance no necesariamente provocaría la necesidad de más

personal experimentado. 90. Solución: a

a. Correcta. Después de diez años, debe evaluarse la eficacia del programa. Si el auditor evalúa los efectos de los datos incompletos, y aclara que no se hace responsable de la confiabilidad, proveerá algo de la evaluación de la eficacia a los lectores sin engañarlos sobre la interpretación de dichos datos.

b. Incorrecta. La organización ya afirmó que los datos están incompletos. Este paso sería redundante.

c. Incorrecta. Muchas veces los auditores necesitan trabajar con datos imperfectos. Un programa que continuó utilizándose durante diez años necesita una evaluación. Siempre y cuando el auditor evalúe los efectos de los datos incompletos y aclare en el informe que no se hace responsable de la confiabilidad, el análisis puede ser útil sin que resulte engañoso.

d. Incorrecta. Ver respuesta “c”. 91. Solución: a

a. Correcta. La mayoría de los perpetradores tratarían de ocultar su robo cargándolo a la cuenta de gastos.

b. Incorrecta. Debitar la cuenta de activos robados iría en la dirección equivocada para ocultar el robo de activos.

c. Incorrecta. Un ingreso decreciente resultaría inusual y se destacaría. d. Incorrecta. Este ingreso no ocultaría el fraude en forma permanente. Simplemente cambiaría el

saldo irreconciliable a otra cuenta de activos. 92. Solución: b

a. Incorrecta. Un grupo tiene mejor oportunidad de perpetrar una irregularidad con éxito que un empleado individual.

b. Correcta. Es probable que un buen sistema de controles internos exponga una irregularidad si es perpetrada por un empleado sin la ayuda de otros.

c. Incorrecta. A menudo, la gerencia puede anular los controles, en forma individual o en grupos. d. Incorrecta. A menudo, la gerencia puede anular los controles, en forma individual o en grupos.

93. Solución: b

a. Incorrecta. Este procedimiento resultaría útil sólo para determinar si la causa se debió a un inventario exagerado.

b. Correcta. Un análisis de operaciones resultaría pertinente para determinar la eficiencia de las operaciones.

c. Incorrecta. Los cambios en el equipamiento pueden representar una mejora en la eficiencia, pero este enfoque no resultaría tan pertinente como el de la respuesta “b”.

d. Incorrecta. Este procedimiento resultaría pertinente para determinar la corrección de las compras de materias primas, pero no proveería ninguna evidencia respecto de la eficiencia de las operaciones.

I - 58

94. Solución: b a. Incorrecta. Los controles de acceso son tangenciales a la cuestión. Los datos autorizados, pero

incorrectos podrían también ser el problema. b. Correcta. Este procedimiento establecería la causa del problema. c. Incorrecta. Proporcionaría información útil, pero no es tan integral como la respuesta “b”. Más

aún, la respuesta “b” provee más información de la causa. d. Incorrecta. Prueba sólo una fuente de la inexactitud de los datos; es decir, el ingreso de los

datos de producción. Se ignoran otras fuentes de posible error. 95. Solución: a

a. Correcta. Es el procedimiento más adecuado porque: (a) el auditor ya determinó que hay una preocupación y (b) este procedimiento provoca la comparación directa de los requerimientos actuales de las partidas con las órdenes de compra generadas. Pueden identificarse las diferencias y tomarse acciones correctivas.

b. Incorrecta. Este procedimiento provee evidencia de que se procesan todos los artículos ingresados. La comparación con las órdenes de compra generadas actualmente no provee evidencia de si se piden las partidas correctas.

c. Incorrecta. El Software general de auditoría es un buen método para identificar un problema de inventario. Sin embargo, el exceso de inventario puede no ser el resultado de una técnica de revisión de la producción. La respuesta “a” aborda la preocupación de auditoría en forma más directa.

d. Incorrecta. Este procedimiento provee evidencia del ingreso de datos en el sistema, pero no determina si los cambios se implementaron en el proceso de producción.

96. Solución: a

a. Correcta. Los resultados de un trabajo de auditoría financiera serían el factor menos pertinente al establecer prioridades en las tareas del auditor.

b. Incorrecta. El fraude es uno de los principales factores que deben considerarse al analizar el riesgo e identificar las actividades de auditoría.

c. Incorrecta. El incremento en los gastos provee un parámetro para la posible exposición o pérdida a la organización.

d. Incorrecta. Las multas impuestas por organismos reguladores podrían representar un riesgo significativo.

97. Solución: a

a. Correcta. Los presupuestos de tiempo deben evaluarse para su revisión después de la encuesta preliminar y de la preparación del programa del trabajo.

b. Incorrecta. Cuando se comprueba una deficiencia, no se requiere ningún trabajo de auditoría adicional.

c. Incorrecta. Las tareas del personal sin experiencia no deben tener ningún efecto en el presupuesto de tiempo.

d. Incorrecta. Las pruebas expandidas no debe tener ningún efecto en el presupuesto de tiempo; el presupuesto ya se habría expandido cuando correspondía.

98. Solución: d

a. Incorrecta. El auditor interno puede ser responsable si se le asigna este trabajo, pero no tiene una responsabilidad definitiva.

b. Incorrecta. El comité de auditoría es responsable de asegurar que se alcancen los objetivos del plan de auditoría anual, pero no es responsable de cada uno de los objetivos del trabajo de auditoría.

c. Incorrecta. El supervisor de auditoría interna puede ser responsable si se le asigna este trabajo, pero no tiene una responsabilidad definitiva.

d. Correcta. De acuerdo con el Consejo para la Práctica 2340-1, el director ejecutivo de auditoría es responsable de la supervisión, en la cual se incluye determinar que se están alcanzando los objetivos del trabajo.

I - 59

99. Solución: b a. Incorrecta. Resultaría apropiado utilizar un programa estandarizado del trabajo para un

ambiente operativo de cambios mínimos. b. Correcta. No resultaría apropiado utilizar un programa estandarizado del trabajo para un

ambiente operativo complejo o con cambios porque los objetivos del trabajo y los pasos del trabajo relacionados pueden ya no ser pertinentes.

c. Incorrecta. Podría utilizarse un programa estandarizado del trabajo para auditar múltiples sucursales con operaciones parecidas.

d. Incorrecta. Resultaría aceptable utilizar un programa estandarizado del trabajo para conducir trabajos de auditorías de inventario consecutivas en el mismo lugar.

100. Solución: a

a. Correcta. Un programa hecho a la medida resultará más pertinente a una operación que un programa generalizado.

b. Incorrecta. Un programa generalizado no puede tomar en cuenta variaciones que resulten del cambio en circunstancias y condiciones variadas.

c. Incorrecta. Un programa generalizado no puede tomar en cuenta variaciones en circunstancias y condiciones.

d. Incorrecta. No debe examinarse cada aspecto de una operación, sino sólo aquellos que probablemente oculten problemas y dificultades.

FIN DE LAS SOLUCIONES PARTE I

El no cumplir con estas instrucciones y con las guías de "Instrucciones para candidatos" puede afectar seriamente

su derecho para recibir los resultados de este examen y su futura participación el programa de Auditor Interno Certificado.

Todos los documentos presentados a la terminación de cualquiera de las

partes de este examen son propiedad exclusiva de The Institute of Internal Auditor, Inc. Los candidatos no puede revelar el contenido de este examen a menos que cuenten con la

autorización del Departamento de Certificación.

II - 1

Preguntas del examen modelo para el Auditor Interno Certificado (CIA)

Parte II – Conducción de

proyectos de auditoría interna

Preguntas del examen modelo parte II: 100

Preguntas del examen CIA real parte II II: 125 (ver explicación en el “prólogo”, página iii)

Tiempo permitido para terminar el examen CIA parte II: 210 minutes

Instrucciones como las que a continuación se enuncian serán las que aparezcan

en la portada de cada examen CIA. Por favor, léanlas con cuidado.

1. Anote su número de candidato en la hoja de respuestas en el espacio proporcionado.

2. No haga anotaciones irrelevantes en la hoja de respuestas.

3. Aségurese de que todo cambio de respuesta sea completamente borrado.

4. Todas las referencias al Marco para la Práctica Profesional se refieren al Marco para la Práctica Profesional del IAI, que incluye las Normas y los Consejos para la Práctica.

Todas las referencias a las Normas se refieren a las Normas Internacionales para el Ejercicio Profesional de la Auditoría Interna resumidas en el Marco para la Práctica Profesional del IAI.

5. Los siguientes términos de muestreo estadístico se utilizan como sinónimos: a. unidad dólar, unidad monetaria, probabilidad proporcional a la medida; b. nivel de confianza y confiabilidad y c. Precisión y tolerancia para el riesgo en el muestreo.

II - 2

1. Un objetivo específico de una auditoría del ciclo de gastos de una compañía es determinar si se recibieron todos los productos pagados y se imputaron a la cuenta correcta. ¿Cuáles de los siguientes objetivos primordiales identificados en las Normas abordaría este objetivo? I. Confiabilidad e integridad de la

información financiera y operativa. II. Cumplimiento de leyes, regulaciones y

contratos. III. Eficacia y eficiencia de las operaciones. IV. Salvaguarda de activos. a. I y II solamente. b. I y IV solamente. c. I, II, y IV solamente. d. II, III, y IV solamente.

2. ¿Cuál de las siguientes situaciones se

permitiría bajo el Código de Ética del IAI? a. Como respuesta a una citación, un

auditor apareció en una corte y reveló información confidencial y relacionada con auditoría que podría dañar a la organización del auditor.

b. Un auditor utilizó información relacionada con auditoría al tomar la decisión de comprar acciones emitidas por la corporación del empleador.

c. Después de elogiar a un empleado en una comunicación de un trabajo de auditoría reciente, un auditor aceptó un obsequio de él.

d. Un auditor no informó observaciones significativas sobre actividad ilegal al consejo porque la gerencia indicó que resolvería la cuestión.

3. Un auditor interno que enfrenta un dilema ético

no explícitamente abordado por el Código de Ética del IAI siempre debe: a. Buscar asesoramiento de un abogado

independiente para determinar las consecuencias personales de las posibles acciones.

b. Tomar acción uniforme con los principios comprendidos en el Código de Ética del IAI.

c. Buscar el asesoramiento del comité de auditoría antes de decidir tomar una acción.

d. Actuar de manera uniforme con el código de ética de la organización empleadora, incluso si dicha acción no resulta uniforme con el Código de Ética del IAI.

1. A specific objective of an audit of a company’s expenditure cycle is to determine if all goods paid for have been received and charged to the correct account. This objective would address which of the following primary objectives identified in the Standards?

I. Reliability and integrity of financial and

operational information. II. Compliance with laws, regulations, and

contracts. III. Effectiveness and efficiency of operations. IV. Safeguarding of assets. a. I and II only. b. I and IV only. c. I, II, and IV only. d. II, III, and IV only.

2. Which of the following would be permissible

under the IIA Code of Ethics? a. In response to a subpoena, an auditor

appeared in a court of law and disclosed confidential, audit-related information that could potentially damage the auditor’s organization.

b. An auditor used audit-related information in a decision to buy stock issued by the employer corporation.

c. After praising an employee in a recent audit engagement communication, an auditor accepted a gift from the employee.

d. An auditor did not report significant observations about illegal activity to the board because management indicated that it would resolve the issue.

3. An internal auditor who encounters an ethical

dilemma not explicitly addressed by the IIA Code of Ethics should always: a. Seek counsel from an independent

attorney to determine the personal consequences of potential actions.

b. Take action consistent with the principles embodied in the IIA Code of Ethics.

c. Seek the counsel of the audit committee before deciding on an action.

d. Act consistently with the employing organization’s code of ethics even if such action would not be consistent with the IIA Code of Ethics.

II - 3

4. Un auditor descubre un plan para exagerar el inventario y así aumentar las ganancias registradas para una división. El auditor tiene evidencia importante de que el gerente divisional era consciente del plan para exagerar el inventario y de que lo había aprobado. Además, existe evidencia de que el gerente puede haber sido responsable de la implementación del plan. El auditor debe: a. Continuar conduciendo entrevistas con

los subordinados hasta que se haga un caso definido, y luego informar el caso al comité de auditoría.

b. Informar a la gerencia senior y al comité de auditoría acerca de los hallazgos y debatir una posible investigación adicional.

c. Informar al gerente divisional sobre las sospechas del auditor y obtener la explicación que el gerente da sobre los hallazgos antes de seguir profundizando sobre la cuestión.

d. Documentar el caso minuciosamente e informar las sospechas al auditor externo para realizar una revisión adicional.

5. ¿En qué fase de la auditoría suelen

desarrollarse las actividades de recolección de datos tales como entrevistar a personal operativo, identificar las normas que deban utilizarse para la evaluación del desempeño y evaluar los riesgos inherentes en las operaciones de un departamento? a. Trabajo de campo. b. Encuesta preliminary. c. Desarrollo del programa de auditoria. d. Examen y evaluación de la evidencia.

6. ¿Cuál de las siguientes situaciones describe

mejor la responsabilidad de un auditor después de observar algunos indicadores de fraude? a. Expandir actividades para determinar si

se garantiza una investigación. b. Informar la posibilidad de fraude a la

gerencia senior y preguntar cómo proseguir.

c. Consultar con el asesor legal externo para determinar el curso de acción por tomar.

d. Informar la cuestión al comité de auditoría y requerir financiamiento para los especialistas externos a fin de ayudar a investigar el posible fraude.

4. An auditor uncovers a plan to overstate inventory and thereby increase reported profits for a division. The auditor has substantial evidence that the divisional manager was aware of and approved the plan to overstate inventory. There is also some evidence that the manager may have been responsible for the implementation of the plan. The auditor should: a. Continue to conduct interviews with

subordinates until a definite case is made, and then report the case to the audit committee.

b. Inform senior management and the audit committee of the findings and discuss possible further investigation.

c. Inform the divisional manager of the auditor’s suspicions and obtain the manager's explanation of the findings before pursuing the matter further.

d. Document the case thoroughly and report the suspicions to the external auditor for further review.

5. Data-gathering activities such as interviewing

operating personnel, identifying standards to be used to evaluate performance, and assessing risks inherent in a department’s operations are typically performed in which phase of an audit engagement? a. Fieldwork. b. Preliminary survey. c. Engagement program development. d. Examination and evaluation of evidence.

6. Which of the following best describes an

auditor’s responsibility after noting some indicators of fraud? a. Expand activities to determine whether an

investigation is warranted. b. Report the possibility of fraud to senior

management and ask how to proceed. c. Consult with external legal counsel to

determine the course of action to be taken.

d. Report the matter to the audit committee and request funding for outside specialists to help investigate the possible fraud.

II - 4

7. ¿Cuál de los siguientes sería el procedimiento menos útil para predecir el monto de cuentas incobrables en una organización? a. Información sobre los índices económicos

que indican un cambio desfavorable en los negocios.

b. Valor de las cuentas por cobrar realmente enviadas a pérdida durante cada uno de los últimos seis meses.

c. Valor total de ventas durante cada uno de los últimos seis meses.

d. Presupuestos escritos del responsable de cobranzas en relación con las futuras cobranzas en efectivo esperadas.

8. Para que sea suficiente, la evidencia de

auditoría debe: a. Encontrarse documentada y con

referencias cruzadas en los papeles de trabajo.

b. Basarse en referencias que se consideren confiables.

c. Encontrarse directamente relacionada con la observación del trabajo e incluir todos los elementos de dicha observación.

d. Resultar lo suficientemente convincente para que una persona prudente llegue a la misma conclusión que el auditor.

9. ¿Cuál de los siguientes ejemplos de evidencia

de auditoría es el más persuasivo? a. Títulos de propiedad de bienes

inmuebles, que se registraron apropiadamente ante un organismo gubernamental.

b. Cheques cancelados escritos por el tesorero y devueltos de un banco.

c. Tarjetas reloj para los empleados, guardadas por un gerente.

d. Facturas de los proveedores archivadas por el departamento de contabilidad.

10. La evidencia competente se define mejor

como evidencia que: a. Se encuentra razonablemente libre de

error y parcialidad y representa fielmente aquello que dice representar.

b. Se obtiene observando a las personas, las propiedades y los eventos.

c. Es suplementaria para otra evidencia ya recopilada y que tiende a fortalecerla o confirmarla.

d. Prueba un hecho intermedio o un grupo de hechos, a partir de los cuales pueden deducirse otros hechos.

7. Which of the following would be least useful in predicting the amount of uncollectible accounts for an organization? a. Published economic indices indicating a

general business downturn. b. Dollar amounts of accounts actually

written off by the organization for each of the past six months.

c. Total monthly sales for each of the past six months.

d. Written forecasts from the credit manager regarding expected future cash collections.

8. To be sufficient, audit evidence should be:

a. Well-documented and cross-referenced in the workpapers.

b. Based on references that are considered reliable.

c. Directly related to the engagement observation and include all of the elements of an engagement observation.

d. Convincing enough for a prudent person to reach the same conclusion as the auditor.

9. Which of the following examples of audit

evidence is the most persuasive? a. Real estate deeds, which were properly

recorded with a government agency. b. Canceled checks written by the treasurer

and returned from a bank. c. Time cards for employees, which are

stored by a manager. d. Vendor invoices filed by the accounting

department. 10. Competent evidence is best defined as

evidence which: a. Is reasonably free from error and bias and

faithfully represents that which it purports to represent.

b. Is obtained by observing people, property, and events.

c. Is supplementary to other evidence already gathered and which tends to strengthen or confirm it.

d. Proves an intermediate fact, or group of facts, from which still other facts can be inferred.

II - 5

11. ¿Cuál de las siguientes opciones representa la evidencia más competente de que las partidas por cobrar realmente existen? a. Confirmaciones positivas. b. Facturas de ventas. c. Informes de recepción. d. Conocimientos de embarque.

12. La siguiente tabla representa la información de

las cuentas por cobrar de una corporación durante un período de tres años: Primer

Año Segundo

Año Tercer

año Cuentas netas por cobrar

como porcentaje del total de los activos 23.4% 27.3% 30.8%

ITO (Rotación de las Cuentas por Cobrar) 6.98 6.05 5.21

Todas las siguientes son explicaciones válidas para estos intercambios excepto que: a. Pudieron haberse registrado ventas

ficticias. b. Los procedimientos de créditos y

cobranzas ya no resultan eficaces. c. Se subestiman las asignaciones para

deudas incobrables. d. Se sobreestimaron los ingresos por

ventas a crédito. 13. La tasa del movimiento de las cuentas por

cobrar de una compañía disminuyó de 7.3 a 4.3 durante los últimos tres años. ¿Cuál es la causa más probable de esta disminución? a. Un incremento en los descuentos

ofrecidos por pagos anticipados. b. Una política de crédito más liberal. c. Un cambio en los pagos netos con

vencimiento de 30 a 25 días. d. Incremento en las ventas de contado.

14. Es probable que al revisar un listado editado

de los cambios de la nómina procesados durante cada ciclo se revele lo siguiente: a. Errores no detectados en las cuotas de

pago de nuevos empleados de la nómina. b. Deducciones inexactas de la nómina. c. Horas de trabajo cargadas a una cuenta

equivocada del sistema de informes de costos.

d. No lograr ofrecer a los empleados una oportunidad de contribuir con su plan de pensiones y jubilaciones.

11. Which of the following represents the most competent evidence that trade receivables actually exist? a. Positive confirmations. b. Sales invoices. c. Receiving reports. d. Bills of lading.

12. The following represents accounts receivable

information for a corporation for a three-year period: Year

One Year Two

Year Three

Net accounts receivable as a percentage of total assets 23.4% 27.3% 30.8%

Accounts receivable turnover ratio 6.98 6.05 5.21

All of the following are plausible explanations for these changes except: a. Fictitious sales may have been recorded. b. Credit and collection procedures have

become ineffective. c. Allowance for bad debts is understated. d. Sales returns for credit have been

overstated. 13. A company’s accounts receivable turnover rate

decreased from 7.3 to 4.3 over the last three years. What is the most likely cause for the decrease? a. An increase in the discount offered for

early payment. b. A more liberal credit policy. c. A change in net payment due from 30 to

25 days. d. Increased cash sales.

14. Reviewing an edit listing of payroll changes

processed during each payroll cycle would most likely reveal: a. Undetected errors in the payroll rates of

new employees. b. Inaccurate payroll deductions. c. Labor hours charged to the wrong

account in the cost reporting system. d. A failure to offer employees an

opportunity to contribute to their pension plan.

II - 6

15. ¿Qué sugiere el siguiente gráfico?

Y= Ingresos por ventas

X= Costos decapacitación

a. Los ingresos por ventas están

inversamente relacionados con los costos de capacitación.

b. El programa de capacitación no resulta eficaz.

c. Los costos de capacitación no afectan a los ingresos por ventas.

d. Los diversos puntos de venta están incorrectamente marcados.

16. ¿Cuáles de las siguientes declaraciones son

correctas en relación con la documentación de papeles de trabajo de auditoría interna para una investigación de fraude?

I. Toda evidencia incriminatoria debe

encontrarse incluida en los papeles de trabajo.

II. Toda evidencia testimonial importante debe revisarse a fin de garantizar que proporcione bases suficientes para las conclusiones alcanzadas.

III. Si se llevan a cabo entrevistas con un sospechoso de fraude, deben incluirse expedientes o declaraciones por escrito en los papeles de trabajo.

a. I solamente. b. II solamente. c. II y III solamente. d. I, II, y III.

17. Los resúmenes de los papeles de trabajo, si

se preparan, pueden utilizarse para: a. Promover la revisión eficiente de los

papeles de trabajo por parte de los supervisores de auditoría interna.

b. Reemplazar los archivos de papeles de trabajo detallados para retención permanente.

c. Servir como comunicación final del trabajo para la gerencia senior.

d. Documentar el total desarrollo de las observaciones y recomendaciones del trabajo.

15. What does the following scattergram suggest?

Y= SalesRevenue

X= Training Costs

a. Sales revenue is inversely related to

training costs. b. The training program is not effective. c. Training costs do not affect sales

revenue. d. Several data points are incorrectly plotted.

16. Which of the following statements are correct

regarding audit engagement workpaper documentation for a fraud investigation?

I. All incriminating evidence should be

included in the workpapers. II. All important testimonial evidence should

be reviewed to ensure that it provides sufficient basis for the conclusions reached.

III. If interviews are held with a suspected perpetrator, written transcripts or statements should be included in the workpapers.

a. I only. b. II only. c. II and III only. d. I, II, and III.

17. Workpaper summaries, if prepared, can be

used to: a. Promote efficient workpaper review by

internal audit supervisors. b. Replace the detailed workpaper files for

permanent retention. c. Serve as an engagement final

communication to senior management. d. Document the full development of

engagement observations and recommendations.

II - 7

18. ¿Cuál de las siguientes situaciones representa evidencia apropiada de la supervisión de los papeles de trabajo?

I. Las iniciales de un supervisor en cada

papel de trabajo. II. Una lista de verificaciones de la revisión

de papeles de trabajo. III. Un memorando que especifique la

naturaleza, el alcance y los resultados de la supervisión de los papeles de trabajo.

IV. Realizar evaluaciones de la calidad de los papeles de trabajo preparados por los auditores.

a. II y IV solamente. b. I, II, y III solamente. c. I, III, y IV solamente. d. I, II, III y IV.

19. Cuál de las siguientes opciones describe de

manera más completa el contenido de los papeles de trabajo? a. Objetivos, procedimientos y conclusiones. b. Propósito, criterios, técnicas y

conclusiones. c. Objetivos, procedimientos, hechos,

conclusiones y recomendaciones. d. Tema, propósito, información de

muestreo y análisis. 20. ¿Cuál de las siguientes situaciones es más

probable que sea el tema de un informe intermedio escrito para el cliente del trabajo? a. Se llevó a cabo el setenta por ciento del

trabajo de auditoría planificado sin observaciones adversas significativas.

b. Los auditores han decidido sustituir los procedimientos de encuestas para algunas de las revisiones detalladas y planificadas de ciertos registros.

c. Se expandió el programa del trabajo por indicaciones de un posible fraude.

d. La quema abierta de una planta subsidiaria establece una posible violación a las reglamentaciones sobre contaminación.

18. Which of the following represents appropriate evidence of supervisory review of engagement workpapers?

I. A supervisor’s initials on each workpaper. II. An engagement workpaper review

checklist. III. A memorandum specifying the nature,

extent, and results of the supervisory review of workpapers.

IV. Performance appraisals that assess the quality of workpapers prepared by auditors.

a. II and IV only. b. I, II, and III only. c. I, III, and IV only. d. I, II, III, and IV.

19. Which of the following most completely

describes the appropriate content of engagement workpapers? a. Objectives, procedures, and conclusions. b. Purpose, criteria, techniques, and

conclusions. c. Objectives, procedures, facts,

conclusions, and recommendations. d. Subject, purpose, sampling information,

and analysis. 20. Which of the following situations is most likely

to be the subject of a written interim report to the engagement client? a. Seventy percent of the planned audit work

has been completed with no significant adverse observations.

b. The auditors have decided to substitute survey procedures for some of the planned detailed review of certain records.

c. The engagement program has been expanded because of indications of possible fraud.

d. Open burning at a subsidiary plant poses a prospective violation of pollution regulations.

II - 8

Utilice la siguiente información para contestar las preguntas 21 a 23.

Una municipalidad de mediana envergadura provee 8,5 mil millones de galones de agua por año para 31.000 clientes. Los medidores de agua se reemplazan al menos cada cinco años para asegurar una facturación exacta. El departamento del agua rastrea el agua sin medir para identificar el consumo que no se está facturando. Hace poco tiempo, el departamento emitió el siguiente informe sobre la actividad del agua:

Actividad Mes uno

Mes dos

Mes tres

1er trimestre real

Meta del 1er trimestre

Medidoresreemplazados 475 400 360 1,235 1,425

Pérdidas registradas 100 100 85 285

Pérdidas reparadas 100 100 85 285 100%

Agua no medida 2% 6% 2% 4% 2%

21. Sobre la base de la actividad informada para

el programa de reemplazo del medidor, un auditor interno llegaría a la siguiente conclusión: a. Se comprenden y cumplen los estándares

operativos establecidos. b. Es probable que se haya tomado

cualquier acción correctiva necesaria durante el trimestre.

c. Deben analizarse y corregirse los desvíos de la meta.

d. Los medidores deben cambiarse cada tres años.

22. Sobre la base de la actividad informada acerca

de las pérdidas reparadas en el primer trimestre, un auditor interno llegaría a la siguiente conclusión: a. Se comprenden y cumplen los estándares

operativos establecidos. b. Deben analizarse los desvíos de la meta

y tomarse las acciones correctivas correspondientes.

c. Debe modificarse el estándar operativo. d. El programa de reparación de pérdidas se

encuentra sobrecargado.

Use the following information to answer questions 21 through 23.

A medium-sized municipality provides 8.5 billion gallons of water per year for 31,000 customers. The water meters are replaced at least every five years to ensure accurate billing. The water department tracks unmetered water to identify water consumption that is not being billed. The department recently issued the following water activity report:

Activity Month One

Month Two

Month Three

Actual 1st Quarter

1st Quarter

Goal Meters

Replaced 475 400 360 1,235 1,425Leaks

Reported 100 100 85 285Leaks

Repaired 100 100 85 285 100%Unmetered

Water 2% 6% 2% 4% 2% 21. Based on the activity reported for the meter

replacement program, an internal auditor would conclude that: a. Established operating standards are

understood and are being met. b. Any corrective action needed has

probably been taken during the quarter. c. Deviations from the goal should be

analyzed and corrected. d. Meters should be changed every three

years. 22. Based on the activity reported for leaks

repaired in the first quarter, an internal auditor would conclude that: a. Established operating standards are

understood and are being met. b. Deviations from the goal should be

analyzed and corrective action should be taken.

c. The operating standard should be changed.

d. The leak-repair program is overstaffed.

II - 9

23. Sobre la base de la actividad informada sobre el agua no medida, un auditor interno llegaría a la siguiente conclusión: a. Se comprenden y cumplen los estándares

operativos establecidos. b. No se garantiza una investigación

adicional sobre el agua no medida. c. Es probable que no se hayan corregido

los desvíos de la meta. d. Debe modificarse el estándar operativo.

24. En la revisión del cumplimiento de ciertas

políticas de la compañía, un auditor descubrió que: (1) El cinco por ciento de los empleado

pertenece a grupos minoritarios. (2) No se contrató a nadie de los grupos

minoritarios en el último año. La conclusión más apropiada a la que el auditor puede llegar es que: a. Existe evidencia insuficiente respecto del

cumplimiento con las políticas establecidas.

b. La división está violando las políticas de la compañía.

c. No pueden auditarse las políticas de la compañía y, por lo tanto, tampoco es posible hacer que se cumplan.

d. Con el cinco por ciento de sus empleados pertenecientes a grupos minoritarios, la división está cumpliendo con eficacia.

25. Deben incluirse las recomendaciones en los

informes de auditoría para: a. Proveer a la gerencia opciones para

abordar los hallazgos de auditoría. b. Asegurar que los problemas se resuelvan

de la manera sugerida por el auditor. c. Minimizar la cantidad de tiempo requerida

para corregir los hallazgos de auditoría. d. Garantizar que se aborden los hallazgos

de auditoría, cualquiera sea el costo. 26. De acuerdo con el Marco para la Práctica

Profesional, ¿cuál de los siguientes forma parte de los requerimientos mínimos para la comunicación final de un trabajo?

I. Antecedentes. II. Propósito del trabajo. III. Alcance del trabajo. IV. Resultados del trabajo. V. Resúmenes.

a. I, II y III solamente. b. I, III y V solamente. c. II, III y IV solamente. d. II, IV y V solamente.

II - 10

23. Based on the activity reported for the unmetered water, an internal auditor would conclude that: a. Established operating standards are

understood and are being met. b. Further audit investigation of unmetered

water is not warranted. c. Deviations from the goal were probably

not corrected. d. The operating standard should be

changed. 24. While testing a division’s compliance with

company affirmative-action policies, an auditor found that: (1) Five percent of the employees are from

minority groups. (2) No one from a minority group has been

hired in the past year. The most appropriate conclusion for the auditor to reach is that: a. Insufficient evidence exists of compliance

with affirmative-action policies. b. The division is violating the company’s

policies. c. The company’s policies cannot be audited

and hence cannot be enforced. d. With five percent of its employees from

minority groups, the division is effectively complying.

25. Recommendations should be included in audit

reports in order to: a. Provide management with options for

addressing audit findings. b. Ensure that problems are resolved in the

manner suggested by the auditor. c. Minimize the amount of time required to

correct audit findings. d. Guarantee that audit findings are

addressed, regardless of cost.

26. According to the Professional Practices Framework, which of the following is part of the minimum requirements for an engagement final communication? I. Background information. II. Purpose of the engagement. III. Engagement scope. IV. Results of the engagement. V. Summaries. a. I, II, and III only. b. I, III, and V only. c. II, III, and IV only. d. II, IV, and V only.

27. ¿Cuál de los siguientes no se consideraría un

objetivo básico de la reunión de cierre de una auditoría? a. Resolver conflictos. b. Debatir las observaciones y

recomendaciones del trabajo. c. Identificar los asuntos de futuras

auditorías. d. Identificar las acciones de la gerencia y

sus respuestas a las observaciones y recomendaciones del trabajo.

28. Durante una revisión de operaciones de

compras, un auditor descubrió que los procedimientos en uso no concordaban con los procedimientos de la compañía. Sin embargo, las pruebas de auditoría revelaron que los procedimientos en uso representaban un aumento en la eficiencia y una disminución en el tiempo de procesamiento, sin un decrecimiento discernible en el control. El auditor debe: a. Informar la falta de cumplimiento de los

procedimientos documentados como una deficiencia operativa.

b. Desarrollar un diagrama de flujo de los nuevos procedimientos e incluirlo en el informe dirigido a la gerencia.

c. Informar el cambio y sugerir que se documente en los procedimientos.

d. Suspender el trabajo hasta que el cliente documente los nuevos procedimientos.

29. La razón principal para emitir informes de

auditoría formales y escritos es: a. Dar una oportunidad de respuesta al

cliente del trabajo. b. Documentar las acciones correctivas

requeridas de la alta gerencia. c. Proporcionar un medio formal por medio

del cual el auditor externo evalúe la posible confianza en la actividad de auditoría interna.

d. Registrar observaciones y acciones correctivas recomendadas.

II - 11

27. Which of the following would not be considered a primary objective of a closing or exit conference? a. To resolve conflicts. b. To discuss the engagement observations

and recommendations. c. To identify concerns for future audit

engagements. d. To identify management’s actions and

responses to the engagement observations and recommendations.

28. During a review of purchasing operations, an

auditor found that procedures in use did not agree with stated company procedures. However, audit tests revealed that the procedures in use represented an increase in efficiency and a decrease in processing time, without a discernible decrease in control. The auditor should: a. Report the lack of adherence to

documented procedures as an operational deficiency.

b. Develop a flowchart of the new procedures and include it in the report to management.

c. Report the change and suggest that the change in procedures be documented.

d. Suspend the completion of the engagement until the engagement client documents the new procedures.

29. The primary reason for having written formal

audit reports is to: a. Provide an opportunity for engagement

client response. b. Document the corrective actions required

of senior management. c. Provide a formal means by which the

external auditor assesses potential reliance on the internal audit activity.

d. Record observations and recommended courses of action.

II - 12

30. ¿Cuál de los siguientes es el mejor enfoque para obtener retroalimentación de los clientes sobre la calidad del trabajo de auditoría interna? a. Formular preguntas durante las

entrevistas de salida y enviar copias de las respuestas documentadas a los clientes.

b. Llamar a los clientes del trabajo después de las entrevistas de salida y enviar copias de las respuestas documentadas a los clientes.

c. Distribuir cuestionarios a los clientes seleccionados poco antes de preparar el informe anual de la actividad de auditoría interna.

d. Proveer cuestionarios a los clientes al comenzar cada trabajo y solicitar que los completen y entreguen después de los trabajos.

31. Cuando se conduce una evaluación de

desempeño de un auditor interno con promedio bajo, no resulta apropiado: a. Notificar al auditor interno sobre la

próxima evaluación varios días antes. b. Utilizar un lenguaje objetivo e imparcial. c. Utilizar generalizaciones. d. Documentar la evaluación.

32. Un auditor está considerado para participar en

el programa de revisión de los controles de nómina de sueldos de una compañía que recientemente transfirió su procesamiento de información a una contratista de servicios. ¿Qué acción debe tomar el auditor tomando en cuenta la decisión anterior? a. Revisar los controles de la nómina tanto

en la compañía como con el contratista de servicios.

b. Revisar únicamente los controles en la compañía respecto a la información enviada y recibida del contratista de servicios.

c. Revisar únicamente los controles relativos a los pagos al contratista de servicios basados en el contrato.

d. Cancelar la auditoría ya que el procesamiento se lleva a cabo fuera de la organización.

30. Which of the following is the best approach for obtaining feedback from engagement clients on the quality of internal audit work? a. Ask questions during the exit interviews

and send copies of the documented responses to the clients.

b. Call engagement clients after the exit interviews and send copies of the documented responses to the clients.

c. Distribute questionnaires to selected engagement clients shortly before preparing the internal audit annual activity report.

d. Provide questionnaires to engagement clients at the beginning of each engagement and request that the clients complete and return them after the engagements.

31. When conducting a performance appraisal of

an internal auditor who has been a below-average performer, it is not appropriate to: a. Notify the internal auditor of the upcoming

appraisal several days in advance. b. Use objective, impartial language. c. Use generalizations. d. Document the appraisal.

32. An auditor is scheduled to audit payroll

controls for a company that has recently outsourced its processing to an information service bureau. What action should the auditor take, considering the outsourcing decision? a. Review the controls over payroll in both

the company and the service bureau. b. Review only the company's controls over

data sent to and received from the service bureau.

c. Review only the controls over payments to the service bureau based on the contract.

d. Cancel the engagement, because the processing is being performed outside of the organization.

II - 13

33. Después de obtener documentación convincente de una conducta no ética por parte del vicepresidente de quien depende el director ejecutivo de auditoría, el director debe: a. Conducir una investigación para

determinar en qué medida el vicepresidente participa en los actos no éticos.

b. Confrontar al vicepresidente con los hechos antes de proseguir.

c. Programar una auditoría de la función del negocio involucrada.

d. Informar los hechos al director general y al comité de auditoría.

34. ¿Cuál de los siguientes procedimientos de

control resultaría menos eficaz para evitar un fraude originado por la emisión de órdenes de compra a proveedores ficticios? a. Requerir que todas las compras se

realicen a partir de una lista autorizada de proveedores, que se administre independientemente de la persona que coloque el pedido.

b. Requerir que únicamente se pague a los proveedores pre-autorizados, sobre la base de la producción real.

c. Requerir que se hagan contratos con todos los proveedores más importantes que suministran los materiales de producción.

d. Requerir que el total de las compras de todos los proveedores de un mes no excedan el total de compras presupuestado para ese mismo mes.

35. Cuando se entrevista a un individuo

sospechoso de fraude, el entrevistador debe: a. Garantizar que el supervisor del

sospechoso se encuentre presente durante la entrevista.

b. Cerrar la puerta con llave para asegurarse de que nadie interrumpa la entrevista.

c. Prestar atención a las palabras que elija el sospechoso para expresarse.

d. Preguntar si el sospechoso cometió el fraude.

33. Upon obtaining factual documentation of unethical business conduct by the vice president to whom the chief audit executive (CAE) reports, the CAE should: a. Conduct an investigation to determine the

extent of the vice president’s involvement in the unethical acts.

b. Confront the vice president with the facts before proceeding.

c. Schedule an audit of the business function involved.

d. Report the facts to the chief executive officer and the audit committee.

34. Which of the following control procedures

would be the least effective in preventing frauds in which purchase orders are issued to fictitious vendors? a. Require that all purchases be made from

an authorized vendor list maintained independently of the individual placing the purchase order.

b. Require that only preapproved vendors be paid for purchases, based on actual production.

c. Require contracts with all major vendors from whom production components are purchased.

d. Require that total purchases from all vendors for a month not exceed the total budgeted purchases for that month.

35. When interviewing an individual suspected of a

fraud, the interviewer should: a. Ensure the suspect’s supervisor is

present during the interview. b. Lock the door to ensure no one will

interrupt the interview. c. Pay attention to the wording choices of

the suspect. d. Ask if the suspect committed the fraud.

II - 14

36. ¿Qué frase describe mejor un proceso de auto-evaluación de controles basado en el control? a. Evaluar, actualizar y hacer más eficientes

los procesos de control seleccionados. b. Examinar en qué medida los controles

funcionan correctamente al administrar los riesgos clave.

c. Analizar la brecha entre el diseño y los marcos de control.

d. Determinar la eficacia de los controles en función de los costos.

37. La determinación de reducciones en costos es

más probable que sea un objetivo de: a. Una auditoría a programas. b. Una auditoría financiera. c. Una auditoría de cumplimiento. d. Una auditoría operativa.

38. Un auditor está considerando desarrollar un

cuestionario para investigar actitudes de los empleados hacia los procedimientos de control. ¿Cuál de los siguientes representa el criterio menos importante al diseñar el cuestionario? a. Deben formularse las preguntas de

manera tal que quienes respondan puedan darles una interpretación válida.

b. Deben formularse las preguntas de manera confiable, para que midan lo que se esperaba que midieran.

c. Debe evitarse un cuestionario demasiado largo para aumentar la proporción de respuestas.

d. Deben formularse las preguntas de manera tal que un “no” como respuesta indique la presencia de un problema.

39. Resulta típico que un trabajo de auditoría de

desempeño implique: a. Revisar la información de los estados

financieros, la cual debe incluir la adecuación de los diversos tratamientos contables.

b. Realizar pruebas de cumplimiento de las políticas, los procedimientos, las leyes y las reglamentaciones.

c. Evaluar el ambiente comparándolo con los criterios establecidos.

d. Evaluar las estructuras organizacionales y departamentales, que incluyan evaluaciones de flujos del proceso.

36. Which phrase best describes a control-based control self-assessment process? a. Evaluating, updating, and streamlining

selected control processes. b. Examining how well controls are working

in managing key risks. c. Analyzing the gap between control design

and control frameworks. d. Determining the cost-effectiveness of

controls. 37. Determination of cost savings is most likely to

be an objective of: a. Program audit engagements. b. Financial audit engagements. c. Compliance audit engagements. d. Operational audit engagements.

38. An auditor is considering developing a

questionnaire to research employee attitudes toward control procedures. Which of the following represents the least important criteria in designing the questionnaire? a. Questions should be worded to ensure a

valid interpretation by the respondents. b. Questions should be reliably worded so

that they measure what was intended to be measured.

c. The length of the questionnaire should be minimized in order to increase the response rate.

d. Questions should be worded such that a "No" answer indicates a problem.

39. A performance audit engagement typically

involves: a. Review of financial statement information,

including the appropriateness of various accounting treatments.

b. Tests of compliance with policies, procedures, laws, and regulations.

c. Appraisal of the environment and comparison against established criteria.

d. Evaluation of organizational and departmental structures, including assessments of process flows.

II - 15

40. El departamento de auditoría interna participa en el proceso de valuación (due diligence work) para cierta adquisición que una compañía se encuentra considerando. Uno de los objetivos de auditoría es determinar si las cuentas por pagar de la empresa a ser adquirida contienen todos los pasivos existentes. ¿Cuál de los siguientes procedimientos de auditoría no resultaría relevante para este objetivo? a. Examinar la documentación soporte de

los desembolsos de efectivo subsecuentes (después del periodo) y verificar la fecha del pasivo.

b. Enviar confirmaciones de saldos, incluyendo los de las cuentas que aparecen en cero, a los proveedores con quienes normalmente la compañía realiza transacciones.

c. Seleccionar una muestra de cuentas del listado de cuentas por pagar y verificar los reportes de recepción, órdenes de compra y facturas que las sustentan.

d. Seguir el rastro a los reportes de recepción emitidos antes del fin del periodo hasta las facturas de proveedores relacionadas y el listado de cuentas por pagar.

41. Un equipo de auditoría interna está realizando

una auditoría de debido cuidado (due diligence) a fin de evaluar planes para una posible fusión/adquisición. ¿Cuál de las siguientes sería la razón menos válida para que una compañía adquiera o se fusione con otra? a. Para diversificar los riesgos. b. Para responder a una política del

gobierno. c. Para reducir los costos de mano de obra. d. Para incrementar los precios de las

acciones.

40. An engagement objective is to determine if a company’s accounts payable contain all outstanding liabilities. Which of the following audit procedures would not be relevant for this objective? a. Examine supporting documentation of

subsequent (after-period) cash disbursements and verify period of liability.

b. Send confirmations, including zero-balance accounts, to vendors with whom the company normally does business.

c. Select a sample of accounts payable from the accounts payable listing and verify the supporting receiving reports, purchase orders, and invoices.

d. Trace receiving reports issued before the period end to the related vendor invoices and accounts payable listing.

41. An internal audit team is performing a due

diligence audit to assess plans for a potential merger/acquisition. Which of the following would be the least valid reason for a company to merge with or acquire another company? a. To diversify risk. b. To respond to government policy. c. To reduce labor costs. d. To increase stock prices.

II - 16

42. ¿Cuál de los siguientes procedimientos resultaría apropiado para probar si el incremento de los costos de un proyecto de construcción fue provocado por el contratista al querer justificar inadecuadamente que dichos incrementos se originaron por pedidos de cambios en el contrato?

I. Verificar que el contratista no haya

cargado costos ya facturados en el contrato original simulando que se originaron por los pedidos de cambios.

II. Determinar si el contratista facturó por trabajos detallados en el contrato original que se cancelaron como resultado de los pedidos de cambios.

III. Verificar que los pedidos de cambios hayan sido aprobados por la gerencia.

a. I solamente. b. III solamente. c. I y II solamente. d. I y III solamente.

43. ¿Cuál de los siguientes sería el punto de

partida más apropiado para una evaluación de cumplimiento de requerimientos de la licencia del software para una organización con más de 15.000 puestos de trabajo de sistemas? a. Determinar si la instalación del software

se controla centralmente o se distribuye en toda la organización.

b. Determinar qué paquetes de software se instalaron en las computadoras de la organización y el número de cada paquete instalado.

c. Determinar cuántas copias de cada paquete de software compró la organización.

d. Determinar qué mecanismos se instalaron para supervisar el uso del software.

42. Which of the following procedures would be appropriate for testing whether cost overruns on a construction project were caused by the contractor improperly accounting for costs related to contract change orders?

I. Verify that the contractor has not charged

change orders with costs that have already been billed to the original contract.

II. Determine if the contractor has billed for original contract work that was canceled as a result of change orders.

III. Verify that the change orders were properly approved by management.

a. I only. b. III only. c. I and II only. d. I and III only.

43. Which of the following would be the most appropriate starting point for a compliance evaluation of software licensing requirements for an organization with more than 15,000 computer workstations? a. Determine if software installation is

controlled centrally or distributed throughout the organization.

b. Determine what software packages have been installed on the organization’s computers and the number of each package installed.

c. Determine how many copies of each software package have been purchased by the organization.

d. Determine what mechanisms have been installed for monitoring software usage.

II - 17

44. Un departamento de transporte lleva el inventario de vehículos y sus registros de mantenimiento en una base de datos. ¿Cuál de los siguientes procedimientos de auditoría es más apropiado para evaluar la exactitud de la información de la base de datos? a. Verificar una muestra de los registros

extraídos de la base de datos mediante documentación soporte.

b. Remitir lotes de transacciones de prueba a través del sistema actual y verificarlos con los resultados esperados.

c. Simular el procesamiento normal mediante el uso de programas de prueba.

d. Utilizar el rastreo de los programas para mostrar cómo y en qué secuencia se procesan las instrucciones del programa en el sistema.

45. Las auditorías de desarrollo de sistemas

incluyen revisiones en varios puntos para asegurar que el desarrollo se encuentre controlado y administrado en forma apropiada. Las revisiones deberán incluir todo lo siguiente excepto: a. Conducir un estudio de factibilidad

técnica respecto del software, del hardware y de los recursos técnicos disponibles.

b. Examinar el nivel de participación del usuario en cada etapa de la implementación.

c. Verificar el uso de controles y técnicas de aseguramiento de calidad para el desarrollo, la conversión y la prueba de los programas.

d. Determinar si el sistema, el usuario y la documentación de las operaciones se ajustan a las Normas formales.

46. ¿Cuál de las siguientes opciones debe

revisarse antes de diseñar cualquier elemento del sistema que utilice un enfoque “top down” (de arriba hacia abajo) para desarrollar nuevos sistemas? a. Los tipos de sistemas de procesamiento

utilizados por los competidores. b. El equipo de computación necesario para

el sistema. c. Las necesidades de información de los

gerentes para la planificación y el control. d. Los controles que se apliquen al sistema

en uso.

44. A transportation department maintains its vehicle inventory and maintenance records in a database. Which of the following audit procedures is most appropriate for evaluating the accuracy of the database information? a. Verify a sample of the records extracted

from the database with supporting documentation.

b. Submit batches of test transactions through the current system and verify with expected results.

c. Simulate normal processing by using test programs.

d. Use program tracing to show how, and in what sequence, program instructions are processed in the system.

45. Systems development audit engagements

include reviews at various points to ensure that development is properly controlled and managed. The reviews should include all of the following except: a. Conducting a technical feasibility study on

the available hardware, software, and technical resources.

b. Examining the level of user involvement at each stage of implementation.

c. Verifying the use of controls and quality assurance techniques for program development, conversion, and testing.

d. Determining if system, user, and operations documentation conforms to formal standards.

46. Which of the following should be reviewed

before designing any system elements in a top-down approach to new systems development? a. Types of processing systems used by

competitors. b. Computer equipment needed by the

system. c. Information needs of managers for

planning and control. d. Controls in place over the current system.

II - 18

47. Al conducir una auditoría de la capacidad de recuperación de desastres de una organización, ¿cuál de las siguientes consideraría un auditor la debilidad más seria? a. Las pruebas utilizan documentos de

recuperación. b. Los contratos tienen dos años de

antigüedad. c. Los medios de respaldo se almacenan en

el lugar correspondiente. d. Sólo algunos sistemas se prueban

anualmente. 48. Si se utilizan transferencias electrónicas de

fondos (ETF) para pagar facturas de proveedores, ¿cuál de los siguientes procedimientos de auditoría asistidas por computador utilizaría un auditor para determinar si se hicieron pagos duplicados?

I. Identificación de transacciones ETF para

el mismo proveedor por el mismo importe. II. Extracción de transacciones ETF con

códigos de proveedor no autorizados. III. Prueba de razonabilidad de las

transacciones ETF. IV. Búsqueda de transacciones ETF con

números duplicados de órdenes de compra.

a. I y II solamente. b. I y IV solamente. c. II y III solamente. d. III y IV solamente.

49. Durante una auditoría operativa, un auditor

comparó la tasa de rotación de inventarios de una subsidiaria con los estándares establecidos para este tipo de actividad con el fin de: a. Evaluar la exactitud de los informes

financieros internos. b. Probar los controles diseñados para

salvaguardar los activos. c. Determinar el cumplimiento de los

procedimientos corporativos con respecto a niveles de inventarios.

d. Evaluar el desempeño e indicar dónde se puede necesitar un trabajo adicional de auditoría.

47. In conducting an audit of an organization’s disaster recovery capability, which of the following would an auditor consider to be the most serious weakness? a. Tests utilize recovery scripts. b. Hot-site contracts are two years old. c. Backup media are stored on-site. d. Only a few systems are tested annually.

48. If electronic funds transfer (EFT) is used to pay

vendor invoices, which of the following computer-assisted audit procedures would an auditor use to determine if any payments were made twice? I. Identification of EFT transactions to the

same vendor for the same dollar amount. II. Extraction of EFT transactions with

unauthorized vendor codes. III. Testing of EFT transactions for

reasonableness. IV. Searching for EFT transactions with

duplicate purchase order numbers. a. I and II only. b. I and IV only. c. II and III only. d. III and IV only.

49. During an operational audit engagement, an

auditor compared the inventory turnover rate of a subsidiary with established industry standards in order to: a. Evaluate the accuracy of internal financial

reports. b. Test controls designed to safeguard

assets. c. Determine compliance with corporate

procedures regarding inventory levels. d. Assess performance and indicate where

additional audit work may be needed.

II - 19

50. Una organización utiliza intercambio electrónico de datos (EDI) y sistemas en línea en lugar de generar documentos en papel para las órdenes de compra, los informes de recepción y las facturas. ¿Cuál de los siguientes procedimientos de auditoría utilizaría un auditor para determinar si las facturas se pagan exclusivamente por los bienes recibidos y a los precios aprobados? a. Utilizar una muestra estadística de los

principales proveedores y buscar los importes pagados por facturas específicas.

b. Usar un software genérico de auditoría para seleccionar una muestra de los pagos y comparar órdenes de compra, facturas e informes de recepción guardados en la computadora, utilizando una referencia común.

c. Tomar una muestra de unidades monetarias de las cuentas por pagar y confirmar los importes directamente con los proveedores

d. Utilizar un software genérico de auditoría para identificar todos los recibos de un día en particular y rastrear los informes de recepción correspondientes con los cheques que se emitieron.

51. ¿Cuál de las siguientes afirmaciones es cierta

sobre benchmarking? a. Suele realizarse comparando el

desempeño de la organización con el de sus principales competidores.

b. Puede realizarse tanto mediante comparaciones cualitativas como cuantitativas.

c. Suele encontrarse limitado a las operaciones de manufactura y los procesos de producción.

d. Suele realizarse comparando el desempeño de una organización con el de las mejores organizaciones.

50. An organization uses electronic data interchange and on-line systems rather than paper-based documents for purchase orders, receiving reports, and invoices. Which of the following audit procedures would an auditor use to determine if invoices are paid only for goods received and at approved prices? a. Select a statistical sample of major

vendors and trace the amounts paid to specific invoices.

b. Use generalized audit software to select a sample of payments and match purchase orders, invoices, and receiving reports stored on the computer using a common reference.

c. Select a monetary-unit sample of accounts payable and confirm the amounts directly with the vendors.

d. Use generalized audit software to identify all receipts for a particular day and trace the receiving reports to checks issued.

51. Which of the following is true of

benchmarking? a. It is typically accomplished by comparing

an organization’s performance with the performance of its closest competitors.

b. It can be performed using either qualitative or quantitative comparisons.

c. It is normally limited to manufacturing operations and production processes.

d. It is accomplished by comparing an organization’s performance to that of the best-performing organizations.

II - 20

52. Si una institución financiera estuviera exagerando sus ingresos al considerar la mayor parte de los préstamos como intereses y sólo una parte menor como amortización del capital, ¿cuál de los siguientes procedimientos de auditoría sería el menos eficaz para detectar este error? a. Llevar a cabo una revisión analítica

comparando el ingreso por intereses de este período como un porcentaje de la cartera de préstamos con el mismo porcentaje del período previo.

b. Utilizar una instalación integrada de pruebas (ITF, por sus siglas en inglés) y remitir pagos de intereses correspondientes a varios préstamos de la cartera de la ITF para determinar si están registrados correctamente.

c. Utilizar datos de prueba y remitir pagos de intereses correspondientes a varios préstamos de la cartera de prueba para determinar si están registrados correctamente.

d. Utilizar un software genérico de auditoría para tomar una muestra aleatoria de los pagos de préstamos realizados durante el período, calculando los importes correctos por registrar y rastrear los registros que se hicieron con las diversas cuentas.

53. Un hospital está evaluando la compra de

software para integrar un nuevo sistema contable de costos con su sistema contable financiero existente. ¿Cuál de las siguientes opciones describe la manera más eficaz en que la actividad de auditoría interna puede participar en el proceso de compras? a. La actividad de auditoría interna evalúa si

las especificaciones de desempeño son coherentes con las necesidades del hospital.

b. La actividad de auditoría interna evalúa si el diseño de aplicación cumple con las Normas de documentación y desarrollo internos.

c. La actividad de auditoría interna determina si se valida y revisa el modelo prototipo con los usuarios antes de que comience el uso de producción.

d. La actividad de auditoría interna no tiene participación alguna, pues el sistema ya se desarrolló externamente.

52. If a financial institution overstated revenue by charging too much of each loan payment to interest income and too little to repayment of principal, which of the following audit procedures would be least likely to detect the error? a. Performing an analytical review by

comparing interest income this period as a percentage of the loan portfolio with the interest income percentage for the prior period.

b. Using an integrated test facility (ITF) and submitting interest payments for various loans in the ITF portfolio to determine if they are recorded correctly.

c. Using test data and submitting interest payments for various loans in the test portfolio to determine if they are recorded correctly.

d. Using generalized audit software to select a random sample of loan payments made during the period, calculating the correct posting amounts, and tracing the postings that were made to the various accounts.

53. A hospital is evaluating the purchase of

software to integrate a new cost accounting system with its existing financial accounting system. Which of the following describes the most effective way for the internal audit activity to be involved in the procurement process? a. The internal audit activity evaluates

whether performance specifications are consistent with the hospital's needs.

b. The internal audit activity evaluates whether the application design meets internal development and documentation standards.

c. The internal audit activity determines whether the prototyped model is validated and reviewed with users before production use begins.

d. The internal audit activity has no involvement since the system has already been developed externally.

II - 21

54. La gerencia senior de una entidad solicitó que la actividad de auditoría interna ofreciera capacitación continua sobre control interno para todo el personal gerencial, lo cual se aborda mejor mediante: a. Un contrato formal de trabajo de

consultoría. b. Un contrato informal de trabajo de

consultoría. c. Un contrato especial de trabajo de

consultoría. d. Un contrato de emergencia de trabajo de

consultoría. 55. Una actividad de consultoría apropiadamente

desempeñada por la función de auditoría interna implica: a. Diseñar sistemas de control. b. Redactar un borrador de los

procedimientos para los sistemas de control.

c. Revisar sistemas de control antes de su implementación.

d. Instalar sistemas de control. 56. Como parte de una investigación preliminar de

la función de compras, un auditor leyó el manual departamental de políticas y procedimientos concluyendo que éste describía correctamente las etapas del proceso y que también contenía un diseño adecuado del control interno. El siguiente objetivo de la auditoría era determinar la eficacia operativa de los controles internos. ¿Cuál procedimiento resultaría más apropiado para cumplir con este objetivo de auditoría? a. Preparar un diagrama de flujo. b. Preparar un narrativo del sistema. c. Llevar a cabo una prueba de controles. d. Llevar a cabo una prueba sustantiva.

57. Un auditor está revisando un plan de la

organización para desarrollar una tarjeta de registro de desempeño. ¿Cuál de las siguientes medidas posibles debe recomendar el auditor que se excluya de la tarjeta de registro de desempeño? a. Innovación de los productos. b. Participación en el Mercado. c. Satisfacción del cliente. d. Desarrollo del empleado.

54. Senior management of an entity has requested that the internal audit activity provide ongoing internal control training for all managerial personnel. This is best addressed by: a. A formal consulting engagement

agreement. b. An informal consulting engagement

agreement. c. A special consulting engagement

agreement. d. An emergency consulting engagement

agreement.

55. A consulting activity appropriately performed by the internal audit function is: a. Designing systems of control. b. Drafting procedures for systems of

control. c. Reviewing systems of control before

implementation. d. Installing systems of control.

56. As part of a preliminary survey of the

purchasing function, an auditor read the department's policies and procedures manual. The auditor concluded that the manual described the processing steps well and contained an appropriate internal control design. The next engagement objective was to determine the operating effectiveness of internal controls. Which procedure would be most appropriate in meeting this objective? a. Prepare a flowchart. b. Prepare a system narrative. c. Perform a test of controls. d. Perform a substantive test.

57. An auditor reviewing an organization’s plan for

developing a performance scorecard. Which of the following potential performance measures should the auditor recommend excluding from the performance scorecard? a. Product innovation. b. Market share. c. Customer satisfaction. d. Employee development.

II - 22

58. Un departamento de ventas ha distribuido gratuitamente artículos costosos junto con las ventas de nuevos productos, a fin de estimular la demanda. La promoción parece exitosa, pero la gerencia piensa que el costo puede ser demasiado elevado y solicitó una revisión por parte de la actividad de auditoría interna. ¿Cuál de los siguientes procedimientos de auditoría resultaría menos útil para determinar la eficacia de la promoción? a. Comparar las ventas de los productos

realizadas durante el período promocional con las ventas realizadas durante un período no promocional similar.

b. Comparar el costo unitario de los productos vendidos antes y durante el período promocional.

c. Analizar el ingreso y el costo marginales durante el período promocional, comparado con el período anterior a la promoción.

d. Revisar los parámetros que tiene el departamento de ventas para determinar que la promoción fue un éxito.

59. Los representantes de ventas de una

compañía manufacturera reciben un reintegro del 100 por ciento de las facturas de sus teléfonos celulares. Los costos varían de manera significativa de un representante a otro y de un mes a otro, lo cual complica los procesos de presupuesto y pronóstico. La gerencia solicitó que los auditores internos desarrollaran un método para controlar dichos costos. ¿Cuál de las siguientes opciones se incluiría de manera más apropiada en el alcance del proyecto de consultoría? a. Realizar una auto-evaluación de control

que incluya a los representantes de ventas.

b. Desarrollar un benchmarking con otros usuarios de teléfonos celulares.

c. Realizar una revisión en el proceso del negocio de las rutinas de compras y obligaciones por liquidar.

d. Considerar las medidas de desempeño y el diseño de los procesos de presupuesto y pronóstico.

60. Una organización desea mejorar sus medidas

de desempeño para una nueva línea de negocio. ¿Qué tipo de benchmarking es más probable a fin de suministrar información útil para este propósito? a. Funcional. b. Competitivo. c. Genérico. d. Interno.

58. A sales department has been giving away expensive items in conjunction with new product sales to stimulate demand. The promotion seems successful, but management believes the cost may be too high and has asked for a review by the internal audit activity. Which of the following procedures would be the least useful to determine the effectiveness of the promotion? a. Comparing product sales during the

promotion period with sales during a similar non-promotion period.

b. Comparing the unit cost of the products sold before and during the promotion period.

c. Performing an analysis of marginal revenue and marginal cost for the promotion period, compared to the period before the promotion.

d. Performing a review of the sales department’s benchmarks used to determine the success of a promotion.

59. Sales representatives for a manufacturing

company are reimbursed for 100 percent of their cellular telephone bills. Cellular telephone costs vary significantly from representative to representative and from month to month, complicating the budgeting and forecasting processes. Management has requested that the internal auditors develop a method for controlling these costs. Which of the following would most appropriately be included in the scope of the consulting project? a. Control self-assessment involving sales

representatives. b. Benchmarking with other cellular

telephone users. c. Business process review of procurement

and payables routines. d. Performance measurement and design of

the budgeting and forecasting processes. 60. An organization wants to improve on its

performance measures for a new business line. Which type of benchmarking is most likely to provide information useful for this purpose? a. Functional. b. Competitive. c. Generic. d. Internal.

II - 23

61. Uno de los objetivos de auditoría para una empresa manufacturera es verificar que todos los reprocesos sean revisados por el ingeniero de producción. ¿Cuál de los siguientes procedimientos de auditoría proporcionaría la mejor evidencia para cumplir este objetivo? a. Rastrear una muestra de entradas en la

bitácora de reprocesos hasta la acción correctiva que se haya tomado.

b. Rastrear una muestra de órdenes de reproceso hasta las entradas en la bitácora de reprocesos.

c. Rastrear una muestra de entradas en la bitácora de revisiones hasta las órdenes de reproceso.

d. Rastrear una muestra de órdenes de reproceso hasta las entradas en la bitácora de revisiones.

62. En una auditoría de cuentas por pagar se

descubrió que los individuos responsables de mantener el archivo maestro de proveedores podían también ingresar las facturas de los proveedores en el sistema. Durante la reunión de cierre, la gerencia aceptó corregir este problema. Cuando desarrolle un trabajo de seguimiento de cuentas por pagar, el auditor debe constatar que la gerencia: a. Haya transferido a los individuos que

mantenían el archivo maestro de proveedores a otro departamento para garantizar que se hayan segregado las responsabilidades de manera apropiada.

b. Haya comparado los archivos maestros de los proveedores y empleados para determinar si se agregó algún proveedor no autorizado.

c. Haya modificado el sistema de controles de acceso para evitar que los empleados ingresen facturas y aprueben pagos.

d. Haya modificado el sistema de cuentas por pagar a fin de evitar que los individuos que mantenían el archivo maestro de proveedores ingresen facturas.

61. One of the audit objectives for a manufacturing company is to verify that all rework is reviewed by the production engineer. Which of the following audit procedures would provide the best evidence for meeting this objective? a. Trace a sample of entries in the rework

log to remedial action taken. b. Trace a sample of rework orders to

entries in the rework log. c. Trace a sample of entries in the review

log to rework orders. d. Trace a sample of rework orders to

entries in the review log. 62. An audit of accounts payable found that the

individuals responsible for maintaining the vendor master file could also enter vendor invoices into the accounts payable system. During the exit conference, management agreed to correct this problem. When performing a follow-up engagement of accounts payable, the auditor should expect to find that management had: a. Transferred the individuals who

maintained the vendor master file to another department to ensure responsibilities were appropriately segregated.

b. Compared the vendor and employee master files to determine if any unauthorized vendors had been added to the vendor master file.

c. Modified the access control system to prevent employees from both entering invoices and approving payments.

d. Modified the accounts payable system to prevent individuals who maintained the vendor master file from entering invoices.

II - 24

63. En una auditoría se descubrió que el costo de algunos materiales instalados en los proyectos de capital se transfirió a la cuenta de inventarios porque se excedió el presupuesto de capital. ¿Cuál de las siguientes sería una técnica apropiada para que utilice la actividad de auditoría interna a fin de supervisar esta situación? a. Identificar varianzas entre las cuentas

capitalizadas todos los meses y el presupuesto de capital.

b. Analizar una muestra de transacciones de capital de cada trimestre para detectar casos en los cuales el material instalado se haya transferido al inventario.

c. Revisar todos los asientos de diario que reflejen que se transfirieron costos de las cuentas de capital a las de inventario.

d. Comparar los recibos de inventario con los débitos realizados a la cuenta de inventario e investigar las discrepancias.

64. Una auditoría del departamento de quejas de

una organización determinó que se había emitido gran cantidad de pagos duplicados debido a problemas en el sistema de procesamiento. Durante la reunión de cierre, el vicepresidente del departamento en cuestión informó a los auditores que de inmediato se intentaría recuperar los pagos duplicados y que se intensificaría el sistema de procesamiento en seis meses para corregir los problemas. Sobre la base de esta respuesta, el director ejecutivo de auditoría debe: a. Ajustar el alcance de la próxima auditoría

del departamento de quejas regularmente programada para evaluar los controles aplicados dentro del sistema de procesamiento.

b. Supervisar el estado de las acciones correctivas y programar un trabajo de seguimiento cuando corresponda.

c. Programar un trabajo de seguimiento en seis meses para evaluar el estado de las acciones correctivas.

d. Debatir los hallazgos con el comité de auditoría y solicitarle que determine la acción de seguimiento apropiada.

63. An audit found that the cost of some material installed on capital projects had been transferred to the inventory account because the capital budget had been exceeded. Which of the following would be an appropriate technique for the internal audit activity to use to monitor this situation? a. Identify variances between amounts

capitalized each month and the capital budget.

b. Analyze a sample of capital transactions each quarter to detect instances in which installed material was transferred to inventory.

c. Review all journal entries that transferred costs from capital to inventory accounts.

d. Compare inventory receipts with debits to the inventory account and investigate discrepancies.

64. An audit of an organization’s claims

department determined that a large number of duplicate payments had been issued due to problems in the claims processing system. During the exit conference, the vice president of the claims department informed the auditors that attempts to recover the duplicate payments would be initiated immediately and that the claims processing system would be enhanced within six months to correct the problems. Based on this response, the chief audit executive should: a. Adjust the scope of the next regularly

scheduled audit of the claims department to assess controls within the claims processing system.

b. Monitor the status of corrective action and schedule a follow-up engagement when appropriate.

c. Schedule a follow-up engagement within six months to assess the status of corrective action.

d. Discuss the findings with the audit committee and ask the committee to determine the appropriate follow-up action.

II - 25

65. Como parte de un programa de auto-inspección de protección ambiental, seguridad y salud, quienes conducen las inspecciones son un miembro del personal del protección ambiental, seguridad y salud y el gerente operativo de un área de trabajo determinada. Si no puede corregirse una deficiencia de inmediato, el miembro del personal del protección ambiental, seguridad y salud la ingresa a una base de datos accesible para todos los departamentos mediante una LAN o red del área local. El gerente del protección ambiental, seguridad y salud utiliza la base datos para proveer a la gerencia senior informes trimestrales de las actividades con respecto a las acciones correctivas. Durante la revisión del programa de auto-inspección, un auditor observa que el gerente operativo ingresa la información de cierre y afirma que ya finalizó la acción correctiva. ¿Qué cambio en el sistema de control compensaría este posible conflicto de intereses? a. No resulta necesario ningún control

adicional porque la gerencia senior revisa el informe trimestral y provee una vigilancia adecuada en esta situación.

b. No resulta necesario ningún control adicional porque los que implementan una acción correctiva se encuentran en la mejor posición para evaluar si resultó adecuada y si finalizó esa acción.

c. Después de ingresar el cierre en el sistema, es necesario verificarlo solicitando la revisión por parte del miembro del personal del protección ambiental, seguridad y salud del equipo de inspección original.

d. El secretario del departamento del protección ambiental, seguridad y salud debe responsabilizarse de ingresar toda la información del sistema de rastreo sobre la base de los memos redactados por el gerente operativo.

65. As part of a manufacturing company’s environmental, health, and safety (EHS) self-inspection program, inspections are conducted by a member of the EHS staff and the operational manager for a given work area or building. If a deficiency cannot be immediately corrected, the EHS staff member enters it into a tracking database that is accessible to all departments via a local area network. The EHS manager uses the database to provide senior management with quarterly activity reports regarding corrective action. During review of the self-inspection program, an auditor notes that the operational manager enters the closure information and affirms that corrective action is complete. What change in the control system would compensate for this potential conflict of interest? a. No additional control is needed because

the quarterly report is reviewed by senior management, providing adequate oversight in this situation.

b. No additional control is needed because those implementing a corrective action are in the best position to evaluate the adequacy and completion of that action.

c. After closure is entered into the system, review by the EHS staff member of the original inspection team should be required in order to verify closure.

d. The EHS department secretary should be responsible for entering all information in the tracking system based on memos from the operational manager.

II - 26

66. ¿Cuál de las siguientes declaraciones describe mejor la responsabilidad de la función de auditoría interna para actividades de seguimiento relacionadas con un trabajo anterior? a. Los auditores internos deben determinar

si se tomó acción correctiva y están logrando los resultados deseados o si la gerencia asumió el riesgo de no tomar dicha acción correctiva.

b. Los auditores internos deben determinar si la gerencia inició la acción correctiva, pero no son responsables de determinar si con dicha acción se están logrando los resultados deseados. Esa determinación es responsabilidad de la gerencia.

c. El director ejecutivo de auditoría es responsable de programar actividades de seguimiento sólo si la gerencia senior o el comité de auditoría le da instrucciones para hacerlo. De lo contrario, el seguimiento es totalmente discrecional.

d. Ninguna de las anteriores. 67. ¿Cuál de las siguientes describe la acción por

tomar más apropiada con respecto a una observación repetida de violaciones a la política de la compañía sobre licitaciones competitivas? a. La comunicación final del trabajo debe

hacer notar que esta observación se hizo en una auditoría anterior.

b. Durante la reunión de cierre de auditoría, debe informarse a la gerencia que la observación no fue corregida.

c. El director ejecutivo de auditoría debe determinar si la gerencia asumió el riesgo de no tomar acciones correctivas.

d. El director ejecutivo de auditoría debe determinar si debe informarse esta situación al auditor externo y a cualquier organismo regulador.

68. Cuando se conduce el seguimiento de

auditoría de un hallazgo relacionado con las rutinas de administración de caja, ¿cuál de las siguientes no resulta necesario considerar? a. Se eliminó el riesgo inherente como

resultado de haber resuelto la situación. b. Con los pasos tomados se está

solucionando la situación revelada en el hallazgo.

c. Se implementaron controles para impedir o detectar la repetición del hallazgo.

d. Se acumularon beneficios como resultado de haber resuelto la situación.

66. Which of the following statements best describes the internal audit function’s responsibility for follow-up activities related to a previous engagement? a. Internal auditors should determine if

corrective action has been taken and is achieving the desired results or if management has assumed the risk of not taking the corrective action.

b. Internal auditors should determine if management has initiated corrective action, but they have no responsibility to determine if the action is achieving the desired results. That determination is management's responsibility.

c. The chief audit executive is responsible for scheduling follow-up activities only if directed to do so by senior management or the audit committee. Otherwise, follow-up is entirely discretionary.

d. None of the above. 67. Which of the following describes the most

appropriate action to be taken concerning a repeated observation of violations of company policy pertaining to competitive bidding? a. The engagement final communication

should note that this same condition had been reported in the prior engagement.

b. During the exit interview, management should be made aware that the violation has not been corrected.

c. The chief audit executive should determine whether management or the board has assumed the risk of not taking corrective action.

d. The chief audit executive should determine whether this condition should be reported to the external auditor and any regulatory agency.

68. When conducting audit follow-up of a finding

related to cash management routines, which of the following does not need to be considered? a. Inherent risk has been eliminated as a

result of resolution of the condition. b. The steps being taken are resolving the

condition disclosed by the finding. c. Controls have been implemented to deter

or detect a recurrence of the finding. d. Benefits have accrued to the entity as a

result of resolving the condition.

II - 27

69. A través de una revisión de seguimiento se descubrió que no se había corregido una significativa debilidad de control interno. El director ejecutivo de auditoría conversó sobre esta cuestión con la gerencia senior, que le expresó al director su voluntad de aceptar el riesgo. El director ejecutivo de auditoría: a. No debe hacer nada más porque la

gerencia es responsable de decidir tomar la acción apropiada como respuesta a las observaciones y recomendaciones informadas del trabajo.

b. Debe iniciar una investigación de fraude para determinar si los empleados aprovecharon la deficiencia de control interno.

c. Debe informar a la gerencia senior que resulta necesario corregir la debilidad y programar otra revisión de seguimiento.

d. Debe evaluar las razones por las cuales la gerencia senior decidió aceptar el riesgo e informar su decisión al consejo.

70. Un comité de auditoría se muestra preocupado

porque la gerencia no está abordando todas las observaciones y recomendaciones de auditoría interna. ¿Qué debe hacer el comité de auditoría para abordar esta situación? a. Solicitar a los gerentes que suministren

planes de acción detallados con fechas específicas para abordar las observaciones y recomendaciones de auditoría.

b. Solicitar a todos los gerentes que confirmen cuándo tomaron acción.

c. Solicitar que el director ejecutivo de auditoría informe por qué no se tomó acción.

d. Solicitar al director ejecutivo de auditoría que establezca procedimientos para supervisar el progreso.

71. Al interrogar a individuos sospechosos de

fraude se debe: a. Cumplir con una orden predeterminada. b. Cubrir más de un tema o tópico. c. Ir de lo general a lo específico. d. Llevar al individuo a dar una respuesta

deseada.

69. A follow-up review found that a significant internal control weakness had not been corrected. The chief audit executive (CAE) discussed this matter with senior management and was informed of management’s willingness to accept the risk. The CAE should: a. Do nothing further because management

is responsible for deciding the appropriate action to be taken in response to reported engagement observations and recommendations.

b. Initiate a fraud investigation to determine if employees had taken advantage of the internal control weakness.

c. Inform senior management that the weakness must be corrected and schedule another follow-up review.

d. Assess the reasons that senior management decided to accept the risk and inform the board of senior management’s decision.

70. An audit committee is concerned that

management is not addressing all internal audit observations and recommendations. What should the audit committee do to address this situation? a. Require managers to provide detailed

action plans with specific dates for addressing audit observations and recommendations.

b. Require all managers to confirm when they have taken action.

c. Require the chief executive officer to report why action has not been taken.

d. Require the chief audit executive to establish procedures to monitor progress.

71. Questions used to interrogate individuals

suspected of fraud should: a. Adhere to a predetermined order. b. Cover more than one subject or topic. c. Move from the general to the specific. d. Direct the individual to a desired answer.

II - 28

72. Un auditor de una compañía importante de ventas al por menor sospecha que existe fraude en el inventario de tres tiendas cuyo costo de ventas es alto. ¿Cuál de las siguientes actividades de auditoría proporcionaría la evidencia más persuasiva de lo anterior? a. Utilizar una instalación integrada de

prueba (integrated test facility-ITF) para comparar las transacciones de ventas individuales con las transacciones de prueba enviadas a través de ITF. Investigar todas las diferencias.

b. Entrevistar a los tres gerentes de tienda para determinar si sus explicaciones respecto a las diferencias observadas son similares, y entonces comparar dichas explicaciones con las del gerente de sección.

c. Programar sorpresivamente una auditoría de inventarios que incluya una toma física de las existencias. Investigar las áreas de mermas de inventario.

d. Tomar una muestra de los precios de las tiendas en lo individual y compararlos con las ventas manejadas a través de la caja registradora, por esos mismos artículos.

73. Un director ejecutivo de auditoría (DEA)

sospecha que varios empleados utilizaron computadoras de escritorio para beneficio personal. Al conducir una investigación, la razón primordial por la cual el DEA eligió un auditor de sistemas de información legal es que dicho profesional poseería: a. Conocimientos del sistema

computarizado que le permitirían realizar una evaluación más amplia del uso y abuso de las computadoras.

b. Conocimientos de lo que constituye evidencia aceptable en una corte.

c. Destrezas analíticas superiores que facilitarían la identificación del abuso de las computadoras.

d. Destrezas para administrar documentación superior y actuar en la organización que faciliten la presentación de hallazgos a la gerencia senior y al consejo.

72. An auditor for a major retail company suspects that inventory fraud is occurring at three stores that have high cost of goods sold. Which of the following audit activities would provide the most persuasive evidence that fraud is occurring? a. Use an integrated test facility (ITF) to

compare individual sales transactions with test transactions submitted through the ITF. Investigate all differences.

b. Interview the three individual store managers to determine if their explanations about the observed differences are the same, and then compare their explanations to that of the section manager.

c. Schedule a surprise inventory audit to include a physical inventory. Investigate areas of inventory shrinkage.

d. Select a sample of individual store prices and compare them with the sales entered on the cash register for the same items.

73. A chief audit executive (CAE) suspects that

several employees have used desktop computers for personal gain. In conducting an investigation, the primary reason that the CAE chose to engage a forensic information systems auditor rather than using the organization’s information systems auditor is that a forensic information systems auditor would possess: a. Knowledge of the computing system that

would enable a more comprehensive assessment of the computer use and abuse.

b. Knowledge of what constitutes evidence acceptable in a court of law.

c. Superior analytical skills that would facilitate the identification of computer abuse.

d. Superior documentation and organization skills that would facilitate in the presentation of findings to senior management and the board.

II - 29

74. Si un auditor interno está entrevistando a tres individuos, uno de los cuales es sospechoso de haber cometido un fraude, ¿cuál de los siguientes constituye el enfoque menos eficaz? a. Pedir a cada individuo que prepare una

declaración escrita en la cual explique sus acciones.

b. Tomar el rol de alguien que busca la verdad.

c. Escuchar con atención lo que cada entrevistado tiene para decir.

d. Intentar lograr que el sospechoso confiese.

75. ¿Cuál de las siguientes acciones es más

probable que sea considerada como indicador de posible fraude? a. El reemplazo del equipo gerencial en

condiciones hostiles. b. Una rápida rotación de los ejecutivos

financieros de la organización. c. Una expansión rápida hacia nuevos

mercados. d. Una auditoría gubernamental de las

declaraciones de impuestos de la organización.

76. ¿Qué técnica de auditoría asistida por

computadora utilizaría un auditor para identificar a un empleado ficticio o que ya no trabaja para una compañía? a. Efectuar una simulación paralela de

cálculos de pagos por nómina. b. Realizar pruebas de excepción para las

deducciones de pagos por nómina. c. Volver a calcular los pagos netos. d. Codificar y rastrear los cambios de las

tasas de impuestos de los pagos por nómina.

74. If an internal auditor is interviewing three individuals, one of whom is suspected of committing a fraud, which of the following is the least effective approach? a. Ask each individual to prepare a written

statement explaining his or her actions. b. Take the role of one seeking the truth. c. Listen carefully to what each interviewee

has to say. d. Attempt to get the suspected individual to

confess. 75. Which of the following is most likely to be

considered an indication of possible fraud? a. The replacement of the management

team after a hostile takeover. b. Rapid turnover of the organization’s

financial executives. c. Rapid expansion into new markets. d. A government audit of the organization’s

tax returns. 76. What computer-assisted audit technique would

an auditor use to identify a fictitious or terminated employee? a. Parallel simulation of payroll calculations. b. Exception testing for payroll deductions. c. Recalculations of net pay. d. Tagging and tracing of payroll tax-rate

changes.

II - 30

77. ¿Cuál de las siguientes no se consideraría una situación que indique una mayor probabilidad de fraude? a. La gerencia delegó a sus subordinados la

autoridad de efectuar compras hasta un cierto límite monetario.

b. Una persona ocupó el mismo puesto de administrar dinero en efectivo por un largo período sin ninguna rotación de responsabilidades.

c. Una persona que maneja valores negociables es responsable de realizar las compras, registrarlas e informar a la gerencia sobre las ganancias, las pérdidas y cualquier anomalía.

d. La asignación de responsabilidades y la obligación de informar en el departamento de cuentas por cobrar no resulta clara.

78. La motivación más común para cometer

fraudes de tipo administrativo es la existencia de: a. Vicios, como el hábito del juego. b. Insatisfacción laboral. c. Presiones financieras en la organización. d. El reto de cometer el delito perfecto.

79. ¿Cuál de las siguientes situaciones indicarían

que algún fraude podría estarse perpetrando en un departamento de mercadotecnia? a. No existe documentación sobre gastos

por montos considerables, hechos con un nuevo proveedor.

b. Aparentemente existe un gerente cuyo estilo de vida es más elevado del que le podría proporcionar el sueldo de un gerente de mercadotecnia.

c. El ambiente de control puede describirse como “muy pobre”; sin embargo, esta actitud es justificada por la administración con el argumento de que así se requiere para efectos de propiciar la creatividad.

d. Todas las anteriores. 80. ¿Qué plan de muestreo no requiere de

muestras adicionales una vez que se encuentra el primer error? a. Muestreo estratificado. b. Muestreo de atributos. c. Muestreo de parar o seguir. d. Muestreo de descubrimiento.

77. Which of the following would not be considered a condition that indicates a higher likelihood of fraud? a. Management has delegated the authority

to make purchases under a certain dollar limit to subordinates.

b. An individual has held the same cash-handling job for an extended period without any rotation of duties.

c. An individual handling marketable securities is responsible for making the purchases, recording the purchases, and reporting any discrepancies and gains or losses to senior management.

d. The assignment of responsibility and accountability in the accounts receivable department is not clear.

78. The most common motivation for management

fraud is the existence of: a. Vices, such as a gambling habit. b. Job dissatisfaction. c. Financial pressures on the organization. d. The challenge of committing the perfect

crime. 79. Which of the following would indicate that fraud

may be taking place in a marketing department? a. There is no documentation for some fairly

large expenditures made to a new vendor. b. A manager appears to be living a lifestyle

that is in excess of what could be provided by a marketing manager's salary.

c. The control environment can best be described as "very loose." However, this attitude is justified by management on the grounds that it is needed for creativity.

d. All of the above. 80. Which sampling plan requires no additional

sampling once the first error is found? a. Stratified sampling. b. Attributes sampling. c. Stop-or-go sampling. d. Discovery sampling

II - 31

81. El auditor interno de un banco desea saber si todos los préstamos están suficientemente respaldados por una garantía, si su antigüedad está determinada en función de los pagos vencidos y si están clasificados adecuadamente como actuales o no actuales. El mejor procedimiento de auditoría para alcanzar este objetivo sería: a. Utilizar un software genérico de auditoría

para conocer el archivo de préstamos totales, clasificarlo según el último pago vencido y tomar una muestra estadística estratificada de la población antigua y actual. Examinar cada uno de los préstamos seleccionados en cuanto a su garantía adecuada y a su antigüedad.

b. Tomar una muestra de bloques de todos los préstamos que excedan de cierto importe y determinar si se encuentran clasificados en forma adecuada. Verificar la antigüedad y clasificación de cada uno de los préstamos aprobados.

c. Tomar una muestra de descubrimiento de todas las solicitudes de préstamos para determinar si cada una de ellas contiene una declaración de garantía.

d. Tomar una muestra de los pagos correspondientes efectuados en la cartera de préstamos y rastrearlos para verificar si los pagos se aplican correctamente. Por cada préstamo identificado, examinar la solicitud para determinar que ésta cuenta con una garantía adecuada.

82. Una diferencia importante entre un muestreo

estadístico y un muestreo de juicio es que con un muestreo estadístico: a. No se requiere juicio; todo se calcula por

medio de fórmulas. b. Se puede utilizar una muestra más

pequeña. c. Se obtienen resultados más precisos. d. Se pueden realizar estimaciones de la

población con una confiabilidad mensurable.

83. ¿En cuál de los siguientes planes de muestreo

la variabilidad de los valores monetarios de los elementos de una población afecta al tamaño de la muestra? a. Muestreo de atributos. b. Muestreo de unidades monetarias. c. Muestreo de la media por unidad. d. Muestreo de descubrimiento.

81. A bank internal auditor wishes to determine whether all loans are supported by sufficient collateral, properly aged regarding current payments, and accurately categorized as current or noncurrent. The best audit procedure to accomplish these objectives would be to: a. Use generalized audit software to read

the total loan file, age the file by last payment due, and extract a statistical sample stratified by the current and aged population. Examine each loan selected for proper collateralization and aging.

b. Select a block sample of all loans in excess of a specified dollar limit and determine if they are current and properly categorized. For each loan approved, verify aging and categorization.

c. Select a discovery sample of all loan applications to determine whether each application contains a statement of collateral.

d. Select a sample of payments made on the loan portfolio and trace them to loans to see if the payments are properly applied. For each loan identified, examine the loan application to determine that the loan has proper collateralization.

82. An important difference between a statistical

and a judgmental sample is that with a statistical sample: a. No judgment is required because

everything is computed according to a formula.

b. A smaller sample can be used. c. More accurate results are obtained. d. Population estimates with measurable

reliability can be made. 83. Variability of the dollar amount of individual

items in a population affects sample size in which of the following sampling plans? a. Attributes sampling. b. Dollar-unit sampling. c. Mean-per-unit sampling. d. Discovery sampling.

II - 32

84. El siguiente diagrama de barra X es un ejemplo de la salida de una aplicación de computadora utilizada por una compañía de seguros de salud a fin de supervisar los montos de las facturas de los médicos para diversos procedimientos quirúrgicos:

Los datos detallados en el diagrama representan: a. Una variable aleatoria. b. Una variable anormal. c. Una variable normal. d. Una variable cíclica.

85. El error estándar de una muestra refleja:

a. El error en la población proyectada basado en errores de la muestra.

b. El índice promedio de error en los elementos de la muestra.

c. El grado de varianza en los elementos de la muestra.

d. El error en la población que el auditor pueda aceptar.

86. Un auditor conduce una encuesta de

percepciones y creencias de los empleados con respecto al plan de cuidado de la salud de una organización. El mejor enfoque para seleccionar una muestra sería: a. Centrar la atención en las personas que

probablemente respondan, de manera tal que pueda obtenerse una muestra mayor.

b. Centrarse en los gerentes y supervisores porque pueden también reflejar las opiniones de los empleados de sus departamentos.

c. Utilizar muestras estratificadas en las cuales los estratos se definan por estado civil, datos de familia, edad y pago que reciben: sueldos o por hora.

d. Utilizar las muestras monetarias de las unidades de acuerdo con los sueldos de los empleados.

84. The following X-bar chart is an example of the output from a computer application used by a health insurance company to monitor physician bill amounts for various surgical procedures:

The data plotted on the chart represents: a. Random variation. b. Abnormal variation. c. Normal variation. d. Cyclic variation.

85. The standard error of a sample reflects:

a. The projected population error based on errors in the sample.

b. The average rate of error in the sample. c. The degree of variation in sample items. d. The error in the population that the auditor

can accept. 86. An auditor is conducting a survey of

perceptions and beliefs of employees concerning an organization’s health-care plan. The best approach to selecting a sample would be to: a. Focus on people who are likely to

respond so that a larger sample can be obtained.

b. Focus on managers and supervisors because they can also reflect the opinions of the people in their departments.

c. Use stratified sampling where the strata are defined by marital and family status, age, and salaried or hourly status.

d. Use monetary-unit sampling according to employee salaries.

Upper Limit XXXXX

XX

XXX X X XXXX

XX X XXX XXX Expected Mean X XXX X XXX X XXX

X X X XX XX X X X X

XLower Limit

Límite sup. XXX XX

XX

XXX X X XXXX

XX X XXX XXX Media esper. X XXX X XXX X XXX

X X X XX XX X X X X

X Límite inf.

II - 33

87. ¿Cuál de las siguientes no constituye una ventaja de enviar un cuestionario de control interno antes de un trabajo de auditoría? a. El cliente del trabajo puede utilizar el

cuestionario para una auto-evaluación antes de la visita del auditor.

b. El cuestionario ayudará al cliente a comprender el alcance del trabajo.

c. La preparación del cuestionario ayudará al auditor a planificar el alcance del trabajo y a organizar la información que deba recopilarse.

d. El cliente del trabajo responderá sólo las preguntas formuladas, sin dar información adicional.

88. ¿Cuál de las siguientes técnicas podría

utilizarse a fin de estimar el desvío estándar para un plan de muestreo? a. Estimación de proporciones. b. Prueba piloto. c. Regresión. d. Muestreo de descubrimiento.

89. Tal como se usa en la verificación del

programa de cuentas por pagar, ¿cuál de las siguientes cuestiones se describe mejor como una prueba analítica? a. Comparar las partidas del programa con

el mayor de cuentas a pagar o el archivo de facturas no pagadas.

b. Comparar los saldos del balance con los saldos del balance de años anteriores.

c. Comparar las confirmaciones recibidas de acreedores seleccionados con el mayor de cuentas por pagar.

d. Examinar las facturas de los proveedores que corresponden a los elementos seleccionados del programa.

90. El uso de una revisión analítica para verificar

la validez de varios gastos operativos no resultaría un enfoque ventajoso si: a. El auditor observara indicadores claros de

un fraude específico que involucre estas cuentas.

b. Las operaciones son relativamente estables y no cambiaron mucho el año pasado.

c. Un auditor deseara identificar las transacciones grandes, extraordinarias o inusuales que se realizaron durante el año.

d. Los gastos de operación varían en relación con otros gastos operativos, pero no en relación con los ingresos.

87. Which of the following is not an advantage of sending an internal control questionnaire prior to an audit engagement? a. The engagement client can use the

questionnaire for self-evaluation prior to the auditor’s visit.

b. The questionnaire will help the engagement client understand the scope of the engagement.

c. Preparing the questionnaire will help the auditor plan the scope of the engagement and organize the information to be gathered.

d. The engagement client will respond only to the questions asked, without volunteering additional information.

88. Which of the following techniques could be

used to estimate the standard deviation for a sampling plan? a. Ratio estimation. b. Pilot sample. c. Regression. d. Discovery sampling.

89. As used in the verification of an accounts

payable schedule, which of the following is best described as an analytical test? a. Comparing the items on the schedule with

the accounts payable ledger or unpaid voucher file.

b. Comparing the balance on the schedule with the balances of prior years.

c. Comparing confirmations received from selected creditors with the accounts payable ledger.

d. Examining vendors' invoices in support of selected items on the schedule.

90. The use of an analytical review to verify the

correctness of various operating expenses would not be a preferred approach if: a. An auditor notes strong indicators of a

specific fraud involving these accounts. b. Operations are relatively stable and have

not changed much over the past year. c. An auditor would like to identify large,

unusual, or non-recurring transactions during the year.

d. Operating expenses vary in relation to other operating expenses, but not in relation to revenue.

II - 34

91. ¿Cuál de los siguientes procedimientos analíticos debe utilizar un auditor para determinar si un cambio en el ingreso por inversiones que se produjo durante el año en curso se debió a cambios en la estrategia de inversión, cambios en la mezcla de carteras o a otros factores? a. Regresión lineal simple de los cambios en

el ingreso por inversiones ocurridos durante los últimos cinco años para determinar su naturaleza.

b. Análisis de las proporciones de los cambios ocurridos mes por mes en la cartera de inversiones.

c. Análisis de las tendencias de los cambios en el ingreso por inversiones como porcentaje del total de activos y los activos de inversión durante los últimos cinco años.

d. Análisis de regresión múltiple mediante el uso de variables independientes relacionadas con la naturaleza de la cartera de inversión y las condiciones del mercado.

92. ¿Cuál de los siguientes procedimientos

proporcionaría la mejor evidencia de la eficacia de la función de otorgamiento de créditos? a. Observar el proceso. b. Revisar la tendencia de las cancelaciones

de cuentas por cobrar. c. Preguntar al gerente de créditos sobre la

eficacia de la función. d. Verificar la evidencia de la aprobación de

créditos en una muestra de pedidos de clientes.

91. Which of the following analytical review procedures should an auditor use to determine if a change in investment income during the current year was due to changes in investment strategy, changes in portfolio mix, or other factors? a. Simple linear regression of investment

income changes over the past five years to determine the nature of the changes.

b. Ratio analysis of changes in the investment portfolio on a monthly basis.

c. Trend analysis of changes in investment income as a percentage of total assets and of investment assets over the past five years.

d. Multiple regression analysis using independent variables related to the nature of the investment portfolio and market conditions.

92. Which of the following procedures would provide the best evidence of the effectiveness of a credit-granting function? a. Observe the process. b. Review the trend in receivables write-offs. c. Ask the credit manager about the

effectiveness of the function. d. Check for evidence of credit approval on

a sample of customer orders.

II - 35

93. Un gerente de producción solicitó demasiadas materias primas para enviarlas a una compañía de su propiedad. El gerente falsificaba documentos de recepción y aprobaba las facturas para su pago. ¿Cuál de los siguientes procedimientos de auditoría tendría mayor probabilidad de detectar este fraude? a. Tomar una muestra de los pagos y

comparar las órdenes de compra, los informes de recepción, las facturas y las copias de los cheques.

b. Tomar una muestra de los pagos y confirmar con los proveedores los importes adquiridos, los precios de compra y las fechas de embarque.

c. Observar el área de recepción y contar el material recibido; comparar los conteos con los informes de recepción elaborados por el personal de recepción.

d. Realizar pruebas analíticas comparando la producción, el material adquirido y los niveles de inventario de las materias primas; investigar las diferencias.

94. Cuando se utiliza un proceso racional de toma

de decisiones, el próximo paso después de definir el problema es: a. Desarrollar soluciones alternativas. b. Identificar niveles aceptables de riesgo. c. Reconocer la brecha entre la realidad y

las expectativas. d. Confirmar las hipótesis.

95. Un diagrama de flujo de las actividades del

proceso y los controles puede proporcionar: a. Información respecto a dónde puede

ocurrir un fraude. b. Información respecto a la magnitud de un

fraude acaecido. c. Una indicación de dónde ha ocurrido un

fraude en algún proceso. d. Ninguna información relacionada con la

prevención de fraudes. 96. ¿Cuál de los siguientes factores resulta

menos necesario para lograr un grupo de trabajo que tenga éxito en la auto-evaluación de control? a. Tecnología para recuento de votos. b. Capacitación para facilitadores. c. Planificación previa. d. Dinámicas de grupo.

93. A production manager ordered excessive raw materials for delivery to a separate company owned by the manager. The manager falsified receiving documents and approved the invoices for payment. Which of the following audit procedures would most likely detect this fraud? a. Select a sample of cash disbursements

and compare purchase orders, receiving reports, invoices, and check copies.

b. Select a sample of cash disbursements and confirm the amount purchased, purchase price, and date of shipment with the vendors.

c. Observe the receiving dock and count materials received; compare the counts to receiving reports completed by receiving personnel.

d. Perform analytical tests, comparing production, materials purchased, and raw materials inventory levels; investigate differences.

94. When using a rational decision-making

process, the next step after definition of the problem is: a. Developing alternative solutions. b. Identifying acceptable levels of risk. c. Recognizing the gap between reality and

expectations. d. Confirming hypotheses.

95. A flowchart of process activities and controls

may provide: a. Information on where fraud could occur. b. Information on the extent of a past fraud. c. An indication of where fraud has occurred

in a process. d. No information related to fraud

prevention. 96. Which of the following factors is least essential

to a successful control self-assessment workshop? a. Voting technology. b. Facilitation training. c. Prior planning. d. Group dynamics.

II - 36

97. Una empresa manufacturera utiliza un sistema de planificación de requerimiento de materiales (MRP, por sus siglas en inglés) para rastrear el inventario, las órdenes y los requerimientos de materias primas. La evaluación de una auditoría preliminar indica que el inventario de la organización está reducido. Con el uso de un software de auditoría, ¿qué condiciones debe buscar un auditor en la base de datos de la MRP para respaldar esta hipótesis?

I. Items a costo cero. II. Cantidades negativas en stock. III. Cantidades pedidas que excedan los

requerimientos. IV. Tiempos de entrega que excedan los

estimados. a. I y II solamente. b. I y IV solamente. c. II y IV solamente. d. III y IV solamente.

98. Una organización provee tarjetas de crédito

para uso comercial a empleados seleccionados. La compañía de tarjetas de crédito provee un archivo de computadora de todas las transacciones realizadas por los empleados de la organización. Un auditor planifica utilizar un software genérico de auditoría para seleccionar las transacciones pertinentes a fin de realizar pruebas. ¿Cuál de las siguientes no se identificaría con facilidad mediante el uso del software genérico de auditoría? a. Transacciones de montos elevados. b. Transacciones fraudulentas. c. Transacciones para usuarios específicos

de la tarjeta. d. Proveedores utilizados por cada usuario

de la tarjeta.

97. A manufacturer uses a materials requirements planning (MRP) system to track inventory, orders, and raw material requirements. A preliminary audit assessment indicates that the organization’s inventory is understated. Using audit software, what conditions should the auditor search for in the MRP database to support this hypothesis? I. Item cost set at zero. II. Negative quantities on hand. III. Order quantity exceeding requirements. IV. Inventory lead times exceeding delivery

schedule. a. I and II only. b. I and IV only. c. II and IV only. d. III and IV only.

98. An organization provides credit cards to

selected employees for business use. The credit card company provides a computer file of all transactions by employees of the organization. An auditor plans to use generalized audit software to select relevant transactions for testing. Which of the following would not be readily identified using generalized audit software? a. High-dollar transactions. b. Fraudulent transactions. c. Transactions for specific cardholders. d. Suppliers used by each cardholder.

II - 37

99. Los auditores internos suelen hacer un diagrama de flujo del sistema de controles, en el cual crean referencias para las descripciones narrativas de ciertas actividades. Es un procedimiento apropiado para: a. Determinar si el sistema alcanza los

objetivos gerenciales establecidos. b. Documentar que el sistema cumple con

los requerimientos internacionales de auditoría.

c. Determinar si el sistema resulta confiable para producir información exacta.

d. Obtener la comprensión necesaria para probar la eficacia del sistema.

100. Comparado con un diagrama de flujo vertical,

¿cuál de las siguientes aseveraciones es cierta sobre los diagramas de flujo horizontales? a. Proporciona más espacio para

descripciones escritas que se relacionen con los símbolos.

b. Proporciona un enfoque más preciso de la asignación de tareas y de revisiones independientes de desempeño.

c. En general, es más largo. d. No cruza líneas departamentales.

FIN DE LAS PREGUNTAS PARTE II POR FAVOR OBSERVE: La parte II del examen CIA actual constará de 125 preguntas de examen. Entre las 125 preguntas, se incluirán hasta 25 preguntas no registradas, que se utilizarán para propósitos de investigación. Dichas preguntas no registradas se insertarán sin identificar entre las registradas. Por lo tanto, los candidatos deben responder a las 125 preguntas lo mejor posible.

99. Internal auditors often flowchart a control system and reference the flowchart to narrative descriptions of certain activities. This is an appropriate procedure to: a. Determine whether the system meets

established management objectives. b. Document that the system meets

international auditing requirements. c. Determine whether the system can be

relied upon to produce accurate information.

d. Gain the understanding necessary to test the effectiveness of the system.

100. Compared to a vertical flowchart, which of the

following is true of a horizontal flowchart? a. It provides more room for written

descriptions that parallel the symbols. b. It brings into sharper focus the

assignment of duties and independent checks on performance.

c. It is usually longer. d. It does not cross departmental lines.

END OF PART II QUESTIONS PLEASE NOTE: The actual CIA exam Part II will contain 125 exam questions. The 125 questions will include up to 25 unscored questions, which will be used for research purposes. These unscored questions will be interspersed with the scored questions and will not be identified as unscored questions. Candidates should therefore answer all 125 questions to the best of their ability.

II - 38

Soluciones a la

Parte II – Cómo conducir el trabajo de auditoría interna

Las soluciones y explicaciones sugeridas para la parte I de las preguntas del Examen modelo para Auditor Interno Certificado se proporcionan en las siguientes páginas.

La tabla que a continuación se presenta asocia los números de las preguntas de la parte I con los temas

evaluados:

Tema Evaluado Número de Pregunta

Conducción de trabajos 1 – 31

Conducción de trabajos específicos 32 – 61

Monitoreo de los resultados de los trabajos 62 – 70

Elementos del conocimiento del fraude 71 – 80

Herramientas de los trabajos 81 – 100

II - 39

1. Solución: b (I y IV solamente) I. Correcta. De acuerdo con la Norma de Implementación 2110.A2: “La actividad de auditoría

interna debe evaluar las exposiciones de riesgo referidas a gobierno, operaciones y sistemas de información de la organización, con relación a lo siguiente: • Confiabilidad e integridad de la información financiera y operativa, • Eficacia y eficiencia de las operaciones, • Protección de activos, • Cumplimiento de leyes, regulaciones y contratos.” El objetivo específico del trabajo de determinar si los productos se imputan a la cuenta apropiada abordarían el objetivo con respecto a la confiabilidad e integridad de la información.

II. Incorrecta. El objetivo específico del trabajo descripto no aborda el cumplimiento. III. Incorrecta. El objetivo específico del trabajo descripto puede abordar la eficacia de las

operaciones, pero no aborda la eficiencia. IV. Correcta. El objetivo específico del trabajo de determinar si se recibieron todos los productos

pagados abordaría la cuestión relacionada con la protección de activos. 2. Solución: a

a. Correcta. Los auditores deben mostrar lealtad a la organización, pero no deben participar en ninguna actividad ilegal. En consecuencia, los auditores deben respetar las citaciones legales.

b. Incorrecta. La Regla de Conducta 3.2 prohíbe a los auditores utilizar información de auditoría para beneficio personal.

c. Incorrecta. La Regla de Conducta 2.2 prohíbe a los auditores aceptar cualquier cosa que pueda perjudicar o que aparentemente pueda perjudicar su juicio profesional.

d. Incorrecta. La Regla de Conducta 1.3 prohíbe a los auditores participar a sabiendas de una actividad ilegal o de actividades inapropiadas. El Consejo para la Práctica 1210.A2-1.10 expresa que las observaciones significativas de actividad ilegal deben informarse al comité de auditoría.

3. Solución: b

a. Incorrecta. El auditor debe actuar en forma coherente con el espíritu comprendido en el Código de Ética del IAI. No resultaría práctico buscar asesoramiento de un abogado independiente para todas las decisiones éticas. La ética es un concepto moral y profesional, no sólo un concepto legal.

b. Correcta. Es coherente con los conceptos comprendidos en el Código de Ética del IAI. c. Incorrecta. No resultaría práctico buscar el asesoramiento del comité de auditoría para todos

los posibles dilemas. Más aún, el consejo que dé el comité podría no ser coherente con las normas de la profesión.

d. Incorrecta. Si las normas de la compañía no son coherentes con las normas de la profesión o no se encuentran a su altura, el auditor interno profesional debe guiarse por las normas de la profesión.

4. Solución: b

a. Incorrecta. El auditor tiene suficiente evidencia para llamar la atención de la gerencia y permitirle decidir qué método va a utilizar para seguir investigando.

b. Correcta. Es la respuesta correcta, de acuerdo con el Consejo para la Práctica 1210.A2-1.6. c. Incorrecta. No hay necesidad de informar a la gerencia divisional sobre las sospechas de

auditoría. Resultaría apropiado entrevistar a la gerencia divisional, pero primordialmente para recolectar datos.

d. Incorrecta. La responsabilidad del auditor es informar dentro de la organización.

II - 40

5. Solución: b a. Incorrecta. Las actividades descriptas deben llevarse a cabo antes de que se realice el trabajo

de campo. b. Correcta. Normalmente, estas actividades se logran durante la fase de investigación preliminar. c. Incorrecta. Estas actividades deben llevarse a cabo antes de que pueda desarrollarse el

programa del trabajo. d. Incorrecta. Las actividades descriptas deben llevarse a cabo antes de que pueda examinarse o

evaluarse la evidencia. 6. Solución: a

a. Correcta. Es una acción apropiada, de acuerdo con el Consejo para la Práctica 1210.A2-1.13. b. Incorrecta. En primer lugar, el auditor debe expandir el trabajo para determinar la existencia de

fraude antes de informar la cuestión a la alta gerencia. En este momento, el auditor sólo tiene sospechas de fraude, dadas las banderas rojas. Debe seguir trabajando antes de consultar con la gerencia, el asesor legal externo o el comité de auditoría.

c. Incorrecta. Ver respuesta “b”. d. Incorrecta. Ver respuesta “b”.

7. Solución: d

a. Incorrecta. Si bien estas estadísticas podrían no resultar tan pertinentes como algunas de las otras elecciones de conjuntos de datos, los datos tendrían gran validez, al haberse recopilado y publicado a partir de una fuente independiente.

b. Incorrecta. Los montos en dólares de este grupo de datos serían objetivos y válidos, y representan las experiencias reales de la organización.

c. Incorrecta. Estos montos incluirían las ventas de contado así como las ventas a crédito, y la inclusión de las ventas de contado reduciría la pertinencia de estos datos al modelo. Sin embargo, estos montos en este grupo de datos también representan las experiencias reales de la organización y, en consecuencia, tienen un alto grado de validez.

d. Correcta. La evidencia de opinión no tiene tanta validez como la evidencia de hecho. Además, la fuente de la evidencia puede ejercer una influencia, que debe considerar el auditor interno cuando evalúe la validez de estos datos.

8. Solución: d

a. Incorrecta. Este es un aspecto mecánico de la evidencia; no tiene relación específica con ninguna de las características de la evidencia.

b. Incorrecta. Esta es una cualidad de la competencia de la evidencia. c. Incorrecta. Esta es una cualidad de la pertinencia de la evidencia. d. Correcta. Esta es una de las cualidades de la suficiencia de la evidencia.

9. Solución: a

a. Correcta. Terceras partes son las que generan esta información, lo cual es independiente de las operaciones del área auditada y, en consecuencia, tiene la evidencia de mayor peso.

b. Incorrecta. Se considera información interna-externa, iniciada por el área auditada y sujeta a distorsiones.

c. Incorrecta. Se considera información interna, generada por el área de auditoría y sujeta a distorsiones.

d. Incorrecta. Se considera información externa-interna. Si bien se inicia externamente, la mantiene el área auditada y, en consecuencia, puede distorsionarse.

II - 41

10. Solución: a a. Correcta. La evidencia competente es evidencia confiable y la mejor que se obtiene por medio

del uso de técnicas apropiadas de auditoría. b. Incorrecta. Es la definición de evidencia física. No toda la evidencia física es necesariamente

competente; en realidad, la calidad de la competencia suele asociarse más con la evidencia documental.

c. Correcta. Es la definición de evidencia corroborativa. Mientras la evidencia corroborativa puede resultar competente, mucha evidencia competente es primordial más que suplementaria.

d. Incorrecta. Esta es la definición de evidencia circunstancial. La evidencia circunstancial no es necesariamente evidencia competente.

11. Solución: a

a. Correcta. La confirmación de un cliente es la evidencia más confiable de que existe una cuenta por cobrar.

b. Incorrecta. Una factura no es particularmente confiable ya que no se desarrolla externamente ni considera pagos subsiguientes.

c. Incorrecta. No es una evidencia de una cuenta por cobrar. d. Incorrecta. No es tan confiable como una confirmación y no confirma la existencia continua de

una cuenta por cobrar. 12. Solución: d

a. Incorrecta. Las ventas ficticias serían una posible respuesta, ya que generarían cuentas incobrables adicionales que no necesariamente se ven reflejadas en las asignaciones para deudas incobrables.

b. Incorrecta. Los procedimientos de créditos y cobranzas ineficaces serían una posible respuesta, ya que podrían contribuir a incrementos en las cuentas incobrables que no necesariamente se ven reflejadas en las asignaciones para deudas incobrables.

c. Incorrecta. Una asignación subestimada para deudas incobrables sería una posible respuesta, ya que contribuiría a sobreestimaciones en cuentas por cobrar netas y a reducciones en la proporción de rotación de cuentas por cobrar.

d. Correcta. Los ingresos por ventas a crédito sobreestimados no serían una posible respuesta, ya que subestimarían (no sobreestimarían) las cuentas por cobrar, lo cual provocaría saldos de cuentas por cobrar más bajos (no más altos) como porcentaje de activos totales.

13. Solución: b

a. Incorrecta. Tendría el efecto contrario. b. Correcta. Con una política liberal de crédito, los clientes se tomarían más tiempo para pagar

(365/4,3 comparado con 365/7,3). c. Incorrecta. Tendría el efecto contrario. d. Incorrecta. No es pertinente, dado que las ventas al contado no tienen impacto alguno.

14. Solución: a

a. Correcta. Sólo una categoría como la de nuevo empleado generaría un cambio en la nómina. b. Incorrecta. La computadora hace los cálculos aritméticos. No es un cambio y no aparecería en

la lista. c. Incorrecta. Estos datos deben provenir del sistema de informes de tiempo (tarjeta horaria u hoja

de cantidad de horas trabajadas). No es un cambio aplicable a la nómina. d. Incorrecta. Esto no es aplicable a un listado de cambios en la nómina.

II - 42

15. Solución: c a. Incorrecta. El diagrama de dispersión no muestra una relación entre los costos de capacitación

y los ingresos por ventas. b. Incorrecta. El diagrama de dispersión no muestra información sobre la eficacia del programa de

capacitación. c. Correcta. El diagrama de dispersión sugiere que los costos de capacitación y las ventas no

están relacionados. d. Incorrecta. No hay nada que indique un dato incorrecto en este gráfico.

16. Solución: d (I, II, y III)

I, II, III. Correcta. Todos los papeles de trabajo deben contener información pertinente que respalde las observaciones y recomendaciones.

17. Solución: a

a. Correcta. Al reunir los grupos de papeles de trabajo, los resúmenes proveen un flujo ordenado y lógico y facilitan la revisión del supervisor.

b. Incorrecta. Los resúmenes forman parte de un archivo de papeles de trabajo, no un reemplazo. c. Incorrecta. A la alta gerencia se le entrega el informe completo o se le provee un resumen

ejecutivo o una reseña del contenido del informe. d. Incorrecta. Las observaciones de auditoría se desarrollan mediante el uso de los atributos de

una observación. (Ver Consejos para la Práctica 2410-1.6 y 7.) 18. Solución: b (I, II y III solamente)

I, II, III. Correcta. El Marco para la Práctica Profesional especifica que los puntos I, II y III son enfoques aceptables para documentar la supervisión de los papeles de trabajo.

IV. Incorrecta. Si bien las evaluaciones de desempeño podrían mencionar las revisiones de los papeles de trabajo, no representan suficiente evidencia de revisión.

19. Solución: c

a. Incorrecta. La respuesta está incompleta porque ignora los hechos (la evidencia) y las recomendaciones.

b. Incorrecta. Esta respuesta está incompleta porque ignora la evidencia y las recomendaciones. c. Correcta. Esta respuesta es la más completa de todas las opciones. d. Incorrecta. Esta respuesta está incompleta porque ignora la evidencia, los objetivos, las

conclusiones y las recomendaciones. 20. Solución: d

a. Incorrecta. No hay necesidad de consideraciones previas en esta situación. b. Incorrecta. Los cambios en la metodología del auditor no son de particular importancia para el

cliente del trabajo. c. Incorrecta. Las indicaciones de un posible fraude no se comunicarían al cliente del trabajo. d. Correcta. Dicha situación requeriría atención inmediata.

21. Solución: c

a. Incorrecta. La cantidad real de medidores reemplazados es menor que la meta, la cual, en consecuencia, no se está alcanzando.

b. Incorrecta. Aparentemente, no se tomó la acción correctiva, ya que los reemplazos reales no alcanzaron la meta.

c. Correcta. No se alcanzó la meta y se necesita acción correctiva. De acuerdo con el Consejo para la Práctica 2100-1.1, los auditores internos se comprometen a evaluar y mejorar la eficacia del control. La determinación de si se identifican, analizan y comunican los desvíos de los estándares operativos a los responsables de tomar acción correctiva constituye una manera de lograr esta función.

d. Incorrecta. No puede determinarse mediante la información dada.

II - 43

22. Solución: a a. Correcta. Se está reparando la misma cantidad de pérdidas informadas. Se está alcanzando la

meta establecida. La Norma de Implementación 2120.A3 expresa que los auditores internos deben revisar operaciones y programas para cerciorarse de la medida en la cual los resultados son coherentes con las metas y los objetivos establecidos a fin de determinar si las operaciones y los programas se están implementando o realizando de la manera que se lo proponían.

b. Incorrecta. No hay desvíos de la meta. c. Incorrecta. El estándar operativo no debe cambiarse, ya que no hay problema en cumplir con

él, y el estándar no puede ser más alto que la cantidad de pérdidas registradas. d. Incorrecta. La dotación del personal no puede evaluarse sobre la base de la información

suministrada. 23. Solución: b

a. Incorrecta. El porcentaje real de agua no medida es mayor que la meta, la cual, en consecuencia, no se alcanzó.

b. Correcta. El Consejo para la Práctica 2320-1.3 expresa que los procedimientos de auditoría analíticos ayudan a los auditores internos a identificar las situaciones que puedan requerir procedimientos de auditoría adicionales. Como el tercer mes tiene el estándar adecuado, es probable que se haya corregido el desvío en el segundo mes; en consecuencia, no se garantiza el trabajo de auditoría adicional.

c. Incorrecta. Aparentemente, se corrigió el desvío del segundo mes. d. Incorrecta. No hay evidencia de que el estándar operativo resulte inapropiado.

24. Solución: a

a. Correcta. Sin el conocimiento de los lineamientos de cumplimiento, no puede llegarse a una conclusión razonable.

b. Incorrecta. El hecho de que no se haya contratado ninguna minoría durante el último año no es pertinente hasta que se conozca el total de contrataciones en el período.

c. Incorrecta. Una política de acción afirmativa es claramente auditable. d. Incorrecta. No puede llegarse a esta conclusión sin conocer la política actual de la compañía.

25. Solución: a

a. Correcta. Las recomendaciones representan las opciones que se encuentran disponibles para la gerencia.

b. Incorrecta. Los problemas deben resolverse de la manera que la gerencia estime apropiada, no el auditor.

c. Incorrecta. Dar recomendaciones puede hacer que la gerencia reduzca los costos/el tiempo para abordar hallazgos de auditoría, pero no hay garantías.

d. Incorrecta. Ver respuesta “c”. 26. Solución: c (II, III y IV solamente)

II, III, IV. Correcta. El Consejo para la Práctica 2410-1.1 expresa que las comunicaciones finales del trabajo deben contener, como mínimo, su propósito, su alcance y sus resultados.

I, V. Incorrecta. Los antecedentes y resúmenes no son elementos requeridos de una comunicación final del trabajo.

27. Solución: c

a. Incorrecta. Resolver conflictos es un objetivo de la reunión de cierre de la auditoría. b. Incorrecta. Hablar de las observaciones del trabajo para llegar a un acuerdo sobre los hechos

es un objetivo de la reunión de cierre de la auditoría. c. Correcta. Identificar los asuntos de futuras auditorías no es un objetivo primario de la reunión

de la auditoría. d. Incorrecta. Determinar los planes de acción y las respuestas de la gerencia es un objetivo de la

reunión de cierre de la auditoría.

II - 44

28. Solución: c a. Incorrecta. Los procedimientos no representan una deficiencia, ya que la eficiencia mejoró sin

disminuir el control. b. Incorrecta. Un diagrama de flujo no es la mejor manera de documentar porque no aborda la

eficiencia. c. Correcta. Representa un cambio en el proceso que debe llamar la atención de la gerencia y

documentarse. d. Incorrecta. Debe finalizarse el trabajo.

29. Solución: d

a. Incorrecta. El cliente de un trabajo debe tener la oportunidad de responder antes de escribir el informe.

b. Incorrecta. Los auditores internos hacen recomendaciones; no presentan requerimientos. c. Incorrecta. Cuando resulte apropiado, los auditores externos revisarían papeles de trabajo para

alcanzar este fin. d. Correcta. Los informes de auditoría deben presentar el propósito, el alcance y los resultados de

un trabajo. 30. Solución: d

a. Incorrecta. La retroalimentación por parte de los clientes se logra mejor si completan un cuestionario diseñado con ese propósito. Dichos cuestionarios facilitan el desarrollo de medidas y tendencias útiles de desempeño de la calidad.

b. Incorrecta. Ver respuesta “a”. c. Incorrecta. El cuestionario debe entregarse al cliente al comienzo del trabajo para que lo

complete después de finalizar dicho trabajo. Distribuir los cuestionarios mucho después de finalizar el trabajo resultaría menos útil porque la información no estaría fresca en la mente del cliente.

d. Correcta. Es una mejor práctica entregar el cuestionario al cliente al comienzo del trabajo, ya sea en forma rutinaria o periódica, para completar después de finalizar dicho trabajo. Entonces el cliente comprende con claridad las medidas de calidad utilizadas por la actividad de auditoría interna y el auditor interno, quien puede observar requerimientos y expectativas específicos antes de que comience el trabajo. Luego, el cliente puede evaluar la calidad del trabajo de auditoría interna mientras se desarrolla, y completar el cuestionario después del trabajo, lo cual estimula a un proceso continuo de monitoreo de la calidad y a la obtención de respuestas por parte del cliente durante todo el trabajo.

31. Solución: c

a. Incorrecta. En una evaluación de desempeño a un auditor interno con promedio bajo, resulta apropiado y conveniente notificar al empleado sobre la próxima evaluación, utilizar un lenguaje objetivo y documentar la evaluación.

b. Incorrecta. Ver respuesta “a”. c. Correcta. No resulta apropiado utilizar generalizaciones cuando se realiza una evaluación de

desempeño a un auditor interno con promedio bajo. En cambio, el evaluador debe citar información específica y prepararse para respaldar las afirmaciones con evidencia.

d. Incorrecta. Ver respuesta “a”.

II - 45

32. Solución: a a. Correcta. Los controles de la contratista de servicios y la organización del usuario resultan

importantes para los controles de la función global de la nómina. b. Incorrecta. Los controles internos de la compañía de servicios de información y la organización

del usuario interactúan entre sí; entonces, ambas deben revisarse. c. Incorrecta. Cambiaría el alcance del trabajo. d. Incorrecta. Si bien el procesamiento se realiza fuera de la organización, la contratista de

servicios externos de información es una extensión de los sistemas de información de la organización. En realidad, el riesgo puede ser más alto, ya que una organización externa controla parte del entorno de control interno. Asimismo, el cambio reciente incrementa el riesgo de la compañía, de la misma manera que la complejidad de la comunicación entre la organización y la contratista de servicios.

33. Solución: d

a. Incorrecta. Es una decisión de la gerencia. b. Incorrecta. Es una responsabilidad de la gerencia. c. Incorrecta. El director ejecutivo de auditoría debe informar la violación a la alta gerencia o al

consejo antes de programar cualquier actividad de auditoría. d. Correcta. Al descubrir un fraude o una conducta no ética, el director ejecutivo de auditoría debe

informar a la gerencia ejecutiva y al comité de auditoría. 34. Solución: d

a. Incorrecta. Sería un procedimiento eficaz porque evitaría que se agregue una compañía ficticia a la lista de proveedores autorizados.

b. Incorrecta. Sería eficaz porque no se pagaría a un proveedor si las partidas no se utilizaran en la producción real.

c. Incorrecta. También sería eficaz porque garantizaría que todos los proveedores son autorizados.

d. Correcta. Sería menos eficaz porque controla la cantidad total de gastos, pero no controla dónde se colocan las órdenes de compra o si se recibe la misma cantidad de artículos comprados.

35. Solución: c

a. Incorrecta. Es poco probable que el entrevistado confiese o suministre otra información útil si el supervisor está presente.

b. Incorrecta. La entrevista debe tener lugar en una sala que permita privacidad, pero no debe haber barreras físicas, ni siquiera una puerta cerrada con llave, para evitar que el sospechoso se vaya si lo desea.

c. Correcta. La elección de las palabras, tales como cambios en el uso de pronombres y verbos, puede indicar que hay áreas de deshonestidad o fabricación.

d. Incorrecta. Durante una entrevista de admisión, el entrevistador debe mostrarse seguro de que el sospechoso cometió el fraude. En consecuencia, el entrevistador debe preguntar cómo cometió el fraude, no si lo cometió.

36. Solución: b

a. Incorrecta. Esta frase describe mejor un enfoque basado en un proceso, si bien los procesos de control no son los únicos que se revisan en este enfoque.

b. Correcta. Un enfoque basado en el control se concentra en determinar en qué medida los controles funcionan correctamente al administrar los riesgos. En general, los riesgos y controles clave se identifican antes del taller.

c. Incorrecta. Si bien el diseño de control podría compararse con los marcos de control en un enfoque basado en el control, no describe el proceso en forma adecuada. Es más probable que un proceso basado en el control examine la brecha entre el diseño y la eficacia de los controles al administrar los riesgos.

d. Incorrecta. Podría hablarse de la eficacia en función de los costos en un taller de auto-evaluación de controles basado en el control, pero no es lo primordial en este proceso.

II - 46

37. Solución: d a. Incorrecta. Las auditorías de programas se dirigen al cumplimiento de objetivos de programas. b. Incorrecta. Las auditorías financieras se dirigen a la exactitud de los registros financieros. c. Incorrecta. La auditoría de cumplimiento se dirige al cumplimiento de requerimientos, entre los

cuales se incluyen los legales y regulatorios. d. Correcta. Es más probable que la auditoría operativa se dirija a la determinación de ahorros en

los costos, centrándose en la economía y la eficiencia. 38. Solución: d

a. Incorrecta. La validez y confiabilidad de cada pregunta resulta extremadamente importante. b. Incorrecta. Ver respuesta "a". c. Incorrecta. Cuando los cuestionarios son demasiado largos, las personas tienden a no

completarlos. d. Correcta. Las preguntas pueden ser de opción múltiple, llenar los espacios en blanco, ensayos,

escalas Likert, etc. 39. Solución: c

a. Incorrecta. Los trabajos de auditorías financieras incluyen la revisión de información financiera. b. Incorrecta. Los trabajos de auditorías de cumplimiento incluyen el examen de procedimientos

de control y su cumplimiento. c. Correcta. Los trabajos de auditorías de desempeño incluyen la revisión del desempeño en

contraposición con la fijación de criterios. d. Incorrecta. Los trabajos de auditorías operativas incluyen la revisión de estructuras

organizacionales y departamentales. 40. Solución: c

a. Incorrecta. Este procedimiento está diseñado para identificar los pagos en el caso de los pasivos que no se incluyen en el período anterior, pero que se pagaron en el período subsiguiente.

b. Incorrecta. Este procedimiento está diseñado para identificar montos no incluidos en las cuentas por pagar. Las cuentas con saldos que aparecen en cero deben verificarse como parte del proceso.

c. Correcta. Este procedimiento no provee evidencia respecto de pasivos no registrados. d. Incorrecta. El rastreo de los informes de recepción emitidos antes del fin del período hasta las

facturas y el listado de cuentas por pagar está diseñado para asegurar que estos envíos se incluyan en las cuentas por pagar.

41. Solución: d

a. Incorrecta. La diversificación de riesgos es una razón frecuente para que una compañía se fusione con otra o la adquiera.

b. Incorrecta. La acción de responder a una política de gobierno es una razón frecuente para que una compañía se fusione con otra o la adquiera.

c. Incorrecta. La reducción de costos de mano de obra es una razón frecuente para que una compañía se fusione con otra o la adquiera.

d. Correcta. El incremento de los precios de las acciones no es una razón frecuente para que una compañía se fusione con otra o la adquiera porque este efecto podría lograrse a través de otros métodos que beneficien directamente el desempeño de la compañía.

42. Solución: c (I y II solamente)

I. Correcta. Aborda la aplicación apropiada de los costos como resultado de los pedidos de cambios.

II. Correcta. Aborda la exactitud de los costos como resultado de los pedidos de cambios. III. Incorrecta. Este procedimiento prueba si la compañía aprobó el trabajo realizado por el

contratista, pero no prueba si el contratista justificó los costos relacionados con el trabajo de manera apropiada.

II - 47

43. Solución: a a. Correcta. El punto lógico de partida es determinar el(los) punto(s) de control. Luego, puede

evaluarse la evidencia del cumplimiento de los requerimientos de la licencia. b. Incorrecta. Antes de dar este paso, un auditor determinaría, en primer lugar, si la instalación se

controla centralmente, porque afectaría a la manera en que el auditor se cercioraría de la información del software instalado.

c. Incorrecta. Ayudaría al auditor a determinar si el software se compró en forma legítima, pero aún así, el auditor necesitaría comenzar por determinar dónde se instala el software, y la respuesta “a” sería un punto de partida más útil.

d. Incorrecta. Supervisar el uso no resultaría tan importante como determinar los procesos de instalación cuando se evalúa el cumplimiento de los requerimientos de licencia.

44. Solución: a

a. Correcta. La verificación es la técnica más usada para comprobar la exactitud de la información mantenida por un sistema, ya sea manual o automatizado.

b. Incorrecta. Probar el programa no comprobará la exactitud de los datos en la base de datos. c. Incorrecta. Simular un procesamiento normal probaría el programa, pero no la exactitud de los

datos. d. Incorrecta. El rastreo requeriría que se insertara una codificación adicional en los programas

del sistema de la base de datos. 45. Solución: a

a. Correcta. Debe llevarse a cabo un estudio de factibilidad en la etapa de análisis de sistemas. b. Incorrecta. Es importante la participación de los usuarios en el proceso de desarrollo en varias

etapas. c. Incorrecta. Esto asegura la calidad en el proceso de desarrollo en varias etapas. d. Incorrecta. Sin buena documentación, un sistema de información puede ser difícil, si no

imposible, de operar, mantener o utilizar. 46. Solución: c

a. Incorrecta. El procesamiento utilizado por los competidores puede no ser pertinente o resultar totalmente desconocido.

b. Incorrecta. En primer lugar, debe ponerse énfasis en los propósitos y necesidades del nuevo sistema, no en el equipamiento.

c. Correcta. Las necesidades y los objetivos de la información de los usuarios deben ser primordiales.

d. Incorrecta. Los controles relacionados con el sistema anterior (en uso) pueden no ser pertinentes ni importantes.

47. Solución: c

a. Incorrecta. El uso de guiones es una práctica común para los eventos en serie. b. Incorrecta. Los contratos de recuperación no se actualizan con tanta frecuencia. c. Correcta. El no lograr almacenar los medios de respaldo en el lugar correspondiente constituye

una debilidad de control muy seria. d. Incorrecta. En general, el acceso de tiempo limitado de prueba sólo permitirá probar algunos

sistemas. 48. Solución: b (I y IV solamente)

I, IV. Correcta. Estas pruebas pueden identificar los pagos duplicados. II, III. Incorrecta. La selección de transacciones con códigos de proveedores no autorizados y la

prueba de razonabilidad de las transacciones no identifican los pagos duplicados.

II - 48

49. Solución: d a. Incorrecta. La comparación con los estándares de la industria no probará la exactitud de los

informes internos. b. Incorrecta. La comparación con los estándares de la industria no probará los controles

diseñados para proteger el inventario. c. Incorrecta. La comparación con los estándares de la industria no probará el cumplimiento. d. Correcta. Un procedimiento analítico de esa índole proveerá una indicación de la eficiencia y

eficacia de la administración del inventario por parte de la subsidiaria. 50. Solución: b

a. Incorrecta. Este procedimiento sólo provee datos con respecto a si los pagos coinciden con las facturas. No provee datos con respecto a si las cantidades facturadas son correctas.

b. Correcta. Ayudaría al auditor a determinar que es probable que se hayan cotejado los tres segmentos de datos antes de realizar el pago.

c. Incorrecta. Igual que la respuesta “a”: sólo provee datos con respecto a si los pagos coinciden con las facturas. No provee datos con respecto a si en realidad se recibieron los bienes.

d. Incorrecta. Sólo provee datos de un día. Si bien coteja los artículos recibidos con los pagados, no provee datos con respecto a si las facturaciones son correctas.

51. Solución: d

a. Incorrecta. El benchmarking implica una comparación con los líderes de la industria o las operaciones “al nivel mundial”. El benchmarking utiliza cifras al nivel de la industria (para proteger la confidencialidad de la información suministrada por las organizaciones participantes) o cifras de las organizaciones que cooperan.

b. Incorrecta. El benchmarking requiere mediciones, que implican comparaciones cuantitativas. c. Incorrecta. El benchmarking puede aplicarse a todas las áreas funcionales de una compañía.

En realidad, como las operaciones de manufactura suelen ser específicas para la industria, mientras que las cosas como el procesamiento de una orden o el pago de una factura no lo son, existe una mayor oportunidad de mejorar aprendiendo de los líderes globales.

d. Correcta. Ver respuesta "a". 52. Solución: a

a. Correcta. Sería el procedimiento menos eficaz porque: (1) proporciona una comparación con el período anterior, en el cual es probable que se haya sufrido por el mismo problema; y (2) es una prueba global.

b. Incorrecta. Utilizar un centro de pruebas integradas aquí sería un muy buen procedimiento porque la preocupación es saber si la tasa de interés se calculó correctamente.

c. Incorrecta. Utilizar datos de prueba resultaría muy eficaz porque se provee una prueba directa del cálculo de la tasa de interés.

d. Incorrecta. Sería el procedimiento más eficaz porque el auditor está tomando una muestra detallada de las transacciones reales.

53. Solución: a

a. Correcta. La actividad de auditoría interna debe asegurar la existencia de especificaciones de desempeño coherentes con las necesidades del hospital porque pueden provocarse especificaciones incompletas o erróneas en la adquisición de software no utilizable o condiciones imprevisibles en el contrato con el proveedor de software.

b. Incorrecta. La actividad de auditoría interna no puede asegurar que el diseño de aplicación cumpla con las normas de documentación y desarrollo internos porque un grupo externo con diferentes normas ya desarrolló el sistema.

c. Incorrecta. No existe un prototipo en la compra de software patentado. d. Incorrecta. Para sistemas desarrollados externamente, el único paso omitido o abreviado del

ciclo de vida de desarrollo es la programación del sistema real. Todas las otras fases se mantienen, incluso si se modifican.

II - 49

54. Solución: a a. Correcta. Dicha capacitación debe planificarse y es de naturaleza continua. Debe estar sujeta a

un contrato de consultoría formal y escrito para asegurar que se reconozcan y cumplan las necesidades y expectativas de aquellos que se capaciten.

b. Incorrecta. Este tipo de contrato se aplica más a tareas de rutina. c. Incorrecta. Este tipo de contrato se aplica más a acuerdos especiales que se realizan de vez en

cuando o sólo una vez. d. Incorrecta. Este tipo de contrato se aplica más a trabajos no planificados.

55. Solución: c

a. Incorrecta. Se presume que el diseño de sistemas perjudica la objetividad de la auditoría. b. Incorrecta. Se presume que la redacción de un borrador de los procedimientos para los

sistemas de control perjudica la independencia. c. Correcta. De acuerdo con el Consejo para la Práctica 1130.A1-1.4, la revisión de sistemas,

incluso antes de la implementación, es una actividad apropiadamente realizada por la función de auditoría interna y no perjudica la objetividad.

d. Incorrecta. Se presume que la instalación de sistemas de controles perjudica la independencia. 56. Solución: c

a. Incorrecta. Los diagramas de flujo son los más apropiados para estudiar el diseño de control interno. El objetivo de auditoría es determinar si los controles se encuentran en su lugar y resultan eficaces, lo cual indica la necesidad de probar los controles.

b. Incorrecta. Los narrativos del sistema son sumamente apropiados para estudiar el diseño de control interno. El objetivo de auditoría es determinar si los controles se encuentran en su lugar y resultan eficaces, lo cual indica la necesidad de probar los controles.

c. Correcta. Las pruebas de los controles, también conocidas como pruebas de cumplimiento, ayudan al auditor a determinar si los controles se siguen y resultan efectivos. Por ejemplo, una política puede requerir que el gerente apruebe todas las grandes transacciones. Como prueba de los controles, el auditor puede tomar una muestra de las grandes transacciones y revisar para constatar si se obtuvo la aprobación del gerente y si la transacción propuesta cumple con todos los criterios que se suponía que debía verificar el gerente.

d. Incorrecta. Las pruebas sustantivas son para determinar si se alcanzó un objetivo y no necesariamente prueban los controles internos.

57. Solución: a

a. Correcta. Las innovaciones en la fabricación de productos o servicios no suelen prestarse a una medición continua de desempeño.

b. Incorrecta. Los resultados clave en la participación en el mercado influyen en la posición competitiva de la organización.

c. Incorrecta. Los resultados clave en la satisfacción al cliente ayudan a predecir las futuras ventas.

d. Incorrecta. Los resultados clave en el desarrollo del empleado ayudan a predecir la habilidad de atraer y retener a los buenos empleados.

58. Solución: b

a. Incorrecta. Esta comparación ayudaría a resaltar la eficacia de la promoción en aumentar las ventas.

b. Correcta. No hay indicación de que haya cambiado el costo de los productos vendidos. El desafío es abordar la eficacia de la promoción.

c. Incorrecta. Es el análisis clave, ya que mostraría el alcance de los ingresos adicionales en contraposición con el costo.

d. Incorrecta. Resultaría útil porque el departamento de ventas puede tener información útil sobre nuevos clientes y repetir compras.

II - 50

59. Solución: c a. Incorrecta. Ni la auto-evaluación de control ni la medición de desempeño abordarán el objetivo

de la gerencia de controlar los costos. b. Incorrecta. Si bien el benchmarking puede tener algo de aplicación, no constituye la

herramienta más apropiada. c. Correcta. Una revisión de los procesos del negocio (BPR, por sus siglas en inglés) evalúa el

desempeño de los procesos administrativos y financieros, tales como los que se encuentran dentro de las compras y las cuentas por pagar. La BPR considera la eficacia y eficiencia del proceso, lo cual incluye la presencia de controles apropiados, para mitigar el riesgo del negocio. Como el objetivo es controlar los costos de los teléfonos celulares, la BPR constituye la herramienta apropiada para utilizar en esta área.

d. Incorrecta. Ver respuesta “a”. 60. Solución: a

a. Correcta. La comparación con organizaciones que realizan funciones relacionadas dentro de la misma área tecnológica provee información sobre lo que se está logrando en cualquier lugar en la nueva línea de negocio.

b. Incorrecta. La comparación con los mejores competidores se centra en el desempeño en organizaciones relacionadas como un todo y es probable que incluya algunas actividades no relacionadas con la nueva línea de negocio.

c. Incorrecta. La comparación de los procesos que son casi iguales cualquiera sea la industria (tal como el procesamiento de documentos) no resultaría tan útil como la comparación de procesos que son similares en la función.

d. Incorrecta. La comparación con lo mejor de la misma organización puede resultar engañosa, ya que no provee información sobre lo que se está logrando fuera de la organización en la nueva línea de negocio.

61. Solución: d

a. Incorrecta. Este procedimiento sólo considera los trabajos del reproceso que requieren acción correctiva. No todas las órdenes del reproceso revisadas por el ingeniero requerirán acción correctiva.

b. Incorrecta. Esta prueba resultaría útil para verificar que todo el reproceso se registre en el diario del sistema, pero no provee ninguna evidencia de que se haya revisado el trabajo.

c. Incorrecta. Como este procedimiento comienza sólo con trabajos del reproceso que se revisaron, no resultaría útil para encontrar trabajos que no se revisaron.

d. Correcta. La mejor evidencia de todo el trabajo realizado es el conjunto de formularios de órdenes del reproceso y la mejor evidencia de lo que se revisó es las entradas en el diario del sistema de revisión. Para determinar si se revisó todo el reproceso, el auditor debe comenzar con la población de todo el reproceso que se realizó (es decir, los formularios de las órdenes del reproceso) y rastrear para obtener la evidencia de que se revisó (es decir, el registro de revisiones).

62. Solución: d

a. Incorrecta. La transferencia de los empleados no es necesaria ni resolvería el problema de control.

b. Incorrecta. Puede ayudar a detectar problemas anteriores, pero no crea un control para abordar futuros problemas.

c. Incorrecta. No abordaría el problema porque no incluye el archivo maestro del proveedor. d. Correcta. Es la única opción que corregirá la deficiencia identificada durante la auditoría.

II - 51

63. Solución: c a. Incorrecta. Las varianzas no identificarían los costos transferidos al inventario. b. Incorrecta. Analizaría una muestra de todas las transacciones de capital mientras la respuesta

“c” abordaría todas las transferencias más específicamente. c. Correcta. Se centraría en el problema de transferencias inapropiadas. d. Incorrecta. No habría recibos de inventario para las transferencias; en consecuencia, comenzar

con recibos de inventario no sería un método eficaz para supervisar esta situación. 64. Solución: b

a. Incorrecta. Como el hallazgo es significativo, la actividad de auditoría interna no esperaría hasta la próxima auditoría regularmente programada para evaluar el estado de la acción correctiva.

b. Correcta. La actividad de auditoría interna debe supervisar el estado de la acción correctiva. Debe programarse un trabajo de seguimiento cuando se hayan completado lo suficiente los cambios en el sistema de procesamiento de reclamos para permitir las pruebas de adecuación y eficacia.

c. Incorrecta. Si bien la gerencia indicó que las correcciones deben finalizarse en seis meses, éste puede no ser el caso. Como resultado, la actividad de auditoría interna debe supervisar el estado de la acción correctiva y programar un trabajo de seguimiento cuando resulte apropiado.

d. Incorrecta. Si bien debe hablarse de los hallazgos con el comité de auditoría por su importancia, el director ejecutivo de auditoría debe determinar el alcance y la oportunidad de un trabajo de seguimiento sobre la base de información disponible.

65. Solución: c

a. Incorrecta. Si bien es probable que la alta gerencia utilice el informe para preguntar por qué ciertas acciones correctivas pueden estar atrasadas con respecto al programa, no tiene manera de saber si las acciones correctivas mostradas como finalizadas en realidad lo están.

b. Incorrecta. Mientras los gerentes operativos pueden ser en realidad los que más saben sobre la acción correctiva, se prefiere la verificación independiente.

c. Correcta. Si existe un paso en el proceso en el cual alguien independiente del área que se inspecciona pueda evaluar la adecuación y finalización de la acción correctiva, se minimiza la posibilidad de un fraude en la información de cierre.

d. Incorrecta. No resulta inapropiado que el personal ambiental, de seguridad y salud ingrese los resultados de la inspección inicial. El hecho de que el secretario ingrese los datos de cierre no mejora los controles, ya que todavía no hay ninguna revisión independiente. Asimismo, resulta menos eficiente y oportuno que hacer ingresar los datos directamente en el campo.

66. Solución: a

a. Correcta. Se expresa en el Consejo para la Práctica 2500.A1-1. b. Incorrecta. Se contradice con la respuesta "a" y el Consejo para la Práctica 2500.A1-1. c. Incorrecta. La Norma de Implementación 2500.A1 expresa que debe tomarse una acción de

seguimiento. No depende de directivas de la gerencia ni del comité de auditoría. d. Incorrecta. Ver respuesta "a".

67. Solución: c

a. Incorrecta. Esta acción resulta insuficiente; ver respuesta “c”. b. Incorrecta. Esta acción resulta insuficiente; ver respuesta “c”. c. Correcta. La gerencia puede decidir asumir el riesgo de no corregir una situación registrada por

el costo u otras consideraciones. d. Incorrecta. Esta acción resultaría inapropiada; ver respuesta “c”.

II - 52

68. Solución: a a. Correcta. Resulta apropiado evaluar si los pasos tomados están resolviendo la situación, si se

implementaron los controles apropiados y si se acumularon los beneficios para la entidad. No es necesario, sin embargo, asegurar que se eliminó el riesgo inherente. (Sólo podría lograrse eliminando el uso de efectivo, lo cual es irreal.)

b. Incorrecta. Ver respuesta “a”. c. Incorrecta. Ver respuesta “a”. d. Incorrecta. Ver respuesta“a”.

69. Solución: d

a. Incorrecta. Ver respuesta “d”. b. Incorrecta. Ver respuesta “d”. c. Incorrecta. Ver respuesta “d”. d. Correcta. La alta gerencia puede decidir aceptar el riesgo debido al costo o a otras

consideraciones. El director ejecutivo de auditoría debe evaluar el motivo fundamental de la alta gerencia y luego informar al consejo la decisión de la gerencia.

70. Solución: d

a. Incorrecta. La gerencia es responsable de asegurar que se tome acción de acuerdo con todas las observaciones y recomendaciones de auditoría interna, pero puede llevar tiempo completar algunas acciones y no resulta práctico esperar que todo se resuelva cuando se reúna un comité de auditoría.

b. Incorrecta. Ver respuesta “a”. c. Incorrecta. Ver respuesta “a”. d. Correcta. El director ejecutivo de auditoría es responsable de establecer los procedimientos

para que la gerencia vigile el progreso de acuerdo con todas las observaciones y recomendaciones de auditoría interna. El comité de auditoría debe introducir esta responsabilidad por escrito en su estatuto, y debe informarse el progreso en cada reunión del comité de auditoría.

71. Solución: c

a. Incorrecta. La respuesta del entrevistador puede sugerir una pregunta de seguimiento que debe formularse antes de hacer la siguiente pregunta planificada.

b. Incorrecta. Puede resultar confuso para quien responde. c. Correcta. Debe obtenerse la información general primero antes de buscar los detalles. d. Incorrecta. El entrevistador debe evitar las preguntas que sugieran las respuestas.

72. Solución: c

a. Incorrecta. La ITF sólo provee evidencia sobre la corrección del procesamiento computarizado. No resultaría pertinente a la razón fundamental hipotética para los datos operativos.

b. Incorrecta. Las entrevistas proveen una forma débil de evidencia y serían mejores si el auditor, en primer lugar, tuviera evidencia esencial respaldada por documentos.

c. Correcta. Si se produjera este tipo de fraude, se provocaría una reducción de inventario. El recuento de inventario sorpresivo sería una técnica de auditoría eficaz.

d. Incorrecta. Habría un problema con la reducción de inventario, no con la determinación de si los artículos se ingresaron en la caja registradora o se examinaron.

II - 53

73. Solución: b a. Incorrecta. Es probable que el auditor de sistemas de información de la organización tenga un

mayor conocimiento de los sistemas computarizados de la organización. b. Correcta. La característica distintiva de la auditoría legal es el conocimiento necesario para

declarar como testigo experto en una corte. Si bien el auditor legal puede poseer los otros atributos detallados, el auditor de sistemas de información de la organización puede también poseer estas destrezas o elementos de conocimiento.

c. Incorrecta. Un auditor legal no necesariamente tendría destrezas analíticas u organizacionales superiores a las del auditor de la organización.

d. Incorrecta. Ver respuesta “c”. 74. Solución: d

a. Incorrecta. Es una buena técnica para utilizar en una entrevista durante una investigación de fraude.

b. Incorrecta. Ver respuesta "a". c. Incorrecta. Ver respuesta "a". d. Correcta. El auditor debe evitar dar la impresión de que busca una confesión o convicción.

75. Solución: b

a. Incorrecta. No es inusual y, por sí mismo, no es un indicador de posible fraude. b. Correcta. Se considera una “bandera roja”, lo cual indica posible fraude. c. Incorrecta. No es inusual y, por sí mismo, no es un indicador de posible fraude. d. Incorrecta. No es inusual y, por sí mismo, no es un indicador de posible fraude.

76. Solución: b

a. Incorrecta. En una simulación paralela, los datos procesados por el sistema del cliente del trabajo se reprocesan a través del programa del auditor para determinar si el resultado obtenido coincide con el resultado generado por el sistema del cliente. Esta técnica podría identificar problemas con el procesamiento del cliente, pero no identificaría a un empleado ficticio o que ya no trabaja para una compañía.

b. Correcta. Este tipo de programa de técnicas de auditoría asistida por computadora (CAAT, por sus siglas en inglés) puede identificar a empleados que en general no tengan deducciones.

c. Incorrecta. Un programa de CAAT puede recalcular montos tales como pago bruto, pago neto, impuestos y otras deducciones y períodos de licencias acumulados o utilizados. Estos cálculos pueden ayudar a determinar si el programa de pagos por nómina funciona correctamente o si se alteraron los archivos de los empleados, pero no identificaría a un empleado ficticio o que ya no trabaja para una compañía.

d. Incorrecta. En este tipo de programa de CAAT, ciertas transacciones reales están “rotuladas” y, a medida que avanzan en el sistema, se crea un archivo de datos que rastrea el procesamiento a través del sistema y permite a un auditor que subsiguientemente revise ese procesamiento. Sin embargo, no identificaría a un empleado ficticio o que ya no trabaja para una compañía.

77. Solución: a

a. Correcta. Es un procedimiento de control aceptable que apunta a limitar el riesgo mientras promueve la eficiencia. No se considera, por sí mismo, una condición que indique una mayor posibilidad de fraude (una bandera roja).

b. Incorrecta. La falta de rotación de deberes o capacitación cruzada para trabajos sensibles se identifica como una bandera roja.

c. Incorrecta. Sería un ejemplo de segregación inapropiada de deberes, lo cual se identifica como una bandera roja.

d. Incorrecta. Se identifica como una bandera roja.

II - 54

78. Solución: c a. Incorrecta. Ver respuesta “c”. b. Incorrecta. Ver respuesta “c”. c. Correcta. El fraude de la gerencia beneficia a las organizaciones más que a los individuos; en

consecuencia, la existencia de presiones financieras es la motivación más común. Los perpetradores de la gerencia tratan de hacer que sus estados financieros parezcan más atractivos por las presiones financieras de los acuerdos de préstamos restrictivos, una situación de efectivo deficiente, la pérdida de clientes significativos, etc.

d. Incorrecta. Ver respuesta “c”. 79. Solución: d

a. Incorrecta. Se considera un posible síntoma de fraude, pero lo mismo ocurre con las otras situaciones.

b. Incorrecta. Ver respuesta “a”. c. Incorrecta. Ver respuesta “a”. d. Correcta. Las transacciones no respaldadas, el derroche y los entornos de control débiles se

consideran síntomas de fraude que deben intensificar la conciencia del auditor en cuanto al posible fraude.

80. Solución: d

a. Incorrecta. El muestreo estratificado es un procedimiento de muestreo variable. Su objetivo primordial es estimar un valor de población particular mediante el uso de los resultados de una muestra. No concierne a errores en la población y, en consecuencia, no se detendría cuando se encuentra el primer error.

b. Incorrecta. El muestreo de atributos da como resultado un estimado de la proporción de ocurrencia de alguna característica en una población. Se determina el tamaño de la muestra para estimar esta proporción con el nivel deseado de aseguramiento; en consecuencia, se requiere el tamaño de toda la muestra, cualquiera sea el momento en que ocurra el error.

c. Incorrecta. El muestreo de parar o seguir es un procedimiento de muestreo secuencial en el cual el paso siguiente se determina por los resultados del paso anterior. Sin embargo, una vez iniciado el paso, se lleva a cabo hasta finalizarlo. Por ejemplo, suponga que un auditor toma una muestra, evalúa los resultados y determina que se requieren elementos adicionales para la muestra. Cada fase de la muestra se conduce sin hacer referencia al momento en que se observa el primer error.

d. Correcta. El objetivo del muestreo de descubrimiento es proveer un nivel específico de aseguramiento de que una muestra dará al menos un ejemplo de un atributo si la proporción de ocurrencia de ese atributo dentro de la población se encuentra dentro de un límite específico o sobrepasa ese límite. La decisión de auditoría se toma de inmediato una vez observado el primer error.

81. Solución: a

a. Correcta. Es el mejor procedimiento porque toma una muestra del archivo total de préstamos y de las pruebas para determinar que el préstamo se clasifique en cuanto a sus categorías, garantías y antigüedad.

b. Incorrecta. Esta muestra sólo aborda préstamos grandes y no prueba garantías en forma apropiada.

c. Incorrecta. Es un procedimiento de auditoría ineficiente porque brinda muestras de solicitudes de préstamos, no de préstamos aprobados.

d. Incorrecta. Sería un procedimiento ineficaz porque se basa sólo en préstamos para los cuales se están efectuando los pagos. No incluye los préstamos que deberían haberse clasificado en categorías de manera diferente porque no se están efectuando los pagos.

II - 55

82. Solución: d a. Incorrecta. El juicio se necesita para los niveles de confianza y la definición de unidades de

muestras. b. Incorrecta. Una muestra estadística puede terminar en una muestra menor o mayor. c. Incorrecta. No hay manera de determinar qué método produciría mayor exactitud. d. Correcta. La única manera de tener confiabilidad mensurable (expresada en términos de

intervalos de confianza) es utilizar una muestra estadística. 83. Solución: c

a. Incorrecta. El muestreo de atributos no se utiliza para las pruebas de montos y, en consecuencia, la variabilidad en los montos no es una cuestión al determinar el tamaño de la muestra.

b. Incorrecta. El muestreo de unidades monetarias neutraliza la variabilidad definiendo la unidad del muestreo como una unidad monetaria individual.

c. Correcta. La variabilidad afecta al desvío estándar. Cuanto mayor el desvío estándar, mayor es el tamaño de la muestra que se requiere para lograr un nivel específico de precisión y confianza.

d. Incorrecta. El objetivo de la muestra de descubrimiento es seleccionar elementos hasta que quede al menos uno con una característica en particular, tal como la evidencia de fraude.

84. Solución: b

a. Incorrecta. En los casos posteriores, existe un patrón definido de incremento. Dicho patrón excede el límite superior de control de manera persistente.

b. Correcta. El modelo en el cual se basan la media esperada y los límites superior e inferior excluye el hecho de que se excede el límite superior de control en forma persistente. Se requiere investigar.

c. Incorrecta. El límite superior se excede en gran medida. De acuerdo con el modelo de planificación, es anormal.

d. Incorrecta. Las partes del diagrama parecen cíclicas, pero una característica significativa es la anormalidad que excede el límite superior de control.

85. Solución: c

a. Incorrecta. El error estándar no es una proyección del error de una población. b. Incorrecta. El error estándar no es una medición de los errores de una muestra. c. Correcta. El error estándar es una función del desvío estándar, que es una medición de la

varianza promedio de la media muestral. El error estándar se usa para calcular la precisión y el intervalo de confianza. Cuanto mayor sea el error estándar, más amplio será el intervalo.

d. Incorrecta. La cantidad de error que el auditor estaría dispuesto a aceptar (el error tolerable) es decisión del auditor; no es el resultado de cálculos estadísticos. La cantidad de error tolerable no tiene efecto en el error estándar.

86. Solución: c

a. Incorrecta. Es probable que esta muestra de conveniencia ponga énfasis en personas con mucho tiempo disponible a expensas de los empleados clave que están demasiado ocupados con trabajo de la compañía para responder.

b. Incorrecta. Los gerentes y supervisores suelen no tener las mismas necesidades y percepciones que sus subordinados y, asimismo, pueden percibir los puntos de vista de sus empleados de manera errónea.

c. Correcta. Como es probable que diferentes empleados tengan diferentes situaciones, necesidades y experiencias, el muestreo estratificado podría asegurar mejor que una muestra representativa da resultado.

d. Incorrecta. Este enfoque provocaría una cantidad desproporcionada de empleados altamente remunerados que pueden no tener las mismas necesidades que los que cobran sueldos bajos.

II - 56

87. Solución: d a. Incorrecta. Responder al cuestionario ayudará al cliente del trabajo a identificar áreas en las

cuales los procedimientos sean débiles o no se encuentren apropiadamente documentados. b. Incorrecta. El cuestionario comunicará las áreas que el auditor planifica evaluar. c. Incorrecta. El auditor podría utilizar la preparación del cuestionario para organizar la

información que debe recopilarse. d. Correcta. La información adicional resulta útil para el auditor.

88. Solución: b

a. Incorrecta. La estimación de razón es un tipo de plan de muestreo de variables. No es una técnica para la estimación del desvío estándar.

b. Correcta. Los auditores utilizan una muestra piloto para estimar el desvío estándar de una población. Ello permite al auditor estimar el intervalo de confianza que se lograría con la muestra, y por lo tanto, ayudaría al auditor a decidir el tamaño de muestra por seleccionar.

c. Incorrecta. Los auditores utilizan la regresión para proyectar saldos de cuentas u otras poblaciones.

d. Incorrecta. El muestreo de descubrimiento es un tipo de plan de muestreo, no una técnica para la estimación del desvío estándar.

89. Solución: b

a. Incorrecta. Es una prueba detallada, no un estudio de relaciones. b. Correcta. Entre las pruebas analíticas se incluyen las comparaciones con los montos

presupuestados, las operaciones anteriores y las similares. c. Incorrecta. Es una prueba detallada, no un estudio de relaciones. d. Incorrecta. Es una prueba detallada, no un estudio de relaciones.

90. Solución: a

a. Correcta. Si el auditor ya sospecha que hay fraude, resultaría apropiada una auditoría con un enfoque más dirigido.

b. Incorrecta. Los datos operativos relativamente estables constituyen un buen escenario para utilizar la revisión analítica.

c. Incorrecta. La revisión analítica resultaría útil para identificar si se realizaron transacciones grandes, extraordinarias o inusuales.

d. Incorrecta. La revisión analítica sólo necesita tener cuentas relacionadas con otras o con datos independientes. No requiere que estén relacionadas con los ingresos.

91. Solución: d

a. Incorrecta. La regresión lineal simple resultaría útil, pero no proveería el mismo aporte que el análisis de regresión múltiple (por ejemplo, la división de acciones en acciones de alto valor inestable y bajo valor inestable, de acuerdo con la medición del mercado Beta).

b. Incorrecta. El análisis de la razón provee algún aporte, pero sólo se encuentra diseñado para proveer datos sobre la composición relativa de los instrumentos que devengan intereses en contraposición con la inversión en acciones. Puede recopilarse más información mediante la regresión múltiple.

c. Incorrecta. El análisis de tendencias sólo verifica que tuvo lugar un cambio y muestra la amplia naturaleza del cambio. No provee aportes sobre las causas del cambio en los ingresos por inversiones.

d. Correcta. Sería el mejor enfoque porque permite al auditor captar información sobre las posibles causas del cambio en los ingresos por inversiones.

II - 57

92. Solución: b a. Incorrecta. La observación proveerá evidencia sobre si el personal de créditos sigue los

procedimientos mientras se los observa. Sin embargo, como saben que los están controlando, es probable que hagan lo que creen que deben hacer, no lo que normalmente hacen.

b. Correcta. El propósito de la función de otorgamiento de créditos es minimizar las cancelaciones de las cuentas por cobrar mientras que al mismo tiempo se aceptan las ventas que probablemente terminen en cobranzas. El revisar la tendencia de las cancelaciones de las cuentas por cobrar proveerá algún aporte concerniente a su minimización.

c. Incorrecta. Las respuestas del gerente de créditos carecerá de objetividad, un atributo clave de la evidencia competente.

d. Incorrecta. Los límites de crédito pueden fijarse en un nivel demasiado alto o no revisarse apropiadamente en forma periódica. La existencia de la aprobación de créditos no detectará estos problemas.

93. Solución: d

a. Incorrecta. Como se falsificaron los documentos, todos los documentos de respaldo coincidirían con cada desembolso en efectivo.

b. Incorrecta. Los proveedores confirmarían todas las transacciones, porque se realizaron todas. c. Incorrecta. Como las órdenes fraudulentas se embarcan a otro lugar, los procedimientos de

recepción en la dársena parecerían correctos. d. Correcta. Como los materiales se embarcan y utilizan en otra empresa, las comparaciones

analíticas mostrarían un incremento sin explicación en los materiales usados. 94. Solución: b

a. Incorrecta. Ver respuesta “b”. b. Correcta. El proceso racional de toma de decisiones implica:

• Reconocer la brecha entre la realidad y las expectativas (“c”). • Definir el problema (dado en el origen). • Evaluar el nivel de riesgo aceptable asociado con una decisión en particular (“b”). • Buscar y evaluar las soluciones del problema (“a”). • Elegir una solución. • Implementar la solución y los resultados de la medición.

c. Incorrecta. Ver respuesta “b”. d. Incorrecta. Ver respuesta “b”.

95. Solución: a

a. Correcta. Mediante la indicación de debilidades de control, los diagramas de flujo muestran dónde puede haber fraude.

b. Incorrecta. Los diagramas de flujo no proveen evidencias de la magnitud del fraude. c. Incorrecta. Se necesitarían otros procedimientos para detectar dónde hay fraude. d. Incorrecta. Los diagramas de flujo proveen evidencia de dónde puede haber fraude. En

consecuencia, los diagramas de flujo ayudan a evitarlo. 96. Solución: a

a. Correcta. Las formas manuales de registrar puntos de vista y dar respuestas grupales resultan eficaces; la tecnología para el recuento de votos puede incrementar la eficiencia, pero no resulta esencial para el éxito.

b. Incorrecta. La auto-evaluación de control (CSA, por sus siglas en inglés) requiere destrezas para los facilitadores.

c. Incorrecta. La CSA requiere una planificación cuidadosa. d. Incorrecta. Los facilitadores de la CSA deben comprender y administrar las dinámicas de

grupo.

II - 58

97. Solución: a (I y II solamente) I. Correcta. Si no hay un valor monetario en la base de datos para el inventario existente, el

inventario podría estar reducido. II. Correcta. Las verificaciones inadecuadas o los préstamos/reembolsos no controlados podrían

provocar cantidades negativas en stock, con lo cual el inventario podría estar reducido. III. Incorrecta. Si las cantidades pedidas exceden los requerimientos, podría provocarse un

incremento en el inventario, pero en sí mismas, no harían que el inventario se vea reducido o incrementado.

IV. Incorrecta. No tendría ningún impacto en la valoración del inventario. 98. Solución: b

a. Incorrecta. Podría utilizarse un software genérico de auditoría para buscar transacciones inusuales, tales como las que exceden un monto específico.

b. Correcta. Es altamente improbable que el sistema de cuentas por pagar contenga evidencia suficiente de transacciones fraudulentas. Podría utilizarse un software genérico de auditoría para explorar las banderas rojas, pero no las identificaría en forma particular.

c. Incorrecta. Los datos de las transacciones podrían filtrarse utilizando un software genérico de auditoría.

d. Incorrecta. Podría hacerse un resumen de los proveedores utilizados por los usuarios de tarjeta con un software genérico de auditoría.

99. Solución: d

a. Incorrecta. Se necesitaría una prueba más directa para alcanzarlos. b. Incorrecta. Las normas internacionales de auditoría interna no requieren el uso de diagramas

de flujo. c. Incorrecta. Se necesitaría una prueba más directa para alcanzarlos. d. Correcta. Es la razón por la cual los auditores emplean diagramas de flujo con referencias para

las descripciones narrativas. 100. Solución: b

a. Incorrecta. En general, un diagrama de flujo vertical se diseña para proveer descripciones escritas.

b. Correcta. Si se pone énfasis en el flujo de procesos entre departamentos y/o personas, se muestra con mayor claridad cualquier separación inapropiada de deberes y falta de verificaciones independientes en el desempeño.

c. Incorrecta. En general, es más corto, porque no se provee espacio para descripciones escritas. d. Incorrecta. Sigue una transacción desde que comienza hasta que se archiva,

independientemente de las líneas departamentales. FIN DE LAS SOLUCIONES PARTE II

El no cumplir con estas instrucciones y con las guías de "Instrucciones para candidatos" puede afectar seriamente

su derecho para recibir los resultados de este examen y su futura participación el programa de Auditor Interno Certificado.

Todos los documentos presentados a la terminación de cualquiera de las

partes de este examen son propiedad exclusiva de The Institute of Internal Auditor, Inc. Los candidatos no puede revelar el contenido de este examen a menos que cuenten con la

autorización del Departamento de Certificación.

III - 1

Preguntas del examen modelo para el Auditor Interno Certificado (CIA)

Parte III – Análisis de los negocios y tecnología

de la información

Preguntas del examen modelo parte III: 100

Preguntas del examen CIA real parte III: 125 (ver explicación en el “prólogo”, página iii)

Tiempo permitido para terminar el examen CIA parte III: 210 minutes

Instrucciones como las que a continuación se enuncian serán las que aparezcan

en la portada de cada examen CIA. Por favor, léanlas con cuidado.

1. Anote su número de candidato en la hoja de respuestas en el espacio proporcionado.

2. No haga anotaciones irrelevantes en la hoja de respuestas

3. Aségurese de que todo cambio de respuesta sea completamente borrado.

4. Todas las referencias al Marco para la Práctica Profesional se refieren al Marco para la Práctica Profesional del IAI, que incluye las Normas y los Consejos para la Práctica. Todas las referencias a las Normas se refieren a las Normas Internacionales para el Ejercicio Profesional de la Auditoría Interna resumidas en el Marco para la Práctica Profesional del IAI.

III - 2

1. Los costos de calidad en los cuales se incurrió para detectar unidades que no se ajustan a las especificaciones del producto se conocen como: a. Costos de prevención. b. Costos de evaluación. c. Costos de reproceso. d. Costos de fallas.

2. El uso de equipos de gestión de calidad total

es importante porque: a. Los equipos bien conducidos pueden

resultar sumamente creativos y abordar problemas complejos mejor que los individuos.

b. Los equipos son más rápidos para tomar decisiones; en consecuencia, ayudan a reducir el tiempo del ciclo.

c La motivación de los empleados es mayor para los integrantes de equipos que para quienes contribuyen en forma individual.

d. El uso de equipos elimina la necesidad de supervisión; en consecuencia, contribuye a la rentabilidad de la compañía.

3. ¿Cuál de los siguientes es el elemento

principal de las normas ISO 9000: 2000 sobre sistemas de gestión de calidad? a. El principio que mejoró la satisfacción del

empleado llevará a un incremento de la productividad.

b. La actitud y las acciones del consejo y de la gerencia respecto de la significatividad del control dentro de la organización.

c. La evaluación del riesgo que determina que no se alcanzan los objetivos.

d. Un requerimiento para que las organizaciones supervisen la información sobre la satisfacción del cliente como medida de desempeño.

1. The costs of quality that are incurred in detecting units of product that do not conform to product specifications are referred to as: a. Prevention costs. b. Appraisal costs. c. Rework costs. d. Failure costs.

2. The use of teams in total quality management

is important because: a. Well-managed teams can be highly

creative and are able to address complex problems better than individuals can.

b. Teams are quicker to make decisions, thereby helping to reduce cycle time.

c Employee motivation is higher for team members than for individual contributors.

d. The use of teams eliminates the need for supervision, thereby allowing a company to become leaner and more profitable.

3. Which of the following is a major element of

the ISO 9000:2000 quality management system standards? a. The principle that improved employee

satisfaction will lead to increased productivity.

b. The attitude and actions of the board and management regarding the significance of control within the organization.

c. The assessment of the risk that objectives are not achieved.

d. A requirement for organizations to monitor information on customer satisfaction as a measure of performance.

III - 3

4. El gerente de ventas de una constructora de yates desarrolló la siguiente tabla para la producción anual y las ventas:

Demanda 10 20 30 50 Probabilidad 0.1 0.2 0.5 0.2

Yates Construidos Utilidades esperadas 10 10 10 10 10 20 0 20 20 20 30 -10 10 30 30 50 -30 -10 10 50

De acuerdo con la tabla, ¿cuántos yates deben construirse? a. 10. b. 20. c. 30. d. 50.

5. En el pronóstico del nivel de inventario de una

empresa, todo lo siguiente resulta útil excepto: a. El conocimiento del comportamiento de

los ciclos del negocio. b. Las asignaciones contables internas de

los costos a diferentes segmentos de la firma.

c. La información de las variaciones estacionales de la demanda.

d. La modelación econométrica. 6. El proceso de agregar recursos para reducir

tiempos de actividad seleccionados en el camino crítico de la programación de proyectos se denomina: a. Estallido (crashing). b. La técnica Delphi. c. Análisis ABC. d. Una solución “branch and bound”.

7. La siguiente información se refiere a un

proyecto:

Actividad Tiempo (dias) Actividad

Precedente Inmediata A 5 None B 3 None C 4 A D 2 B E 6 C, D

La fecha límite más cercana para el proyecto es: a. 11 días. b. 14 días. c. 15 días. d. 20 días.

4. The sales manager for a builder of custom yachts developed the following conditional table for annual production and sales:

Demand 10 20 30 50 Probability 0.1 0.2 0.5 0.2

Yachts Built Expected Profit 10 10 10 10 10 20 0 20 20 20 30 -10 10 30 30 50 -30 -10 10 50

According to the table, how many yachts should be built? a. 10. b. 20. c. 30. d. 50.

5. All of the following are useful for forecasting the needed level of inventory except: a. Knowledge of the behavior of business

cycles. b. Internal accounting allocations of costs to

different segments of the company. c. Information about seasonal variations in

demand. d. Econometric modeling.

6. The process of adding resources to shorten

selected activity times on the critical path in project scheduling is called: a. Crashing. b. The Delphi technique. c. ABC analysis. d. A branch-and-bound solution.

7. The following information applies to a project:

Activity Time (days) Immediate Predecessor

A 5 None B 3 None C 4 A D 2 B E 6 C, D

The earliest completion time for the project is: a. 11 days. b. 14 days. c. 15 days. d. 20 days.

III - 4

8. ¿Cuál de las siguientes técnicas permitiría a un fabricante de recursos limitados maximizar sus ganancias? a. La técnica Delphi. b. Suavización exponencial. c. Análisis de regression. d. Programación lineal.

9. Una forma de limitar las demoras en

producción causadas por las fallas del equipamiento y su reparación es: a. Programar la producción sobre la base de

la capacidad. b. Un plan de mantenimiento basado en un

análisis de las órdenes de reparación. c. Autorizar previamente el pago de

mantenimiento de equipos y horas extras. d. Establecer un programa de

mantenimiento preventivo para todo el equipo de producción.

10. Para eliminar el efecto de variaciones

estacionales de una serie temporal, los datos originales deben: a. Incrementarse por el factor estacional. b. Reducirse por el factor estacional. c. Multiplicarse por el factor estacional. d. Dividirse por el factor estacional.

11. Una ventaja de utilizar códigos de barras en

lugar de otros medios de identificación es que: a. Se controla el movimiento de todos los

lotes. b. El movimiento de los lotes se registra en

forma fácil y rápida. c. Los proveedores pueden utilizar los

mismos números de lotes. d. Los proveedores utilizan los mismos

métodos de identificación. 12. Una técnica apropiada para planificar y

controlar inventarios de fabricación, tales como materias primas, componentes y montajes menores cuya demanda depende del nivel de producción es: a. La planificación de requerimientos de

materiales. b. El análisis de regresión. c. La presupuestación del capital. d. La programación lineal.

8. Which of the following will allow a manufacturer with limited resources to maximize profits? a. The Delphi technique. b. Exponential smoothing. c. Regression analysis. d. Linear programming.

9. A means of limiting production delays caused

by equipment breakdown and repair is to: a. Schedule production based on capacity

planning. b. Plan maintenance activity based on an

analysis of equipment repair work orders. c. Pre-authorize equipment maintenance

and overtime pay. d. Establish a preventive maintenance

program for all production equipment. 10. To remove the effect of seasonal variation

from a time series, original data should be: a. Increased by the seasonal factor. b. Reduced by the seasonal factor. c. Multiplied by the seasonal factor. d. Divided by the seasonal factor.

11. An advantage of using bar codes rather than

other means of identification of parts used by a manufacturer is that: a. The movement of all parts is controlled. b. The movement of parts is easily and

quickly recorded. c. Vendors can use the same part numbers. d. Vendors use the same identification

methods. 12. An appropriate technique for planning and

controlling manufacturing inventories, such as raw materials, components, and sub-assemblies, whose demand depends on the level of production is: a. Materials requirements planning. b. Regression analysis. c. Capital budgeting. d. Linear programming.

III - 5

13. Si mediante una política de compras “just-in-time” (justo-a-tiempo) se logran reducir los costos totales de inventario de una compañía manufacturera, ¿cuál de las siguientes combinaciones es más probable? a. Un aumento en los costos de compras y

una disminución de los costos de faltantes de stock.

b. Un aumento en los costos de compras y una disminución de los costos de calidad.

c. Un aumento en los costos de calidad y una disminución en los costos de las órdenes de compra.

d. Un aumento en los costos de faltantes de stock y una disminución en los costos de transporte.

14. En un modelo de cantidad económica de

pedidos (EOQ, por sus siglas en inglés) tanto los costos por orden como los costos de tenencia son estimados. Si dichos estimados se modifican para determinar en qué medida los cambios afectan a la EOQ óptima, dicho análisis debe denominarse: a. Modelo de pronósticos. b. Análisis de sensibilidad. c. Análisis del método del camino crítico. d. Análisis de decisión.

15. La venta de una película para usar con una

cámara es un ejemplo de una de las siguientes estrategias de fijación de precios por combinación de productos. ¿Cuál? a. Fijación de precios de sub-productos. b. Fijación de precios de productos

opcionales. c. Fijación de precios de productos cautivos. d. Fijación de precios por paquete de

productos. 16. Una estrategia de marketing competitivo en la

cual una empresa se especializa en servir a los clientes pasados por alto o ignorados por competidores importantes se denomina: a. Estrategia de líderes del mercado. b. Estrategia de desafío del mercado. c. Estrategia de seguidores del mercado. d. Estrategia de nichos del mercado.

13. If a just-in-time purchasing policy is successful in reducing the total inventory costs of a manufacturing company, which of the following combinations of cost changes would be most likely to occur? a. An increase in purchasing costs and a

decrease in stockout costs. b. An increase in purchasing costs and a

decrease in quality costs. c. An increase in quality costs and a

decrease in ordering costs. d. An increase in stockout costs and a

decrease in carrying costs. 14. In an economic order quantity (EOQ) model,

both the costs per order and the holding costs are estimates. If those estimates are varied to determine how much the changes affect the optimal EOQ, such analysis would be called a: a. Forecasting model. b. Sensitivity analysis. c. Critical path method analysis. d. Decision analysis.

15. When film is sold for use with a camera, this is

an example of which of the following product mix pricing strategies? a. By-product pricing. b. Optional product pricing. c. Captive product pricing. d. Product bundle pricing.

16. A competitive marketing strategy in which a

firm specializes in serving customers overlooked or ignored by major competitors is called a: a. Market leader strategy. b. Market challenger strategy. c. Market follower strategy. d. Market niche strategy.

III - 6

17. ¿Cuál de los siguientes procedimientos de contratación de personal proporciona mayor control sobre la exactitud de la información contenida en una solicitud de empleo? a. A los solicitantes se les requiere

acompañar copias no oficiales de sus certificados junto con la solicitud, como verificación de sus credenciales académicas.

b. La organización contratante llama al último empleo de cada finalista para verificar la duración en el empleo y el puesto desempeñado.

c. Cartas de recomendación que certifiquen la actuación del solicitante deben ser enviadas por correo directamente a la organización contratante, en vez de ser presentadas por el solicitante.

d. A los solicitantes se les requiere firmar para expresar que la información que aparece en la solicitud es verídica y correcta, como confirmación de su veracidad.

18. Un tablero de comando se relaciona, en primer

lugar, con: a. El personal. b. La estructura. c. La estrategia. d. Los sistemas.

19. La práctica de registrar pagos por anticipado

de clientes como pasivos es una aplicación: a. De la presunción de empresa en marcha. b. De la presunción de la unidad monetaria. c. del principio del costo histórico. d. Del principio de reconocimiento de

ingresos. 20. ¿Cuál de los siguientes casos es un ejemplo

de pasivo contingente? a. Una tienda minorista de un shopping

paga al arrendador un alquiler mínimo mensual más un porcentaje acordado de ventas.

b. Una compañía se niega a pagar la factura por los honorarios de auditoría anual porque el monto parece ser superior al acordado con el socio del estudio contable.

c. Una compañía incrementa el impuesto a las ganancias a pagar en los estados financieros preliminares.

d. Un arrendatario está de acuerdo con reembolsar al arrendador por un déficit en el valor residual de los activos bajo contrato de arrendamiento.

17. Which of the following hiring procedures provides the most control over the accuracy of information submitted on an employment application? a. Applicants are required to submit

unofficial copies of their transcripts along with the application as verification of their educational credentials.

b. The hiring organization calls the last place of employment for each finalist to verify the employment length and position held.

c. Letters of recommendation which attest to the applicant’s character must be mailed directly to the hiring organization rather than being submitted by the applicant.

d. Applicants are required to sign that the information on the applicant is true and correct as a confirmation of the truth of the information in the application.

18. A balanced scorecard is primarily concerned

with: a. Staff. b. Structure. c. Strategy. d. Systems.

19. The practice of recording advanced payments

from customers as liabilities is an application of the: a. Going concern assumption. b. Monetary unit assumption. c. Historic cost principle. d. Revenue recognition principle.

20. Which of the following is an example of a

contingent liability? a. A retail store in a shopping mall pays the

lessor a minimum monthly rent plus an agreed-upon percentage of sales.

b. A company is refusing to pay the invoice for the annual audit because it seems higher than the amount agreed upon with the public accounting firm's partner.

c. A company accrues income tax payable in its interim financial statements.

d. A lessee agrees to reimburse a lessor for a shortfall in the residual value of an asset under lease.

III - 7

21. ¿Cuál de los siguientes debe ser parte de cualquier modelo de riesgos que incluya valuación de inventario? a. Políticas de garantía de productos. b. Política de fijación de precios para los

proveedores. c. Gastos de reducción de inventario. d. Pronósticos anuales de ventas.

22. Una compañía utiliza la depreciación en línea

recta para propósitos de información financiera, pero utiliza la depreciación acelerada para propósitos de impuestos. ¿Cuál de los siguientes saldos de cuentas sería más bajo en los estados financieros utilizados para propósitos tributarios que en los estados financieros utilizados para propósitos generales? a. Depreciación acumulada. b. Efectivo. c. Utilidades retenidas. d. Activos fijos brutos.

23. Mediante un plan definido de pensiones de

contribuciones, se informa <Lista A> en el balance general sólo si la cantidad con la que la organización contribuyó al fideicomiso de pensiones es <Lista B> la requerida.

Lista A Lista B a. Un activo Mayor que b. Un activo Igual a c. Un pasivo Mayor que d. Un pasivo Igual a

24. Si en un contrato de arrendamiento (leasing) el

propietario transfiere substancialmente todos los beneficios y riesgos del activo al arrendatario, el valor del activo se reconoce en los libros del arrendatario como un activo <Lista A> y el contrato de arrendamiento es un contrato <Lista B>.

Lista A Lista B a. Tangible De capital b. Intangible De capital c. Tangible Operativo d. Intangible Operativo

25. ¿Cuál de los siguientes valores es probable

que tenga el menor riesgo? a. Los bonos de renta. b. Los debentures o bonos sin garantía

específica. c. Los debentures subordinados. d. Los bonos de garantía hipotecaria.

21. Which must be part of any risk model involving inventory valuation? a. Product warranty policies. b. Vendor pricing policies. c. Inventory shrinkage expense. d. Annual sales forecasts.

22. A company uses straight-line depreciation for

financial reporting purposes, but uses accelerated depreciation for tax purposes. Which of the following account balances would be lower in the financial statements used for tax purposes than it would be in the general purpose financial statements? a. Accumulated depreciation. b. Cash. c. Retained earnings. d. Gross fixed assets.

23. Under a defined contribution pension plan,

<List A> is reported on the balance sheet only if the amount the organization has contributed to the pension trust is <List B> the amount required.

List A List B a. An asset Greater than b. An asset Equal to c. A liability Greater than d. A liability Equal to

24. If a lease agreement transfers substantially all of the benefits and risks of ownership of the asset to the lessee, the asset value is recognized on the lessee’s books as <List A> asset and the lease is <List B> lease.

List A List B a. A tangible A capital b. An intangible A capital c. A tangible An operating d. An intangible An operating

25. Which of the following securities is likely to

have the least risk? a. Income bonds. b. Debentures. c. Subordinated debentures. d. First-mortgage bonds.

III - 8

26. Una compañía estadounidense y una alemana compraron las mismas acciones en una bolsa europea y las tuvieron durante un año. Si durante ese período, el valor del euro se debilitó frente al dólar, al comparar con el rendimiento de la compañía europea, el de la compañía estadounidense será: a. Menor. b. Mayor. c. Igual. d. No es posible determinarlo por la

información disponible. 27. En una oferta de fusión de dos niveles, los

accionistas reciben una cantidad mayor por acción si: a. Aceptan comprar bonos recién emitidos

en la empresa combinada. b. Aceptan volver a vender a la compañía

cualquier bono que posean en este momento.

c. Ofrecen sus existencias más tarde. d. Ofrecen sus existencias más temprano.

28. ¿Qué combinación de razones puede

utilizarse para obtener el rendimiento del capital? a. La razón del valor del mercado con

respecto al valor de libros y la razón del valor de la deuda total con respecto al activo total.

b. La razón del precio con respecto a las utilidades, las utilidades por acción y el margen de utilidad neta.

c. La razón del precio con respecto a las utilidades y el rendimiento de los activos.

d. El margen de utilidad neta, la rotación de activos y el multiplicador del capital.

26. A U.S. company and a European company purchased the same stock on a European stock exchange and held the stock for one year. If the value of the euro weakened against the U.S. dollar during the period, in comparison with the European company’s return, the U.S. company’s return will be: a. Lower. b. Higher. c. The same. d. Indeterminate from the information

provided. 27. In a two-tier merger offer, shareholders receive

a higher amount per share if they: a. Agree to purchase newly issued bonds in

the combined firm. b. Agree to sell back to the firm any bonds

that they currently own. c. Tender their stock later. d. Tender their stock earlier.

28. Which combination of ratios can be used to

derive return on equity? a. Market value to book value ratio and total

debt to total assets ratio. b. Price to earnings ratio, earnings per

share, and net profit margin. c. Price to earnings ratio and return on

assets. d. Net profit margin, asset turnover, and

equity multiplier.

III - 9

29. ¿Cuál de las siguientes declaraciones es verdad con respecto al impacto de la inflación en el análisis de razones financieras? a. La inflación impacta sólo en aquellas

razones computadas a partir de las cuentas del balance.

b. La inflación impacta en el análisis de las razones de una empresa a través del tiempo, pero no en el análisis comparativo de empresas con distinto tiempo de antigüedad.

c. La inflación impacta en el análisis de razones de una empresa a través del tiempo, así como en el análisis comparativo de empresas con distinto tiempo de antigüedad.

d. La inflación impacta en el análisis comparativo de empresas con distinto tiempo de antigüedad, pero no en el análisis de razones financieras de una empresa a través del tiempo.

30. La diferencia entre la tasa de rendimiento

requerida en una inversión riesgosa dada y la de una inversión libre de riesgos con el mismo rendimiento es: a. La prima de riesgo. b. El coeficiente de variación. c. El error estándar de medición. d. El coeficiente beta.

31. Las decisiones de la estructura del capital

implican determinar las proporciones de financiamiento por medio: a. De la deuda de corto o largo plazo. b. De la deuda o del capital. c. Del activo de corto o largo plazo. d. De las utilidades acumuladas o acciones

comunes. 32. Si se venden bonos con descuento y se utiliza

el método de amortización denominado de interés efectivo, los gastos por intereses: a. Se incrementarán de un período a otro. b. Se mantendrán constantes de un período

a otro. c. Serán iguales al pago en efectivo de los

intereses en cada período. d. Será menor que el pago en efectivo de

los intereses en cada período.

29. Which of the following is true about the impact of price inflation on financial ratio analysis? a. Inflation impacts only those ratios

computed from balance sheet accounts. b. Inflation impacts financial ratio analysis

for one firm over time, but not comparative analysis of firms of different ages.

c. Inflation impacts financial ratio analysis for one firm over time, as well as comparative analysis of firms of different ages.

d. Inflation impacts comparative analysis of firms of different ages, but not financial ratio analysis for one firm over time.

30. The difference between the required rate of

return on a given risky investment and that of a risk-free investment with the same expected return is the: a. Risk premium. b. Coefficient of variation. c. Standard error of measurement. d. Beta coefficient.

31. Capital structure decisions involve determining

the proportions of financing from: a. Short-term or long-term debt. b. Debt or equity. c. Short-term or long-term assets. d. Retained earnings or common stock.

32. If bonds are sold at a discount and the

effective interest method of amortization is used, interest expense will: a. Increase from one period to another. b. Remain constant from one period to

another. c. Equal the cash interest payment each

period. d. Be less than the cash interest payment

each period.

III - 10

33. Si se fija un alto porcentaje de los costos totales de una empresa, el impacto operativo de la empresa: a. Será alto. b. Será bajo. c. No cambiará. d. No puede determinarse.

34. Al comparar dos empresas, si todo lo demás

se mantiene igual, la empresa con el coeficiente más alto de pago de dividendos tendrá: a. Un costo marginal de capital más alto. b. Una menor relación de deuda. c. Un programa de oportunidad de

inversiones más alto. d. Un precio mayor de la tasa de ganancias.

35. ¿Por qué una compañía mantendría un saldo

de caja compensado? a. Para efectuar pagos y cobranzas de

rutina. b. Para pagar servicios bancarios. c. Para proveer una reserva en caso de

imprevistas fluctuaciones de los flujos de caja.

d. Para aprovechar oportunidades de compras económicas que puedan surgir.

36. La teoría de los mercados eficientes implica

que los precios de los valores: a. No representan una buena estimación de

los futuros flujos de caja. b. Son justos y reflejan toda la información

disponible al público. c. No constituyen el mejor parámetro para

las decisiones corporativas de índole financiera.

d. Son siempre menores que su precio justo.

33. If a high percentage of a firm’s total costs is fixed, the firm’s operating leverage will be: a. High. b. Low. c. Unchanged. d. Unable to be determined.

34. When comparing two companies, if all else is

equal, the company that has a higher dividend payout ratio will have a: a. Higher marginal cost of capital. b. Lower debt ratio. c. Higher investment opportunity schedule. d. Higher price to earnings ratio.

35. Why would a company maintain a

compensating cash balance? a. To make routine payments and

collections. b. To pay for banking services. c. To provide a reserve in case of

unforeseen fluctuations in cash flows. d. To take advantage of bargain purchase

opportunities that may arise. 36. The efficient markets theory implies that

securities prices are: a. Not a good estimate of future cash flows. b. Fair and a reflection of all publicly

available information. c. Not the best benchmark for corporate

financial decisions. d. Always less than their fair value.

III - 11

Utilice la siguiente gráfica para responder las preguntas 37 y 38.

37. Si todo lo demás se mantiene igual, las

empresas con mayores márgenes de utilidades requieren menos financiamiento adicional para cualquier tasa de crecimiento en ventas. Si el margen de utilidad de una empresa aumentara, la línea de fondos requeridos cambiaría: a. Hacia arriba y con menor pendiente. b. Hacia arriba y con mayor pendiente. c. Hacia abajo y con menor pendiente. d. Hacia abajo y con mayor pendiente.

38. La línea de fondos requeridos no pasa a

través del vértice de origen a menos que la empresa tenga: a. Una política de pago de dividendos del

100 por ciento. b. Una política de pago de dividendos del

cero por ciento. c. Una tasa de crecimiento en ventas del

100 por ciento. d. Una tasa de crecimiento en ventas del

cero por ciento.

Use the following graph to answer questions 37 through 38.

37. If all else is equal, firms with higher profit

margins require less additional financing for any sales growth rate. If the profit margin of a company increased, the funds-needed line would shift: a. Up and become less steep. b. Up and become more steep. c. Down and become less steep. d. Down and become more steep.

38. The funds-needed line does not pass through

the origin unless the firm has a: a. 100 percent dividend payout policy. b. Zero percent dividend payout policy. c. 100 percent sales growth rate. d. Zero percent sales growth rate.

Additional Financing Needed ($)

Sales Growth (%)

Funds Needed Financiamiento Adicional Requerido ($)

Fondos Requeridos

Crecimiento en Ventas (%)

III - 12

Utilice la siguiente información para responder las preguntas 39 y 40. El 1 de enero, una compañía no tiene saldo de apertura de inventario. Se realizan las siguientes compras durante el año: Unidades Costo Compradas de Unidades 1 de enero 5,000 $10.00 1 de abril 5,000 $9.00 1 de julio 5,000 $8.00 1 de octubre 5,000 $7.50

Hay 10.000 unidades en el inventario del 31 de diciembre. 39. Si la compañía utiliza el método de valuación

“primero entrado, primero salido” (FIFO, por sus siglas en inglés) el saldo final del inventario será: a. $77,500. b. $85,000. c. $86,250. d. $95,000.

40. Si la compañía utiliza el método de valuación

“último entrado, último salido” (LIFO; por sus siglas en inglés) el costo de los productos vendidos para el año será: a. $77,500. b. $86,250. c. $87,500. d. $95,000.

41. El valor económico de una compañía

aumentará después de un aumento en: a. El flujo de efectivo neto. b. El riesgo sistemático. c. El riesgo no sistemático. d. La tasa de descuento.

42. Los deterioros anormales:

a. No se espera que ocurran cuando se usan costos estándar.

b. No son en general controlables por parte del supervisor de producción.

c. Son el resultado de normas no realistas de producción.

d. No se espera que ocurran en condiciones eficientes de operación.

43. ¿Cuál de los siguientes es un costo del

producto para una compañía manufacturera? a. El seguro del edificio de sedes

corporativas. b. Los impuestos de propiedad de una

fábrica. c. La depreciación del automóvil de un

vendedor. d. El sueldo de un gerente de ventas.

III - 13

Use the following information to answer questions 39 through 40.

On January 1, a company has no opening inventory balance. The following purchases are made during the year: Units Unit Purchased Cost January 1 5,000 $10.00 April 1 5,000 $9.00 July 1 5,000 $8.00 October 1 5,000 $7.50

There are 10,000 units in inventory on December 31. 39. If the company uses the first-in, first-out (FIFO)

method of inventory valuation, the ending inventory balance will be: a. $77,500. b. $85,000. c. $86,250. d. $95,000.

40. If the company uses the last-in, first-out (LIFO)

method of inventory valuation, cost of goods sold for the year will be: a. $77,500. b. $86,250. c. $87,500. d. $95,000.

41. The economic value of a firm will rise following

an increase in: a. Net cash flow. b. Systematic risk. c. Unsystematic risk. d. The discount rate.

42. Abnormal spoilage is:

a. Not expected to occur when standard costs are used.

b. Not usually controllable by the production supervisor.

c. The result of unrealistic production standards.

d. Not expected to occur under efficient operating conditions.

43. Which of the following is a product cost for a

manufacturing company? a. Insurance on the corporate headquarters

building. b. Property taxes on a factory. c. Depreciation on a salesperson’s vehicle. d. The salary of a sales manager.

III - 14

Utilice la siguiente información para responder las preguntas 44 y 45. Una empresa que tiene un 18 por ciento de costo de capital, considera los siguientes proyectos (al 1º de enero del año 1):

Ene. 1, AñorUno,

Egresos enefectivo

(000’s de $)

Dic. 31, AñorCinco,

Ingresos enefectivo

(000’s de $)

Tasa internade

rendimientodel proyecto

Proyecto A $3,500 $7,400 15%Proyecto B $4,000 $9,950 ?

Valor presente de $1 al final de “N” períodos N 12% 14% 15% 16% 18% 20% 22% 4 .6355 .5921 .5718 .5523 .5158 .4823 .4230 5 .5674 .5194 .4972 .4761 .4371 .4019 .3411 6 .5066 .4556 .4323 .4104 .3704 .3349 .2751 44. Con el uso del método de valor presente neto, dicho valor para el proyecto A es:

a. $(316,920). b. $(265,460). c. $0. d. $316,920.

45. La tasa interna de rendimiento del proyecto B

llega casi al: a. 15 por ciento. b. 18 por ciento. c. 20 por ciento. d. 22 por ciento.

46. Un presupuesto flexible es una expresión

cuantitativa de un plan que: a. Se desarrolla para el nivel real de

producción alcanzado durante el período del presupuesto.

b. Está constituido por el presupuesto de ingresos y sus cédulas soporte del período.

c. Se centra en los costos de las actividades necesarias para producir y vender productos y servicios durante un período presupuestal.

d. Proyecta costos sobre la base de futuras mejoras en las prácticas y los procedimientos ya existentes durante un período presupuestal.

Use the following information to answer questions 44 through 45.

A firm with an 18 percent cost of capital is considering the following projects (on January 1 of year one):

Jan. 1, YearOne,

Cash Outflow(000’s Omitted)

Dec. 31, YearFive,

Cash Inflow(000’s Omitted)

ProjectInternal Rate

of ReturnProject A $3,500 $7,400 15%Project B $4,000 $9,950 ?

Present Value of $1 Due at the End of “N” Periods N 12% 14% 15% 16% 18% 20% 22% 4 .6355 .5921 .5718 .5523 .5158 .4823 .4230 5 .5674 .5194 .4972 .4761 .4371 .4019 .3411 6 .5066 .4556 .4323 .4104 .3704 .3349 .2751 44. Using the net present value method, project

A’s net present value is: a. $(316,920). b. $(265,460). c. $0. d. $316,920.

45. Project B’s internal rate of return is closest to: a. 15 percent. b. 18 percent. c. 20 percent. d. 22 percent.

46. A flexible budget is a quantitative expression of

a plan that: a. Is developed for the actual level of output

achieved for the budget period. b. Is comprised of the budgeted income

statement and its supporting schedules for a budget period.

c. Focuses on the costs of activities necessary to produce and sell products and services for a budget period.

d. Projects costs on the basis of future improvements in existing practices and procedures during a budget period.

III - 15

47. Una compañía tiene divisiones de ventas en el país X y el país Y. La compañía vende un solo producto, y producir le cuesta $20 por unidad. No existen barreras comerciales o tarifas entre los tres países. La información específica para cada país es la siguiente:

País

Tasa Corporativa de Impuesto

Precio de Venta antes de Impuesto

Cantidad Vendida

Costos Adicionales

Home 50% $30 1,500 $10,000 X 60% $40 1,000 $12,500 Y 40% $35 2,000 $11,000

Cuando vende productos a su(s) división(es) de ventas en <Lista A>, la compañía debe fijar el <Lista B> precio de transferencia posible.

Lista A Lista B a. Países X y Y Más alto b. Países X y Y Más bajo c. País X Más alto d. Pais Y Más alto

48. El presupuesto maestro:

a. Muestra los resultados pronosticados y reales.

b. Sólo contiene costos controlables. c. Puede utilizarse para determinar las

variaciones de los costos de fabricación. d. Contiene el presupuesto operativo.

49. Una compañía produjo y vendió 100,000

unidades de un componente con un costo variable de $20 por unidad. De las unidades producidas, 1,200 no cumplieron con las especificaciones de tolerancia de la compañía y fueron reprocesadas a un costo de $12 por unidad. Estas unidades se vendieron como “segundas” de fábrica a $45 cada una y las unidades de primera calidad se vendieron a $50. Si la compañía hubiera implantado un programa de aseguramiento de la calidad para asegurarse de que todas las unidades se produjeron conforme a las especificaciones, el incremento en el margen de contribución de este componente habría sido: a. $14,400. b. $20,400. c. $21,600. d. $39,600.

47. A domestic company has sales divisions in country X and country Y. The company sells only one product, which costs $20 per unit to produce. There are no trade barriers or tariffs among the three countries. Information specific to each of the three countries is as follows:

Country Corporate Tax Rate

Before-Tax Selling Price

Quantity Sold

Additional Costs

Home 50% $30 1,500 $10,000 X 60% $40 1,000 $12,500 Y 40% $35 2,000 $11,000 When selling items to its sales division(s) in <List A>, the company should set the <List B> allowable transfer price.

List A List B a. Countries X and Y Highest b. Countries X and Y Lowest c. Country X Highest d. Country Y Highest

48. A master budget:

a. Shows forecasted and actual results. b. Contains only controllable costs. c. Can be used to determine manufacturing

cost variances. d. Contains the operating budget.

49. A company produced and sold 100,000 units

of a component with a variable cost of $20 per unit. Of the units produced, 1,200 failed the company's tolerance specifications and were reworked at a cost of $12 per unit. Reworked units were sold as factory seconds at $45 each, and first-quality units were sold at $50 each. If the company had implemented a quality assurance program to ensure that all units produced conformed to specifications, the increase in the company's contribution margin from this component would have been: a. $14,400. b. $20,400. c. $21,600. d. $39,600.

III - 16

50. Una compañía tiene capacidad ociosa en activos fijos relacionados con su proceso de producción. Si en un año dado estos activos sólo se utilizaron al 80 por ciento de capacidad y ese año el nivel de ventas fue de $2,000,000, el nivel de ventas a la capacidad total es: a. $1,600,000. b. $2,000,000. c. $2,500,000. d. $10,000,000.

Utilice la siguiente información para responder las preguntas 51 y 52.

Una empresa cosecha, empaca y despacha todo lo que produce. Opera con tres líneas de empaque. A continuación, se presenta un resumen de sus costos de inventario de producto terminado: Gastos por sueldos de empleados en

línea de empaque $150,000

Gastos por sueldos de supervisión de la línea de empaque

$90,000

Gastos por sueldos de control de calidad

$30,000

Gastos de empaque $15,000Gastos por electricidad $3,000Gastos por depreciación $66,000 51. Los costos de las líneas de empaque se

acumularían en parte por: a. Registrar los gastos de la nómina por

categoría de puestos. b. Calcular los gastos de depreciación. c. Preparar estados financieros mensuales. d. Pronosticar faltantes de material

mensualmente. 52. Al finalizar el período de informes, se

empacaron y despacharon 600.000 unidades. No queda inventario disponible. Si la empresa emplea el método de costo por procesos, el costo por unidad sería de: a. $0,197 b. $0,275 c. $0,315 d. $0,590

50. A company has excess capacity in production-related fixed assets. If in a given year these fixed assets were being used to only 80 percent of capacity and the sales level in that year was $2,000,000, the full capacity sales level is: a. $1,600,000. b. $2,000,000. c. $2,500,000. d. $10,000,000.

Use the following information to answer questions 51 through 52.

A company harvests, packs, and ships all of its own produce. The company operates three packing lines. A summary of completed inventory costs is as follows: Packing-line employee salary expense $150,000Packing-line supervision salary expense $90,000Quality control salary expense $30,000Packing crates expense $15,000Electricity expense $3,000Depreciation expense $66,000 51. Costs for the packing lines would be

accumulated in part by: a. Recording payroll expense by employee

job category. b. Computing depreciation expense. c. Producing monthly financial statements. d. Forecasting monthly material shortages.

52. At the end of the reporting period, 600,000

units had been packed and shipped. No inventory remained on hand. If the company used process costing, the cost per unit would be: a. $0.197 b. $0.275 c. $0.315 d. $0.590

III - 17

53. ¿Cuál de las siguientes sería una base razonable para asignar los costos de manejo de material a las unidades producidas en un sistema de costos basado en actividades? a. Producción por año. b. Cantidad de componentes por unidad

completa. c. Cantidad de tiempo requerido para

producir una unidad. d. Cantidad de costos fijos aplicados a cada

unidad completa. 54. A menudo, el método de utilidad o beneficio

residual se prefiere sobre el método de retorno de la inversión (ROI) como una evaluación del rendimiento porque: a. El método de utilidad residual es una

medida a lo largo del tiempo, en tanto que el ROI representa los resultados por un solo período.

b. El método de utilidad residual se concentra en maximizar el dinero absoluto de utilidad, en lugar de considerar sólo un porcentaje del rendimiento, como lo hace el ROI.

c. LA tasa imputada de interés utilizada en el cálculo de la utilidad residual se puede derivar más fácilmente que la tasa objetivo que se compara con el ROI, una vez que ésta se ha calculado.

d. La inversión promedio se emplea con el método de utilidad residual en tanto que la inversión a fin de año se utiliza para calcular el ROI.

55. ¿Cuál de los siguientes costos no son

relevantes en una decisión respecto de una orden especial de fabricación? a. Costos incrementales. b. Costos de oportunidad. c. Costos que se desembolsan. d. Costos en los cuales ya se incurrió.

56. ¿Cuál de las siguientes es una herramienta de

política monetaria que el banco central de una nación podría usar para estabilizar la economía durante un período de inflación? a. Vender títulos públicos. b. Reducir los requisitos de reserva de los

bancos. c. Reducir las tasas de descuento de los

bancos. d. Procurar que suban las tasas de

impuestos.

53. Which of the following would be a reasonable basis for allocating the material handling costs to the units produced in an activity-based costing system? a. Number of production runs per year. b. Number of components per completed

unit. c. Amount of time required to produce one

unit. d. Amount of overhead applied to each

completed unit. 54. Residual income is often preferred over return

on investment (ROI) as a performance evaluation because: a. Residual income is a measure over time

while ROI represents the results for a single time period.

b. Residual income concentrates on maximizing absolute dollars of income rather than a percentage return as with ROI.

c. The imputed interest rate used in calculating residual income is more easily derived than the target rate that is compared to the calculated ROI.

d. Average investment is employed with residual income while year-end investment is employed with ROI.

55. Which of the following costs are not relevant in a special-order decision? a. Incremental costs. b. Opportunity costs. c. Outlay costs. d. Sunk costs.

56. Which of the following is a tool of monetary

policy that a nation’s central bank could use to stabilize the economy during an inflationary period? a. Selling government securities. b. Lowering bank reserve requirements. c. Lowering bank discount rates. d. Encouraging higher tax rates.

III - 18

57. Si un país establece cuotas para superar déficits comerciales crónicos, es probable que: a. Suban los índices de desempleo y de

producción. b. Suban los índices de desempleo y bajen

los de productividad. c. Bajen los índices de desempleo y suban

los de productividad. d. Bajen los índices de desempleo y

productividad. 58. ¿Cuál de las siguientes declaraciones

describe correctamente la introducción de créditos fiscales gubernamentales en las inversiones? a. Las inversiones corporativas tendrán

mayores valores presentes netos que si no existiera el crédito fiscal. Todo lo demás queda igual.

b. Los créditos fiscales en las inversiones están diseñados para restringir la inflación.

c. Los créditos fiscales en las inversiones incrementan los costos de inversión y, con todo lo demás igual, reducen el nivel de la inversión corporativa.

d. Los créditos fiscales en las inversiones son impuestos que suelen gravarse en proyectos individuales más que en grupos de proyectos.

59. Las tarifas sobre ingresos están diseñadas

para: a. Desarrollar nuevas oportunidades de

exportación. b. Proporcionar al gobierno ingresos

fiscales. c. Restringir la cantidad de un producto

básico que puede importarse en un período dado.

d. Estimular a las empresas extranjeras para que limiten el monto de sus exportaciones a un país en particular.

60. A una compañía le interesa contratar servicios

de mantenimiento por un año con cuatro años opcionales. Las especificaciones requieren que el posible contratista brinde ciertos servicios de limpieza a determinados intervalos. ¿Cuál de los siguientes es el mejor tipo de contrato para este requerimiento? a. De costos reembolsables. b. De entrega indefinida. c. De precios fijos. d. De tiempo y materials.

57. If a country uses trade quotas to overcome chronic trade deficits, the most likely outcome would be that: a. Unemployment and productivity rates will

rise. b. Unemployment rates will rise and

productivity rates will decline. c. Unemployment rates will decline and

productivity rates will rise. d. Unemployment and productivity rates will

decline. 58. Which of the following correctly describes the

introduction of a government tax credit on investments? a. Corporate investments will have higher

net present values, all else equal, than without the tax credit.

b. Tax credits on investments are designed to restrain inflation.

c. Tax credits on investments increase investment costs, and all else equal, reduce the level of corporate investment.

d. Tax credits on investments are taxes that are typically levied on individual projects rather than on groups of projects.

59. Revenue tariffs are designed to:

a. Develop new export opportunities. b. Provide the government with tax

revenues. c. Restrict the amount of a commodity that

can be imported in a given period. d. Encourage foreign companies to limit the

amount of their exports to a particular country.

60. A company would like to contract for janitorial

services for one year with four option years. The specifications require the potential contractor to perform certain cleaning services at specified intervals. Which of the following is the best contract type for this requirement? a. Cost-reimbursable. b. Indefinite delivery. c. Fixed-price. d. Time-and-materials.

III - 19

61. ¿Cuál de los siguientes se refiere a impuestos que no necesariamente tienen una mayor participación absoluta en un aumento de los ingresos? a. Progresivos. b. Proporcionales. c. Regresivos. d. Fijos.

62. Diferencias temporarias y permanentes entre

ingresos imponibles e ingresos financieros antes de impuestos difieren en que: a. Las diferencias temporarias no afectan a

montos imponibles ni deducibles. b. Sólo las diferencias permanentes tienen

consecuencias de impuestos diferidos. c. Sólo las diferencias temporarias tienen

consecuencias de impuestos diferidos. d. Las diferencias temporarias incluyen

items que forman parte de los ingresos financieros antes de impuestos, pero nunca de los ingresos imponibles.

63. Un impuesto al valor agregado se cobra sobre

la base de: a. La diferencia entre el valor de las ventas

de una compañía y el valor de sus compras a otras compañías nacionales.

b. La diferencia entre el precio de venta de un bien raíz y el precio que la compañía pagó originalmente por dicho bien.

c. El valor de las ventas de una compañía comparado con el de empresas relacionadas.

d. La ganancia obtenida de las ventas de una compañía.

64. El producto bruto nacional disminuirá con un

aumento en: a. Los gastos de consumo. b. Las importaciones. c. Las exportaciones. d. La inflación.

61. Which of the following refers to taxes that do not necessarily take a larger absolute share of an increase in income? a. Progressive. b. Proportional. c. Regressive. d. Flat.

62. Temporary and permanent differences

between taxable income and pre-tax financial income differ in that: a. Temporary differences do not give rise to

future taxable or deductible amounts. b. Only permanent differences have

deferred tax consequences. c. Only temporary differences have deferred

tax consequences. d. Temporary differences include items that

enter into pre-tax financial income but never into taxable income.

63. A value-added tax is collected on the basis of:

a. The difference between the value of a company’s sales and the value of its purchases from other domestic companies.

b. The difference between the selling price of real property and the price the company originally paid for the property.

c. The value of a company’s sales to related companies.

d. The profit earned on a company’s sales. 64. The gross national product will fall following an

increase in: a. Consumption expenditures. b. Imports. c. Exports. d. Inflation.

III - 20

65. ¿Cuál de los siguientes no es un control de salida típico? a. Revisión de los registros de procesos

computarizados para determinar si todos los trabajos se ejecutaron en forma apropiada.

b. Cotejar los datos de entrada con la información existente en los archivos maestros y colocar los items que no concuerdan en un archivo en suspenso.

c. Conciliar los informes de salida en forma periódica para asegurarse de que los totales, los formatos y los detalles críticos estén correctos y concuerden con los datos de entrada.

d. Mantener procedimientos formales y documentación que especifiquen a los receptores autorizados de los informes de salida, de las verificaciones o de otros documentos críticos.

66. La minimización de la probabilidad de

ediciones no autorizadas de programas de producción, del lenguaje de control de trabajos y del software del sistema operativo puede lograrse mejor con: a. Revisiones de acceso a la base de datos. b. Revisiones de cumplimiento. c. Buenos procedimientos de control de

cambios. d. Un software eficaz de seguridad de la

red. 67. ¿Qué control, una vez implementado, ayudaría

mejor a lograr el objetivo de control mediante el cual un sistema puede tener la capacidad de mantener la responsabilidad de los usuarios por las funciones que desarrollan? a. Corte programado. b. Hardware redundante. c. Registro de actividades. d. Registro de errores en las transacciones.

68. Los sistemas de procesamiento de imágenes

pueden reducir el volumen de papel que circula dentro de una organización. Para minimizar la probabilidad de que los usuarios confíen en imágenes equivocadas, la gerencia debe asegurarse de que existan controles apropiados para mantener la: a. Legibilidad de los datos de la imagen. b. Accesibilidad de los datos de la imagen. c. Integridad de los datos indexados. d. Secuencia inicial de los datos indexados.

65. Which of the following is not a typical output control? a. Reviewing the computer processing logs

to determine that all of the correct computer jobs executed properly.

b. Matching input data with information on master files and placing unmatched items in a suspense file.

c. Periodically reconciling output reports to make sure that totals, formats, and critical details are correct and agree with input.

d. Maintaining formal procedures and documentation specifying authorized recipients of output reports, checks, or other critical documents.

66. Minimizing the likelihood of unauthorized

editing of production programs, job control language, and operating system software can best be accomplished by: a. Database access reviews. b. Compliance reviews. c. Good change control procedures. d. Effective network security software.

67. Which control, when implemented, would best

assist in meeting the control objective that a system have the capability to hold users accountable for functions performed? a. Programmed cutoff. b. Redundant hardware. c. Activity logging. d. Transaction error logging.

68. Image processing systems have the potential

to reduce the volume of paper circulated throughout an organization. To reduce the likelihood of users relying on the wrong images, management should ensure that appropriate controls exist to maintain the: a. Legibility of image data. b. Accessibility of image data. c. Integrity of index data. d. Initial sequence of index data.

III - 21

69. Tanto los usuarios como la gerencia aprueban la propuesta inicial, las especificaciones de diseño, el plan de conversión y el plan de pruebas de un sistema de información. Este es un ejemplo de: a. Controles de implementación. b. Controles de hardware. c. Controles de las operaciones de la

computadora. d. Controles de seguridad de los datos.

70. El dispositivo utilizado para conectar redes

que no son iguales es: a. Una puerta (gateway). b. Un puente (bridge). c. Un ruteador (router). d. Un concentrador de cables (wiring

concentrator). 71. La interrupción total del procesamiento en un

sistema de tecnología de información distribuida se puede minimizar por medio del uso de: a. Informes de excepción. b. La protección de fallas suave (fail –soft

protection). c. Respaldo y recuperación. d. Seguridad en los archivos de datos.

72. Para evitar el ingreso de datos no válidos, un

banco añadió un dígito extra al final de cada número de cuenta y sometió el nuevo número a un algoritmo. Esta técnica se conoce como: a. Reconocimiento óptico de caracteres. b. Dígito de verificación. c. Verificación de dependencia. d. Verificación de formato.

73. ¿Con cuál de las siguientes acciones

comienza una implementación exitosa de intercambio electrónico de datos (EDI por sus siglas en inglés)? a. “Mapeo” (Mapping) de los procesos de

trabajo y de los flujos que sustentan las metas de la organización.

b. Compra de hardware nuevo para el sistema EDI.

c. Selección de proveedores confiables para el software de traducción y comunicación.

d. Estandarización de los datos y los formatos de transacciones.

69. Both users and management approve the initial proposal, design specifications, conversion plan, and testing plan of an information system. This is an example of: a. Implementation controls. b. Hardware controls. c. Computer operations controls. d. Data security controls.

70. A device used to connect dissimilar networks

is a: a. Gateway. b. Bridge. c. Router. d. Wiring concentrator.

71. A total interruption of processing throughout a

distributed information technology system can be minimized through the use of: a. Exception reporting. b. Fail-soft protection. c. Backup and recovery. d. Data file security.

72. To avoid invalid data input, a bank added an

extra number at the end of each account number and subjected the new number to an algorithm. This technique is known as: a. Optical character recognition. b. A check digit. c. A dependency check. d. A format check.

73. Successful electronic data interchange (EDI)

implementation begins with which of the following? a. Mapping the work processes and flows

that support the organization’s goals. b. Purchasing new hardware for the EDI

system. c. Selecting reliable vendors for translation

and communication software. d. Standardizing transaction formats and

data.

III - 22

74. Una compañía que utiliza el intercambio electrónico de datos (EDI) estableció una práctica para rastrear los avisos de recepción de sus socios comerciales y para emitir mensajes de advertencia si dichos avisos no ocurrían dentro de un período razonable. ¿Qué riesgo intentaba abordar la empresa por medio de esta práctica? a. Las transacciones que no se originaron a

partir de un socio comercial legítimo pueden introducirse dentro de la red del EDI.

b. A veces, la transmisión de transacciones mediante el EDI a los socios comerciales puede fallar.

c. Puede haber desacuerdos entre las partes respecto de cómo las transacciones mediante el EDI se ajustan a determinado contrato.

d. Es probable que los datos no se procesen en forma exacta y completa con el software del EDI.

75. ¿Qué técnica podría utilizarse para prevenir el

ingreso de caracteres alfabéticos en un número de identificación numérica? a. Una verificación de existencia. b. Un dígito de verificación. c. Una verificación de dependencia. d. Una verificación de formato.

76. Un fabricante de equipo electrónico complejo

tal como osciloscopios y microscopios ha estado despachando sus productos con manuales impresos muy voluminosos, pero desea reducir el costo de producir y enviar esta documentación. De los siguientes, el mejor medio que el fabricante puede utilizar es: a. Escribir una vez y leer muchas. b. Una cinta de audio digital. c. Un disco compacto/memoria de sólo

lectura. d. Salida de computadora a microformato.

77. En una organización de gran envergadura, el

mayor riesgo de no tener la mesa de ayuda de un centro de información equipada con el personal adecuado es: a. Tener gran dificultad al realizar auditorías

de aplicación. b. Disponer de la documentación

inadecuada para los sistemas de aplicación.

c. Gran probabilidad de usar un código de programas no autorizado.

d. Errores persistentes en la interacción de los usuarios con los sistemas.

74. A company using electronic data interchange (EDI) made it a practice to track the functional acknowledgments from trading partners and to issue warning messages if acknowledgments did not occur within a reasonable length of time. What risk was the company attempting to address by this practice? a. Transactions that have not originated

from a legitimate trading partner may be inserted into the EDI network.

b. Transmission of EDI transactions to trading partners may sometimes fail.

c. There may be disagreement between the parties as to whether the EDI transactions form a legal contract.

d. EDI data may not be accurately and completely processed by the EDI software.

75. What technique could be used to prevent the

input of alphabetic characters into an all numeric identification number? a. An existence check. b. A check digit. c. A dependency check. d. A format check.

76. A manufacturer of complex electronic

equipment such as oscilloscopes and microscopes has been shipping its products with thick paper manuals but wants to reduce the cost of producing and shipping this documentation. Of the following, the best medium for the manufacturer to use to accomplish this is: a. Write-once-read-many. b. Digital audio tape. c. Compact disc/read-only memory. d. Computer-output-to-microform.

77. In a large organization, the biggest risk in not

having an adequately staffed information center help desk is: a. Increased difficulty in performing

application audits. b. Inadequate documentation for application

systems. c. Increased likelihood of use of

unauthorized program code. d. Persistent errors in user interaction with

systems.

III - 23

78. ¿Qué tecnología se requiere para convertir un documento impreso en un archivo de computadora? a. Reconocimiento óptico de caracteres. b. Intercambio electrónico de datos. c. Lector óptico de código de barras. d. Unificación y fusion.

79. Los programas de utilitarios pueden emplearse

para leer archivos que contienen todos los códigos de acceso del usuario autorizados para un servidor. Un control para prevenir esta situación es: a. Usar claves de acceso criptografiadas

internamente. b. Jerarquizar las claves de acceso. c. Utilizar claves de acceso para iniciar. d. Considerar una red par-a-par.

80. ¿Cuál de los siguientes es un programa

malicioso, cuyo propósito es reproducirse mediante la red e impedir el acceso al servidor utilizando los recursos del sistema en forma excesiva? a. Bomba lógica. b. Virus. c. Gusano. d. Virus troyano.

81. Se puede prevenir la alteración de registros en

línea sin autorización usando: a. Verificación clave. b. Verificaciones de secuencias por

computadora. c. Mapeo por computadora. d. Controles de acceso a la base de datos.

82. ¿Cuál de las siguientes afirmaciones es

verdadera respecto de la seguridad del correo electrónico?

I. El correo electrónico no puede ser más

seguro que el sistema dentro del cual opera.

II. Los mensajes confidenciales de correo electrónico deben almacenarse en el servidor como correo electrónico durante el mismo tiempo que documentos impresos de similares características.

III. En organizaciones grandes, puede haber muchos administradores de correo electrónico y lugares con diversos niveles de seguridad.

a. I solamente. b. I y II solamente. c. I y III solamente. d. II y III solamente.

78. What technology is needed in order to convert a paper document into a computer file? a. Optical character recognition. b. Electronic data interchange. c. Bar-code scanning. d. Joining and merging.

79. Utility programs can be used to read files

which contain all authorized access user codes for a server. A control to prevent this is: a. Internally encrypted passwords. b. A password hierarchy. c. Log-on passwords. d. A peer-to-peer network.

80. Which of the following is a malicious program,

the purpose of which is to reproduce itself throughout the network and produce a denial of service attack by excessively utilizing system resources? a. Logic bomb. b. Virus. c. Worm. d. Trojan horse.

81. Unauthorized alteration of on-line records can

be prevented by employing: a. Key verification. b. Computer sequence checks. c. Computer matching. d. Database access controls.

82. Which of the following statements are correct

regarding electronic mail security?

I. Electronic mail can be no more secure than the computer system on which it operates.

II. Confidential electronic mail messages should be stored on the mail server as electronic mail for the same length of time as similar paper-based documents.

III. In larger organizations, there may be several electronic mail administrators and locations with varying levels of security.

a. I only. b. I and II only. c. I and III only. d. II and III only.

III - 24

83. ¿Cuál de la siguientes medidas representará el menor nivel de seguridad para información confidencial almacenada en una computadora portátil (notebook)? a. Encriptar los archivos de datos de la

computadora portátil. b. Instalar un código de acceso para el

protector de pantallas de la computadora portátil.

c. Utilizar una computadora portátil con un disco rígido removible.

d. Guardar la computadora portátil en un estuche con llave cuando no se use.

84. ¿Cuál de las siguientes sería la mayor

preocupación del auditor al revisar la política respecto de la venta a terceras personas de las computadoras personales usadas de la compañía? a. Si los archivos eliminados del disco duro

se borraron completamente. b. Si la computadora tiene virus. c. Si todo el software de la computadora

cuenta con licencia. d. Si la computadora cuenta con software de

emulación de terminal. 85. ¿Cuál de los siguientes elementos no sería

adecuado considerar en el diseño físico de un centro de datos? a. Evaluación de posibles riesgos

provenientes de autopistas y líneas de ferrocarril.

b. Uso de sistemas biométricos de acceso. c. Diseño de tablas de autorización para el

acceso al sistema operativo. d. Inclusión de un sistema de energía sin

interrupción y protección contra sobrecargas.

86. El uso ineficiente de equipos de computación

puede controlarse por a. Planes de contingencia. b. Estudios de factibilidad de sistemas. c. Planificación de capacidad. d. Informes de excepción.

83. Which of the following would provide the least security for sensitive data stored on a notebook computer? a. Encrypting data files on the notebook

computer. b. Using password protection for the screen-

saver program on the notebook computer. c. Using a notebook computer with a

removable hard disk drive. d. Locking the notebook computer in a case

when not in use. 84. Which of the following would be of greatest

concern to an auditor reviewing a policy regarding the sale of a company’s used personal computers to outside parties? a. Whether deleted files on the hard disk

drive have been completely erased. b. Whether the computer has viruses. c. Whether all software on the computer is

properly licensed. d. Whether there is terminal emulation

software on the computer. 85. Which of the following would not be

appropriate to consider in the physical design of a data center? a. Evaluation of potential risks from railroad

lines and highways. b. Use of biometric access systems. c. Design of authorization tables for

operating system access. d. Inclusion of an uninterruptible power

supply system and surge protection. 86. Inefficient usage of excess computer

equipment can be controlled by: a. Contingency planning. b. System feasibility studies. c. Capacity planning. d. Exception reporting.

III - 25

87. Una compañía electrónica decidió implementar un nuevo sistema a través del uso de técnicas de desarrollo rápido de aplicaciones. ¿Cuál de los siguientes se incluiría en el desarrollo del nuevo sistema? a. Diferir la necesidad de documentación de

sistemas hasta que se hayan completado los módulos finales.

b. Retirar de los equipos de desarrollo las responsabilidades de administración de proyectos.

c. Crear el sistema módulo por módulo hasta que se complete.

d. Usar técnicas de desarrollo por objetos para minimizar el uso de código previo.

88. Un banco está desarrollando un sistema

computarizado para ayudar a evaluar las aplicaciones de los préstamos. El personal de sistemas de información entrevista a los garantes hipotecarios a fin de extraer su conocimiento y sus procesos de decisión para el ingreso al sistema computarizado. El sistema completo debe poder procesar información de la misma manera que los garantes hipotecarios y hacer recomendaciones finales con respecto a las decisiones en cuanto a los préstamos. Este enfoque se denomina: a. Sistema experto. b. Red neural. c. Agente inteligente. d. Lógica confusa.

89. Las pruebas de la aceptación del usuario

resultan más importantes en un proceso de desarrollo orientado hacia el objeto que en un ambiente tradicional por las implicancias: a. De la ausencia de documentación de

diseño. b. De la falta de un sistema de rastreo para

los cambios. c. De la posibilidad de una supervisión

continua. d. De la posesión de propiedades en las

jerarquías. 90. La mejor evidencia de un plan de

contingencias eficaz es: a. No tener ninguna interrupción del

procesamiento durante el último año. b. Disponer de amplia documentación del

plan. c. Tener la firma del plan por parte del

departamento de auditoria interna. d. Tener una prueba exitosa del plan.

87. An electronics company has decided to implement a new system through the use of rapid application development techniques. Which of the following would be included in the development of the new system? a. Deferring the need for system

documentation until the final modules are completed.

b. Removing project management responsi-bilities from the development teams.

c. Creating the system module by module until completed.

d. Using object development techniques to minimize the use of previous code.

88. A bank is developing a computer system to

help evaluate loan applications. The information systems (IS) staff interview the bank's mortgage underwriters to extract their knowledge and decision processes for input into the computer system. The completed system should be able to process information the same as do the underwriters and make final recommendations regarding loan decisions. This approach is called: a. An expert system. b. A neural network. c. An intelligent agent. d. Fuzzy logic.

89. User acceptance testing is more important in

an object-oriented development process than in a traditional environment because of the implications of the: a. Absence of design documentation. b. Lack of a tracking system for changes. c. Potential for continuous monitoring. d. Inheritance of properties in hierarchies.

90. The best evidence that contingency planning is

effective is to have: a. No processing interruptions during the

past year. b. Comprehensive documentation of the

plan. c. Signoff on the plan by the internal audit

activity. d. Successful testing of the plan.

III - 26

91. ¿Cuál de las siguientes acciones abordaría mejor el problema de que los datos que se cargan de una computadora de escritorio pueden estar equivocados? a. Deben hacerse copias de seguridad de la

computadora central en forma periódica. b. Dos personas deben estar presentes

cuando la computadora de escritorio esté cargando datos.

c. La computadora central debe someter la información a los mismos controles de edición y rutinas de validación que se requerirían para el ingreso de datos “en línea”.

d. Debe requerirse a los usuarios que revisen una muestra al azar de los datos procesados.

92. Un password es un ejemplo de:

a. Control físico. b. Control de edición. c. Control digital. d. Control de acceso.

93. El evitar que alguien con suficiente destreza

técnica pueda impedir los procedimientos de seguridad y hacer cambios en los programas de producción se logra mejor por medio de: a. La revisión de informes de trabajos

terminados. b. La comparación de programas de

producción con copias controladas en forma independiente.

c. La producción periódica de datos de prueba.

d. La segregación apropiada de tareas. 94. Para reducir la exposición de seguridad

cuando se transmite información propietaria sobre líneas de comunicación, una compañía debe usar: a. Modems asincrónicos. b. Técnicas de autenticación. c. Técnicas de rellamada (call-back). d. Dispositivos criptográficos.

91. Which of the following actions would best address a concern that data uploaded from a desktop computer may be erroneous? a. The mainframe computer should be

backed up on a regular basis. b. Two persons should be present at the

desktop computer when it is uploading data.

c. The mainframe computer should subject the data to the same edits and validation routines that on-line data entry would require.

d. Users should be required to review a random sample of processed data.

92. A password is an example of:

a. A physical control. b. An edit control. c. A digital control. d. An access control.

93. Preventing someone with sufficient technical

skill from circumventing security procedures and making changes to production programs is best accomplished by: a. Reviewing reports of jobs completed. b. Comparing production programs with

independently controlled copies. c. Running test data periodically. d. Providing suitable segregation of duties.

94. To reduce security exposure when transmitting proprietary data over communication lines, a company should use: a. Asynchronous modems. b. Authentication techniques. c. Call-back procedures. d. Cryptographic devices.

III - 27

95. ¿Cuál de los siguientes controles de seguridad evitaría mejor el acceso no autorizado a información sensible a través de una terminal que no se encuentra en uso conectada directamente a una computadora central? a. El uso de un protector de pantalla con

contraseña o password b. El uso de párrafos de puestos de trabajo c. La encriptación de datos de archivos d. La desconexión automática de usuarios

inactivos 96. ¿Cuál de las siguientes puestas a punto del

acceso resulta adecuada en un ambiente computarizado?

Actualizar el acceso

para los datos de producción

Actualizar el acceso para los programas de producción

¿Los tienen los usua-rios?

¿Los tienen los programado-res de las aplicaciones?

¿Los tienen los usua-rios?

¿Los tienen los programado-res de las aplicaciones?

a. Sí No No No b. Sí No No Sí c. No Sí Sí No d. No Sí Sí Sí

97. La mejor manera de mantener la seguridad de

las bibliotecas de programas de computadora es: a. Instalar un sistema de registro para el

acceso a programas. b. Supervisar el acceso físico a los medios

de la librería de programas. c. Restringir el acceso físico y lógico. d. Denegar el acceso a terminales remotas.

98. ¿Qué interfase de lenguaje utilizaría un

administrador de base de datos para establecer la estructura de las tablas de la base de datos? a. Lenguaje de definición de datos. b. Lenguaje de control de datos. c. Lenguaje de manipulación de datos. d. Lenguaje de consulta de datos.

95. Which of the following security controls would best prevent unauthorized access to sensitive data through an unattended data terminal directly connected to a mainframe? a. Use of a screen-saver with a password. b. Use of workstation scripts. c. Encryption of data files. d. Automatic log-off of inactive users.

96. Which of the following access setups is

appropriate in a computer environment?

Update Access for Production Data

Update Access for Production Programs

Users Have?

Application Programmers Have?

Users Have?

Application Programmers Have?

a. Yes No No No b. Yes No No Yes c. No Yes Yes No d. No Yes Yes Yes

97. Computer program libraries can best be kept

secure by: a. Installing a logging system for program

access. b. Monitoring physical access to program

library media. c. Restricting physical and logical access. d. Denying access from remote terminals.

98. What language interface would a database

administrator use to establish the structure of database tables? a. Data definition language. b. Data control language. c. Data manipulation language. d. Data query language.

III - 28

99. Es probable que las prestaciones de consulta para un sistema de base de datos incluyan todos los siguientes excepto: a. Capacidad de salida gráfica. b. Diccionario de acceso de datos. c. Un verificador de validación de datos. d. Una interfase de consulta por ejemplo.

100. El aspecto más difícil al utilizar recursos de

internet es: a. Realizar la conexión física. b. Localizar la mejor fuente de información. c. Conseguir el equipo que se requiere. d. Obtener la autorización para acceder.

FIN DE LAS PREGUNTAS PARTE III POR FAVOR OBSERVE: La parte III del examen CIA actual constará de 125 preguntas de examen. Entre las 125 preguntas, se incluirán hasta 25 preguntas no registradas, que se utilizarán para propósitos de investigación. Dichas preguntas no registradas se insertarán sin identificar entre las registradas. Por lo tanto, los candidatos debe responder a las 125 preguntas lo mejor posible.

99. Query facilities for a database system would most likely include all of the following except: a. Graphical output capability. b. Data dictionary access. c. A data validity checker. d. A query-by-example interface.

100. The most difficult aspect of using Internet

resources is: a. Making a physical connection. b. Locating the best information source. c. Obtaining the equipment required. d. Getting authorization for access.

END OF PART III QUESTIONS PLEASE NOTE: The actual CIA exam Part III will contain 125 exam questions. The 125 questions will include up to 25 unscored questions, which will be used for research purposes. These unscored questions will be interspersed with the scored questions and will not be identified as unscored questions. Candidates should therefore answer all 125 questions to the best of their ability.

III - 29

Soluciones a la

Parte III – Análisis del negocio y tecnología de la información

Las soluciones y explicaciones sugeridas para la parte I de las preguntas del Examen modelo para Auditor Interno Certificado se proporcionan en las siguientes páginas.

La tabla que a continuación se presenta asocia los números de las preguntas de la parte I con los temas

evaluados:

Tema Evaluado Número de Pregunta

Procesos del negocio 1 – 18

Contabilidad financiera y finanzas 19 – 41

Contabilidad administrativa 42 – 55

Asuntos reguladores, legales y economía 56 – 64

Tecnología de la información 65 – 100

III - 30

1. Solución: b a. Incorrecta. Se incurre en costos de prevención para evitar la fabricación de productos que no

se ajusten a las especificaciones. b. Correcta. Se incurre en costos de evaluación (tales como el mantenimiento de equipos de

prueba y la prueba destructiva) para detectar qué productos no se ajustan a las especificaciones.

c. Incorrecta. Se incurre en costos de reproceso, un tipo de costo de fallas, cuando se detecta un producto que no se ajusta y se realizan correcciones.

d. Incorrecta. Se incurre en costos de fallas en la reparación de productos que no se ajustan. 2. Solución: a

a. Correcta. Los equipos pueden utilizar el conocimiento y las destrezas diversas de todos sus miembros.

b. Incorrecta. Los equipos suelen resultar ineficientes y costosos. c. Incorrecta. Si bien la motivación de los empleados puede ser alta en los equipos, no siempre se

traduce directamente en la mejora de la calidad. d. Incorrecta. Si bien puede reducirse la necesidad de supervisión, no se elimina.

3. Solución: d

a. Incorrecta. Este elemento no forma parte de las normas ISO 9000. En las normas ISO se sostiene que si se siguen los ocho principios de administración que subyacen en las normas ISO 9000 se logrará mejorar la satisfacción del empleado.

b. Incorrecta. Es el ambiente de control como se define en el glosario de las Normas de The IIA; no hay referencia directa a ningún concepto de esa índole en las normas ISO 9000.

c. Incorrecta. El enfoque de las normas ISO 9000 no adopta un enfoque de evaluación de riesgos; dicho enfoque es el que subyace en la auditoría interna.

d. Correcta. Es uno de los cambios más importantes de las normas ISO 9000 implementados en la revisión del año 2000.

4. Solución: c

Para lograr las máximas ganancias esperadas, deben construirse 30 yates. Para cada nivel de producción, multiplique la probabilidad de la demanda por la ganancia esperada: a. Incorrecta. Computación: 0,1($10)+0,2($10)+0,5($10)+0,2($10) = $10. b. Incorrecta. Computación: 0,1($0)+0,2($20)+0,5($20)+0,2($20) = $18. c. Correcta. Computación: 0,1(-$10)+0,2($10)+0,5($30)+0,2($30) = $22. d. Incorrecta. Computación: 0,1(-$30)+0,2(-$10)+0,5($10)+0,2($50) = $10.

5. Solución: b

a. Incorrecta. El conocimiento del comportamiento de los ciclos del negocio, la comprensión de las variaciones estacionales por la demanda del producto y el uso de los modelos econométricos pueden resultar valiosos cuando se pronostican las compras requeridas del inventario.

b. Correcta. Las asignaciones contables internas de costos a diferentes segmentos de la compañía son asignaciones arbitrarias de costos en los cuales ya se incurrió y que no tienen nada que ver con pronosticar la demanda.

c. Incorrecta. Ver respuesta "a". d. Incorrecta. Ver respuesta "a".

6. Solución: a

a. Correcta. El estallido (crashing) es el proceso de agregar recursos para reducir tiempos de actividad en el camino crítico de la programación de proyectos.

b. Incorrecta. La técnica Delphi es un enfoque cualitativo de pronóstico. c. Incorrecta. El análisis ABC es un modelo de inventario. d. Incorrecta. La solución “branch-and-bound” es una solución de programación entera.

III - 31

7. Solución: c a. Incorrecta. Once días es la fecha límite más cercana, no la más lejana para terminar el

proyecto. b. Incorrecta. Catorce días suman 5 + 3 + 6, pero no es un camino para terminar. c. Correcta. Los dos caminos son 5 + 4 + 6 = 15 días y 3 + 2 + 6 = 11 días. El camino más largo

y, en consecuencia, la fecha límite más cercana, es 15 días. d. Incorrecta. Veinte días es la suma de todos los tiempos de actividad.

8. Solución: d

a. Incorrecta. La técnica Delphi es un método de pronóstico cualitativo que obtiene pronósticos a través del consenso grupal.

b. Incorrecta. La suavización exponencial es una técnica de pronóstico que utiliza valores de series del pasado para llegar a valores pronosticados.

c. Incorrecta. El análisis de regresión es una técnica estadística utilizada para desarrollar pronósticos basados en la relación entre dos o más variables.

d. Correcta. La programación lineal es una técnica matemática para maximizar o minimizar un objetivo dado sujeto a ciertas limitaciones. Es la técnica correcta para optimizar el problema de los recursos limitados.

9. Solución: d

a. Incorrecta. Programar la producción sobre la base de la utilización de la capacidad ignora otros factores importantes tales como las demandas.

b. Incorrecta. La presupuestación de las actividades del departamento de mantenimiento sobre la base de órdenes de trabajo anteriores no evitará las fallas ni reparaciones del equipamiento.

c. Incorrecta. Las autorizaciones continuas de las órdenes de trabajo y el tiempo extra no abordarán el problema que se presente.

d. Correcta. Un programa de mantenimiento preventivo reducirá las fallas y reparaciones del equipamiento.

10. Solución: d

a. Incorrecta. Ver respuesta “d”. b. Incorrecta. Ver respuesta “d”. c. Incorrecta. Ver respuesta “d”. d. Correcta. Si los datos originales (con cuatro tendencias) se dividen por norma estacional, el

componente estacional se factorea fuera de los datos. 11. Solución: b

a. Incorrecta. El movimiento de los lotes puede perderse si no se registra con algún método de identificación.

b. Correcta. Una razón para utilizar códigos de barra en lugar de otros medios de identificación es que se registra el movimiento de los lotes con mínimos costos de mano de obra.

c. Incorrecta. Cada proveedor tiene su propio esquema de numeración de lotes, que es probable que no corresponda al esquema del comprador.

d. Incorrecta. Cada proveedor tiene su propio método de identificación, aunque los proveedores que trabajan en la misma actividad suelen cooperar para minimizar la cantidad de sistemas de códigos de barra que pueden utilizar.

12. Solución: a

a. Correcta. La planificación de requerimientos de materiales (MRP) es una técnica de planificación y controles para administrar los inventarios de manufactura según la demanda.

b. Incorrecta. El análisis de regresión es un procedimiento estadístico para estimar la relación entre variables.

c. Incorrecta. La presupuestación del capital se utiliza para analizar y evaluar las inversiones de capital a largo plazo.

d. Incorrecta. La programación lineal es una técnica matemática para maximizar o minimizar un objetivo dado sujeto a ciertas limitaciones.

III - 32

13. Solución: d a. Incorrecta. El proveedor puede solicitar una concesión en su precio de venta, que aumentaría

los costos de compra del fabricante. Sin embargo, la compañía manufacturera recibirá menos materiales en cualquier momento dado, incrementando la probabilidad de faltantes de stock, lo cual provocaría un aumento en los costos de los faltantes de stock.

b. Incorrecta. El proveedor puede solicitar una concesión en su precio de venta, que aumentaría los costos de compra del fabricante. Sin embargo, el costo de la calidad no necesariamente se vería afectado por el sistema de compras “just in time” (justo a tiempo).

c. Incorrecta. Con menos órdenes de compra procesadas por el fabricante, es probable que disminuyan los costos de las órdenes. Sin embargo, el costo de la calidad no necesariamente se vería afectado por el sistema de compras “just in time” (justo a tiempo).

d. Correcta. En esta situación, la compañía recibirá menos materiales en cualquier momento dado, incrementando la probabilidad de faltantes de stock, lo cual provocaría un aumento de los costos de los faltantes de stock. Al mismo tiempo, el inventario promedio será menor, lo cual provocará una reducción de los costos de transporte.

14. Solución: b

a. Incorrecta. Los modelos de pronóstico implican proyectar los datos a través del tiempo o desarrollar los modelos de regresión cuando los datos de la serie temporal no se encuentran disponibles.

b. Correcta. Un análisis de sensibilidad para la cantidad económica de pedidos (EOQ) implica variar los costos de tenencia por unidad y/o los costos de las órdenes a fin de determinar cuánto afectan los cambios a la EOQ óptima.

c. Incorrecta. El método del camino crítico implica programar los proyectos. d. Incorrecta. El análisis de decisión implica seleccionar la mejor opción de las alternativas.

15. Solución: c

a. Incorrecta. No hay costos adicionales en los cuales se haya incurrido más que de almacenamiento y despacho para el desarrollo de este producto. En consecuencia, el fabricante obtendrá ganancias de cualquier precio sobre el costo de almacenamiento y despacho.

b. Incorrecta. Los productos opcionales son aquellos ofrecidos para la venta junto con el producto principal. Es improbable que tengan un costo de producción cero; en consecuencia, el vendedor debe recibir un precio que exceda los costos de almacenamiento y despacho para dichos productos.

c. Correcta. Los productos cautivos son aquellos que deben utilizarse junto con el producto principal. Los vendedores suelen obtener su dinero de los productos cautivos, en lugar de obtenerlo del producto principal que se vende a un bajo precio. Por lo tanto, los productos cautivos tendrán un precio mucho mayor que los costos de almacenamiento y despacho.

d. Incorrecta. Los paquetes de productos son combinaciones de productos que se venden juntos a un precio reducido, tales como los abonos de temporada para un teatro. Los productos se ofrecen en paquetes a fin de promocionar la venta de ciertos items que los consumidores quizá no comprarían de otra manera. El precio combinado del paquete debe ser lo suficientemente bajo para estimular a los consumidores para que compren el paquete, pero es necesario que se recuperen los costos de producción y se genere alguna ganancia para el vendedor. El precio debe exceder los costos de almacenamiento y despacho.

III - 33

16. Solución: d a. Incorrecta. Las estrategias de líderes del mercado son empleadas por los principales

competidores que dominan un mercado. b. Incorrecta. Las estrategias de desafío del mercado son utilizadas por las compañías más

destacadas, que atacan a los competidores de manera agresiva para obtener mayor participación en el mercado.

c. Incorrecta. Las estrategias de seguidores del mercado son usadas por las compañías más destacadas que siguen las ofertas, las cotizaciones y los programas del mercado del competidor.

d. Correcta. La especialización en servir a los clientes pasados por alto o ignorados por competidores importantes es una estrategia de nichos del mercado. Dicha estrategia aborda el mercado, los clientes, los productos o líneas de mezcla de marketing.

17. Solución: b a. Incorrecta. El solicitante provee sus antecedentes, lo cual provoca una pérdida de

independencia. Además, como los antecedentes no son oficiales, resulta muy fácil cambiar la información y enviar una fotocopia de los antecedentes alterados.

b. Correcta. Representa una verificación independiente del empleo, ya que la organización contratante está desarrollando el proceso de verificación.

c. Incorrecta. No hay nada que impida a los solicitantes escribir las cartas por sí mismos, proveer información fraudulenta sobre el remitente de las cartas y despacharlas por correo.

d. Incorrecta. Si un solicitante va a mentir sobre la información que provee, no hay razón para creer que no va a firmar con su propio nombre para certificar los datos fraudulentos. No constituye una verificación independiente.

18. Solución: c

a. Incorrecta. Si bien un tablero de comando debe desarrollarse con el personal en mente, la meta primordial es la alineación de las medidas de desempeño con la estrategia.

b. Incorrecta. La estructura debe crearse después de desarrollar el tablero de comando para asegurar que las responsabilidades, la competencia y las medidas resultan apropiadas para lograr las estrategias acordadas.

c. Correcta. El tablero de comando es primero una herramienta para asistir a la organización en describir y clarificar su estrategia y luego la alineación de sus medidas de desempeño con esa estrategia.

d. Incorrecta. Los sistemas constituyen un medio para alcanzar los objetivos que se establecieron después de desarrollar el tablero de comando.

19. Solución: d

a. Incorrecta. La presunción de empresa en marcha es que la compañía durará mucho tiempo, lo cual no se relaciona directamente con la práctica de registrar como pasivos los ingresos no ganados.

b. Incorrecta. La presunción de la unidad monetaria es que el dinero es el común denominador mediante el cual se conduce la actividad económica y que la unidad monetaria provee una base apropiada para considerar la medición y el análisis. No se relaciona directamente con la práctica de registrar como pasivos los ingresos no ganados.

c. Incorrecta. El principio del costo histórico es el requerimiento de que la mayor parte de los activos y pasivos se justifiquen e informen sobre la base del precio de adquisición. No se relaciona directamente con la práctica de registrar como pasivos los ingresos no ganados.

d. Correcta. Como el monto recibido en efectivo todavía no se obtuvo como ganancia, resulta apropiado registrar el pago por adelantado como pasivo de la compañía. Constituye un ejemplo del principio de reconocimiento de ingresos, que sostiene que los ingresos no deben reconocerse hasta que no estén devengados.

III - 34

20. Solución: d a. Incorrecta. No hay incertidumbre respecto del importe del pago del alquiler. Los gastos por

alquiler pueden acumularse a medida que se producen las ventas. b. Incorrecta. Se recibió un servicio y la compañía debe un monto. El monto no es contingente en

un acontecimiento futuro. La compañía puede acumular el importe que esperaba que mostrase la factura.

c. Incorrecta. A partir de la fecha de los estados financieros a fecha intermedia, se paga el impuesto a los réditos porque se generaron ganancias. No hay incertidumbre respecto del importe o de la oportunidad del pago a partir de la fecha de los estados financieros a fecha intermedia.

d. Correcta. Es una garantía. El pasivo es contingente pues el arrendador no recibe todo el valor residual de un tercero.

21. Solución: c

a. Incorrecta. Las políticas de garantía no forman parte de la valuación del inventario. b. Incorrecta. Las políticas de fijación de precios para los acreedores no tienen impacto alguno

sobre la valuación de inventario hasta que se compren los productos. El precio en el momento de la compra es el único que importa en la valuación de inventario y los cambios en las políticas de fijación de precios para los acreedores no necesariamente impactan en la valuación.

c. Correcta. La magnitud de la pérdida de inventario por reducción debe considerarse en los modelos de riesgos que incluyan valuación de inventario.

d. Incorrecta. Los pronósticos de ventas no afectan a la valuación de inventario. 22. Solución: c

a. Incorrecta. El saldo de la depreciación acumulada sería más alto en los estados financieros utilizados para propósitos tributarios, ya que los gastos de depreciación más altos se informarían por depreciación acelerada más que por depreciación en línea recta.

b. Incorrecta. Los gastos de depreciación no son en efectivo. El saldo en efectivo no se ve afectado por el método de depreciación que se utiliza.

c. Correcta. Mediante el uso de la depreciación acelerada, los gastos de depreciación son más altos y, el ingreso neto, más bajo. En consecuencia, las utilidades retenidas serían más bajas en los estados financieros utilizados para propósitos tributarios que en los utilizados para propósitos generales sobre la base de la depreciación en línea recta.

d. Incorrecta. El costo histórico de los activos fijos se registra en la cuenta de activos fijos brutos. El costo histórico de los activos no se ve afectado por el método de depreciación que se utiliza.

23. Solución: a

a. Correcta. Mediante un plan definido de contribuciones, la compañía informa un activo en el balance sólo si la contribución al fideicomiso de pensiones es mayor que la contribución definida solicitada.

b. Incorrecta. Un activo se informa sólo si la contribución excede la contribución solicitada. Si la contribución real es igual a la solicitada, no se informa ningún activo.

c. Incorrecta. La compañía informaría un pasivo en el balance sólo si la contribución fuera menor y no mayor que el monto solicitado.

d. Incorrecta. La compañía no informaría un pasivo en el balance si contribuyera con el monto solicitado al fideicomiso de pensiones.

III - 35

24. Solución: a a. Correcta. Los activos donados por usufructo se reconocen como activos tangibles. Cuando en

los contratos de arrendamiento se transfieren los beneficios y riesgos del propietario del activo al arrendatario, el contrato de arrendamiento es un contrato de capital, ya que constituye esencialmente una forma de financiamiento, o de capital, para el arrendatario.

b. Incorrecta. Los activos donados por usufructo no se consideran intangibles. c. Incorrecta. Si en un contrato de arrendamiento el propietario transfiere substancialmente todos

los beneficios y riesgos del propietario del activo al arrendatario, luego el contrato es un contrato de capital, no operativo.

d. Incorrecta. Ver respuestas "b" y "c". 25. Solución: d

a. Incorrecta. Los bonos de renta sólo pagan interés si se obtiene interés. b. Incorrecta. Los debentures son bonos sin caución. c. Incorrecta. Los debentures subordinados se encuentran sujetos a otra deuda. d. Correcta. Los bonos de garantía hipotecaria se encuentran respaldados por activos fijos.

26. Solución: a

a. Correcta. Como el rendimiento de la compañía estadounidense se encuentra adversamente afectado y, el de la compañía europea, no, el de la estadounidense será definitivamente más bajo que el de la europea.

b. Incorrecta. El rendimiento de la compañía estadounidense se encuentra adversamente afectado por el movimiento de la tasa de cambio.

c. Incorrecta. El rendimiento de la compañía estadounidense se encuentra directamente afectado por el movimiento de la tasa de cambio, mientras que el de la europea, no.

d. Incorrecta. Ver respuesta “a”. 27. Solución: d

a. Incorrecta. Una oferta “de dos niveles” incluye dos precios de oferta diferentes para las acciones adquiridas. Las condiciones de la adquisición de acciones no se relacionan con la emisión o la recompra de bonos en la compañía.

b. Incorrecta. Ver respuesta “a”. c. Incorrecta. Es lo opuesto de la respuesta correcta. d. Correcta. En una oferta de dos niveles, los accionistas siente una necesidad incontrolable de

vender temprano al licitador a una oferta mayor del precio de la acción para aquellos que licitan sus acciones temprano.

III - 36

28. Solución: d a. Incorrecta. La razón del valor del mercado con respecto al valor de libros es el valor del

mercado de la acción ordinaria por acción derivada dividiendo el valor de libros de la acción ordinaria por cantidad promedio de acciones. Ni esta razón ni la razón de la deuda total con respecto a la razón del activo total provee información alguna sobre el ingreso neto disponible para los accionistas, lo cual es necesario para calcular el rendimiento del capital.

b. Incorrecta. La razón del precio con respecto a las utilidades es la razón del precio de mercado de la acción dividida por las utilidades obtenidas por acción; dichas utilidades son el ingreso neto disponible para los accionistas dividido por la cantidad promedio de acciones; y el margen neto de utilidades es la ganancia neta dividida por las ventas. Si bien las tres razones contienen mucha información sobre la cuenta del capital, ninguna provee información sobre el valor de libros de la acción ordinaria, lo cual es necesario para calcular el rendimiento del capital.

c. Incorrecta. La razón con respecto a las utilidades es la razón del precio del mercado de la acción dividida por las utilidades obtenidas por acción y la razón del rendimiento del activo es el ingreso neto dividido por el activo. Ninguna de estas dos razones provee información sobre el valor de libros de la acción ordinaria, lo cual es necesario para calcular el rendimiento del capital.

d. Correcta. Estas tres razones abarcan la ecuación simple de Du Pont: Ingreso Neto Disponible para Los Accionistas x Ventas x Activo Total = Rendimiento del Capital Ventas Activo Total Acción Ordinaria al Valor de Libros El activo total y las ventas se cancelan en la multiplicación, dejando el ingreso neto disponible para los accionistas dividido por la acción ordinaria al valor de libros, que es igual al rendimiento del capital.

29. Solución: c

a. Incorrecta. La inflación también distorsiona los gastos de depreciación, los costos de inventario y las utilidades.

b. Incorrecta. La inflación impacta en ambos aspectos. c. Correcta. La inflación impacta en ambos aspectos. d. Incorrecta. La inflación impacta en ambos aspectos.

30. Solución: a

a. Correcta. La prima de riesgo es la porción del rendimiento esperado atribuido al riesgo incrementado.

b. Incorrecta. El coeficiente de variación representa la desviación estándar del rendimiento de una inversión dividida por el rendimiento medio.

c. Incorrecta. El error estándar representa una medición de la variabilidad en el rendimiento de la inversión.

d. Incorrecta. El coeficiente beta representa la sensibilidad del rendimiento de la inversión al rendimiento del mercado.

31. Solución: b

a. Incorrecta. Esta respuesta ignora el capital. b. Correcta. Tanto la deuda como el capital son factores en la estructura del capital de una

compañía. c. Incorrecta. La decisión no incluye directamente el activo. d. Incorrecta. La decisión incluye el capital, pero no se centra en el tipo de capital utilizado.

III - 37

32. Solución: a a. Correcta. Los gastos por intereses son iguales al valor de inactividad del pasivo al comienzo

del período por la tasa de interés efectivo. El valor de inactividad del pasivo es igual al valor nominal del bono menos el descuento. Como el descuento se amortiza durante la vida útil del bono, el valor de inactividad aumenta. En consecuencia, los gastos por intereses aumenta durante la vida útil del bono.

b. Incorrecta. Ver respuesta “a”. c. Incorrecta. Los gastos por intereses exceden el pago de intereses en efectivo. El exceso es el

monto del descuento amortizado en cada período. d. Incorrecta. Ver respuesta “c”.

33. Solución: a

a. Correcta. En terminología de negocios, hablar de un alto grado de impacto operativo, mientras otras cosas se mantienen constantes, significa que un cambio relativamente pequeño en las ventas provocará un gran cambio en los ingresos operativos. En consecuencia, si un alto porcentaje del costo total de una empresa es fijo, se dice que la empresa tiene un alto grado de impacto operativo.

b. Incorrecta. Lo opuesto es verdad; ver respuesta “a”. c. Incorrecta. Ver respuesta “a”. d. Incorrecta. Ver respuesta “a”.

34. Solución: a

a. Correcta. Cuanto más alta sea la razón del pago de dividendos, mayor será la rapidez con que se agoten las utilidades no distribuidas y la compañía debe buscar un financiamiento más costoso, fuera del capital, lo cual impulsa el costo marginal del capital.

b. Incorrecta. La razón de la deuda se computa dividiendo las deudas totales por el activo total. La razón del pago de dividendos no impacta en la razón de la deuda.

c. Incorrecta. Las oportunidades de inversión disponibles para la compañía no se determinan por el nivel de pago de dividendos.

d. Incorrecta. Lo opuesto es verdad. La razón del precio con respecto a las utilidades se computa dividiendo el precio por acción por las utilidades obtenidas por acción, de manera que una compañía con una razón más alta de pago de dividendos tendría una razón más baja de precio con respecto a las utilidades.

35. Solución: b

a. Incorrecta. El saldo de caja mantenido para realizar pagos y recaudaciones de rutina se denomina saldo de transacciones.

b. Correcta. El saldo de caja denominado saldo compensado es el dinero que se dejó en la cuenta de cheques del banco a fin de compensar al banco por los servicios que provee.

c. Incorrecta. El saldo de caja mantenido como reserva para las fluctuaciones del flujo de fondos se denomina saldo preventivo.

d. Incorrecta. Es el saldo de caja especulativo que se mantiene para que la firma pueda aprovechar cualquier oportunidad de compra de liquidación que pueda surgir.

36. Solución: b

a. Incorrecta. Los precios de los valores son una buena estimación de futuros flujos de caja mediante esta teoría.

b. Correcta. El mercado está continuamente ajustándose a la nueva información y actuando para corregir errores de cotizaciones.

c. Incorrecta. Los precios de los valores constituyen el mejor parámetro mediante esta teoría. d. Incorrecta. Los precios de los valores son iguales a su valor equitativo según lo perciben los

inversores.

III - 38

37. Solución: c a. Incorrecta. Un mayor margen de utilidades reduciría el financiamiento adicional necesario, de

acuerdo con lo expresado en la pregunta. El resultado sería un cambio hacia abajo, no hacia arriba, en la línea de los fondos.

b. Incorrecta. Ver respuesta “a”. c. Correcta. Un mayor margen de utilidades reduciría el financiamiento adicional necesario,

cambiando la línea de los fondos hacia abajo. d. Incorrecta. La línea tendría una menor, no una mayor pendiente, si la empresa tuviera un

mayor margen de utilidades. 38. Solución: a

a. Correcta. Si todas las utilidades se pagan como dividendos, luego no hay utilidades no distribuidas. El crecimiento de todas las ventas debe financiarse por fuentes espontáneas o externas.

b. Incorrecta. La línea de los fondos sólo atraviesa el origen en el caso especial en que todas las utilidades se paguen como dividendos.

c. Incorrecta. La línea de los fondos es un gráfico de la relación entre las tasas de crecimiento de las ventas y las necesidades adicionales de financiamiento. No se dibuja sólo para un punto o un nivel del crecimiento de las ventas.

d. Incorrecta. Si bien la tasa de crecimiento de las ventas sería cero en el momento en que la línea de los fondos atraviesa el origen, los fondos necesarios pueden no ser cero cuando el crecimiento de las ventas es cero.

39. Solución: a

a. Correcta. Mediante la valuación de inventario primero entrado, primero salido (FIFO), se presume que las 10.000 unidades del inventario final fueron los últimos items que se compraron. El costo de dichas unidades es: 5,000 unidades @ $7.50 + 5,000 unidades @ $8 = $37,500 + $40,000 = $77,500.

b. Incorrecta. Esta solución es el saldo del inventario final mediante el método de identificación específico si se detecta que las unidades que quedan en el inventario de fin de año fueron compradas el 1 de abril y el 1 de julio: 5,000 unidades @ $9 + 5,000 unidades @ $8 = $45,000 + $40,000 = $85,000.

c. Incorrecta. Esta solución es el saldo del inventario final mediante el método de costo promedio. El costo promedio de todos los items comprados se utiliza para calcular el saldo del inventario final. El costo promedio de los items comprados es: [$10 (5,000) + $9 (5,000) + $8 (5,000) + $7.50 (5,000)] / 20,000 = $8.625 por unidad, de manera que se asigna un valor de $86.250 a 10,000 unidades.

d. Incorrecta. Esta solución es el saldo del inventario final mediante el método de valuación de inventario último entrado, primero salido (LIFO). Se presume que los últimos items comprados son los primeros en venderse, de manera que se asigna el costo de las primeras compras a los items que quedan en el inventario: 5,000 unidades @ $10 + 5,000 unidades @ $9 = $50,000 + $45,000 = $95,000.

III - 39

40. Solución: a a. Correcta. Mediante el método de inventario último entrado, primero salido (LIFO), se presume

que las 10.000 unidades vendidas durante el año fueron las últimas compradas. El costo de los productos vendidos durante el año se calcula como: 5.000 unidades @ $7,50 + 5.000 unidades @ $8 = $37.500 + $40.000 = $77.500.

b. Incorrecta. Ésta es la solución si se utiliza el método de costo promedio. El costo promedio de todos los items comprados es de $8.625 por unidad, de manera que se asigna un costo de $86.250 a las 10.000 unidades vendidas.

c. Incorrecta. Ésta es la solución si se utiliza el método de identificación específico y si se detecta que las unidades que quedan en el inventario de fin de año fueron compradas el 1 de abril y el 1 de julio. Luego, los items vendidos se habrían comprado el 1 de enero y el 1 de octubre y el costo de los productos vendidos durante el año se calcula como: 5.000 unidades @ $10 + 5.000 unidades @ $7,50 = $50.000 + $37.500 = $87.500.

d. Incorrecta. Ésta es la solución si se utiliza el método primero entrado, primero salido (FIFO). Mediante el FIFO, se presume que se vendieron los items más viejos, de manera que el costo de los productos vendidos durante el año se calcula como: 5.000 unidades @ $10 + 5.000 unidades @ $9 = $50.000 + $45.000 = $95.000.

41. Solución: a

a. Correcta. El valor de una compañía se encuentra dado por la expresión: N

V = Σ FCt t=1 (1 + k) t en la cual V es el valor, FC es el flujo de fondos neto, k es la tasa de descuento (costo del

capital), y t es el tiempo. Lo que sigue es que el valor aumentará a medida que se incremente el FC.

b. Incorrecta. Un incremento en el riesgo sistemático (o del mercado) aumentará el costo global del capital y, en consecuencia, aumentará la K, la tasa de descuento. Como resultado, el valor de la compañía disminuirá.

c. Incorrecta. Un incremento en el riesgo no sistemático (o específico de la compañía) puede diversificarse y no afectará al valor de la compañía.

d. Incorrecta. Un incremento en la tasa de descuento reducirá el valor de la compañía. 42. Solución: d

a. Incorrecta. Los deterioros anormales no constituyen una función del sistema de costos; es una función del proceso de producción.

b. Incorrecta. Los deterioros anormales puede ser el resultado de cualquiera de una variedad de circunstancias que, en general, son controlables por los supervisores de primera línea.

c. Incorrecta. Los deterioros anormales puede ser el resultado de una variedad de condiciones o circunstancias, que no necesariamente se relacionan con las normas.

d. Correcta. No se espera que se produzcan los deterioros anormales en condiciones operativas eficientes. No constituye una parte inherente del proceso de producción.

43. Solución: b

a. Incorrecta. El seguro del edificio de sedes corporativas no es un costo de producción y, en consecuencia, es un costo de período.

b. Correcta. Los impuestos de propiedad de una fábrica constituyen un costo de producto. c. Incorrecta. La depreciación del automóvil de un vendedor no es un costo de producción y, en

consecuencia, es un costo de período. d. Incorrecta. El sueldo de un gerente de ventas no es un costo de producción y, en

consecuencia, es un costo de período.

III - 40

44. Solución: b a. Incorrecta. Esta respuesta descuenta el flujo de fondos a la tasa correcta de descuento (18%)

pero por cuatro años en lugar de cinco, y además resta la salida de fondos del ingreso de fondos en lugar de lo contrario.

b. Correcta. El ingreso de caja al 31 de diciembre del 2000 está a cinco años del momento de la evaluación y el valor actual neto de fondos utiliza el costo de capital de la firma del 18%. El factor para el 18% de cinco años es 0,4371. $7.400.000 por 0.4371 es igual a $3.234.540, el cual menos $3.500.000 (valor actual de los desembolsos) es igual a $265.460.

c. Incorrecta. Esta respuesta no puede ser calculada utilizando la tabla de dólares y valores dada. d. Incorrecta. Esta respuesta descuenta el ingreso de fondos a la tasa de descuento correcta

(18%) pero por cuatro años en vez de cinco. 45. Solución: c

a. Incorrecta. Ver respuesta “c”. b. Incorrecta. Ver respuesta “c”. c. Correcta. El veinte por ciento es la tasa del rendimiento que equipara los ingresos y egresos de

fondos. El valor actual del 20 por ciento durante cinco años es .4019 que, multiplicado por $9.950.000, es igual a $3,998,905. En consecuencia, el valor presente neto del proyecto se aproxima a $0 mediante el uso de la tasa del 20 por ciento.

d. Incorrecta. Ver respuesta “c”. 46. Solución: a

a. Correcta. Es la definición de un presupuesto flexible. b. Incorrecta. Es la definición de un presupuesto operativo. c. Incorrecta. Es la definición de presupuestación basada en la actividad. d. Incorrecta. Es la definición de presupuestación de Kaizen.

47. Solución: c

a. Incorrecta. El país X tiene una tasa tributaria más alta que el país local y el país Y tiene una tasa tributaria más baja. Los incentivos de la fijación de precios de transferencia serán diferentes para las dos divisiones de ventas.

b. Incorrecta. Ver respuesta “a”. c. Correcta. El país X tiene una tasa tributaria más alta que el país local; por lo tanto, el incentivo

es transferir las utilidades desde el país X. La división de ventas del país X será menos rentable si se le cobra un precio de transferencia más alto.

d. Incorrecta. El país Y tiene una tasa tributaria más baja que el país local; por lo tanto, el incentivo es transferir las utilidades hasta el país Y cobrando un precio de transferencia más bajo a esa división de ventas.

48. Solución: d

a. Incorrecta. El presupuesto maestro no contiene resultados reales. b. Incorrecta. El presupuesto maestro refleja todos los costos aplicables esperados, ya sean

controlables por los gerentes individuales o no. c. Incorrecta. El presupuesto maestro no se encuentra estructurado para permitir la determinación

de las variaciones de los costos de fabricación, lo cual se logra mediante el uso del presupuesto flexible y los resultados reales.

d. Correcta. El presupuesto operativo es un elemento esencial del presupuesto maestro.

III - 41

49. Solución: b a. Incorrecta. Esta respuesta sólo tiene en cuenta el costo de reproceso y excluye el ingreso que

se perdió cuando las unidades se vendieron como “segundas” de fábrica: $12 x 1.200 = 14.400.

b. Correcta. Esta respuesta incluye correctamente tanto el costo de reproceso como el ingreso que se perdió cuando las unidades se vendieron como “segundas” de fábrica en lugar de hacerlo como unidades de primera calidad: [$12 + ($50 - $45)] x 1.200 = 20.400.

c. Incorrecta. Esta respuesta computa lo que habría representado el margen de contribución si los items reprocesados se hubieran vendido como unidades de primera calidad: [$50 - ($20 + $12)] x 1.200 = 21.600.

d. Incorrecta. Esta respuesta computa el margen de contribución incorrectamente para las unidades reprocesadas, pues ignora los costos variables de $20 por unidad:($45 - $12) x 1.200 = 39.600.

50. Solución: c

a. Incorrecta. Representa el 80 por ciento de las ventas reales, calculadas como .80 ($2.000.000) = $1.600.000.

b. Incorrecta. Representa las ventas reales. c. Correcta. Las ventas totales de capacidad ociosa se calculan de la siguiente manera: Ventas reales / Porcentaje de capacidad al cual se operó con los activos fijos. = $2.000.000/.80 = $2.500.000. d. Incorrecta. Representa las ventas reales divididas por la proporción de la capacidad inutilizada

en lugar de la utilizada o $2.000.000/.2 = $10.000.000. 51. Solución: a

a. Correcta. La acumulación de costos se realiza mediante sistemas contables que organizan datos mediante un catálogo apropiado. Se acumulan los costos reales en lugar de los costos que se predicen.

b. Incorrecta. La computación de gastos de depreciación no organizaría datos clasificados en categorías.

c. Incorrecta. La producción de estados financieros no organizaría datos clasificados en categorías.

d. Incorrecta. El pronóstico de los faltantes de materiales no organizaría los datos clasificados en categorías.

52. Solución: d

a. Incorrecta. El costo por procesos no asigna costos por línea de empaque. ($150.000 + 90.000 + 30.000 + 15.000 + 3.000 + 66.000 = $354.000/600.000 = .59/3 = $0.197).

b. Incorrecta. Los costos por proceso incluyen todos los costos. ($150.000 + 15.000 = 165.000/600.000 = $0,275).

c. Incorrecta. El costo por procesos incluye todos los costos. ($90.000 + 30.000 + 3.000 + 66.000 = 189.000/600.000 = $0,315).

d. Correcta. El costo por procesos es el costo promedio por unidad producida, o el costo total dividido por la cantidad de unidades. ($150.000 + 90.000 + 30.000 + 15.000 + 3.000 + 66.000 = $354.000/600.000 = $0,59).

III - 42

53. Solución: b a. Incorrecta. Esta base de asignación se relaciona con los costos de los lotes y no con los costos

de las unidades individuales. b. Correcta. Existe una relación causal directa entre la cantidad de componentes de un producto

terminado y la cantidad de costos materiales de manejo en los cuales se incurrió. c. Incorrecta. Esta base de asignación es la base tradicional para asignar gastos generales a las

unidades producidas cuando el proceso de producción se desarrolla con gran intensidad de mano de obra.

d. Incorrecta. No es una base de asignación, sino el resultado del proceso de asignación cuando se determinan los costos de los productos.

54. Solución: b

a. Incorrecta. Es incorrecta porque ambas medidas representan los resultados para un solo período.

b. Correcta. La utilidad residual se concentra en las utilidades en exceso del rendimiento mínimo deseado. Con el ROI, un segmento puede rechazar un proyecto que se excede del rendimiento mínimo si el proyecto reduce el ROI global de los segmentos. Por ejemplo, un proyecto que obtiene un ROI del 22 %, que resulta mayor que la tasa objetivo del 20 %, podría rechazarse si el segmento está ganando el 25 %, porque el proyecto reducirá el ROI del segmento, lo cual no ocurriría con la utilidad residual.

c. Incorrecta. No es correcta porque la tasa objetivo para el ROI es la misma que la imputada de los interés utilizada en el cálculo de la utilidad residual.

d. Incorrecta. Es incorrecta porque las inversiones promedio deben emplearse en ambos métodos. En todo caso, la base de inversión utilizada para ambos métodos sería la misma.

55. Solución: d

a. Incorrecta. Los costos incrementales son pertinentes si ocurren en el futuro. b. Incorrecta. Los costos de oportunidad (beneficios) son pertinentes si ocurren en el futuro. c. Incorrecta. Los costos que se desembolsan son pertinentes si ocurren en el futuro. d. Correcta. Los costos en los cuales ya se incurrió nunca son pertinentes porque ocurrieron en el

pasado. 56. Solución: a

a. Correcta. La venta de títulos públicos es de contracción, pues retira dinero de circulación. b. Incorrecta. La reducción de los requisitos de reserva abastecería la economía porque los

bancos prestarían más dinero. c. Incorrecta. La reducción de las tasas de descuento abastecería la economía porque se

estimularía a que se realicen préstamos. d. Incorrecta. Es una política fiscal, no una monetaria.

57. Solución: d

a. Incorrecta. Ver respuesta "d". b. Incorrecta. Ver respuesta "d". c. Incorrecta. Ver respuesta "d". d. Correcta. Con los cuotas, se preservarán los empleos locales; en consecuencia, se reducirá el

desempleo. Como se preservarán los empleos para las industrias ineficientes (menos eficientes que los competidores extranjeros), se reducirán los índices de productividad porque no se especializarán en aquellos productos con los cuales tienen ventajas comparativas.

III - 43

58. Solución: a a. Correcta. Los créditos fiscales en las inversiones son deducciones de los impuestos

corporativos. El resultado es un costo de inversión más bajo y valores presentes netos del proyecto más altos, con todo lo demás igual.

b. Incorrecta. Los créditos fiscales son reducciones de los impuestos corporativos reales y, como se encuentran disponibles más utilidades para los dividendos, no se restringiría la inflación.

c. Incorrecta. Lo opuesto es verdad. d. Incorrecta. Los créditos fiscales en las inversiones no constituyen impuestos gravados en

proyectos. 59. Solución: b

a. Incorrecta. Ver respuesta “b”. b. Correcta. En general, las tarifas sobre ingresos se aplican sobre productos que no se fabrican

en el ámbito local. Su propósito es proporcionar ingresos fiscales al gobierno. c. Incorrecta. Las cuotas de importación se encuentran diseñadas para restringir la cantidad de un

producto básico que puede importarse en un período dado. d. Incorrecta. Las restricciones voluntarias de exportaciones, que tienen el mismo efecto que las

cuotas de importación, estimulan a las empresas extranjeras a que limiten sus exportaciones a un país en particular.

60. Solución: c

a. Incorrecta. Los contratos de costos reembolsables se utilizan cuando los requerimientos resultan complejos y los costos no pueden identificarse ni estimarse con facilidad.

b. Incorrecta. Los contratos de entregas indefinidas se utilizan sólo cuando los suministros y/o servicios de futuras entregas no se conocen en el momento de subscribirse el contrato.

c. Correcta. Los contratos de precios fijos se utilizan cuando se encuentran bien definidos los requerimientos, pueden identificarse las inseguridades y estimarse los costos, y existe una competencia adecuada.

d. Incorrecta. Los contratos de tiempo y materiales se utilizan cuando no resulta posible estimar la duración exacta del trabajo en el momento de ubicar el contrato.

61. Solución: c

a. Incorrecta. Los impuestos progresivos, para los cuales la tasa tributaria promedio aumenta a medida que se incrementan los ingresos, consiguen tanto un porcentaje mayor de utilidades como un monto absoluto mayor de ingresos a medida que éstos se incrementan.

b. Incorrecta. Las tasas proporcionales, para las cuales una tasa tributaria promedio es constante para todos los niveles de ingresos, siempre consiguen un monto absoluto mayor de ingresos a medida que éstos se incrementan.

c. Correcta. Los impuestos regresivos son aquellos para los cuales se reduce la tasa tributaria promedio a medida que se incrementan los ingresos. Consiguen un menor porcentaje de ingresos a medida que éstos se incrementan; por lo tanto, no conseguirán un monto absoluto mayor de ingresos a medida que éstos se incrementan.

d. Incorrecta. Un impuesto fijo tendría la misma tasa tributaria de porcentajes cualquiera sea el ingreso y, en consecuencia, conseguiría un monto absoluto mayor de ingresos a medida que éstos se incrementan.

III - 44

62. Solución: c a. Incorrecta. Son las diferencias temporarias las que afectan a montos imponibles o deducibles

unos años más tarde, cuando se recuperan los montos informados de los activos y se fijan los montos informados de los pasivos.

b. Incorrecta. Las diferencias temporarias tienen consecuencias de impuestos diferidos mientras que las diferencias permanentes, no. Las diferencias permanentes afectan sólo a al período en el cual ocurren.

c. Correcta. Las diferencias permanentes no tienen consecuencias de impuestos diferidos porque afectan sólo al período en el cual ocurren. Las diferencias permanentes incluyen (1) items que forman parte de los ingresos financieros antes de impuestos, pero nunca de los ingresos imponibles y (2) items que forman parte de los ingresos imponibles, pero nunca de ingresos financieros antes de impuestos. En contraste, las diferencias temporarias provocan montos imponibles o deducibles un(os) año(s) más tarde, cuando se recuperan los montos informados de los activos y se fijan los montos informados de los pasivos. Por lo tanto, las diferencias temporarias sí tienen consecuencias de impuestos diferidos, mientras que las diferencias permanentes, no.

d. Incorrecta. Las diferencias permanentes, y no las temporarias, incluyen items que forman parte de los ingresos financieros antes de impuestos, pero nunca de ingresos imponibles.

63. Solución: a

a. Correcta. Un impuesto al valor agregado se cobra sobre la base del valor creado por la compañía. Se mide como la diferencia entre el valor de sus ventas y sus compras.

b. Incorrecta. Es una descripción de cómo calcular el impuesto sobre ganancias de capital. c. Incorrecta. Es una descripción de un precio de transferencia interna. d. Incorrecta. Es una descripción de cómo calcular el impuesto a las ganancias.

64. Solución: b

a. Incorrecta. Por definición, el producto nacional bruto (PNB) = C + I + G + X, donde C es consumo, I es inversión, G es gobierno y X es exportaciones netas (exportaciones menos importaciones). En consecuencia, el PNB aumentará con un incremento en el consumo.

b. Correcta. Un incremento en las importaciones provocará una caída en las exportaciones netas y el PNB.

c. Incorrecta. Un incremento en las exportaciones aumentará el PNB. d. Incorrecta. Un incremento en la inflación aumentará el PNB.

65. Solución: b

a. Incorrecta. La revisión de los registros de procesos computarizados es un control de salida para asegurar que los datos sean exactos y completos.

b. Correcta. La combinación de los datos de entrada con la información del archivo maestro o de los archivos en suspenso es un control de procesamiento, no un control de salida, para asegurar que los datos sean completos y exactos durante la actualización.

c. Incorrecta. La conciliación periódica de los informes de salida es un control de salida para asegurar que los datos sean exactos y completos.

d. Incorrecta. El mantenimiento de los procedimientos y la documentación formales que especifican los receptores autorizados constituye un control de salida para asegurar una distribución apropiada.

III - 45

66. Solución: c a. Incorrecta. Con frecuencia, el propósito de las revisiones de la base de datos es determinar si

(1) los usuarios obtuvieron el acceso a las áreas de la base de datos para las cuales no tienen autorización y (2) los usuarios autorizados pueden acceder a la base de datos utilizando programas que les suministren privilegios no autorizados para ver y/o modificar información.

b. Incorrecta. El propósito de las revisiones de cumplimientos es determinar si una organización cumplió con los procedimientos y reglamentos internos y externos.

c. Correcta. El control de cambios del programa abarca: (1) mantener los registros de las autorizaciones de cambios, modificaciones de códigos y resultados de pruebas; (2) adherirse a una metodología de desarrollo de los sistemas (con documentación inclusive); (3) autorizar las alteraciones de las interfases de las subsidiarias y las sedes y (4) restringir el acceso a la fuente autorizada y los códigos ejecutables.

d. Incorrecta. El propósito del software de seguridad de la red es proveer controles lógicos en toda la red.

67. Solución: c

a. Incorrecta. Los controles de los cortes programados mitigan el riesgo de registrar transacciones en el período equivocado.

b. Incorrecta. El hardware redundante es un control sobre el malfuncionamiento del hardware. c. Correcta. Los registros de las actividades proveen una pista de auditoría de la actividad del

usuario. d. Incorrecta. Los registros de los errores en las transacciones controlan las transacciones en

lugar de la actividad de los usuarios. 68. Solución: c

a. Incorrecta. La legibilidad de los datos de imágenes es importante para su uso, pero independiente del uso de la imagen equivocada.

b. Incorrecta. La exactitud de los datos de imágenes es importante para su uso, pero independiente del uso de la imagen equivocada.

c. Correcta. Si los datos del índice para los sistemas de procesamiento de imágenes es probable que los usuarios confíen en las imágenes equivocadas.

d. Incorrecta. El mantenimiento de la secuencia inicial de los datos de índice puede no resultar posible ya que los datos de imágenes se encuentran modificados y las imágenes se agregan / quitan.

69. Solución: a

a. Correcta. Los controles de implementación ocurren en el proceso de desarrollo de los sistemas en diversos puntos para asegurar que la implementación se encuentra apropiadamente controlada y administrada.

b. Incorrecta. Los controles de hardware aseguran que el hardware de la computadora se halla físicamente seguro y verifica que el equipo funcione en forma adecuada.

c. Incorrecta. Los controles de las operaciones de la computadora se aplican al trabajo del departamento de computación y ayuda a asegurar que los procedimientos programados sean consistentes y se apliquen en forma correcta al almacenamiento y procesamiento de datos.

d. Incorrecta. Los controles de seguridad de los datos aseguran que los archivos de datos, ya sean en disco o en cinta, no se encuentren sujetos a un acceso, cambio o destrucción no autorizados.

III - 46

70. Solución: a a. Correcta. Una puerta (gateway) que suele implementarse mediante un software traduce entre

dos o más familias diferentes de protocolos y posibilita las conexiones entre redes diferentes. b. Incorrecta. Un puente (bridge) une los segmentos de las redes de manera tal que parezcan ser

un solo segmento físico. c. Incorrecta. Un ruteador (router) conecta dos o más segmentos de redes, de manera tal que los

segmentos mantengan sus identidades lógicas separadas. d. Incorrecta. Un concentrador de cables (wiring concentrator) acepta un cableado de pares

trenzados de cada una de las diversas computadoras personales en la misma red del área local.

71. Solución: b

a. Incorrecta. Los informes de excepción pueden utilizarse para controlar la corrección y oportunidad de las actualizaciones, pero no pueden minimizar el impacto de una interrupción.

b. Correcta. La capacidad de continuar procesando en todos los sitios excepto en uno que no resulte funcional se denomina protección de fallas suave (fail-soft protection), una ventaja de los sistemas distribuidos.

c. Incorrecta. Los procedimientos de respaldo se utilizan para evitar el proceso de recuperación introduciendo cualquier cambio erróneo en el sistema después de detectar las fallas en la computadora.

d. Incorrecta. La seguridad en los archivos de datos se utiliza para evitar cambios no autorizados en los archivos de datos.

72. Solución: b

a. Incorrecta. Ver respuesta “b”. b. Correcta. Un dígito de verificación es un número de referencia extra que sigue un código de

identificación y guarda una relación matemática con los otros dígitos. Este dígito extra se ingresa con los datos. El código de identificación puede encontrarse sujeto a un algoritmo y compararse con el dígito de verificación.

c. Incorrecta. Ver respuesta “b”. d. Incorrecta. Ver respuesta “b”.

73. Solución: a

a. Correcta. Los beneficios marcados se producen cuando el intercambio electrónico de datos se encuentra ligado a los esfuerzos estratégicos que alteran, no reflejan, las prácticas anteriores. La aplicación del EDI a un proceso ineficiente provoca que se continúen haciendo las cosas mal, pero con mayor rapidez.

b. Incorrecta. El requisito previo para el éxito con el EDI es comprender la misión de la empresa y los procesos y flujos que respaldan sus metas, seguido de la cooperación con los socios externos. Los problemas del hardware aparecen después.

c. Incorrecta. Antes de aplicar la tecnología del EDI a la empresa, el EDI debe verse como parte de una solución integrada global para los requerimientos organizacionales.

d. Incorrecta. El EDI no es una solución en sí misma. En lugar de pensar en cómo vender las transacciones en todas partes, una compañía debe, en primer lugar, pensar sobre todo el proceso desde el inicio hasta el fin.

III - 47

74. Solución: b a. Incorrecta. Para abordar esta cuestión, debe evitarse el acceso no autorizado al sistema del

EDI, los procedimientos deben encontrarse en su lugar para asegurar el uso eficaz de claves de acceso y deben mantenerse la integridad de datos y la privacidad mediante el uso de medidas de encriptación y autenticación.

b. Correcta. El rastreo de los agradecimientos funcionales de los clientes, cuando se requiera, ayudará a asegurar la transmisión exitosa de las transacciones del EDI.

c. Incorrecta. Deben existir acuerdos contractuales entre la compañía y los socios comerciales del EDI.

d. Incorrecta. El riesgo de que los datos del EDI puedan no estar completos y exactamente procesados se encuentra primordialmente controlado por el sistema.

75. Solución: d

a. Incorrecta. Ver respuesta “d”. b. Incorrecta. Ver respuesta “d”. c. Incorrecta. Ver respuesta “d”. d. Correcta. Con una verificación de formato, la computadora constata las características del

contenido de los caracteres, la longitud o el signo de los campos de datos individuales. 76. Solución: c

a. Incorrecta. El sistema de escritura única y lectura múltiple (WORM, por sus siglas en inglés) es una técnica de almacenamiento que suele utilizarse como medio de archivo.

b. Incorrecta. La cinta de audio digital (DAT, por sus siglas en inglés) se utiliza primordialmente como medio de respaldo en los sistemas de imágenes y como archivo maestro para el CD-ROM.

c. Correcta. Resultaría más económico producir y embarcar un disco compacto / la memoria de sólo lectura (CD-ROM) que reproducir los voluminosos libros de texto e imágenes. Es probable que los usuarios de equipamiento electrónico tengan acceso a las lectoras de CD-ROM en las computadoras personales para poder utilizar la documentación del CD-ROM.

d. Incorrecta. La salida de computadora a microformato (COM, por sus siglas en inglés) se utiliza para el acceso frecuente a documentos archivados tales como cheques cancelados en aplicaciones bancarias.

77. Solución: d

a. Incorrecta. Las auditorías de aplicación deben tener casi la misma dificultad con o sin una mesa de ayuda equipada con el personal adecuado.

b. Incorrecta. La preparación de la documentación constituye una función de desarrollo, no una de mesa de ayuda.

c. Incorrecta. La probabilidad del uso de un código de programas no autorizado es una función de control de cambios, no una de mesa de ayuda.

d. Correcta. El riesgo más grande de no tener una mesa de ayuda equipada con el personal adecuado es que, sin saberlo, los usuarios seguirán cometiendo errores en su interacción con los sistemas de información.

78. Solución: a

a. Correcta. El reconocimiento óptico de caracteres (OCR, por sus siglas en inglés) convierte las imágenes de los documentos en papel, como los lee un dispositivo de scanner, en archivos de texto.

b. Incorrecta. Ver respuesta “a”. c. Incorrecta. Ver respuesta “a”. d. Incorrecta. Ver respuesta “a”.

III - 48

79. Solución: a a. Correcta. Las claves de acceso internamente encriptadas son controles diseñados para evitar

que los usuarios vean el archivo de la clave de acceso con la aplicación de un software de utilidad.

b. Incorrecta. Las jerarquías de las claves de acceso representan un conjunto de códigos de autorización interrelacionados para distinguir entre los privilegios de acción tales como leer, agregar o eliminar registros.

c. Incorrecta. La claves de acceso para iniciar representan los códigos de acceso al sistema típico para la autorización del usuario inicial.

d. Incorrecta. Una red par-a-par es un sistema que confía en una serie de microcomputadoras iguales para el procesamiento.

80. Solución: c

a. Incorrecta. Una bomba lógica es un mecanismo para lanzar algún ataque al sistema, que se activa cuando se presenta una condición en particular (por ejemplo, cierta fecha u operación del sistema).

b. Incorrecta. Un virus es el fragmento de un código (no un programa independiente) que se reproduce adjuntándose a otro programa.

c. Correcta. Un gusano es un programa independiente que se reproduce copiándose de un sistema a otro por una red y consume recursos de la computadora y la red.

d. Incorrecta. Un virus troyano es un programa independiente que parece realizar una función útil, pero oculta otro programa no autorizado dentro de él.

81. Solución: d

a. Incorrecta. La verificación de claves asegura la exactitud de los campos seleccionados requiriendo que una persona diferente vuelva a verificarlos.

b. Incorrecta. Las verificaciones de secuencia aseguran la integridad de la carga o descarga de datos verificando el uso de números de serie de documentos preasignados.

c. Incorrecta. El mapeo por computadora supone el control de archivos seleccionados de datos de entrada con información mantenida en suspenso o en un archivo maestro.

d. Correcta. Los usuarios pueden obtener acceso a las bases de datos desde las terminales sólo a través de procedimientos establecidos de reconocimiento y autorización; en consecuencia, se impide el acceso no autorizado.

82. Solución: c (I y III solamente)

I. Correcta. Es una afirmación verdadera. II. Incorrecta. Un mail confidencial no debe retenerse en el servidor una vez que el usuario lo

descargó en su computadora personal. III. Correcta. Es una afirmación verdadera.

83. Solución: b

a. Incorrecta. La encriptación de datos proporciona una adecuada seguridad para las computadoras personales.

b. Correcta. La protección de las claves de acceso para un programa de protección de pantalla puede eludirse con facilidad.

c. Incorrecta. Los discos rígidos removibles proporcionarían una adecuada seguridad. d. Incorrecta. La seguridad se alcanza guardando la computadora bajo llave en un estuche.

III - 49

84. Solución: a a. Correcta. Si bien la mayoría de los programas borran los punteros de archivo, no remueven los

datos residuales. La compañía debe utilizar dispositivos especiales que borren completamente los datos, lo cual es importante por la naturaleza confidencial de los datos de las microcomputadoras.

b. Incorrecta. Esto podría crear una responsabilidad para la compañía si un virus destruye datos o programas de la parte compradora. Sin embargo, la parte compradora debe utilizar un software anti-virus para detectar y eliminar cualquier virus. El tema, si bien es importante, no es tan serio como el de la respuesta “a”.

c. Incorrecta. La parte compradora tiene la responsabilidad de asegurar que todo su software tenga las licencias adecuadas. Si la compañía no demostró que todo su software tenía las licencias adecuadas, esto podría crear una responsabilidad. Sin embargo esta responsabilidad no es tan seria como lo implicado en la respuesta “a”.

d. El software de emulación de terminal está ampliamente disponible. 85. Solución: c

a. Incorrecta. Los riesgos externos deben ser evaluados para determinar la ubicación del centro. b. Incorrecta. Los sistemas de acceso biométricos controlan el acceso físico al centro de datos. c. Correcta. Las tablas de autorización para el acceso al sistema operativo está dirigido al control

lógico no al control físico. d. Incorrecta. Los sistemas de energía sin interrupción y protección contra sobrecargas están

incluidos en el diseño del centro de datos. 86. Solución: c

a. Incorrecta. El plan de contingencia se refiere a un acuerdo para lugares de procesamiento alternativos en el caso de una falla en el equipamiento.

b. Incorrecta. El estudio de factibilidad es una de las fases en el ciclo de vida de desarrollo de los sistemas.

c. Correcta. El plan debe incluir: metas y objetivos, un inventario de la capacidad actual y una estimación de las necesidades futuras.

d. Incorrecta. Los informes de excepción están dirigidos a resaltar problemas y llevarlos a la atención de la administración.

87. Solución: c

a. Incorrecto. La documentación del sistema no se elimina ni se posterga utilizando una aplicación de rápido desarrollo.

b. Incorrecta. La administración de proyectos implica el desarrollo de equipos. c. Correcta. El nuevo sistema se desarrollaría módulo por módulo. d. Incorrecta. Las técnicas de desarrollo por objetos podría no ser útil; si así fuera, se

incrementaría el uso del código anterior. 88. Solución: a

a. Correcta. Un sistema experto es un programa de computadora que se centra en el conocimiento y capta la pericia de un ser humano en dominios limitados del conocimiento.

b. Incorrecta. Una red neural es un software que trata de emular los patrones de procesamiento del cerebro biológico.

c. Incorrecta. Los agentes inteligentes son programas de software que utilizan una base de conocimiento incorporado o aprendido a fin de llevar a cabo tareas específicas, repetitivas y predecibles para un usuario individual, un proceso de negocios o una aplicación de software. En Internet, un agente inteligente es, en general, un programa que recolecta información o realiza algún otro servicio sin la presencia inmediata del usuario y sobre la base de un programa regular.

d. Incorrecta. La lógica confusa es inteligencia artificial que se basa en las reglas y tolera la imprecisión mediante el uso de términos no específicos denominados funciones de membresía para solucionar problemas.

III - 50

89. Solución: d a. Incorrecta. En lugar de la ausencia de documentos tradicionales de diseño, los items tales

como el modelo de negocios, la narrativa de las funciones del proceso, las pantallas iterativas de desarrollo, los procesos e informes computarizados y las guías de las descripciones de los productos se llevan a cabo en un desarrollo orientado hacia el objeto, pero la existencia de documentos específicos no afectan a la importancia de las pruebas de aceptación del usuario.

b. Incorrecta. En general, los sistemas de desarrollo orientados hacia el objeto no incluyen los de rastreo para los cambios hechos a los objetos y jerarquías.

c. Incorrecta. Como los sistemas orientados hacia el objeto en general se desarrollan en los ambientes del cliente/servidor, existe la posibilidad de que se realice una supervisión continua del uso del sistema, pero la supervisión continua es habitual durante la operación del sistema, no durante el desarrollo.

d. Correcta. Las pruebas de aceptación del usuario son más importantes en el desarrollo orientado hacia el objeto por el hecho de que todos los objetos de una clase heredan las propiedades de la jerarquía, lo cual significa que los cambios en un objeto pueden afectar a otros. Ello incrementa la importancia de las pruebas de aceptación del usuario para verificar el correcto funcionamiento de todo el sistema.

90. Solución: d

a. Incorrecta. La ausencia de interrupciones en el proceso no indica nada sobre las interrupciones que puedan ocurrir en el futuro, especialmente sobre aquellas que no están bajo el control de la organización.

b. Incorrecta. Un plan de contingencia puede incluir documentación completa, pero hasta que se pruebe, la organización no tiene idea de su eficacia.

c. Incorrecta. La firma del auditor es un indicador de la calidad del plan, pero hasta que éste se pruebe, la organización no tiene indicación de su eficacia.

d. Correcta. La única forma de conocer si el plan de contingencia es eficaz es probarlo simulando una interrupción o conduciendo una prueba completa de los procedimientos de recuperación.

91. Solución: c

a. Incorrecta. Esta práctica es un control adecuado, pero no está dirigido a la cuestión de la integridad de los datos cargados. Los archivos de respaldo no pueden evitar ni detectar problemas en la carga de los datos; sólo pueden ayudar a corregir errores causados por una carga deficiente.

b. Incorrecta. Este control puede ser útil para evitar el fraude en la carga de los datos, pero es poco útil para evitar errores.

c. Correcta. Esto podría evitar errores de datos. d. Incorrecta. Este control es detectivo por naturaleza, pero el error ya pudo haber causado

informes erróneos y decisiones de administración equivocadas. Hacer que los usuarios traten de encontrar errores en la carga de datos sería costoso.

92. Solución: d

a. Incorrecta. Los controles físicos limitan el acceso a un área y no incluyen claves de acceso. b. Incorrecta. Los controles de edición prueban la validez de los datos. c. Incorrecta. Los controles digitales son ejemplos de controles físicos. d. Correcta. Las claves de acceso constituyen una forma de controles dado que limitan el acceso

a los sistemas y a la información almacenada en ellos.

III - 51

93. Solución: d a. Incorrecta. La revisión de los trabajos procesados revelará el acceso, pero no lo evitará. b. Incorrecta. La comparación de programas de producción y copias controladas revelará los

cambios, pero no los evitará. c. Incorrecta. La producción periódica de datos de prueba detectará los cambios, pero no los

evitará. d. Correcta. Cuando las responsabilidades están separadas, los usuarios no pueden obtener un

conocimiento detallado de los programas y los operadores no pueden obtener acceso no supervisado a los programas de producción.

94. Solución: d

a. Incorrecta. Los modems asincrónicos manejan el flujo de datos desde los dispositivos periféricos hasta un procesador central.

b. Incorrecta. Las técnicas de autenticación confirman que los usuarios válidos tengan acceso al sistema.

c. Incorrecta. Los procedimientos de rellamada se usan para asegurar que las llamadas entrantes provengan de lugares autorizados.

d. Correcta. Los dispositivos criptográficos protegen los datos en transmisión a través de las líneas de comunicación.

95. Solución: d

a. Incorrecta. Normalmente, las terminales de datos no utilizan protectores de pantalla. b. Incorrecta. El uso de párrafos de terminal (“scripting”) son programas para automatizar

procesos tales como el inicio o arranque de la computadora. c. Incorrecta. La encriptación de archivos no impedirá ver sus datos en una terminal que no se

encuentre en uso. d. Correcta. La baja automática de usuarios inactivos puede impedir ver datos sensibles en una

terminal que no se encuentre en uso. 96. Solución: a

a. Correcta. Los usuarios necesitan actualizar datos mediante programas de aplicación. b. Incorrecta. Los programadores de aplicación no deben efectuar cambios en los programas de

producción. Deben presentarlos a la unidad de control de cambios. c. Incorrecta. Los programadores de aplicación jamás deben tener acceso a la actualización de

los datos de producción. Los usuarios no tienen necesidad alguna de cambiar los programas de producción.

d. Incorrecta. Ver respuestas "b" y "c". 97. Solución: c

a. Incorrecta. La instalación de un sistema de registros (“logs”) para el acceso a los programas permitiría la detección de acceso no autorizado, pero no lo impediría.

b. Incorrecta. La supervisión del acceso físico a las bibliotecas de programas controlaría sólo el acceso físico no autorizado.

c. Correcta. Restringir el acceso físico y lógico asegura las bibliotecas de programas contra el uso no autorizado en forma personal o remota por medio de las terminales.

d. Incorrecta. Denegar todo acceso remoto a través de terminales sería probablemente ineficiente y no aseguraría las bibliotecas de programas contra el acceso físico.

III - 52

98. Solución: a a. Correcta. El lenguaje de definición de datos (DDL) se utiliza para definir (o determinar) la base

de datos. b. Incorrecta. El lenguaje de control de datos (DCL) se utiliza para especificar privilegios y reglas

de seguridad. c. Incorrecta. El lenguaje de manipulación de datos (DML) proporciona a los programadores un

medio para actualizar la base de datos. d. Incorrecta. El lenguaje de consulta de datos (DQL) se utiliza para consultas que surjan en un

momento dado. 99. Solución: c

a. Incorrecta. La mayoría de las herramientas de consulta (query) incluyen la capacidad de presentar los resultados de dichas consultas en forma gráfica.

b. Incorrecta. Las herramientas de consulta incluyen diccionario de acceso de datos porque es la manera de saber qué atributos de la tabla presentar ante los usuarios.

c. Correcta. La característica menos probable de una herramienta de consulta sería un chequeo de validación de datos porque el sistema de la base de datos ya puso en vigor cualquier restricción a la validez en el momento en que la información se insertó en la base de datos. Cualquier chequeo de validación adicional sería una función del programa de aplicación de un usuario más que una consulta.

d. Incorrecta. Las herramientas de consulta suelen tener una interfase de consulta por ejemplo. 100. Solución: b

a. Incorrecta. No hay limitación en la cantidad de puertos de acceso. b. Correcta. El aspecto más difícil del uso de los recursos de Internet es localizar la mejor

información, dada la gran cantidad de fuentes de información. c. Incorrecta. El único equipamiento requerido para el acceso a los recursos de Internet es una

computadora, un módem, una línea telefónica u otras líneas de acceso y un software básico de computación.

d. Incorrecta. Como rutina, las organizaciones proporcionan acceso a Internet para sus empleados, y los individuos pueden obtener acceso a través de subscripciones individuales a proveedores del servicio comercial.

FIN DE LAS SOLUCIONES PARTE III

El no cumplir con estas instrucciones y con las guías de "Instrucciones para candidatos" puede afectar seriamente

su derecho para recibir los resultados de este examen y su futura participación el programa de Auditor Interno Certificado.

Todos los documentos presentados a la terminación de cualquiera de las

partes de este examen son propiedad exclusiva de The Institute of Internal Auditor, Inc. Los candidatos no puede revelar el contenido de este examen a menos que cuenten con la

autorización del Departamento de Certificación.

IV - 1

Preguntas del examen modelo para el Auditor Interno Certificado (CIA)

Parte IV – Destrezas de la

administración del negocio

Preguntas del examen modelo parte IV: 100

Preguntas del examen CIA real parte IV: 125 (ver explicación en el “prólogo”, página iii)

Tiempo permitido para terminar el examen CIA parte IV: 210 minutes

Instrucciones como las que a continuación se enuncian serán las que aparezcan

en la portada de cada examen CIA. Por favor, léanlas con cuidado.

1. Anote su número de candidato en la hoja de respuestas en el espacio proporcionado.

2. No haga anotaciones irrelevantes en la hoja de respuestas

3. Aségurese de que todo cambio de respuesta sea completamente borrado.

4. Todas las referencias al Marco para la Práctica Profesional se refieren al Marco para la Práctica Profesional del IAI, que incluye las Normas y los Consejos para la Práctica. Todas las referencias a las Normas se refieren a las Normas Internacionales para el Ejercicio Profesional de la Auditoría Interna resumidas en el Marco para la Práctica Profesional del IAI.

IV - 2

1. Cuando las empresas compiten en diferentes ubicaciones geográficas o tienen múltiples líneas de productos que no necesariamente se superponen, la manera más eficaz de responder a una acción agresiva de un competidor sin hacer movimientos destructivos ni contraataques es la siguiente: a. Despistar al competidor para que tome o

no una acción. b. Hacer un anuncio previo de los

movimientos que se proponen hacer. c. Iniciar un movimiento en el mercado

cuando el competidor se muestra fuerte. d. Iniciar movimientos agresivos directos.

2. El componente clave de una organización

configurada como componente profesional es el siguiente: a. Punto estratégico. b. Tecnoestructura. c. Administración de la gerencia de línea. d. Núcleo operativo.

3. Una compañía manufacturera produce

utensilios de plástico para un segmento del mercado en particular al menor costo posible. La compañía está buscando un costo de: a. Estrategia de liderazgo. b. Estrategia de enfoque. c. Estrategia de diferenciación. d. Estrategia de contención.

4. Según el modelo de Porter, la acción

concurrente de las fuerzas competitivas básicas determina: a. Las utilidades a largo plazo y la

intensidad de la competencia en la industria.

b. Las barreras que posibles actuantes deban enfrentar para ingresar en la industria.

c. La rivalidad dentro de la industria. d. La estrategia que una compañía debe

seguir para alcanzar sus objetivos. 5. ¿Cuál de las siguientes es una definición

empresaria orientada al mercado en contraposición con una definición orientada al producto? a. Fabricamos acondicionadores de aire y

calderas. b. Suministramos energía. c. Producimos películas. d. Vendemos ropa.

1. When firms compete in different geographical locations or have multiple product lines that do not necessarily overlap, the most effective way of responding to an aggressive move by a competitor without directly triggering destructive moves and countermoves is to: a. Mislead the competitor into taking or not

taking an action. b. Make a prior announcement of intended

moves. c. Initiate a move in the market where the

competitor is strong. d. Initiate direct aggressive moves.

2. The key component of an organization

configured as a professional bureaucracy is the: a. Strategic apex. b. Technostructure. c. Line administration. d. Operating core.

3. A manufacturing company produces plastic

utensils for a particular market segment at the lowest possible cost. The company is pursuing a cost: a. Leadership strategy. b. Focus strategy. c. Differentiation strategy. d. Containment strategy.

4. The concurrent action of basic competitive

forces as defined by Porter’s model determines the: a. Long-term profitability and the

competency intensity of the industry. b. Barriers that potential players must face

to enter the industry. c. Rivalry within the industry. d. Strategy that a company should follow to

achieve its objectives. 5. Which of the following is a market-oriented

definition of a business versus a product-oriented definition of a business? a. We make air conditioners and furnaces. b. We supply energy. c. We produce movies. d. We sell clothing.

IV - 3

6. ¿Cuál de las siguientes afirmaciones describe mejor la sinergia en un mercado? a. La transferencia de tecnología de un

producto a otro. b. Un paquete de productos distribuidos

mediante los mismos canales. c. La producción de múltiples productos en

una instalación. d. El uso de destrezas complementarias de

administración para lograr ingresar en un nuevo mercado.

7. ¿Cuál de las siguientes es una estrategia que

las compañías pueden utilizar para estimular la innovación?

I. Tener como fuente a los proveedores

más avanzados. II. Establecer programas que recompensen

la iniciativa del empleado. III. Identificar las mejores prácticas de los

competidores como motivadores. IV. Asegurar que siempre se alcancen las

metas de desempeño. a. I solamente. b. II y IV solamente. c. I, II, y III solamente. d. I, II, III, y IV.

8. ¿Cuál de las siguientes es una fuerza básica

que impulsa la competencia de la industria y que, combinada con otras fuerzas competitivas, determina el mejor potencial de utilidades en la industria?

I. La amenaza de nuevos participantes. II. El poder negociador de los proveedores. III. El acceso favorable a las materias

primas. IV. La diferenciación de productos. a. I solamente. b. I y II solamente. c. III y IV solamente. d. I, II, III y IV.

9. ¿Cuál de las siguientes no es una

característica de un ambiente maduro de la industria? a. Consolidación. b. Interdependencia competitiva. c. Demanda en declinación. d. Enfoque estratégico sobre el

impedimento del ingreso de nuevos competidores en el mercado.

6. Which of the following statements best describes a market synergy? a. Technology transfer from one product to

another. b. Bundling of products distributed through

the same channels. c. Production of multiple products at one

facility. d. Use of complementary management skills

to achieve entry into a new market 7. Which of the following is a strategy that

companies can use to stimulate innovation?

I. Source from the most advanced suppliers.

II. Establish employee programs that reward initiative.

III. Identify best practice competitors as motivators.

IV. Ensure that performance targets are always achieved.

a. I only. b. II and IV only. c. I, II, and III only. d. I, II, III, and IV.

8. Which of the following is a basic force that

drives industry competition and which, when combined with other competitive forces, determines the ultimate profit potential in the industry?

I. Threat of new entrants. II. Bargaining power of suppliers. III. Favorable access to raw materials. IV. Product differentiation.

a. I only. b. I and II only. c. III and IV only. d. I, II, III, and IV.

9. Which of the following is not characteristic of a

mature industry environment? a. Consolidation. b. Competitive interdependence. c. Falling demand. d. Strategic focus on deterring entry of new

competitors into the marketplace.

IV - 4

10. ¿En cuál de los siguientes ambientes de la industria la concesión de franquicias y las fusiones horizontales constituyen estrategias comúnmente utilizadas? a. Las industrias emergentes. b. Las industrias en declinación. c. Las industrias fragmentadas. d. Las industrias maduras.

11. ¿Cuál de los siguientes costos necesita

considerar la gerencia para introducir un nuevo producto o sustituir un producto existente por uno nuevo?

I. Costos de reentrenamiento de

empleados. II. Costos de adquirir nuevos equipos

auxiliares. III. Anulaciones de inversiones en tecnología

obsoleta no amortizadas. IV. Requerimientos de capital para el cambio. a. I y III solamente. b. I, II, y IV solamente. c. II, III y IV solamente. d. I, II, III y IV.

12. ¿Cuál de las siguientes sería una ventaja

competitiva global? a. Costos fijos bajos. b. Economías de escala de producción. c. Protección débil para los derechos de

autor. d. Requerimientos intensivos de servicio

local. 13. Los gobiernos restringen el comercio para:

I. Fomentar la seguridad nacional. II. Desarrollar nuevas industrias. III. Proteger industrias en declinación. IV. Incrementar los ingresos tributarios. a. I y IV solamente. b. II y III solamente. c. I, II y III solamente. d. II, III y IV solamente.

14. ¿Cuál de las siguientes es una tendencia

social que afecta a la organización? a. Cambios en los mercados laborales. b. Legislación más estricta para proteger el

ambiente. c. Inflación que aumenta. d. Repuestos de acero en automóviles y

artefactos eléctricos.

10. In which of the following industry environments are franchising and horizontal mergers commonly used strategies? a. Emerging industries. b. Declining industries. c. Fragmented industries. d. Mature industries.

11. Which of the following costs does

management need to consider when introducing a new product or substituting a new product for an existing one?

I. Costs of retraining employees. II. Costs of acquiring new ancillary

equipment. III. Write-offs due to undepreciated

investment in old technology. IV. Capital requirements for changeover.

a. I and III only. b. I, II, and IV only. c. II, III, and IV only. d. I, II, III, and IV.

12. Which of the following would be a source of

global competitive advantage? a. Low fixed costs. b. Production economies of scale. c. Weak copyright protection. d. Intensive local service requirements.

13. Governments restrict trade in order to:

I. Foster national security. II. Develop new industries. III. Protect declining industries. IV. Increase tax revenues.

a. I and IV only. b. II and III only. c. I, II, and III only. d. II, III, and IV only.

14. Which of the following is a social trend

affecting an organization? a. Changes in the labor markets. b. Tougher legislation to protect the

environment. c. Rising inflation. d. Replacements for steel in cars and

appliances.

IV - 5

15. ¿Cuál de los siguientes factores estimularía el ingreso en un mercado existente? a. Subsidio del gobierno para nuevos

inversores. b. Alta diferenciación de productos,

principalmente producida por marcas registradas.

c. Conocimiento de la industria, con altas inversiones en desarrollo.

d. Bajos costos fijos de realización. 16. Una estrategia de integración hacia atrás es

más apropiada cuando los proveedores actuales de la empresa: a. Son altamente confiables. b. No son confiables. c. Se encuentran geográficamente

dispersos. d. Se encuentran geográficamente

concentrados. 17. La producción “justo a tiempo”:

a. Logra que se dependa menos de los proveedores.

b. Reduce el costo de implementar nuevas estrategias.

c. Hace decrecer la flexibilidad de las instalaciones de producción.

d. Incrementa la necesidad de una fuerza laboral confiable.

18. La oportunidad de conceder franquicias surge

de la habilidad de: a. Desarrollar productos. b. Diferenciar productos. c. Estandarizar productos. d. Diversificar productos.

19. La expansión de la capacidad también se

conoce como: a. Penetración en el mercado. b. Desarrollo del mercado. c. Desarrollo de productos. d. Diversificación.

20. Una compañía productora de leche adquiere

sus propios establecimientos de producción y venta. La estrategia de crecimiento adoptada por la compañía puede identificarse como: a. Integración horizontal. b. Integración vertical. c. Diversificación concéntrica. d. Diversificación conglomerada.

15. Which of the following factors would encourage entry into an existing market? a. Governmental subsidy for new investors b. High product differentiation, principally

produced by trademarks c. Knowledge of the industry, with high

investments in development d. Low exit fixed costs

16. A backward integration strategy is most

appropriate when the firm’s current suppliers are: a. Highly reliable. b. Not reliable. c. Geographically dispersed. d. Geographically concentrated.

17. Just-in-time production:

a. Reduces the dependency on suppliers. b. Reduces the cost of implementing new

strategies. c. Decreases production facility flexibility. d. Increases the need for a dependable

workforce. 18. The opportunity for franchising comes from the

ability to: a. Develop products. b. Differentiate products. c. Standardize products. d. Diversify products.

19. Capacity expansion is also referred to as:

a. Market penetration. b. Market development. c. Product development. d. Diversification.

20. A milk-producing company acquires its own

dairy farms to supply milk. The growth strategy adopted by the company can be identified as: a. Horizontal integration. b. Vertical integration. c. Concentric diversification. d. Conglomerate diversification.

IV - 6

21. Una empresa posee una unidad de negocio estratégica (SBU, por sus siglas en inglés) que tiene una baja participación en un mercado de alto crecimiento. Mantener incluso esta baja participación requiere que la empresa se comprometa a una significativa cantidad de efectivo. La empresa podría lograr adoptar una estrategia de construcción para esta unidad si:

I. La SBU muestra un fuerte potencial de

crecimiento y obtiene una significativa participación en el mercado.

II. La empresa puede financiar su crecimiento.

III. La empresa espera un incremento a corto plazo en el flujo de fondos.

IV. La empresa desea renunciar a las ganancias a corto plazo.

a. I solamente. b. II y III solamente. c. III y IV solamente. d. I, II y IV solamente.

22. ¿Cuál de las siguientes afirmaciones es

verdadera con respecto a objetivos organizacionales? a. No se necesitan objetivos para fijar metas

y una dirección. b. La fijación de objetivos suele ser un

resultado de la negociación. c. Los objetivos son declaraciones

generales de dirección de acuerdo con una meta.

d. No resulta necesario actualizar los objetivos en forma continua para reflejar los cambios ambientales.

23. En la matriz de crecimiento-participación del

Grupo de Consultoría de Boston (BCG, por sus siglas en inglés), ¿en qué estrategia se requiere gran cantidad de efectivo e importantes inversiones necesarias para crecer y mantener un posicionamiento competitivo, pero el flujo de fondos neto es generalmente modesto? a. Negocios “vaca”. b. Negocios “interrogante”. c. Negocios “perro”. d. Negocios “estrella”.

24. En el ciclo de vida de un producto, el primer

síntoma de la etapa de declinación es un decrecimiento en: a. Los niveles de inventario de la empresa. b. Las ventas del producto. c. Los costos de producción del producto. d. Los precios del producto.

21. A firm has a strategic business unit (SBU) that has a low market share in a high growth market. To maintain even this low share of the market requires the firm to commit a significant amount of cash. The firm might successfully adopt a build strategy for this unit if the:

I. SBU shows a strong potential to grow and

obtains a significant share of the market. II. Firm can finance its growth. III. Firm expects a short-term increase in

cash flow. IV. Firm is willing to forego short-term

earnings.

a. I only. b. II and III only. c. III and IV only. d. I, II, and IV only.

22. Which of the following statements is true

regarding organizational objectives? a. Objectives are not needed in order to set

targets and direction. b. Objective setting is often a result of

negotiation. c. Objectives are general statements of

direction in line with a goal. d. Objectives need not be continually

updated to reflect environmental changes. 23. In the Boston Consulting Group (BCG) growth-

share matrix, which strategy in the matrix describes large generation of cash and heavy investment needed to grow and maintain competitive positioning but net cash flow is usually modest? a. Cash cows. b. Question marks. c. Dogs. d. Stars.

24. In the product life cycle, the first symptom of

the decline stage is a decline in a: a. Firm’s inventory levels. b. Product’s sales. c. Product’s production cost. d. Product’s prices.

IV - 7

25. Durante la etapa de crecimiento del ciclo de vida de un producto: a. La calidad de los productos es pobre. b. La calidad de los productos mejora en

forma continua. c. Existe poca diferencia entre los productos

que compiten. d. La calidad de los productos se torna más

variable y los productos se diferencian menos.

26. Durante la etapa de introducción de la

innovación de un producto, el crecimiento en las ventas es normalmente lento debido a: a. La promoción costosa de ventas. b. La alta competencia. c. La producción excesiva. d. Las alternativas disponibles.

27. ¿Durante qué etapa del ciclo de vida de un

producto es muy probable que el precio asignado para un producto específico sea más bajo? a. La etapa de introducción. b. La etapa de crecimiento. c. La etapa de madurez. d. La etapa de declinación.

28. ¿En qué etapa del ciclo de vida de un

producto sería mayor la oportunidad de reducir costos? a. La etapa de introducción. b. La etapa de crecimiento. c. La etapa de madurez. d. La etapa de declinación.

29. ¿Cuál de las siguientes es la razón más

significativa por la cual los gobiernos locales y las organizaciones internacionales buscan eliminar convenios de industrias para controlar precios y normas? a. Los precios incrementados de ventas

reducen la cantidad de ingresos tributarios corporativos pagaderos al gobierno.

b. La verdadera competencia mantiene los precios lo más bajos posible, incrementando así la eficiencia en el mercado.

c. Las empresas pequeñas no pueden sobrevivir ni crecer sin protección gubernamental.

d. La estabilidad económica de desarrollar países depende de un mercado libre global.

25. During the growth stage of the product life cycle: a. Quality of products is poor. b. Quality of products continuously

improves. c. There is little difference between

competing products. d. Quality of products becomes more

variable and products are less differentiated.

26. During the introduction stage of an innovative

product, sales growth is normally slow due to: a. Expensive sales promotion. b. High competition. c. Overproduction. d. Available alternatives.

27. The price charged on a consistent basis for a

specific product would most likely be lowest during which stage of the product life cycle? a. Introduction stage. b. Growth stage. c. Maturity stage. d. Decline stage.

28. The opportunity for cost reductions would be

greatest in which stage of the product life cycle? a. Introduction stage. b. Growth stage. c. Maturity stage. d. Decline stage.

29. Which of the following is the most significant

reason that domestic governments and international organizations seek to eliminate cartels? a. Increased sales prices reduce the amount

of corporate tax revenues payable to the government.

b. True competition keeps prices as low as possible, thus increasing efficiency in the marketplace.

c. Small businesses cannot survive or grow without government protection.

d. The economic stability of developing countries depends on a global free market.

IV - 8

30. Cuando una empresa multinacional decide vender sus productos al extranjero, uno de los riesgos que enfrenta es que el gobierno del mercado extranjero podría imputar el “dumping” a la empresa, lo cual ocurre cuando: a. Un producto se vende a diferentes

precios en distintos lugares geográficos. b. Una empresa imputa un costo menor al

que implica hacer un producto para ingresar en la participación en el mercado y obtenerla.

c. Las versiones de baja calidad de un producto se venden al extranjero para poder aprovecharlo.

d. Los precios de transferencia se fijan artificialmente altos para minimizar los pagos de impuestos.

31. Cuando se inician las operaciones

internacionales, una organización debe considerar las dimensiones culturales para evitar malentendidos. ¿Cuál de las siguientes no representa una dimensión cultural reconocida en un ambiente laboral? a. Auto-control. b. Distancia del poder. c. Masculinidad versus feminidad. d. El evitar la inseguridad.

32. ¿Cuál de las siguientes técnicas incrementaría

el entendimiento de una situación compleja y ambigua que enfrenta una organización?

I. “Brainstorming” o tormenta de ideas. II. Encuestas. III. Pensamiento lateral. a. I solamente. b. I y III solamente. c. II y III solamente. d. I, II y III.

33. ¿Cuál de las siguientes representa el

impedimento más significativo para fusionar las bases de datos de los clientes sobrepasando los límites internacionales? a. El tiempo de respuesta. b. Las cuestiones tributarias. c. Las regulaciones sobre la privacidad. d. Los archivos de respaldo y la

recuperación. 34. Los tres factores principales que favorecen la

globalización son los siguientes: a. Cultural, comercial, técnico b. Flexibilidad, proximidad, adaptabilidad. c. Político, tecnológico, social d. Ambición, posicionamiento, organización.

IV - 9

30. When a multinational firm decides to sell its products abroad, one of the risks that it faces is that the government of the foreign market could charge the firm with dumping, which occurs when: a. A product sells at different prices

geographically. b. A firm charges less than it costs to make

a product in order to enter and win market share.

c. Lower quality versions of a product are sold abroad in order to be affordable.

d. Transfer prices are set artificially high in order to minimize tax payments.

31. When initiating international ventures, an

organization should consider cultural dimensions to prevent misunderstandings. Which of the following does not represent a recognized cultural dimension in a work environment? a. Self-control. b. Power distance. c. Masculinity versus femininity. d. Uncertainty avoidance.

32. Which of the following would increase

understanding of a complex and ambiguous situation confronted by an organization?

I. Brainstorming. II. Polling. III. Lateral thinking.

a. I only. b. I and III only. c. II and III only. d. I, II, and III.

33. Which of the following represents the most

significant impediment to merging customer databases across international boundaries? a. Response time. b. Taxation issues. c. Privacy regulations. d. Backup and recovery.

34. The three major factors favoring globalization are: a. Cultural, commercial, technical. b. Flexibility, proximity, adaptability. c. Political, technological, social. d. Ambition, positioning, organization.

IV - 10

35. ¿Cuál de las siguientes orientaciones de la gerencia se caracteriza por los esfuerzos de una organización para adaptar el producto y el programa de marketing a cada ambiente local? a. Etnocéntrica. b. Policéntrica. c. Geocéntrica. d. Regiocéntrica.

36. La globalización asiste en lograr economías de

escala, lo cual constituye: a. Un beneficio de costos. b. Un beneficio de oportunidad. c. Un beneficio de aprendizaje. d. Un beneficio de arbitraje.

37. En algunas regiones del mundo, los negocios

se realizan más a menudo a través de relaciones personales que de contratos legales, lo cual es un ejemplo de: a. Factor cultural. b. Factor commercial. c. Factor técnico. d. Factor legal.

38. Las compañías multinacionales se muestran

mejor posicionadas para administrar sus obligaciones tributarias extranjeras al hacer lo siguiente: a. Declarar menos utilidades en países con

niveles de impuestos más bajos. b. Declarar más utilidades en países con

niveles de impuestos más bajos. c. Imputar precios más altos para

componentes en países con niveles de impuestos más bajos.

d. Imputar precios más bajos para componentes en países con niveles de impuestos más altos.

39. ¿Cuál de las siguientes situaciones describe

una fuente de interrupción en la comunicación que se provoca dentro de una organización debido a un sentido de superioridad mostrado por los miembros de una cultura en particular con respecto a otra? a. Problema de percepción. b. Estereotipos. c. Etnocentrismo. d. El evitar la inseguridad.

40. El nacionalismo, la expropriación y el

terrorismo son los mejores ejemplos de: a. Riesgo económico. b. Riesgo político. c. Riesgo operativo. d. Riesgo ambiental.

35. Which of the following management orientations is characterized by an organization’s efforts to adapt the product and marketing program to each local environment? a. Ethnocentric. b. Polycentric. c. Geocentric. d. Regiocentric.

36. Globalization assists in achieving economies

of scale, which is a: a. Cost benefit. b. Timing benefit. c. Learning benefit. d. Arbitrage benefit.

37. In some regions of the world, business is

conducted more often through personal relationship building than through legal contracts. This is an example of a: a. Cultural factor. b. Commercial factor. c. Technical factor. d. Legal factor.

38. Multinational companies are better poised to

manage their overseas tax liability by: a. Declaring lower profits in countries with

lower taxation levels. b. Declaring higher profits in countries with

lower taxation levels. c. Charging higher prices for components in

countries with lower taxation levels. d. Charging lower prices for components in

countries with higher taxation levels. 39. Which of the following describes a source of

communication breakdown which occurs within an organization due to a sense of superiority by members of a particular culture over another? a. Perceptual problem. b. Stereotyping. c. Ethnocentrism. d. Uncertainty avoidance.

40. Nationalism, expropriation, and terrorism are

best categorized as examples of: a. Economic risk. b. Political risk. c. Operational risk. d. Environmental risk.

IV - 11

41. ¿Cuál de los siguientes es un típico aspecto cultural que hace más difícil la comunicación internacional e intercultural?

I. Las largas distancias entre emisor y

receptor. II. El lenguaje corporal. III. El idioma. IV. La actitud. V. El tiempo. a. I y III solamente. b. II y V solamente. c. I, IV y V solamente. d. II, III, IV y V solamente.

42. La globalización y la localización están

moldeando la estructura competitiva de las industrias. El escenario que contribuye al ambiente más competitivo se presenta cuando: a. Dominan las fuerzas globales. b. Dominan las fuerzas locales. c. Domina una mezcla de fuerzas globales y

locales. d. No dominan ni las fuerzas globales ni las

locales. 43. Para una empresa multinacional, ¿cuál de las

siguientes es una desventaja de la política etnocéntrica de dotación de personal en la cual todas las posiciones gerenciales clave las ocupan empleados de empresas matrices? a. Eleva los costos de compensación,

capacitación y dotación de personal. b. Provoca resentimiento entre los

empleados de la empresa de países receptores.

c. Limita el avance profesional de los empleados de empresas matrices.

d. Aisla las sedes de las subsidiarias extranjeras.

44. Una de las claves para el rediseño exitoso de

los puestos a fin de motivar a los empleados es: a. La formación de equipos de trabajo

autónomos. b. La ampliación de los puestos por medio

de incorporar más tareas similares a las que se desempeñan.

c. La rotación de trabajadores a puestos diferentes para proporcionarles variedad.

d. El cambio de funciones de los puestos de manera que se adapten a las necesidades de crecimiento de cada trabajador.

41. Which of the following is a cultural aspect that typically makes international and intercultural communication more difficult?

I. Long distances between sender and

receiver. II. Body language. III. Language. IV. Attitude. V. Time.

a. I and III only. b. II and V only. c. I, IV, and V only. d. II, III, IV, and V only.

42. Globalization and localization are shaping the

competitive structure of industries. The scenario contributing to the most competitive environment is when: a. Global forces dominate. b. Local forces dominate. c. Mix of global and local forces dominate. d. Neither global nor local forces dominate.

43. For a multinational firm, which of the following

is a disadvantage of an ethnocentric staffing policy in which all key management positions are filled by parent-company nationals? a. It significantly raises the compensation,

training, and staffing costs. b. It produces resentment among the firm’s

employees in host countries. c. It limits career mobility for parent-country

nationals. d. It isolates headquarters from foreign

subsidiaries. 44. One of the keys to successfully redesigning

jobs is: a. Creating autonomous work teams. b. Enlarging jobs by adding more tasks

similar to those being performed. c. Rotating workers to different jobs to

provide them with variety. d. Changing the content of jobs so that they

fit workers’ need for growth.

IV - 12

45. La necesidad de un empleado de auto-realización se satisfaría mediante: a. Suministro de pensiones y jubilaciones

atractivas. b. Asignaciones de nuevas tareas

desafiantes. c. Buenas condiciones laborales. d. Respuestas positivas regulares.

46. ¿Cuál de las siguientes teorías incluye la

aseveración de que los empleados pueden motivarse mediante el logro de aceptación o estima en el lugar de trabajo? a. La teoría de la equidad. b. La teoría de la expectative. c. La teoría de la necesidad de jerarquías. d. La teoría de la fijación de metas.

47. ¿Cuál de las siguientes acciones tomadas por

la gerencia no resultaría eficaz para motivar a un empleado a fin de que logre un desempeño superior? a. Ampliación de los puestos. b. Enriquecimiento del trabajo. c. Seguridad en el trabajo. d. Rotación en el trabajo.

48. ¿Cuál de las siguientes situaciones fortalece e

incrementa una conducta aceptable eliminando o evitando consecuencias no deseadas? a. Refuerzo positivo. b. Refuerzo negativo. c. Premio. d. Castigo.

49. Entre los premios no financieros de un

empleado, un viaje de vacaciones pagas se conoce como: a. Premio social. b. Premio simbólico. c. Premio visual/auditivo. d. Manipulables.

50. La capacitación de los empleados en diversas

funciones constituye un ejemplo de: a. Ampliación de los puestos. b. Rotación. c. Enriquecimiento. d. Rediseño.

45. An employee’s need for self-actualization would be met by: a. Attractive pension provisions. b. Challenging new job assignments. c. Good working conditions. d. Regular positive feedback.

46. Which of the following theories includes the

assertion that employees may be motivated by achievement of acceptance or esteem in the workplace? a. Equity theory. b. Expectancy theory. c. Needs hierarchy theory. d. Goal-setting theory.

47. Which of the following actions taken by

management would not be effective in motivating an employee to superior performance? a. Job enlargement. b. Job enrichment. c. Job security. d. Job rotation.

48. Which of the following strengthens and

increases acceptable behavior by termination or withdrawal of undesirable consequences? a. Positive reinforcement. b. Negative reinforcement. c. Reward. d. Punishment.

49. Among the nonfinancial rewards to an

employee, a paid vacation trip can best be categorized as: a. Social reward. b. Token award. c. Visual/auditory reward. d. Manipulatables.

50. Cross-training of employees in various

functions is an example of job: a. Enlargement. b. Rotation. c. Enrichment. d. Redesign.

IV - 13

51. ¿Cuál de los siguientes es un enfoque de administración para la motivación de los empleados?

I. Proveer retroalimentación sobre el

desempeño. II. Presentar oportunidades para la

responsabilidad. III. Satisfacer necesidades personales. a. III solamente. b. I y II solamente. c. II y III solamente. d. I, II y III.

52. ¿Cuál de los siguientes es un ejemplo de

cómo elegir la información que debe presentarse para asegurar que se vea al presentador de la mejor manera? a. Filtro. b. Percepción selective. c. Emoción. d. Elección del idioma.

53. ¿Cuál de las siguientes es una técnica eficaz

para escuchar activamente?

I. Resumen. II. Aclaración. III. Evaluación. IV. Énfasis. a. I y IV solamente. b. II y III solamente. c. I, II y IV solamente. d. I, II, III y IV.

54. ¿Cuál de los siguientes pasos juega en contra

de escuchar en forma eficaz? a. Escuchar la emoción según la situación. b. Formular buenas preguntas. c. Escuchar los pasos por seguir para

encontrar una solución. d. Ayudar al orador a completar el punto que

desarrolla. 55. ¿De qué tipo de comunicación organizacional

son ejemplos las instrucciones de trabajo, los memos oficiales y los manuales de procedimientos? a. Hacia arriba. b. Hacia abajo. c. Lateral. d. Diagonal.

51. Which of the following is a management approach to motivating employees?

I. Providing performance feedback. II. Presenting opportunities for responsibility. III. Satisfying personal needs.

a. III only. b. I and II only. c. II and III only. d. I, II, and III.

52. Which of the following is an example of

choosing which information is to be presented in order to ensure that the presenter is seen in the best possible light? a. Filtering. b. Selective perception. c. Emotion. d. Language choice.

53. Which of the following is an effective active

listening technique?

I. Summarizing. II. Clarifying. III. Evaluating. IV. Empathizing

a. I and IV only. b. II and III only. c. I, II, and IV only. d. I, II, III, and IV.

54. Which of the following steps works against

effective listening? a. Listening for the emotion in the situation. b. Asking good questions. c. Listening to the steps to reach a solution. d. Helping the speaker to complete the

point. 55. Job instructions, official memos, and

procedures manuals are examples of which type of organizational communication? a. Upward. b. Downward. c. Lateral. d. Diagonal.

IV - 14

56. ¿Cuál de las siguientes se considera una desventaja de la comunicación electrónica?

I. Sobrecarga de información. II. Representación malentendida de los

sentimientos y emociones. III. Tiempo de transmisión reducido. IV. Falta de pistas por escrito. a. I y II solamente. b. II y IV solamente. c. I, II y III solamente. d. I, II, III y IV.

57. ¿Cuál de las siguientes es la mejor medida de

productividad que debe utilizarse para evaluar diversos departamentos de una gran tienda minorista? a. Cantidad de clientes que cada empleado

atiende por día. b. Ingresos por cada metro cuadrado. c. Cantidad de unidades que cada

departamento vende por día. d. Cantidad promedio de unidades que cada

departamento almacena por mes. 58. Después de tres años de utilidades que

decrecían considerablemente a pesar del incremento de las ventas y de una economía creciente, ¿cuál de los siguientes es el curso de acción preferido para que tome un CEO? a. Fijar una meta de cambio completo para

obtener utilidades significativamente crecientes en un período de dos meses.

b. Reducir el personal en un 10 por ciento en cada unidad.

c. Reducir el personal en un 20 por ciento en las funciones que no agregan valor.

d. Estimular la innovación en todos los niveles y utilizar un programa de jubilación temprana para reducir la cantidad de personal.

59. ¿Cuál de las siguientes es un ejemplo de una

medida de eficiencia? a. La tasa o índice de ausencias del

personal. b. La meta de convertirse en un fabricante

líder en el ramo. c. El número de reclamaciones a

aseguradoras que se procesan al día. d. La tasa o índice de reclamaciones de

clientes.

56. Which of the following is considered a disadvantage of electronic communication?

I. Information overload. II. Misrepresentation of feelings and

emotions. III. Reduced transmission time. IV. Lack of paper trail.

a. I and II only. b. II and IV only. c. I, II, and III only. d. I, II, III, and IV.

57. Which of the following is the best measure of

productivity to use to evaluate several departments in a large retail store? a. Number of customers served per

employee per day. b. Revenue per square foot. c. Number of units sold per department per

day. d. Average number of units stocked per

month per department. 58. After three years of steadily decreasing profits

in spite of increased sales and a growing economy, which of the following is the preferred course of action for a chief executive officer to take? a. Set a turnaround goal of significantly

increasing profits within two months. b. Reduce staff by 10 percent in every unit. c. Reduce staff in the nonvalue-adding

functions by 20 percent. d. Encourage innovation at all levels and

use an early retirement program to reduce staff size.

59. Which of the following is an example of an

efficiency measure? a. The rate of absenteeism. b. The goal of becoming a leading

manufacturer. c. The number of insurance claims

processed per day. d. The rate of customer complaints.

IV - 15

60. La alineación de las metas gerenciales con las metas organizacionales requiere:

I. Asignar responsabilidades para las

actividades. II. Delegar la autoridad para realizar tareas

necesarias. III. Establecer obligaciones por cumplir. IV. Medir y evaluar el desempeño. a. I y IV solamente. b. I, II y III solamente. c. II, III y IV solamente. d. I, II, III y IV.

61. ¿Cuál de las siguientes afirmaciones es

generalmente verdad con respecto al tramo de control del gerente? a. Es típico encontrar tramos de control en

organizaciones planas, que tienen pocos niveles jerárquicos.

b. Una organización con tramos estrechos de control necesita más gerentes que uno con tramos más amplios de control.

c. Los tramos más amplios de control implican mayores gastos administrativos y menos auto-administración.

d. Las tramos más amplios de control ayudan a asegurar controles internos y el cumplimiento de políticas adecuadas en toda la organización.

62. ¿En cuál de las siguientes situaciones

resultaría apropiada una reducción en el tramo de control? a. Los gerentes no dedican gran cantidad de

tiempo a la planificación ni a la administración estratégica.

b. Los gerentes deben dedicar gran cantidad de tiempo a la coordinación con otros gerentes.

c. Los subordinados trabajan en la misma área, en lugar de encontrarse geográficamente dispersos.

d. El trabajo realizado por los subordinados es casi idéntico.

60. The alignment of managerial goals with organizational goals usually requires:

I. Assigning responsibility for activities. II. Delegating the authority to perform

necessary tasks. III. Establishing accountability. IV. Measuring and evaluating performance.

a. I and IV only. b. I, II, and III only. c. II, III, and IV only. d. I, II, III, and IV.

61. Which of the following is generally true

regarding a manager’s span of control? a. Narrow spans of control are typically

found in flat organizations, which have few hierarchical levels.

b. An organization with narrow spans of control needs more managers than one with wider spans of control.

c. Wider spans of control mean higher administrative expense and less self-management.

d. Wider spans of control help ensure good internal controls and policy compliance throughout an organization.

62. In which of the following situations would a

narrower span of control be more appropriate? a. Managers do not spend a great deal of

time on planning or strategic management.

b. Managers must spend a great deal of time coordinating with other managers.

c. Subordinates work in the same area, rather than being geographically dispersed.

d. Work performed by subordinates is substantially identical.

IV - 16

63. Una evaluación de desempeño de 360 grados suele requerir aportes de:

I. Empleados que son subordinados. II. Empleados de otras unidades de

negocios. III. Pares y compañeros de equipo. IV. Datos sobre la satisfacción de los

clientes. a. I y IV solamente. b. I, II y III solamente. c. II, III y IV solamente. d. I, II, III y IV.

64. Una organización matricial resultaría más

apropiada para: a. Una compañía que opera un conjunto de

call centers telefónicos geográficamente dispersos que proveen soporte técnico.

b. Una compañía que comienza diversos proyectos de ingeniería y construcción complejos y multidisciplinarios cada año.

c. Una compañía minorista que vende a clientes mediante tiendas múltiples ubicadas en paseos de compras, así como a través de sitios Web y catálogos enviados por correo.

d. Una compañía que provee personal temporario a diversos organismos comerciales y gubernamentales.

65. ¿Cuál de las siguientes no es una ventaja de

la descentralización? a. Resulta más fácil tomar decisiones. b. Se incrementa la motivación de los

gerentes. c. Se logra una mayor uniformidad en las

decisiones. d. Los problemas pueden resolverse de

inmediato. 66. En una estructura organizacional en red:

a. Un empleado informa a dos supervisores. b. La autoridad y responsabilidad se

concentran en la posición superior de la organización.

c. La mano de obra es especializada. d. Las funciones principales del negocio se

subcontratan a proveedores externos.

63. A 360-degree performance appraisal typically requires inputs from:

I. Employees who are subordinates. II. Employees in other business units. III. Peers and teammates. IV. Customer satisfaction data.

a. I and IV only. b. I, II, and III only. c. II, III, and IV only. d. I, II, III, and IV.

64. A matrix organization would be most

appropriate for a: a. Company operating a set of

geographically dispersed telephone call centers that provide technical support.

b. Company that starts several complex, multidisciplinary engineering and construction projects each year.

c. Retail company that sells to customers through multiple stores, located in shopping malls, as well as through a Web site and mailed catalogs.

d. Company that provides temporary staffing to a wide variety of commercial and governmental agencies.

65. Which of the following is not an advantage of

decentralization? a. Decisions are more easily made. b. Motivation of managers increases. c. Greater uniformity in decisions is

achieved. d. Problems can be resolved immediately.

66. A network organizational structure is one in

which: a. An employee reports to two supervisors. b. Authority and responsibility are

concentrated at the top of the organization.

c. Labor is specialized. d. Major business functions are

subcontracted to third-party providers.

IV - 17

67. La departamentalización puede ser:

I. Realizada por funciones. II. Realizada por productos. III. Geográfica. a. I solamente. b. II solamente. c. I y II solamente. d. I, II y III.

68. ¿Cuál de los siguientes puede ser un factor

restrictivo asociado con la toma de decisiones en grupo? a. Los grupos generalmente no analizan los

problemas con la suficiente profundidad. b. Es muy difícil lograr que los individuos

acepten las decisiones tomadas por grupos.

c. Los grupos tienen dificultades cuando tratan de identificar los componentes importantes para tomar decisiones.

d. La responsabilidad se disipa cuando las decisiones son tomadas en grupo.

69. ¿En cuál de las siguientes situaciones podría

la política organizacional tener un impacto significativo? a. Cuando las asignaciones de espacio se

hacen de acuerdo con criterios objetivos. b. Cuando el presupuesto permite

generosos aumentos de sueldo para todos los empleados.

c. Cuando las promociones se basan en la actitud de un empleado.

d. Cuando los resultados del desempeño se expresan con claridad y son objetivos.

70. ¿Cuál de las siguientes afirmaciones es

verdad con respecto a las opiniones grupales? a. Hay una tendencia a aceptar la voluntad

de la mayoría y a ignorar el aporte individual que difiere de la opinión grupal.

b. No se requiere que el grupo llegue a un consenso.

c. El alcance de las opiniones grupales es proporcional al tamaño del grupo.

d. Existen demasiadas alternativas generadas para facilitar la toma de decisiones.

67. Departmentalization may be performed by:

I. Function. II. Product. III. Geography.

a. I only. b. II only. c. I and II only. d. I, II, and III.

68. Which of the following can be a limiting factor

associated with group decision making? a. Groups generally do not analyze

problems in enough depth. b. It is very difficult to get individuals to

accept decisions made by groups. c. Groups have a difficult time identifying the

important components of decision making.

d. Accountability is dispersed when groups make decisions.

69. In which of the following situations would

organizational politics most likely have a significant impact? a. When space allocations are made

according to objective criteria. b. When the budget allows for generous

salary increases for all employees. c. When promotions are based on an

employee’s attitude. d. When performance outcomes are clearly

stated and objective. 70. Which of the following is true regarding

groupthink? a. There is a tendency to conform to the

majority’s will and to ignore relevant individual input that is at variance with group opinion.

b. The group is not required to reach consensus.

c. The extent of groupthink is proportional to the size of the group.

d. There are too many alternatives generated to facilitate decision making.

IV - 18

71. Un coordinador para un gran proyecto de capital utilizó una sesión de “brainstorming” o tormenta de ideas de los gerentes de proyecto para decidir cómo volver a encaminar el proyecto. Una desventaja de este enfoque es que: a. La responsabilidad para tomar la decisión

no resultará clara. b. Sólo se abordarán los factores

situacionales. c. Disminuirá la creatividad. d. disminuirá la diversidad de los puntos de

vista. 72. Un miembro de un equipo que pone el

enfoque en la perspectiva global y les recuerda a otros la visión, misión o meta del equipo asume informalmente el rol de: a. Contribuyente. b. Colaborador. c. Comunicador. d. Desafiante.

73. Al implantar una decisión respecto a cambiar

la composición de los equipos de auditoría, la administración encuentra fuerte resistencia al cambio por parte de los miembros del departamento de auditoría. La razón más probable para esta resistencia es: a. Posibles ineficiencias del nuevo

programa. b. La desintegración de los equipos de

auditoría existentes. c. La insuficiencia de personal en las tareas

incluidas. d. La selección de un enfoque de mayor

costo para llevar a cabo la auditoría. 74. Los equipos pueden formarse mediante todos

los siguientes métodos excepto: a. La participación en una serie de desafíos

externos. b. La incorporación de varios roles

interdependientes. c. La clasificación de la eficacia grupal. d. El ejercer presión directa sobre los

disidentes.

71. A project coordinator for a large capital project used a brainstorming session of the senior project managers to decide how to get the project back on schedule. A disadvantage of this approach is that: a. Responsibility for the decision will be

unclear. b. Only situational factors will be addressed. c. Creativity will be decreased. d. Diversity of views will be decreased.

72. A team member who focuses on the overall

perspective and reminds others of the vision, mission, or goal of the team informally assumes the role of a: a. Contributor. b. Collaborator. c. Communicator. d. Challenger.

73. Following a decision to change the

composition of several work teams, management encounters significant resistance to the change from members of the teams. The most likely reason for the resistance is: a. Possible inefficiencies of the new

arrangement. b. The breakup of existing teams. c. Understaffing for the tasks involved. d. The selection of a more costly approach

to performing the assigned tasks. 74. Teams may be built by all the following

methods except: a. Participating in a series of outdoor

challenges. b. Incorporating a number of interdependent

roles. c. Rating group effectiveness. d. Exerting direct pressure on dissenters.

IV - 19

75. Los grupos, cuando se comparan con los individuos, presentan ventajas y desventajas para tomar decisiones. ¿Cuál de las siguientes afirmaciones es verdadera en relación con las decisiones de grupo?

Ventaja Desventaja a. Responsabilidad Los desacuerdos no

(accountability) salen a la superficie personal por las presiones incrementada. para amoldarse.

b. Mayor aceptación Toma más tiempo de una decisión por llegar a una decisión. los participantes.

c. Toma menos tiempo Falta de llegar a una responsabilidad decisión. (accountability) personal.

d. Mayor diversidad Menor aceptación de en cuanto a una decisión por parte especialización. de los participantes

76. ¿Cuál de las siguientes es vital para mantener

el “empowerment” del equipo? a. Proveer estructura a los miembros del

equipo. b. Vigilar el progreso y ofrecer

retroalimentación oportuna sobre el desempeño.

c. Reducir la autoridad del equipo cuando se cometen errores.

d. Evitar la tensión y el conflicto dentro del equipo.

77. ¿Cuál de los siguientes no es un enfoque

apropiado para formar un equipo? a. Asegurar un equilibrio en los roles

complementarios del equipo. b. Elegir miembros sobre la base de su

necesidad de mejorar sus destrezas. c. Desarrollar valores claros y compartidos. d. Seleccionar miembros del equipo sobre la

base de la manera en que pueden relacionarse entre sí.

78. ¿Cuál de los siguientes indica un equipo de

alto desempeño? a. Orgullo por el líder del equipo. b. Rápido acuerdo con la primera solución

propuesta para los problemas que enfrenta el equipo.

c. Cautela al tomar riesgos. d. Compromiso con el crecimiento personal

de los miembros del equipo.

75. When compared to individuals, groups have advantages and disadvantages for decision making. Which of the following is true regarding group decisions?

Advantage Disadvantage a. Increased personal Disagreements do

accountability not surface because of pressures to conform

b. Increased acceptance More time needed to of a decision by arrive at a decision participants

c. Less time needed to Lack of personal arrive at a decision accountability

d. Increased diversity Reduced acceptance of expertise of decision by participants

76. Which of the following is vital to maintaining

team empowerment? a. Provide structure to team members. b. Monitor progress and offer timely

feedback on performance. c. Reduce authority of the team when

mistakes are made. d. Avoid tension and conflict within the team.

77. Which of the following is not an appropriate

approach to team building? a. Ensuring a balance of complementary

team roles. b. Choosing members based on their need

to improve their skills. c. Developing clear and shared values. d. Selecting team members based on how

they are likely to relate to each other. 78. Which of the following indicates a high-

performance team? a. Pride in the team leader. b. Quick agreement on the first proposed

solution for problems facing the team. c. Cautiousness in risk-taking. d. Commitment to personal growth of team

members.

IV - 20

79. ¿Cuál de los siguientes es un indicador de destrezas interpersonales que resultan necesarias para los miembros del equipo?

I. Mantiene a los superiores, miembros de

equipo y otras partes apropiadas rutinariamente informados sobre los desarrollos significativos.

II. Dedica el suficiente tiempo a cultivar contactos con pares para obtener información oportuna o resolver cuestiones fuera de los canales formales.

III. Asume rutinariamente una cantidad apropiada de trabajo o responsabilidad para proyectos grupales.

a. I solamente. b. I y II solamente. c. II y III solamente. d. I, II y III.

80. ¿Cuál de los siguientes escenarios describe a

una organización en desequilibrio al enfocarse demasiado en la eficiencia más que en la eficacia? a. El trabajo no se termina y se desperdician

recursos. b. El trabajo se termina pero se

desperdician recursos. c. El trabajo no se termina pero los recursos

no se desperdician. d. El trabajo se termina y los recursos no se

desperdician. 81. ¿Cuál de los siguientes no es un principio

eficaz para guiar el uso de técnicas de liderazgo por parte de un gerente? a. Servir como modelo de la conducta

esperada de los demás. b. Valorar las responsabilidades. c. Valorar las diferencias. d. Seguir procedimientos escritos en todo

momento. 82. Los miembros de un equipo de producción

trabajaron bien juntos durante varios años. Sin embargo, hace poco tiempo, hubo varias discusiones entre dos de ellos y otros comenzaron a tomar partido, lo cual provocó un efecto negativo en el desempeño de la producción. El mejor estilo de liderazgo para el gerente en esta situación es: a. Directivo. b. De soporte. c. Participativo. d. Orientado a los logros.

79. Which of the following is an indicator of interpersonal skills that are necessary for members of a team?

I. Routinely keeps superiors, team

members, and other appropriate parties informed of significant developments.

II. Spends sufficient time cultivating contacts with peers to obtain timely information or resolve issues outside formal channels.

III. Routinely assumes an appropriate amount of work or responsibility for group projects.

a. I only. b. I and II only. c. II and III only. d. I, II, and III.

80. Which of the following scenarios illustrates an

organization that has become out of balance by focusing too much on efficiency rather than effectiveness? a. The job is not completed and resources

are wasted. b. The job is completed but resources are

wasted. c. The job is not completed but resources

are not wasted. d. The job is completed and resources are

not wasted. 81. Which of the following is not an effective

principle for guiding a manager’s use of leadership techniques? a. Serve as a model of the behaviour

expected from others. b. Value accountability. c. Value differences. d. Follow written procedures at all times.

82. A production team has worked well together

for several years. However, severe arguments have recently occurred between two members of the team, and other members have begun to take sides, causing a negative effect on production performance. The best leadership style for the manager in this situation is: a. Directive. b. Supportive. c. Participative. d. Achievement-oriented.

IV - 21

83. El gerente de un organismo gubernamental supervisa una sección de empleados administrativos que revisan solicitudes de licencias para su aprobación o rechazo. Los trabajos administrativos se encuentran bien definidos en cuanto a su procedimiento y están sujetos a las regulaciones gubernamentales. En esta situación, ¿cuál sería el mejor estilo de liderazgo para el gerente? a. Directivo. b. De soporte. c. Participativo. d. Orientado a los logros.

84. La creencia de que el liderazgo exitoso ocurre

cuando el estilo del líder coincide con la situación es la base para: a. El enfoque de la contingencia hacia el

liderazgo. b. El modelo de cuadrícula gerencial del

liderazgo. c. Un enfoque de conducta hacia el

liderazgo. d. Un enfoque a las teorías del liderazgo

orientado a los logros. 85. Un empleado es técnicamente extraordinario y

trabaja bien con sus clientes, pero no sabe liderar un equipo muy bien. Para mejorar el desempeño del empleado: a. Debe ponérselo a cargo del proyecto más

grande; la única manera de aprender es realizando la tarea.

b. Debe ponérselo a cargo de pequeños proyectos con logros fijos y un personal totalmente capacitado.

c. Debe enviárselo a capacitarse mediante clases de teoría de administración.

d. Debe dejárselo solo y dársele tareas que acentúen las fortalezas personales y eviten las debilidades personales.

86. ¿Cuál de los siguientes rasgos es el más

importante para tener éxito como gerente de proyecto? a. Conocimientos sobre presupuestos y

temas contables. b. Destrezas de administración de personas,

tales como resolución de conflictos y negociaciones.

c. Análisis estadístico y experiencia en diseño de procesos.

d. Herramientas de administración estratégica y capacitación afín.

83. A manager in a government agency supervises a section of clerical employees who review license applications for approval or denial. The clerical jobs are well-defined procedurally and are subject to government regulations. In this situation, what is the best leadership style for the manager? a. Directive. b. Supportive. c. Participative. d. Achievement-oriented.

84. The belief that successful leadership occurs

when the leader’s style matches the situation is the basis for: a. The contingency approach to leadership. b. The managerial-grid model of leadership. c. A behavioral approach to leadership. d. An achievement-oriented approach to

leadership theories. 85. An employee is technically outstanding and

works well with customers, but is not good at leading a team. To improve the employee’s performance, the employee should be: a. Put in charge of the biggest project; the

only way to learn is by performing the task.

b. Put in charge of small projects with set milestones and a fully trained staff.

c. Sent to school for management theory classes.

d. Left alone and given assignments that accentuate personal strengths and avoid personal weaknesses.

86. Which of the following traits is the most

important in order to succeed as a project manager? a. Budgeting and accounting knowledge. b. People management skills, such as

conflict resolution and negotiation. c. Statistical analysis and process design

experience. d. Strategic management tools and training.

IV - 22

87. ¿Qué tipo de estilo de liderazgo tiene un líder que explica las decisiones y da la oportunidad para aclarar cuestiones? a. De venta. b. De expression. c. Participativo. d. De delegación.

88. Un departamento de auditoría interna adopta

una postura mediante la cual provee capacitación a la gerencia acerca de la conciencia sobre el fraude, en la que se incluye una visión global de la política y línea directa del fraude corporativo. Dicha postura para capacitación demuestra que el departamento de auditoría interna está tomando algunos de los siguientes roles de liderazgo. ¿Cuáles? a. “Pathfinding” o de exploración, que pone

el enfoque en “¿Cuál es nuestro propósito y cómo lo lograremos?”

b. Alineación, que pone el enfoque en “¿Cómo alineamos los sistemas y procesos para lograr nuestro propósito?”

c. “Empowering”, que pone el enfoque en “¿Cómo trabajamos con nuestra gente para que tenga la autoridad, la responsabilidad y el compromiso adecuados para ayudarnos mejor a lograr nuestro propósito?”

d. Modelación, que pone el enfoque en “¿Cómo demostramos los valores para convencer a otros que nos sigan y asuman la responsabilidad para lograr nuestro propósito?”

89. ¿Cuáles de las siguientes son condiciones

para una relación de asesoramiento exitosa?

I. La relación debe tener como objetivo mejorar al asesorado.

II. La relación debe basarse en el crecimiento del asesorado.

III. La relación entre el mentor y el asesorado debe ser voluntaria.

IV. La relación de asesoramiento requiere un ambiente de trabajo positivo.

a. I y II solamente. b. III y IV solamente. c. I, II y III solamente. d. I, II, III y IV.

87. A leader who explains decisions and provides opportunity for clarification is described as having which type of leadership style? a. Selling. b. Telling. c. Participating. d. Delegating.

88. An internal audit department adopts a training

posture that provides training to management on fraud awareness, including an overview of the corporate fraud policy and hotline. This training posture best demonstrates that the internal audit department is taking which of the following leadership roles? a. Pathfinding, which focuses on “What is

our purpose and how will we achieve it?” b. Aligning, which focuses on “How do we

align systems and processes to achieve our purpose?”

c. Empowering, which focuses on “How do we cultivate our people to have the right authority, responsibility, and commitment to help us best achieve our purpose?”

d. Modeling, which focuses on “How do we demonstrate the values to convince others to follow us and take responsibility for achieving our purpose?”

89. Which of the following are conditions for a

successful mentoring relationship?

I. The relationship should be aimed at improvement of the mentee.

II. The relationship should be based on growth of the mentee.

III. The pairing of mentor and mentee should be voluntary.

IV. Mentoring requires a positive work environment.

a. I and II only. b. III and IV only. c. I, II, and III only. d. I, II, III, and IV.

IV - 23

90. ¿Cuál de los siguientes es el estilo de liderazgo en el cual el líder y los seguidores toman decisiones sobre la base del consenso? a. Autocrático. b. Autoritario benevolente. c. Consultivo. d. Participativo.

91. ¿Cuál de las siguientes técnicas de resolución

de conflictos tiene la meta de mantener relaciones armoniosas haciendo prevalecer las necesidades de los demás sobre las nuestras? a. Acomodamiento. b. Compromiso. c. Colaboración. d. Evitar.

92. Cuando realiza una negociación exitosa, un

negociador debe: a. Comprender las implicancias para ambas

partes si la negociación falla. b. Concentrarse sólo en las cuestiones de la

negociación. c. No desviarse de las posiciones

estipuladas. d. Depender de la investigación inicial

preparada para la negociación. 93. ¿Cuál es una desventaja primordial de forzar

a otra parte a aceptar las condiciones de una negociación? a. Daña la relación entre los negociadores. b. No se logran las metas del negociador. c. Se necesita más tiempo para llegar a un

acuerdo. d. Se reduce el respaldo interno para las

tácticas del negociador. 94. Cuando se negocia con una personalidad

analítica, el negociador debe: a. Presentar los hechos y precedentes de

una manera organizada. b. Incitar a la otra parte para lograr un cierre

rápido de las negociaciones. c. Poner el enfoque en crear un vínculo con

la otra parte. d. Incluir cuestiones importantes en la

propuesta de negociación.

90. Which of the following is the leadership style in which the leader and the followers make decisions on the basis of consensus? a. Autocratic. b. Benevolent authoritative. c. Consultative. d. Participative.

91. Which of the following conflict resolution

techniques has the goal of maintaining harmonious relationships by placing another’s needs and concerns above one’s own? a. Accommodation. b. Compromise. c. Collaboration. d. Avoidance.

92. When performing a successful negotiation, a

negotiator should: a. Understand the implications for both sides

if the negotiation fails. b. Concentrate solely on the issues in the

negotiation. c. Not deviate from stated positions. d. Depend on the initial research prepared

for the negotiation. 93. What is a primary disadvantage of forcing

another party to accept terms in a negotiation? a. Damage of the relationship between the

negotiators. b. Lack of achievement of the negotiator’s

goals. c. Increased time involved in reaching an

agreement. d. Reduction in internal support for the

negotiator’s tactics. 94. When negotiating with an analytical

personality, the negotiator should: a. Present facts and precedents in an

organized manner. b. Push the other party for quick closure of

negotiations. c. Focus on creating a bond with the other

party. d. Include unimportant items in the proposal

for bargaining.

IV - 24

95. Mientras se negocia un contrato con el representante de un proveedor, un gerente se enfrenta a una resistencia inesperada. En primer lugar, el gerente debe: a. Intentar determinar la razón de la

resistencia. b. Detener la reunión y abordar las

inquietudes del representante en privado. c. Volver a estipular la posición del gerente

con respecto a la cuestión. d. Determinar los puntos de vista y

requerimientos del representante investigando al proveedor.

96. El surgimiento de un conflicto como resultado

de que un gerente de ventas prometió una entrega a sus clientes que resulta incompatible con los bajos niveles de inventario mantenidos por el gerente de producción constituye un ejemplo de uno de los siguientes tipos de conflicto interpersonal: a. Diferencias personales. b. Deficiencia en la información. c. Incompatibilidad de los roles. d. Estrés ambiental.

97. ¿Cuál de las siguientes afirmaciones con

respecto a los enfoques hacia la resolución de conflictos es correcta? a. Forzar es un estilo de administración de

conflictos en el cual se da mayor importancia a la relación que a las metas individuales, y las metas se abandonan para preservar las relaciones.

b. Evitar, no hacer nada y retirarse es un estilo de administrar conflictos mediante el cual se renuncia a las metas personales y las relaciones. Se considera una solución temporaria porque el problema y el conflicto continúan reapareciendo.

c. El estilo “soothing” ocurre cuando las metas son sumamente importantes mientras la relación es de menor importancia. Las metas se logran a cualquier costo.

d. El compromiso valora las metas y relaciones en gran medida. Los conflictos se ven como problemas por resolverse, y los negociadores buscan una solución que alcanza las metas así como mejora las relaciones.

95. While negotiating a contract with a supplier’s representative, a manager encounters unexpected resistance. The manager should first: a. Attempt to determine the reason behind

the resistance. b. Stop the meeting and address the

representative’s concerns privately. c. Restate the manager’s position regarding

the issue. d. Determine the representative’s views and

requirements by researching the supplier. 96. Conflict arising as a result of a sales manager

making delivery promises to customers that are incompatible with the low inventory levels maintained by the production manager is an example of which of the following types of interpersonal conflict? a. Personal differences. b. Information deficiency. c. Role incompatibility. d. Environmental stress.

97. Which of the following statements regarding

approaches to conflict resolution is correct? a. Forcing is a style of managing conflict

where the relationship is given more importance than individual goals, and goals are conceded in order to preserve relationships.

b. Withdrawing is a style of managing conflict where personal goals and relationships are relinquished. It is regarded as a temporary solution because the problem and conflict continue to reoccur.

c. Smoothing occurs when goals are highly important while the relationship is of minor importance. Goals are achieved at any cost.

d. Compromising highly values goals and relationships. Conflicts are viewed as problems to be solved, and negotiators seek a solution that both achieves goals and improves relationships.

IV - 25

98. Evitar que un conflicto aflore es un ejemplo de seguir una de las siguientes estrategias de administración de conflictos: a. Evitar. b. Desactivación. c. Contención. d. Confrontación.

99. La negociación de valor agregado se

caracteriza por: a. Una parte que se acerca a otra con una

propuesta. b. Una serie de ofertas y contraofertas entre

las partes negociadoras. c. Una parte que se acerca a otra con

diversas propuestas. d. Una parte que rápidamente acepta las

demandas de la otra. 100. ¿En cuál de los siguientes principios se basa

el método de la negociación con principios?

I. Separar a las personas del problema. II. Poner énfasis en las posiciones, no en los

intereses. III. Inventar opciones para beneficio mutuo. IV. Insistir en el uso de criterios subjetivos. a. I y II solamente. b. I y III solamente. c. I, II y III solamente. d. II, III y IV solamente.

FIN DE LAS PREGUNTAS PARTE IV POR FAVOR OBSERVE: La parte IV del examen CIA actual constará de 125 preguntas de examen. Entre las 125 preguntas, se incluirán hasta 25 preguntas no registradas, que se utilizarán para propósitos de investigación. Dichas preguntas no registradas se insertarán sin identificar entre las registradas. Por lo tanto, los candidatos deben responder a las 125 preguntas lo mejor posible.

98. Keeping a conflict from surfacing at all is an example of following which conflict management strategy? a. Avoidance. b. Defusion. c. Containment. d. Confrontation.

99. Added-value negotiation is characterized by:

a. One party approaching another with a proposal.

b. A series of offers and counteroffers between the negotiating parties.

c. One party approaching another with several proposals.

d. One party quickly conceding to the demands of the other.

100. The method of principled negotiation is based

on which of the following principles?

I. Separate the people from the problem. II. Focus on positions, not interests. III. Invent options for mutual gain. IV. Insist on using subjective criteria.

a. I and II only. b. I and III only. c. I, II, and III only. d. II, III, and IV only.

END OF PART IV QUESTIONS PLEASE NOTE: The actual CIA exam Part IV will contain 125 exam questions. The 125 questions will include up to 25 unscored questions, which will be used for research purposes. These unscored questions will be interspersed with the scored questions and will not be identified as unscored questions. Candidates should therefore answer all 125 questions to the best of their ability.

IV - 26

Soluciones a la Parte IV - Destrezas de la

administración del negocio

Las soluciones y explicaciones sugeridas para la parte I de las preguntas del Examen modelo para Auditor Interno Certificado se proporcionan en las siguientes páginas.

La tabla que a continuación se presenta asocia los números de las preguntas de la parte I con los temas

evaluados:

Tema Evaluado Número de Pregunta

Administración estratégica 1 – 28

Entornos globales del negocio 29 – 43

Conducta organizacional 44 – 67

Destrezas administrativas 68 – 90

Negociaciones 91 – 100

IV - 27

1. Solución: c a. Incorrecta. Despistar a otros competidores para que tomen o no una acción a fin de beneficiar a

la empresa es un engaño. El engaño es una forma de señal del mercado que no intenta llevarse a cabo.

b. Incorrecta. La señal del mercado dada por un competidor que provee una indicación directa o indirecta de sus intenciones, motivos, metas o situación interna es un medio indirecto de comunicarse en el mercado y un aporte esencial en el análisis de los competidores.

c. Correcta. El inicio de un movimiento en el mercado donde el competidor es fuerte es un “cross- parry”. El “cross-parry” es una manera eficaz de mostrar desagrado; además, hace que se cierna la amenaza de una represalia más seria.

d. Incorrecta. Los movimientos agresivos directos se realizan para reducir el desempeño de competidores significativos o amenazar a sus metas; además, es probable que se provoque un contraataque.

2. Solución: d

a. Incorrecta. El rol del punto estratégico mediante supervisión directa es limitado, debido a destrezas estandarizadas en una burocracia profesional.

b. Incorrecta. El rol de la tecnoestructura de establecer reglas y procedimientos es limitado, debido a destrezas estandarizadas en una burocracia profesional.

c. Incorrecta. El rol de administración de la gerencia de línea es limitado, debido a destrezas estandarizadas en una burocracia profesional.

d. Correcta. El núcleo operativo es la clave en una burocracia profesional por el énfasis en las destrezas estandarizadas. La burocracia profesional involucra a personas con conjuntos de destrezas estandarizadas y les permite funcionar de una manera no supervisada.

3. Solución: b

a. Incorrecta. El liderazgo en costos es el productor de costos más bajos de la industria como un todo.

b. Correcta. Una estrategia de enfoque de costos se propone ser líder en costos para un segmento del mercado en particular.

c. Incorrecta. La diferenciación de costos se propone proveer un producto a diferentes costos en diferentes segmentos del mercado.

d. Incorrecta. La contención de costos se propone controlar los costos relacionados con un producto o mercado en particular.

4. Solución: a

a. Correcta. El impacto de las cinco fuerzas de Porter determina la intensidad de la competencia y las posibles utilidades de la industria, en la cual las utilidades se miden en términos del rendimiento del capital invertido a largo plazo.

b. Incorrecta. Las barreras de ingreso constituyen una de las cinco fuerzas que deben medirse para definir la intensidad de la competencia y las posibles utilidades.

c. Incorrecta. La rivalidad es una de las cinco fuerzas que deben medirse para definir la intensidad de la competencia y las posibles utilidades.

d. Incorrecta. El análisis de las cinco fuerzas es sólo una etapa en la definición de la estrategia. 5. Solución: b

a. Incorrecta. Es una definición orientada al producto. b. Correcta. Es una definición orientada al mercado en contraposición con la definición orientada

al producto: “vendemos combustible”. c. Incorrecta. Es una definición orientada al producto. d. Incorrecta. Es una definición orientada al producto.

IV - 28

6. Solución: b a. Incorrecta. La transferencia de tecnología constituye la sinergia de tecnología. b. Correcta. Un paquete de productos, la distribución mediante los mismos canales o el uso de la

misma fuerza de ventas constituyen ejemplos de sinergias del mercado. c. Incorrecta. La producción de múltiples productos en una instalación de producción es un

ejemplo de sinergia de costos. d. Incorrecta. El uso de las destrezas complementarias de administración es un ejemplo de

sinergia en la gerencia. 7. Solución: c (I, II y III solamente)

I, II, III. Correcta. Estas estrategias desafiarán a la organización a superarse y mejorar y, además, premiarán a los empleados por utilizar la innovación.

IV. Incorrecta. Poner el enfoque sólo en los resultados del desempeño desalentará a los empleados en cuanto a tomar riesgos y, de esta manera, se obstaculizará la innovación.

8. Solución: b (I y II solamente)

I, II. Correcta. Son dos de las cinco fuerzas básicas que impulsan la competencia de la industria. Las otras son: el poder negociador de los compradores, la amenaza de productos sustitutos y la rivalidad entre competidores actuales.

III. Incorrecta. El desorden en la mano de obra y la escasez de materiales son factores que afectan al acceso a las materias primas y, en consecuencia, pueden afectar a la competencia y las utilidades, pero no se encuentran entre las cinco fuerzas básicas que impulsan la competencia.

IV. Incorrecta. Es una de las seis fuentes principales de las barreras de ingreso. Las otras son las economías de escala, los requerimientos de capital, los costos de reasignación de recursos, el acceso a canales de distribución y las políticas gubernamentales.

9. Solución: c

a. Incorrecta. La consolidación es característica de un ambiente maduro de la industria. b. Incorrecta. La interdependencia competitiva es característica de un ambiente maduro de la

industria. c. Correcta. La demanda en declinación es característica de las industrias decrecientes. d. Incorrecta. El enfoque estratégico sobre el impedimento del ingreso de nuevos competidores

en el mercado es característico de un ambiente maduro de la industria. 10. Solución: c

a. Incorrecta. Ver respuesta “c”. b. Incorrecta. Ver respuesta “c”. c. Correcta. Las estrategias tales como cadena de locales, concesión de franquicias y fusiones

horizontales se utilizan comúnmente en industrias fragmentadas, porque existen pocas barreras de ingreso. Las compañías de las industrias fragmentadas se enfrentan a muchas oportunidades de diferenciación, pero cada oportunidad de ventaja competitiva es pequeña.

d. Incorrecta. Ver respuesta “c”. 11. Solución: d (I, II, III y IV)

I, II, III, IV. Correcta. Entre los costos que la gerencia debe considerar se incluyen los costos de reentrenamiento de empleados; los costos de adquirir nuevo equipamiento auxiliar, las anulaciones debido a inversiones en vieja tecnología no amortizadas, los costos de requerimientos de capital y de investigación y desarrollo para el cambio y los costos de modificar etapas interrelacionadas de producción o aspectos afines de la empresa.

IV - 29

12. Solución: b a. Incorrecta. En general, los costos fijos bajos implican barreras débiles para ingresar y una

consiguiente habilidad por parte de los competidores locales para desafiar a una empresa global de mayor envergadura.

b. Correcta. Hasta el punto en que la producción de cada unidad sea más económica que la última, dicha situación favorece a los productores concentrados grandes en una escala global. (El ejemplo arquetípico es la refinación del petróleo.)

c. Incorrecta. La protección débil para los derechos de autor o la imposición de los derechos de propiedad intelectual posibilitarían que los competidores locales pequeños produzcan en forma eficiente, aunque ilícitamente, en el corto plazo.

d. Incorrecta. Hasta el punto en que un producto requiera un servicio local, esta situación diluye la ventaja de ser un competidor global eficiente y de gran envergadura.

13. Solución: c (I, II y III solamente)

I, II, III. Correcta. Normalmente, el gobierno restringe el comercio para fomentar la seguridad nacional, desarrollar nuevas industrias y proteger industrias decrecientes.

IV. Incorrecta. El incremento de los ingresos tributarios no sería un ímpetu para los gobiernos a fin de restringir el comercio, porque los ingresos tributarios no disminuirían con el comercio reducido.

14. Solución: a

a. Correcta. Es una tendencia social. b. Incorrecta. Es una tendencia política en la cual participa el gobierno. c. Incorrecta. La inflación se encuentra conectada con la economía; de allí que sea una tendencia

económica. d. Incorrecta. Representa una tendencia tecnológica.

15. Solución: a

a. Correcta. Los subsidios para nuevos inversores debilitan las barreras de ingreso de la industria, permitiendo a dichos inversores ingresar en la industria y producir una rivalidad mayor entre más competidores.

b. Incorrecta. La diferenciación de productos se considera una barrera de ingreso que estimula a los posibles nuevos inversores a ingresar en la industria (porque son incapaces de ofrecer un producto comparable) mientras protegen las utilidades de la industria.

c. Incorrecta. El período de aprendizaje de la industria es un activo que los nuevos inversores deben adquirir. En algunos casos, este costo se torna extremadamente alto y puede desalentar a los nuevos inversores en cuanto a ingresar en la industria.

d. Incorrecta. Los costos fijos bajos provocan que los inversores tengan una salida fácil cuando deciden dejar la industria, pero los costos de salida no estimularían en particular el ingreso de nuevos inversores.

16. Solución: b

a. Incorrecta. No hay necesidad de una integración hacia atrás si los proveedores actuales de la empresa son altamente confiables.

b. Correcta. La integración hacia atrás resulta apropiada cuando los proveedores actuales de la empresa no son confiables.

c. Incorrecta. La integración hacia atrás es una forma de integración vertical que involucra la compra de proveedores para reducir la dependencia. La ubicación de los proveedores no es una cuestión.

d. Incorrecta. Ver respuesta “c”.

IV - 30

17. Solución: b a. Incorrecta. La producción “justo a tiempo” (JIT, por sus siglas en inglés) puede lograr que se

dependa más de los proveedores. b. Correcta. La producción JIT puede reducir el costo de implementar nuevas estrategias. c. Incorrecta. La producción JIT incrementará la flexibilidad de las instalaciones de producción. d. Incorrecta. La producción JIT no afectaría a la necesidad de una fuerza laboral confiable.

18. Solución: c

a. Incorrecta. Desarrollar productos implica agregar más valor o características al producto existente. La oportunidad de conceder franquicias se basa en la estandarización de productos.

b. Incorrecta. Diferenciar productos implica que los productos deben ser diferentes en diferentes mercados. En general, no constituye la base para conceder franquicias.

c. Correcta. Estandarizar productos significa mantener el mismo producto o estandarizar la producción, las operaciones y las instalaciones en diferentes lugares o mercados. Esta estandarización provee la oportunidad de conceder franquicias.

d. Incorrecta. Diversificar productos significa abordar productos diferentes, aunque se relacionen o no. En general, la concesión de franquicias se basa en la estandarización de productos, más que en su diferenciación.

19. Solución: a

a. Correcta. La penetración en el mercado es el crecimiento de productos existentes y/o el desarrollo de mercados existentes.

b. Incorrecta. El desarrollo del mercado busca nuevos mercados para los productos actuales. c. Incorrecta. El desarrollo de productos implica el lanzamiento de nuevos productos a mercados

existentes. d. Incorrecta. La diversificación es el lanzamiento de nuevos productos para nuevos mercados.

20. Solución: b

a. Incorrecta. La integración horizontal puede describirse como el agregado de nuevos productos a mercados existentes o de nuevos mercados a productos existentes.

b. Correcta. La integración vertical ocurre cuando una compañía pasa a ser su propio proveedor o distribuidor.

c. Incorrecta. La diversificación concéntrica ocurre cuando una compañía agrega nuevos productos que tienen sinergias tecnológicas con los productos existentes.

d. Incorrecta. La diversificación conglomerada implica hacer nuevos productos para una totalmente nueva clase de clientes.

21. Solución: d (I, II y IV solamente)

I, II, IV. Correcta. Una empresa puede adoptar una estrategia de construcción para este tipo de unidad de negocio estratégica (SBU, por sus siglas en inglés) si la SBU muestra un fuerte potencial de crecimiento, si la empresa desea renunciar a las ganancias a corto plazo y al flujo de fondos y si desea y tiene la capacidad de financiar su crecimiento.

III. Incorrecta. Una empresa que espera un incremento a corto plazo en el flujo de fondos puede adoptar una estrategia de abandono o de cosecha, pero no una estrategia de construcción porque este tipo de SBU necesita gran cantidad de flujo de fondos para financiar su crecimiento.

22. Solución: d

a. Incorrecta. Los objetivos son vitales para fijar metas y establecer un sentido de dirección, sin el cual se pierde tiempo y se incurre en un costo innecesario.

b. Incorrecta. La fijación de objetivos suele involucrar la negociación. c. Incorrecta. Los objetivos son declaraciones más precisas de dirección. d. Correcta. Los objetivos necesitan estar continuamente actualizados para reflejar los cambios

ambientales.

IV - 31

23. Solución: d a. Incorrecta. Un negocio “vaca” genera más fondos de los que requiere, proveyendo fondos a la

corporación para invertir en otras operaciones. b. Incorrecta. Los negocios “interrogante” merecen atención para determinar si la operación es

viable o no. La empresa se encuentra ubicada en una industria creciente, pero no logró una fuerte posición competitiva.

c. Incorrecta. Los negocios “perro” describen una empresa situada en una industria decreciente o de bajo crecimiento y, si no se encuentra fuertemente posicionada, sufrirá el abandono.

d. Correcta. Los negocios “estrella” describen una empresa con una fuerte posición competitiva en la industria. La industria es sólida y las SBUs son sumamente atractivas. El flujo de fondos neto es modesto, ya que la inversión es considerable aunque los negocios “estrella” generen gran cantidad de fondos.

24. Solución: b

a. Incorrecta. El decrecimiento en los niveles de inventario no es el primer síntoma de la etapa de declinación ya que ocurrirá sólo cuando la necesidad de producción decrezca como resultado del decrecimiento en las ventas. Asimismo, un decrecimiento en los niveles de inventario podría ser el resultado de un cambio en la producción “justo a tiempo”.

b. Correcta. Las ventas de la mayoría de las formas y marcas de los productos finalmente caen. El decrecimiento puede ser lento o rápido. Es el primer síntoma de la etapa de declinación.

c. Incorrecta. El decrecimiento en la producción puede ser debido a razones tecnológicas de planta o a la no disponibilidad de materias primas en cualquier etapa del ciclo de vida de un producto.

d. Incorrecta. La alteración en los precios es una decisión de marketing. Esta acción puede tomarse en la etapa de madurez para competir en el mercado. No es un síntoma de la etapa de declinación.

25. Solución: b

a. Incorrecta. Éste es el caso durante la etapa de introducción del ciclo de vida de un producto. b. Correcta. Durante la etapa de crecimiento del ciclo de vida de un producto, su confiabilidad se

torna más importante. c. Incorrecta. Éste es el caso durante la etapa de madurez del ciclo de vida de un producto. d. Incorrecta. Éste es el caso durante la etapa de declinación del ciclo de vida de un producto.

26. Solución: a

a. Correcta. En la etapa de introducción, debe emprenderse la promoción costosa de ventas para educar al consumidor y desarrollar la aceptación del mercado.

b. Incorrecta. En la etapa de introducción, los competidores no tienden a ingresar en el mercado de los productos.

c. Incorrecta. En la etapa de introducción, la producción excesiva es improbable y no afectaría al crecimiento de las ventas.

d. Incorrecta. En la etapa de introducción, no existen muchas alternativas disponibles. 27. Solución: c

a. Incorrecta. Durante la etapa de introducción, hay poca competencia o nada; en consecuencia, los precios son los más altos. Asimismo, los costos son altos en la etapa de introducción.

b. Incorrecta. Durante la etapa de crecimiento, los precios serán más bajos que durante la de introducción, pero no tan bajos como durante la de madurez. Los costos están bajando y se están agregando competidores, pero los costos no son los mínimos y hay la cantidad máxima de competidores posible.

c. Correcta. Durante la etapa de madurez, la competencia es la mayor posible y los costos son los más bajos; por lo tanto, los precios serían los más bajos.

d. Incorrecta. Durante la etapa de declinación, hay pocos competidores; en consecuencia, los precios pueden aumentar. Más aún, los costos aumentan porque el volumen está decreciendo.

IV - 32

28. Solución: b a. Incorrecta. Los costos son altos durante la etapa de introducción y el volumen es bajo. b. Correcta. Durante la etapa de crecimiento, la oportunidad de reducciones de costos está en su

punto máximo porque el volumen se incrementa en una alta proporción; por lo tanto, son más unidades de producción las que tienen costos fijos y se están concretando los beneficios de la curva de aprendizaje.

c. Incorrecta. Como el volumen no está cambiando mucho durante la etapa de madurez, existen menos oportunidades de reducciones de costos.

d. Incorrecta. Es típico que los costos por unidad aumenten durante la etapa de declinación porque el volumen decrece. Asimismo, los incrementos de los costos pueden pasarse a los clientes con mayor facilidad porque existen pocos competidores.

29. Solución: b

a. Incorrecta. Como un precio de ventas incrementado aumentaría las utilidades de la corporación, los ingresos tributarios perdidos mediante la eliminación de convenios para controlar precios y normas serviría como elemento disuasivo de los esfuerzos por eliminar convenios para controlar precios y normas del gobierno.

b. Correcta. Los gobiernos y organizaciones internacionales buscan proteger a los consumidores y la salud de la economía local y global a través de los esfuerzos por eliminar convenios para controlar precios y normas.

c. Incorrecta. Si bien el efecto de las actividades de eliminación de convenios para controlar precios y normas puede dañar pequeñas empresas, el mayor impacto puede producirse sobre la economía global.

d. Incorrecta. Si bien un mercado libre puede contribuir a la estabilidad de desarrollar economías de países, éste no provee una razón precisa para que los esfuerzos locales por eliminar convenios controlen precios y normas en los países industrializados.

30. Solución: b

a. Incorrecta. Es la definición de mercados grises. b. Correcta. Es la definición de “dumping”. c. Incorrecta. Ver respuesta “b”. d. Incorrecta. Ver respuesta “b”.

31. Solución: a

a. Correcta. La supremacía personal o el autocontrol es una dimensión personal más que una cultural.

b. Incorrecta. Una distancia del poder es una dimensión cultural referida a la medida en la que se distribuye el poder irregularmente entre los individuos.

c. Incorrecta. La masculinidad versus la feminidad es una dimensión cultural que se refiere al predominio de la agresividad y adquisición de las cosas (calificado como masculino) versus la preocupación por las personas, los sentimientos y la calidad de vida (calificada como femenina).

d. Incorrecta. La medida en la que alguien se sienta amenazado por situaciones ambiguas y evite el conflicto es una dimensión cultural que se conoce como evitar la inseguridad.

32. Solución: b (I y III solamente)

I. Correcta. Mediante la estimulación de grupos para identificar contribuciones modernas o inusuales, la “brainstorming” o tormenta de ideas ayuda a identificar todos los elementos de un problema y a hacer que su resolución resulte visible.

II. Incorrecta. Mediante la identificación de puntos de vista sin análisis adicional, las encuestas no incrementan el entendimiento de una situación compleja.

III. Correcta. El pensamiento lateral explora diferentes maneras de mirar una cuestión, incrementando, de esta manera, la probabilidad de hallar mejores soluciones.

IV - 33

33. Solución: c a. Incorrecta. El tiempo de respuesta no es una cuestión significativa. b. Incorrecta. Las cuestiones tributarias no son significativas. c. Correcta. Las leyes de privacidad específicas del país pueden resultar muy rigurosas sobre los

límites de los datos del cliente. d. Incorrecta. Los archivos de respaldo y la recuperación no constituyen impedimentos

significativos. 34. Solución: c

a. Incorrecta. Son fuerzas propulsoras de la localización, no de la globalización. b. Incorrecta. Son los tres beneficios de la localización. c. Correcta. Estos factores favorecen la globalización reduciendo las barreras comerciales y el

costo de coordinación, incrementando las economías de escala y estimulando la estandarización y las marcas globales.

d. Incorrecta. Son los factores determinantes de una estrategia de negocio global. 35. Solución: b

a. Incorrecta. El etnocentrismo se caracteriza por una orientación local. La compañía pone el enfoque en su mercado local y considera secundaria la exportación.

b. Correcta. La compañía se adapta al ambiente local mediante una orientación policéntrica. Entre los ejemplos se incluyen diversas subsidiarias de una corporación multinacional.

c. Incorrecta. El geocentrismo se basa en la presunción de que las similitudes del mundo pueden incorporarse para formar una estrategia global.

d. Incorrecta. El regiocentrismo se basa en la presunción de que las similitudes de la región pueden incorporarse en los objetivos o estrategias del negocio.

36. Solución: a

a. Correcta. Los beneficios de costos se obtienen de economías de escala debido a la estandarización de productos y/o procesos, así como al incrementado poder negociador sobre los proveedores de materias primas, componentes y servicios.

b. Incorrecta. Los beneficios de oportunidad se obtienen debido a los enfoques coordinados en el lanzamiento de productos en las primeras etapas del ciclo de vida de un producto.

c. Incorrecta. Los beneficios de aprendizaje surgen de la transferencia coordinada de información, las mejores prácticas y el personal de distintas subsidiarias.

d. Incorrecta. Los beneficios de arbitraje surgen de las ventajas que una compañía global puede obtener utilizando recursos de un país para el beneficio de una subsidiaria de otro país.

37. Solución: a

a. Correcta. Las actitudes, los gustos, la conducta y los códigos sociales constituyen componentes de los factores culturales. En consecuencia, se trata de un caso de factor cultural.

b. Incorrecta. La distribución, customización y respuesta comprenden los factores comerciales. c. Incorrecta. Las normas, la presencia espacial, el transporte y los lenguajes constituyen los

factores técnicos. d. Incorrecta. Las cuestiones de regulación y seguridad nacional constituyen los factores legales.

38. Solución: b

a. Incorrecta. Declarar menos utilidades en países con niveles de impuestos más bajos no hará que se alcance el objetivo tributario.

b. Correcta. Declarar más utilidades en países con niveles de impuestos más bajos hará que se reduzcan las obligaciones tributarias.

c. Incorrecta. Imputar precios más altos hará que se reduzcan tanto los ingresos como las obligaciones tributarias, lo cual no es una situación óptima.

d. Incorrecta. Los precios más bajos provocarán ganancias más altas y mayores obligaciones tributarias.

IV - 34

39. Solución: c a. Incorrecta. Los problemas de percepción se originan en diferencias en la interpretación de la

realidad por parte de cada persona. b. Incorrecta. Los estereotipos constituyen una tendencia de percibir que otra persona pertenece

a una clase o categoría única o particular. c. Correcta. El etnocentrismo lleva a las personas a creer que son las mejores en todo, cualquiera

sea el tema de conversación. Este espíritu se promueve a través de las estructuras de valor y del espíritu nacionalista de las personas.

d. Incorrecta. El evitar la inseguridad se refiere a la medida en la que las personas se sienten amenazadas por situaciones ambiguas y en la que tratan de evitar estas situaciones.

40. Solución: b

a. Incorrecta. El riesgo económico es la probabilidad de que la mala administración económica provoque cambios en el ambiente del negocio del país que perjudicará a las utilidades y a otras metas de la compañía.

b. Correcta. El riesgo político es la probabilidad de que las fuerzas políticas provoquen cambios en el ambiente del negocio del país que perjudiquen a las utilidades y a otras metas de la compañía. El nacionalismo, la expropiación y el terrorismo son todos ejemplos de riesgo político.

c. Incorrecta. El riesgo operativo es la inseguridad de acontecimientos no financieros que pueden provocar el fracaso de la organización y las pérdidas financieras relacionadas.

d. Incorrecta. El riesgo ambiental es la falta de certeza y la severidad del impacto de los posibles peligros ambientales.

41. Solución: d (II, III, IV y V solamente)

I. Incorrecta. Las dificultades de comunicación debido a largas distancias que separan a emisores y receptores se minimizaron mediante la comunicación electrónica tal como el correo electrónico, el fax y la teleconferencia.

II. Correcta. El lenguaje corporal y otras formas de comunicación no verbal pueden tener diferentes significados en diferentes culturas.

III. Correcta. El idioma suele ser una causa de mala interpretación porque las partes pueden no tener dominio del idioma.

IV. Correcta. Las actitudes, tales como los estereotipos, pueden provocar malentendidos. V. Correcta. El tiempo es una barrera para la comunicación internacional porque diferentes

culturas consideran y utilizan el tiempo de manera diferente. 42. Solución: c

a. Incorrecta. Cuando dominan las fuerzas globales, se ignoran la adaptación y la respuesta local. b. Incorrecta. Cuando dominan las fuerzas locales, se ignoran la eficiencia, la velocidad, el

arbitraje y el aprendizaje. c. Correcta. La competitividad no puede lograrse sin lograr los beneficios de la integración y

coordinación globales así como los beneficios de la localización, entre los cuales se incluyen la flexibilidad, la proximidad y el tiempo de respuesta rápida.

d. Incorrecta. Cuando ninguna de las fuerzas domina, no puede obtenerse ningún posicionamiento competitivo.

43. Solución: b

a. Incorrecta. Es una desventaja clave de una política geocéntrica de dotación de personal, no de una política etnocéntrica. Si bien se requieren costos de recolocación y mayor compensación para gerentes exilados de la estrategia etnocéntrica, permite una estructura de compensación global a fin de seguir los niveles nacionales en cada país, reduciendo los costos globales de dotación de personal.

b. Correcta. Es la desventaja clave de una política etnocéntrica de dotación de personal. c. Incorrecta. Una estrategia de política etnocéntrica de dotación de personal limita el avance

profesional de los empleados de países locales, no de empresas matrices. d. Incorrecta. Es una desventaja de una política policéntrica de dotación de personal.

IV - 35

44. Solución: d a. Incorrecta. Los equipos de trabajo autónomos sólo constituyen un tipo de enriquecimiento

laboral y no son aplicables en muchas situaciones. b. Incorrecta. Mientras la ampliación de los puestos para agregar diferentes tareas pueden ser un

factor alentador, la incorporación de más tareas similares no rediseña tareas fundamentalmente y puede resultar desalentadora si las tareas existentes son difíciles o aburridas.

c. Incorrecta. La rotación no rediseña tareas fundamentalmente. d. Correcta. Uno de los principios fundamentales del diseño y rediseño de tareas es tomar en

cuenta las necesidades y destrezas de los empleados y combinarlas con dichas tareas, o cambiarlas para adaptarse a esas necesidades y destrezas.

45. Solución: b

a. Incorrecta. El suministro de pensiones y jubilaciones atractivas satisfaría las necesidades fisiológicas de un empleado.

b. Correcta. Las asignaciones de nuevas tareas desafiantes satisfarían las necesidades de auto-realización de un empleado.

c. Incorrecta. Las buenas condiciones laborales satisfarían las necesidades fisiológicas de un empleado.

d. Incorrecta. Las respuestas positivas regulares satisfarían las necesidades de estima de un empleado.

46. Solución: c

a. Incorrecta. La teoría de la equidad pone el enfoque en el equilibrio entre aportes y resultados. b. Incorrecta. La base de la teoría de la expectativa es que los empleados sólo se motivan para

proveer aportes si creen que se obtendrá un nivel dado de resultados. c. Correcta. La teoría de la necesidad sugiere que los empleados tienen necesidades para cuya

satisfacción se encuentran motivados en el lugar de trabajo. d. Incorrecta. La teoría de fijación de metas se basa en la creencia de que los empleados tienen

deseos y aspiraciones, que dan como resultado metas que dirigen su conducta. 47. Solución: c

a. Incorrecta. La ampliación de los puestos es una extensión horizontal del trabajo, a fin de abarcar una mayor parte del trabajo total de producción.

b. Incorrecta. El enriquecimiento del trabajo es una extensión vertical del trabajo, para crear mayor responsabilidad, el trabajo mismo y el desafío en él.

c. Correcta. La ausencia de seguridad en el trabajo llevaría a la insatisfacción, pero su presencia no llevaría a lograr un desempeño superior o a la satisfacción en el trabajo.

d. Incorrecta. La rotación en el trabajo provocaría una mayor satisfacción porque el empleado tiene una mayor comprensión del proceso laboral.

48. Solución: b

a. Incorrecta. El refuerzo positivo fortalece e incrementa la conducta mediante la presentación de una consecuencia conveniente.

b. Correcta. El refuerzo negativo fortalece la respuesta e incrementa la probabilidad de que vuelvan a evitarse consecuencias no deseadas de cualquier acción.

c. Incorrecta. Un premio es simplemente algo que considera conveniente la persona que lo presenta.

d. Incorrecta. El refuerzo negativo fortalece e incrementa la conducta mientras que el castigo debilita y hace que decrezca la conducta.

IV - 36

49. Solución: b a. Incorrecta. Los premios sociales normalmente incluyen el reconocimiento del logro del

empleado mediante acciones tales como solicitar consejos. b. Correcta. Son normalmente premios no recurrentes que muestran la valoración del rol del

empleado; otros ejemplos similares son los cupones de obsequio, opciones de compra de acciones, horas libres pagas o tickets para almorzar y entradas de teatro.

c. Incorrecta. Entre los ejemplos de premios visuales o auditivos se incluyen una oficina privada, debates en el club del libro o la redecoración del ambiente laboral.

d. Incorrecta. Entre los ejemplos de manipulables se encuentran obsequios, tales como accesorios de escritorio, relojes, trofeos, ropa o anillos.

50. Solución: b

a. Incorrecta. La ampliación de los puestos implica extender un trabajo en forma horizontal. b. Correcta. Cuando el trabajo se torna demasiado rutinario, dicha preparación evita que el

personal se aburra. c. Incorrecta. El enriquecimiento del trabajo implica su expansión vertical, para permitir al

trabajador controlar su planificación, ejecución y evaluación. d. Incorrecta. La variedad de trabajo no es una opción de rediseño del trabajo, sino una función.

51. Solución: d (I, II y III)

I. Correcta. Proveer retroalimentación sobre el desempeño es una parte importante de los enfoques basados en la meta para motivar a los empleados.

II. Correcta. La presentación de oportunidades para la responsabilidad motivará a los empleados ayudándolos a satisfacer la necesidad para el logro.

III. Correcta. La satisfacción de las necesidades personales es un método para motivar a los empleados.

52. Solución: a

a. Correcta. El filtro implica que el emisor de un mensaje manipule la información de manera tal que el receptor la vea más favorable.

b. Incorrecta. La percepción selectiva implica que los receptores interpreten de manera selectiva lo que ven u oyen sobre la base de sus intereses, antecedentes, experiencia y actitudes.

c. Incorrecta. Las emociones afectan a la interpretación del mensaje, no a los contenidos. d. Incorrecta. La elección del idioma implica la selección personal de palabras para comunicar el

mismo mensaje sin distorsionarlo. 53. Solución: c (I, II y IV solamente)

I. Correcta. El resumen hace que se considere el punto de enfoque del debate y clarifica las prioridades.

II. Correcta. La aclaración del contenido o proceso ayuda a los demás a explorar el problema y a hacer sus ideas más concretas.

III. Incorrecta. La evaluación puede abreviar la presentación de los puntos de vista y llevar a otra persona a creer que la mente de quien escucha ya está completa.

IV. Correcta. El énfasis muestra que quien escucha comprende cómo la otra persona siente y la estimula a expresar sus sentimientos.

54. Solución: d

a. Incorrecta. Escuchar las emociones posibilita la detección de emociones fuertes que inhiben la resolución de problemas racionales y la probabilidad de obtener consenso.

b. Incorrecta. Formular preguntas pensadas muestra que alguien está escuchando con suma atención y estimula a las personas a encontrar sus propias soluciones.

c. Incorrecta. Escuchar cómo una persona está resolviendo el problema permite sugerir comentarios sobre el proceso así como sobre el contenido.

d. Correcta. Al interrumpir al orador, aunque sea con buenas intenciones, el receptor puede inhibirlo para comunicarse y es probable que se precipite a llegar a conclusiones no garantizadas.

IV - 37

55. Solución: b a. Incorrecta. Ver respuesta “b”. b. Correcta. Son ejemplos de comunicación hacia abajo. c. Incorrecta. Ver respuesta “b”. d. Incorrecta. Ver respuesta “b”.

56. Solución: a (I y II solamente)

I. Correcta. La sobrecarga de información y la representación malentendida de los sentimientos y emociones se consideran desventajas de la comunicación electrónica. La sobrecarga de información, tal como la de los numerosos mensajes de correos electrónicos, puede llevar a pérdida de tiempo e ineficiencias, y se considera una desventaja de la comunicación electrónica.

II. Correcta. El correo electrónico no puede transmitir con exactitud el sentimiento y el tono que se propone la persona que inicia la comunicación y el receptor puede interpretarlo erróneamente. Se considera una desventaja de la comunicación electrónica.

III. Incorrecta. El tiempo reducido de transmisión se considera un resultado positivo de la comunicación electrónica.

IV. Incorrecta. En general, la comunicación electrónica da como resultado una pista por escrito adecuada (tal como el “correo enviado” guardado).

57. Solución: b

a. Incorrecta. No es la mejor medida para utilizar. Por ejemplo, un departamento que vende libros puede atender a gran cantidad de clientes o vender muchas unidades, pero no significa que resulte más productivo que un departamento que vende muebles.

b. Correcta. Una productividad crítica de interés la constituyen los ingresos por metro cuadrado. El espacio mínimo de la tienda es un recurso limitado cuya productividad debe analizarse.

c. Incorrecta. Ver respuesta “a”. d. Incorrecta. La cantidad de unidades almacenadas en un departamento dado no muestra nada

acerca de la productividad. 58. Solución: d

a. Incorrecta. Esta respuesta ilustra dos de las características del decrecimiento de la organización: la centralización incrementada de la toma de decisiones y la falta de planificación a largo plazo. Es probable que el énfasis exclusivo en los resultados a corto plazo resulte contraproducente.

b. Incorrecta. Otra característica del decrecimiento organizacional la constituyen las reducciones no prioritarias. La “downsizing” (reducción de personal) en sí misma rara vez hace cambiar a la compañía.

c. Incorrecta. Es un método demasiado precario para reducciones prioritarias. La reducción desproporcionada de personal en las funciones de control podría tener consecuencias desastrosas.

d. Correcta. Es una solución a largo plazo que contiene elementos necesarios para enfrentar el decrecimiento organizacional.

59. Solución: c

a. Incorrecta. No es una medida de eficiencia porque no hay ninguna comparación de aportes con resultados.

b. Incorrecta. Es un ejemplo de eficacia, no de eficiencia. c. Correcta. La eficiencia es la proporción del resultado eficaz con respecto al aporte requerido

para lograrlo. Las reclamaciones a aseguradoras que se procesan al día miden el resultado (los reclamos procesados) con respecto al aporte (un día de trabajo).

d. Incorrecta. No es una medida de eficiencia porque no hay ninguna comparación de aportes con resultados.

IV - 38

60. Solución: d (I, II, III y IV) I, II, III, IV. Correcta. Se requieren todas para alinear las metas gerenciales con las

organizacionales. 61. Solución: b

a. Incorrecta. Los tramos estrechos de control suelen encontrarse en organizaciones de gran envergadura con muchos más niveles.

b. Correcta. Los tramos estrechos de control implican que es más baja la proporción de los supervisados (subordinados) con respecto a los que realizan la supervisión (gerentes).

c. Incorrecta. Los tramos estrechos de control implican más gastos administrativos y menos auto-administración.

d. Incorrecta. Los tramos más amplios de control permiten a los subordinados mayor discreción. 62. Solución: b

a. Incorrecta. Si se requiere una planificación considerable, un gerente se beneficiaría de los requerimientos de supervisión reducida. Asimismo, la planificación incrementada implica un ambiente cambiante, en el cual el trabajo de los subordinados irá cambiando con el tiempo, lo cual requerirá un significativo trabajo gerencial para la capacitación.

b. Correcta. Si se requiere una coordinación considerable, un gerente se beneficiaría de los requerimientos de supervisión reducida. Más aún, la coordinación incrementada implica que el trabajo realizado por los subordinados no se encuentra estandarizado.

c. Incorrecta. Los subordinados geográficamente dispersos justificarían un tramo estrecho de control.

d. Incorrecta. Con un trabajo muy similar, los subordinados pueden capacitarse entre sí, proveer respaldo si un subordinado no se encuentra presente y verificar el trabajo de uno y de otro. Más aún, los procedimientos laborales resultan relativamente fáciles de documentar y un gerente puede estar bien informado sobre el trabajo de todos los que hagan algo similar.

63. Solución: d (I, II, III y IV)

I, II, III, IV. Correcta. Todos son aportes típicos para un proceso de evaluación de desempeño de 360 grados.

64. Solución: b

a. Incorrecta. Los individuos a los cuales se les asignó un trabajo en un call center tendría sólo un supervisor, que se encontraría ubicado en ese call center.

b. Correcta. Una organización matricial que designe a especialistas de acuerdo con lo que necesite para diversos proyectos resultaría apropiado para una compañía de esa índole.

c. Incorrecta. Si bien las ventas realizadas a través de múltiples canales requieren coordinación, no hay razón para creer que se trasladaría a personal de un canal de ventas a otro para trabajar en un proyecto, o que informarían a dos supervisores diferentes.

d. Incorrecta. El personal temporario no informaría a cualquier gerente que se encuentre dentro de un organismo temporario; simplemente trabajan para una amplia variedad de empleados, que tienen total autoridad. No necesariamente se trata de personal especializado y, si no resulta imprescindible, no se les paga.

65. Solución: c

a. Incorrecta. La facilidad en la toma de decisiones es una ventaja de la descentralización. b. Incorrecta. El incremento en la motivación de los gerentes es una ventaja de la

descentralización. c. Correcta. La uniformidad incrementada en las decisiones es una ventaja de la centralización. d. Incorrecta. La inmediatez en la resolución de problemas es una ventaja de la descentralización.

IV - 39

66. Solución: d a. Incorrecta. Es una característica distintiva de una estructura matricial. b. Incorrecta. Es una característica de la centralización. c. Incorrecta. Es una característica de la burocracia. d. Correcta. La subcontratación de las funciones principales del negocio a proveedores externos

es una característica de las estructuras en red. 67. Solución: d (I, II y III)

I, II, III. Correcta. La departamentalización puede ser geográfica o realizarse por funciones o productos.

68. Solución: d

a. Incorrecta. En realidad, los grupos pueden analizar problemas con mayor profundidad. b. Incorrecta. En realidad, puede intensificarse la aceptación de decisiones porque, en general,

los participantes dicen que los resultados son “de ellos” en lugar de decir que son “de los demás”.

c. Incorrecta. En realidad, los grupos pueden hacer un mejor trabajo de identificación de componentes importantes.

d. Correcta. Posiblemente, se trata de un problema importante asociado con la toma de decisiones en grupo. Cuando se disipa la responsabilidad, suele perderse. Por ese motivo, el grupo, en general, sólo aconseja, y una persona en particular, tal como un gerente de auditoría, toma la decisión final, con lo cual se logra poner el enfoque en la responsabilidad.

69. Solución: c

a. Incorrecta. El uso de criterios objetivos minimiza el posible impacto de la política organizacional.

b. Incorrecta. Cuando todos los empleados se benefician, la política organizacional tendría menor impacto que si las decisiones se tomaran de manera tal que beneficiaran a algunos empleados y no a otros.

c. Correcta. El uso de criterios subjetivos, tales como la actitud, más que de criterios objetivos, tal como el desempeño de las tareas, permite que la política organizacional tenga un mayor impacto.

d. Incorrecta. Ver respuesta “a”.

70. Solución: a a. Correcta. Las opiniones grupales se refieren a la tendencia de aceptar la voluntad de la

mayoría cuando existe un aporte individual que difiere de la opinión general. El concepto de opiniones grupales se refiere a una toma de decisiones inadecuada en un grupo. Los grupos que toman decisiones grupales no consideran todas las alternativas, y desean obtener la unanimidad a costa de decisiones de calidad. Las opiniones grupales tienen lugar cuando los grupos son altamente cohesivos y se encuentran bajo considerable presión para tomar una decisión de calidad.

b. Incorrecta. El consenso es conveniente, incluso con las opiniones grupales. c. Incorrecta. Las opiniones grupales no se limitan sólo a los grupos de determinado tamaño, y el

grado de opiniones grupales no es proporcional al tamaño del grupo. d. Incorrecta. Existen pocas alternativas o ninguna que aborde las opiniones grupales.

71. Solución: a

a. Correcta. Es una de las desventajas más importantes de la toma de decisiones grupales. b. Incorrecta. Esta tendencia no depende del hecho de que el problema se aborde mediante la

toma de decisiones grupales. c. Incorrecta. Una ventaja de la toma de decisiones grupales mediante la “brainstorming” o

tormenta de ideas es que se incrementa la creatividad. d. Incorrecta. Una ventaja de la toma de decisiones grupales mediante la “brainstorming” es que

se incrementa la diversidad de puntos de vista.

IV - 40

72. Solución: c a. Incorrecta. Un contribuyente es un miembro de un equipo orientado a las tareas que provee al

equipo buena información y datos técnicos y lo incita a fijar metas de alto desempeño. b. Incorrecta. Un colaborador une a todo el equipo y se muestra abierto y flexible a nuevas ideas,

además de desear trabajar fuera del rol definido y compartir el reconocimiento y el crédito con otros miembros.

c. Correcta. Un comunicador es un miembro orientado a las personas, se guía por el proceso y escucha con eficacia, desempeña el rol de facilitador y logra el consenso.

d. Incorrecta. Alguien desafiante se muestra cándido y abierto, cuestiona las metas del equipo, desea discrepar con el líder del equipo y estimula a la toma de riesgos bien concebida.

73. Solución: b

a. Incorrecta. Las quejas sobre “por qué no funcionará” casi siempre representan un obstáculo “aceptable” para un plan que tiene consecuencias de conductas inaceptables.

b. Correcta. Los miembros de grupos de trabajo cohesivos suelen ejercer presión para resistirse a los cambios que amenazan con separar al grupo.

c. Incorrecta. Las cuestiones de que se designe poco o mucho personal para una tarea representan los síntomas de una resistencia al cambio, pero no la causa real ni la raíz del problema.

d. Incorrecta. La mención de los factores de costo también representa un motivo “aceptable” para bloquear la implementación de un nuevo enfoque.

74. Solución: d

a. Incorrecta. Es un método para formar equipos. b. Incorrecta. Es un método para formar equipos. c. Incorrecta. Es un método para formar equipos. d. Correcta. Es un síntoma de opiniones grupales.

75. Solución: b

a. Incorrecta. Las decisiones grupales llevan a una falta de responsabilidad personal. b. Correcta. Una ventaja de la toma de decisiones grupales es que aquellos que participan en un

proceso grupal son los que más aceptan y respaldan las decisiones del grupo. Una desventaja es que lleva más tiempo tomar una decisión en un grupo, en comparación con un individuo que toma una decisión.

c. Incorrecta. Ver respuesta “b”. d. Incorrecta. Ver respuesta “b”.

76. Solución: b

a. Incorrecta. Los miembros buscan una estructura al comienzo del desarrollo del equipo, antes de lograr el “empowerment”.

b. Correcta. El monitoreo y la retroalimentación resultan vitales para mantener el nivel adecuado de “empowerment”.

c. Incorrecta. La tolerancia a problemas y errores forman parte del “empowerment”. d. Incorrecta. La tensión y el conflicto forman una parte normal del desarrollo del equipo.

77. Solución: b

a. Incorrecta. La definición del rol es un enfoque reconocido para formar un equipo. b. Correcta. El desarrollo de destrezas no es un enfoque reconocido y no debe ser el propulsor en

la formación del equipo. c. Incorrecta. El desarrollo de valores es un enfoque reconocido para formar un equipo. d. Incorrecta. Las relaciones interpersonales constituyen un enfoque reconocido para formar un

equipo.

IV - 41

78. Solución: d a. Incorrecta. Los equipos de alto desempeño reconocen el valor de todos los miembros. b. Incorrecta. Refleja la falta de diversidad característica de las opiniones grupales. c. Incorrecta. Los individuos tienden a mostrarse más adversos al riesgo que los equipos. d. Correcta. Este respaldo mutuo es característico de los equipos de alto desempeño.

79. Solución: b (I y II solamente)

I, II. Correcta. Las opciones I y II son elementos de destrezas interpersonales que incluyen la interacción, la cooperación y la habilidad para tratar a las personas.

III. Incorrecta. La opción III es un elemento de participación en equipo. 80. Solución: c

a. Incorrecta. Este escenario ilustra una organización fuera de equilibrio sin enfoque hacia la eficacia o la eficiencia.

b. Incorrecta. Este escenario ilustra una organización fuera de equilibrio sin demasiado enfoque hacia la eficacia.

c. Correcta. Este escenario ilustra una organización fuera de equilibrio que pone demasiado enfoque en la eficiencia.

d. Incorrecta. Este escenario ilustra una organización con perfecto equilibrio entre la eficiencia y la eficacia.

81. Solución: d

a. Incorrecta. El liderazgo recursivo es importante para lograr confianza. b. Incorrecta. Asegura actividades de alto valor. c. Incorrecta. La búsqueda de sinergias en la diversidad es un hábito eficaz de liderazgo. d. Correcta. El enfoque en el proceso interno es un hábito de administración y no de liderazgo.

82. Solución: a

a. Correcta. El liderazgo directivo provee la mayor satisfacción de los subordinados cuando un equipo se enfrenta a un conflicto interno permanente.

b. Incorrecta. El estilo de soporte es mejor cuando las tareas resultan sumamente estructuradas y hay un bajo nivel de estrés y rivalidad.

c. Incorrecta. El estilo participativo es más útil cuando los subordinados sienten que controlan sus propios destinos.

d. Incorrecta. El estilo orientado a los logros resulta óptimo para las situaciones en las cuales las tareas se encuentran ambiguamente estructuradas.

83. Solución: b

a. Incorrecta. El liderazgo directivo provee la mayor satisfacción de los subordinados cuando un equipo se enfrenta a un conflicto interno permanente.

b. Correcta. El estilo de soporte es mejor cuando las tareas resultan sumamente estructuradas y las relaciones de autoridad son claras y burocráticas.

c. Incorrecta. El estilo participativo es más útil cuando los subordinados sienten que controlan sus propios destinos.

d. Incorrecta. El estilo orientado a los logros resulta óptimo para las situaciones en las cuales las tareas se encuentran ambiguamente estructuradas.

IV - 42

84. Solución: a a. Correcta. El enfoque de la contingencia se basa en el principio de que no hay una sola manera

óptima de liderazgo. De acuerdo con ese principio, el estilo o conducta de liderazgo eficaz depende de los requerimientos de la situación. En consecuencia, el estilo del líder debe combinarse con la situación cambiando al líder o su conducta.

b. Incorrecta. El modelo de cuadrícula gerencial se basa en la presunción de que el estilo de gerencia en equipo es el más eficaz, cualquiera sea la situación. En consecuencia, no se adecua a la visión de la contingencia del liderazgo.

c. Incorrecta. El enfoque de conducta no pone el enfoque en la situación ni en la combinación entre el líder y la situación. En cambio, el enfoque está en la identificación de conductas de liderazgo. La cuestión de conductas contingentes (la combinación de conductas con la situación) no es fundamental para el enfoque de conductas.

d. Incorrecta. Un enfoque orientado a los logros implica metas desafiantes y no depende de la situación.

85. Solución: b

a. Incorrecta. El empleado no se encuentra preparado para un desafío importante. Es probable que el empleado fracase y los sentimientos del fracaso se oponen al desarrollo.

b. Correcta. El tamaño del proyecto, la guía firme de logros fijos y la fortaleza de un personal totalmente capacitado ayudará al empleado a tener éxito. Asimismo, el personal ayudará a capacitar al empleado, que debe aprender algunas destrezas de este proyecto y estar preparado para un proyecto un poco más desafiante.

c. Incorrecta. Podría ser un tanto útil, pero en una buena situación, la teoría es menos probable que la experiencia para desarrollar destrezas de liderazgo.

d. Incorrecta. Evitaría los problemas a corto plazo, pero no ayudaría al empleado a desarrollar las destrezas necesarias para seguir adelante.

86. Solución: b

a. Incorrecta. Los proyectos son temporarios y van surgiendo según el caso, y la mayoría de los desafíos implican personas y cuestiones técnicas/tecnológicas.

b. Correcta. Los proyectos se encuentran fuera de la jerarquía o cadena de comando organizacional normal; en consecuencia, un líder de proyecto no puede confiar en una autoridad directa para cumplir lo que deben hacer diversas partes de una organización que están participando en un proyecto.

c. Incorrecta. El análisis estadístico y el diseño de procesos son destrezas que, si fueran necesarias en un proyecto, serían empleadas por un subordinado, no por el gerente del proyecto.

d. Incorrecta. La administración estratégica es a largo plazo, general y comienza con relativamente pocas restricciones distintas de los recursos existentes en toda la organización, mientras la administración de proyectos es a corto plazo y pone el enfoque en un proyecto aprobado con recursos específicos.

87. Solución: a

a. Correcta. Es una descripción de un estilo de liderazgo de venta. b. Incorrecta. Un estilo de liderazgo de expresión provee instrucciones específicas y supervisa el

desempeño atentamente. c. Incorrecta. Un estilo de liderazgo participativo estimula el compartir ideas y facilita la toma de

decisiones. d. Incorrecta. Un estilo de liderazgo de delegación traspasa la responsabilidad de las decisiones y

la implementación.

IV - 43

88. Solución: c a. Incorrecta. El rol de “Pathfinding” o de exploración aborda el propósito, la misión y/o visión. b. Incorrecta. Si bien la conciencia del fraude podría considerarse parte del proceso de alineación,

no es la mejor respuesta porque es sólo un componente de un programa de fraude global. c. Correcta. El “empowering” es la mejor respuesta porque la capacitación para tomar conciencia

del fraude de acuerdo con lo observado en la pregunta provee a los individuos la información y las pautas que necesitan para tomar la responsabilidad y comprometerse a las tareas requeridas para lograr un propósito. La capacitación para tomar conciencia del fraude asegura que conozcan la política y la manera de informar cuestiones sospechosas.

d. Incorrecta. La capacitación no implica la modelación de la conducta. 89. Solución: c (I, II y III solamente)

I. Correcta. Las metas deben incluir un mejor desempeño del trabajo y potencial profesional. II. Correcta. El crecimiento, no la creación de la dependencia, es el resultado deseado. III. Correcta. Muchos programas de asesoramiento fracasaron cuando la relación entre el mentor y

el asesorado no fue voluntaria. IV. Incorrecta. La relación de asesoramiento puede resultar útil en todos los ambientes de trabajo.

90. Solución: d

a. Incorrecta. Con el estilo autocrático, los gerentes imponen decisiones a sus subordinados. b. Incorrecta. Con el estilo autoritario benevolente, el gerente toma todas las decisiones, pero

cree que los subordinados deben estar motivados por los premios y que el liderazgo se produce por una forma condescendiente de la relación amo-sirviente.

c. Incorrecta. Con el estilo consultivo, el gerente consulta con sus subordinados, tiene en cuenta sus puntos de vista y toma la decisión final.

d. Correcta. Con el estilo participativo, el líder y los seguidores toman la decisión en forma democrática mediante un consenso y los superiores tienen total confianza en sus subordinados.

91. Solución: a

a. Correcta. El acomodamiento es una técnica de resolución que tiene la meta de mantener relaciones armoniosas.

b. Incorrecta. Ver respuesta “a”. c. Incorrecta. Ver respuesta “a”. d. Incorrecta. Ver respuesta “a”.

92. Solución: a

a. Correcta. Los negociadores deben evaluar las mejores alternativas tanto para ellos mismos como para las otras partes a fin de determinar su fuerza relativa en el proceso de negociación. Si las alternativas no se encuentran disponibles o no resultan atractivas, una parte se encuentra bajo presión adicional para hacer el trabajo de negociación.

b. Incorrecta. Las cuestiones que se encuentran fuera del conflicto en sí mismo pueden impactar en las negociaciones.

c. Incorrecta. La flexibilidad puede asistir al negociador en la identificación de alternativas. d. Incorrecta. Puede requerirse investigación adicional a fin de comprender las alternativas de la

otra parte en su totalidad para la negociación.

IV - 44

93. Solución: a a. Correcta. En negociaciones futuras, no es muy probable que el oponente forzado trabaje con el

negociador para lograr metas mutuas. Por último, las negociaciones en las cuales una o ambas partes sienten que deben ganar a costa de la otra parte no crean una relación de confianza y cooperación.

b. Incorrecta. El negociador logró las metas de esta negociación. c. Incorrecta. A menudo, un enfoque de colaboración hacia una negociación tardará más debido

al tiempo que llevó comprender las necesidades y preocupaciones de la otra parte para luego resolver la cuestión para beneficio de ambas partes.

d. Incorrecta. Éste puede o no ser el caso, según el estilo de negociación preferido por la organización.

94. Solución: a

a. Correcta. Una persona analítica tiende a considerar los detalles y regirse por información basada en hechos.

b. Incorrecta. El incitar a una persona analítica puede provocar una resistencia incrementada. c. Incorrecta. Las personalidades analíticas toman decisiones basadas en hechos más que en las

emociones. d. Incorrecta. La persona analítica tiende a no disfrutar tanto de los juegos de la negociación

como otros tipos de personalidades. Si la otra parte cree que el negociador engaña, puede no desear cooperar o incluso puede detener las negociaciones por completo.

95. Solución: a

a. Correcta. Si no sabe la razón de la resistencia, el negociador no puede enfrentarla con eficacia. Por ejemplo, el saber si la otra parte está preocupada por la cuestión global o por un tema en particular impactará en la respuesta del negociador.

b. Incorrecta. En general, debe intentarse una resolución en ese momento. Sólo si una o ambas partes de la negociación necesitan tiempo adicional para evaluar nueva información o calmarse deben tomarse un descanso.

c. Incorrecta. El negociador debe, en primer lugar, trabajar con la otra parte para determinar la causa del desacuerdo.

d. Incorrecta. La investigación con respecto a la otra parte debe realizarse antes de la reunión inicial de la negociación.

96. Solución: c

a. Incorrecta. Normalmente, surgen debido a diferencias en el proceso de educación, cultura y tradiciones familiares y socialización. Esta fuente de conflicto suele tornarse sumamente emotiva y se enfrenta a insinuaciones morales.

b. Incorrecta. Esto surge de la interrupción en las comunicaciones de la organización. Esta fuente de conflicto no está cargada de emociones y, una vez corregida, hay poco resentimiento.

c. Correcta. Esto surge del conflicto de roles intra-individuales y del conflicto intergrupal como resultado de las responsabilidades funcionales de los gerentes que son interdependientes en una organización interfuncional.

d. Incorrecta. Éstos se amplifican por un ambiente estresante donde existen recursos escasos o reducidos, “downsizing” (reducción de personal), presiones competitivas o altos grados de incertidumbre.

IV - 45

97. Solución: b a. Incorrecta. Éste es el estilo de “soothing”, en el cual las metas se conceden a favor de la

armonía y porque se cree que el conflicto daña las relaciones. b. Correcta. El estilo de evitar, no hacer nada y retirarse implica alejarse de las cuestiones por las

cuales tiene lugar el conflicto y de las personas involucradas. Se cree que es imposible tratar de resolver conflictos y que resulta más fácil retirarse que enfrentar el conflicto.

c. Incorrecta. Éste es el estilo de forzar, en el cual se obliga a los oponentes a aceptar la solución. No existen preocupaciones con respecto a las necesidades de otras personas y en la negociación no importa si las soluciones resultan aceptables o no.

d. Incorrecta. Es el estilo de confrontación, en el cual se buscan soluciones que satisfagan a ambas partes y mantengan la relación. Asimismo, se describe como un estilo de colaboración o “de beneficio mutuo”. Implica que las partes conflictivas se reúnan cara a cara y colaboren para llegar a un acuerdo que satisfaga a ambas partes.

98. Solución: a

a. Correcta. Una estrategia de evitar se propone resolver el conflicto ignorando o imponiendo una solución. Resulta apropiada si el conflicto resulta trivial o si se necesita tomar una acción rápida para evitar que surja el conflicto.

b. Incorrecta. Con una estrategia de desactivación, se hace un intento por desactivar el conflicto y enfriar las emociones implicadas. Resulta apropiada cuando se necesita una medida temporaria y cuando los grupos tienen una meta importante para ambas partes.

c. Incorrecta. En una estrategia de contención, se permite que aflore algo del conflicto, pero se encuentra cuidadosamente contenido al especificar qué cuestiones deben debatirse y cómo deben resolverse.

d. Incorrecta. En una estrategia de confrontación, se ponen en escena todas las cuestiones y los grupos conflictivos confrontan las cuestiones directamente y se enfrentan entre sí a fin de llegar a una solución satisfactoria para ambas partes.

99. Solución: c

a. Incorrecta. La negociación tradicional comienza cuando una parte aborda a otra con una propuesta que se adecua a sus propias necesidades, pero cuando utiliza una negociación de valor agregado, la primera parte que inicia la negociación proveerá diversas propuestas.

b. Incorrecta. Las negociaciones de valor agregado evitan las ofertas y contraofertas poniendo el enfoque en soluciones beneficiosas para ambas partes.

c. Correcta. Cuando se utilizan técnicas de negociación de valor agregado, la parte que inicia siempre ofrece diversas alternativas para la negociación de manera tal que puedan llegar a una solución beneficiosa para ambas partes.

d. Incorrecta. Cuando se utilizan técnicas de valor agregado, cualquiera de las partes admitirá las demandas de la otra. En cambio, los negociadores de valor agregado deben hacer su mejor oferta al comienzo de las negociaciones.

100. Solución: b (I y III solamente)

I, III. Correcta. El método de negociación con principios se basa en poner el enfoque en intereses básicos, en opciones que satisfagan a ambas partes y en normas justas. Entre los principios básicos se incluyen: • Separar a las personas del problema. • Poner el enfoque en los intereses, no en las posiciones. • Inventar opciones para beneficio mutuo. • Insistir en el uso de criterios objetivos.

II, IV. Incorrecta. Ver respuestas I y III. FIN DE LAS SOLUCIONES PARTE IV